Vous êtes sur la page 1sur 269

.

ÉJ« AÖÞ @ ø BñÓ úG ñÜØ

Mamouni My Ismail
.

☞ : Professeur Agrégé (1995)-Docteur (2009) en Math


☞ : Master 1 (2011) en Sc de l’éducation, Univ. Rouen
☞ : MP-CPGE My Youssef, Rabat
☞ : Enseignant en Classes Prépas depuis 1995
✉ : mamouni.myismail@gmail.com
✍ : mamouni.new.fr
Feuille d’exercices-MP
-MP

P
P

-MP
Feeuui illes ’exercices-M
s-M
P

P
ercices
s-M

s-M
ices
P

P
c e
-M

M
e

ice
i
erc

erc
cic

s-

P
es

erc

ce

M
P
x
ic

d’ exer

-
ui illes s d’ex

ui lles d’ex

es
ci
e

M
c

-
s ’ex

ic
r

er

es P
d

sd
e

ic P c M
ex

ex

r -
s d’

s
lle d

e
Feuilles

c
e
s

-M
cic
r
ex
d’

s
euille

e e
FFeuuille

d’ exer -MP
ex cic
d’ exer -MP s
e

s
e d ’ ic e
ll

illlles ’exerc
s ’ c e s
e

e d i
illlles ’exerc
FF
F

u d P
F

i
e

FFeeuuilles ’exercices-M
u i d P
FFeeuuilles ’exercices-M
F e uilles d
e
e uilles d
F
F e F
Feuilles d’exercices-MP
Feuilles d’exercices-MP F
F euilles
e
F
d
euilles
FFeeuui illes ’exercices-M
F e d
FFeeuui illes ’exercices-M
ui lles d’ex P
FFeuuille

Feeuui illes ’exercices-M

ui lles d’ex
erc
F

P
lle d
F
F
e
F
Feuille d’exercices-MP
Feuilles

i
ui illes s d’ex

ui lles d’ex

s ’ex ces-
lle d erc
e

e
euille

i
MP
ces-
d’
ll

lle d

s ’ex
erc
e

ex
d’ erc MP
ice
s d’

s ’ex

ex
er s-M
ice
d’ exer

d’

ci
er s-M
s

sd
ex

ex

ci
ce P
d
e

er

P
s-
ce
r

M
s-
c

ci
e

erc
ic

P
M
x

ce
erc

erc
cic
e

ercices

P
s

s-
ice
-M

M
e

ices
c
s-M

s-M
P

P
es-M

-MP
P

P
-MP
P

c 2011
i

r
s
✍ M AMOUNI M Y I SMAIL
MAMOUNI . NEW. FR
P ROBLÈMES C ORRIGÉS -MP

i
s

s
u
Sommaire

s
é
Autour de la Comatrice. . . . . . . . . . . . . . . . . . . . . . . . . . . . . . . . . . . . . . . . . . . . . . . 1

u
1

1. 1 Énoncé. . . . . . . . . . . . . . . . . . . . . . . . . . . . . . . . . . . . . . . . . . . . . . . . . . . . . 1

r
2 Endomorphismes nilpotents.. . . . . . . . . . . . . . . . . . . . . . . . . . . . . . . . . . . . . . . . . . . . 3

é
2. 1 Énoncé (cnc 2005, TSI) . . . . . . . . . . . . . . . . . . . . . . . . . . . . . . . . . . . . . . . . . . . . . . 3
2. 2 Corrigé (Pr. Mamouni, CPGE My Youssef, Rabat) . . . . . . . . . . . . . . . . . . . . . . . . . . . . . . . . . . 5

Po r
r
3 Endomorphismes cycliques. . . . . . . . . . . . . . . . . . . . . . . . . . . . . . . . . . . . . . . . . . . . . 7
3. 1 Énoncé (extrait cnc 99, MP) . . . . . . . . . . . . . . . . . . . . . . . . . . . . . . . . . . . . . . . . . . . . 7

u
1 Endomorphismes nilpotents d’indice n et n-1 . . . . . . . . . . . . . . . . . . . . . . . . . . . . . . . . . . . . 10

uor
1. 1 Énoncé. . . . . . . . . . . . . . . . . . . . . . . . . . . . . . . . . . . . . . . . . . . . . . . . . . . . . 11
1. 2 Corrigé, Pr. Mamouni, CPGE My Youssef, Rabat . . . . . . . . . . . . . . . . . . . . . . . . . . . . . . . . . . 13
4 Matrices Stochastiques . . . . . . . . . . . . . . . . . . . . . . . . . . . . . . . . . . . . . . . . . . . . . . 16
4. 1 Énoncé (extrait cnc 2002, TSI) . . . . . . . . . . . . . . . . . . . . . . . . . . . . . . . . . . . . . . . . . . . 17

P
5 Décomposition Spectrale et racine carrée d’un endomorphisme . . . . . . . . . . . . . . . . . . . . . . . . . . . . 19
5. 1 Énoncé (extrait CCP PC 2010) . . . . . . . . . . . . . . . . . . . . . . . . . . . . . . . . . . . . . . . . . . . 19
5. 2 Corrigé (Pr. Blache, CPGE France) . . . . . . . . . . . . . . . . . . . . . . . . . . . . . . . . . . . . . . . . . 23
Recherche de quelques polynômes minimaux . . . . . . . . . . . . . . . . . . . . . . . . . . . . . . . . . . . . 31

r
1

1. 1 Énoncé : (extrait e3a 2008, MP) . . . . . . . . . . . . . . . . . . . . . . . . . . . . . . . . . . . . . . . . . . 31

e
1. 2 Corrigé Pr. Dufait, CPGE France . . . . . . . . . . . . . . . . . . . . . . . . . . . . . . . . . . . . . . . . . 35
Commutant d’une matrice . . . . . . . . . . . . . . . . . . . . . . . . . . . . . . . . . . . . . . . . . . . . . 41
er
6

6. 1 Énoncé : (extrait e3a 2011, MP) . . . . . . . . . . . . . . . . . . . . . . . . . . . . . . . . . . . . . . . . . . 41

m
6. 2 Corrigé, Pr. Stainer, Lycée Clemenceau, Nantes . . . . . . . . . . . . . . . . . . . . . . . . . . . . . . . . . . . 43
1 Diagonalisation du crochet de Lie . . . . . . . . . . . . . . . . . . . . . . . . . . . . . . . . . . . . . . . . . 45
1. 1 Enoncé : CNC 2000, MP . . . . . . . . . . . . . . . . . . . . . . . . . . . . . . . . . . . . . . . . . . . . . 45
m

1. 2 Corrigé, Pr. Mamouni, CPGE My Youssef, Rabat . . . . . . . . . . . . . . . . . . . . . . . . . . . . . . . . . . 49

r
7 Topologie des matrices . . . . . . . . . . . . . . . . . . . . . . . . . . . . . . . . . . . . . . . . . . . . . . 56

o
7. 1 Énoncé (extrait CNC) . . . . . . . . . . . . . . . . . . . . . . . . . . . . . . . . . . . . . . . . . . . . . . 56
or

F
✉: mamouni.myismail@gmail.com

1
i

r
s
P ROBLÈMES C ORRIGÉS -MP ✍ M AMOUNI M Y I SMAIL MAMOUNI . NEW. FR

i
s

s
u
8 Problème du point fixe. . . . . . . . . . . . . . . . . . . . . . . . . . . . . . . . . . . . . . . . . . . . . . . 60

s
2 Topologie des racines carrés d’une matrice. . . . . . . . . . . . . . . . . . . . . . . . . . . . . . . . . . . . . . 62
2. 1 Enoncé : CCP 2005, MP . . . . . . . . . . . . . . . . . . . . . . . . . . . . . . . . . . . . . . . . . . . . . 62

u
2. 2 Corrigé, Pr. Devulder, CPGE France . . . . . . . . . . . . . . . . . . . . . . . . . . . . . . . . . . . . . . . . 66
Algèbres de dimension 2 . . . . . . . . . . . . . . . . . . . . . . . . . . . . . . . . . . . . . . . . . . . . . 71

r
9

é
9. 1 Énoncé : CNC 96 . . . . . . . . . . . . . . . . . . . . . . . . . . . . . . . . . . . . . . . . . . . . . . . . 71
9. 2 Corrigé, Pr. Med. El Fatemi, CPGE CPR Tanger . . . . . . . . . . . . . . . . . . . . . . . . . . . . . . . . . . . 76
10 matrices positives et décomposition de Cholesky. . . . . . . . . . . . . . . . . . . . . . . . . . . . . . . . . . . 82

Po r
r
10 .1Énoncé : CCP 2003, PSI. . . . . . . . . . . . . . . . . . . . . . . . . . . . . . . . . . . . . . . . . . . . . . 82

u
10 .2Corrigé, Pr. J. DEBARBIEUX, Lycée Faidherbe, Lille . . . . . . . . . . . . . . . . . . . . . . . . . . . . . . . . . 88

uor
3 Espaces d’Artin et applications . . . . . . . . . . . . . . . . . . . . . . . . . . . . . . . . . . . . . . . . . . . 95
3. 1 Énoncé : Extrait Centrale 2010, MP . . . . . . . . . . . . . . . . . . . . . . . . . . . . . . . . . . . . . . . . 96
3. 2 Corrigé : Pr. Boujaida, CPGE Rabat . . . . . . . . . . . . . . . . . . . . . . . . . . . . . . . . . . . . . . . . 102
11 Coniques-Quadriques. . . . . . . . . . . . . . . . . . . . . . . . . . . . . . . . . . . . . . . . . . . . . . . 116

P
11 .1Exercice 1 : Extrait e3a 2009, MP . . . . . . . . . . . . . . . . . . . . . . . . . . . . . . . . . . . . . . . . . . 117
11 .2Exercice 2 : Extrait e3a 2004, MP . . . . . . . . . . . . . . . . . . . . . . . . . . . . . . . . . . . . . . . . . . 118
11 .3Corrigé Exericie 1, Pr. Dufait . . . . . . . . . . . . . . . . . . . . . . . . . . . . . . . . . . . . . . . . . . . 121
11 .4Corrigé Exericie 2, Mma Gayout. . . . . . . . . . . . . . . . . . . . . . . . . . . . . . . . . . . . . . . . . . 123

r
12 Fonctions harmoniques . . . . . . . . . . . . . . . . . . . . . . . . . . . . . . . . . . . . . . . . . . . . . . 125
Énoncé : Mines-Ponts 2004, MP . . . . . . . . . . . . . . . . . . . . . . . . . . . . . . . . . . . . . . . . . . 125

e
12 .1

13 Courbes & Surfaces . . . . . . . . . . . . . . . . . . . . . . . . . . . . . . . . . . . . . . . . . . . . . . . . 130


er
Énoncé : e3a 2009, PSI . . . . . . . . . . . . . . . . . . . . . . . . . . . . . . . . . . . . . . . . . . . . . . 130

m
13 .1

13 .2Corrigé : Pr Skler, CPGE France . . . . . . . . . . . . . . . . . . . . . . . . . . . . . . . . . . . . . . . . . . 132


14 Calcul Différentiel, Courbes & Surfaces. . . . . . . . . . . . . . . . . . . . . . . . . . . . . . . . . . . . . . . 136
Problème : extrait X 2004, MP . . . . . . . . . . . . . . . . . . . . . . . . . . . . . . . . . . . . . . . . . . . 136
m

14 .1

Problème : Mines-Ponts 2004, MP . . . . . . . . . . . . . . . . . . . . . . . . . . . . . . . . . . . . . . . . . 138

r
14 .2

14 .3Corrige Problème I : Pr. Duval, CPGE France . . . . . . . . . . . . . . . . . . . . . . . . . . . . . . . . . . . . 141

o
14 .4Corrigé Problème II : Pr. Patte, CPGE France . . . . . . . . . . . . . . . . . . . . . . . . . . . . . . . . . . . . 143
or

F
✉: mamouni.myismail@gmail.com

2
i

r
s
✍ M AMOUNI M Y I SMAIL
MAMOUNI . NEW. FR
P ROBLÈMES C ORRIGÉS -MP

i
s

s
u
15 Calcul Différentiel, Courbes & Surfaces. . . . . . . . . . . . . . . . . . . . . . . . . . . . . . . . . . . . . . . 145

s
Problème : extrait cnc 2007, PSI . . . . . . . . . . .
15 . 1 . . . . . . . . . . . . . . . . . . . . . . . . . . . . . . . 145
Exercice I : extrait e3a 2007, MP . . . . . . . . . . . . . . . . . . . . . . . . . . . . . . . . . . . . . . . . . . 147

é
15 . 2

u
Exercice II : extrait e3a 2008, MP. . . . . . . . . . .
15 . 3 . . . . . . . . . . . . . . . . . . . . . . . . . . . . . . . 149
Corrigé Problème : Pr. Chabchi, CPGE Marrakech . . . . . . . . . . . . . . . . . . . . . . . . . . . . . . . . . . 150

r
15 . 4

Corrigé Exercice I : Pr. Deyris, CPGE France . . . . . . . . . . . . . . . . . . . . . . . . . . . . . . . . . . . . 152

é
15 . 5

Corrigé Exercice II : Pr. Lemaire, CPGE France . . . .


15 . 6 . . . . . . . . . . . . . . . . . . . . . . . . . . . . . . . 153
16 Valeur moyenne d’une fonction . . . . . . . . . . . . . . . . . . . . . . . . . . . . . . . . . . . . . . . . . . 156

Po r
r
Enoncé : CNC 2006, MP . . . . . . . . . . . . . .
16 . 1 . . . . . . . . . . . . . . . . . . . . . . . . . . . . . . . 156
Corrigé : Pr. Mamouni, CPGE Rabat, Maroc. . . . . . . . . . . . . . . . . . . . . . . . . . . . . . . . . . . . . 160

u
16 . 2

17 Transformée de Fourier, de Lebesgue. Séries de Dirichlet . . . . . . . . . . . . . . . . . . . . . . . . . . . . . . . 169

uor
Enoncé : CCP 2011, PSI. . . . . . . . . . . . . . .
17 . 1 . . . . . . . . . . . . . . . . . . . . . . . . . . . . . . . 169
Pr. Verschueren, Lycée Daudet à Nîmes . . . . . . .
17 . 2 . . . . . . . . . . . . . . . . . . . . . . . . . . . . . . . 173
18 Suites et séries de fonctions . . . . . . . . . . . . . . . . . . . . . . . . . . . . . . . . . . . . . . . . . . . . 182
Problème I : Exemple d’étude . . . . . . . . . . . . . . . . . . . . . . . . . . . . . . . . . . . . . . . . . . . 182

P
18 . 1

Problème II : Fonction ζ et η de Riemann.. . . . . . .


18 . 2 . . . . . . . . . . . . . . . . . . . . . . . . . . . . . . . 183
Corrigé : Pr. Deyris, CPGE France . . . . . . . . . .
18 . 3 . . . . . . . . . . . . . . . . . . . . . . . . . . . . . . . 190
19 Équations de Bessel et la fonction Gamma. . . . . . . . . . . . . . . . . . . . . . . . . . . . . . . . . . . . . . 194
Énoncé : CNC 2007, MP . . . . . . . . . . . . . . . . . . . . . . . . . . . . . . . . . . . . . . . . . . . . . 194

r
19 . 1

Corrigé : Pr. Taibi, CPGE Rabat, Maroc . . . . . . . .


19 . 2 . . . . . . . . . . . . . . . . . . . . . . . . . . . . . . . 198

e
20 Fonction Gamma et Psi d’Euler. . . . . . . . . . . . . . . . . . . . . . . . . . . . . . . . . . . . . . . . . . . 207
Problème I : Extrait CCP 2006, MP . . . . . . . . . . . . . . . . . . . . . . . . . . . . . . . . . . . . . . . . . 207
er
20 . 1

Problème II : Extrait e3a 2011, MP . . . . . . . . . . . . . . . . . . . . . . . . . . . . . . . . . . . . . . . . . 210

m
20 . 2

Corrigé Problème II : Pr. Patte. . . . . . . . . . . .


20 . 3 . . . . . . . . . . . . . . . . . . . . . . . . . . . . . . . 212
21 Ev. Euclidiens : Les "must" Classiques. . . . . . . . . . . . . . . . . . . . . . . . . . . . . . . . . . . . . . . . 214
Matrice de Gram. . . . . . . . . . . . . . . . . . . . . . . . . . . . . . . . . . . . . . . . . . . . . . . . . 214
m

21 . 1

Décomposition polaire . . . . . . . . . . . . . . . . . . . . . . . . . . . . . . . . . . . . . . . . . . . . . . 215

r
21 . 2

Endomorphismes normaux. . . . . . . . . . . . .
21 . 3 . . . . . . . . . . . . . . . . . . . . . . . . . . . . . . . 216

o
Décomposition de Cholesky, QR. . . . . . . . . . .
21 . 4 . . . . . . . . . . . . . . . . . . . . . . . . . . . . . . . 216
or

F
✉: mamouni.myismail@gmail.com

3
i

r
s
P ROBLÈMES C ORRIGÉS -MP ✍ M AMOUNI M Y I SMAIL MAMOUNI . NEW. FR

i
s

s
u
21 .5F 6= E mais F⊥ = {0E }. . . . . . . . . . . . . . . . . . . . . . . . . . . . . . . . . . . . . . . . . . . . . . 216

s
Rayleigh, Courant Fischer . . . . . . . . . . . . . . . . .
21 .6 . . . . . . . . . . . . . . . . . . . . . . . . . . . 217
22 Réduction simultanée et décomposition de Cholesky . . . . . . . . . . . . . . . . . . . . . . . . . . . . . . . . . 218

u
Enoncé : Extrait CCP 2011, MP . . . . . . . . . . . . . . .
22 .1 . . . . . . . . . . . . . . . . . . . . . . . . . . . 218
23 Phénomène de Gibbs . . . . . . . . . . . . . . . . . . . . . . . . . . . . . . . . . . . . . . . . . . . . . . . 224

r
Énoncé : CCP 2006, MP . . . . . . . . . . . . . . . . . . . . . . . . . . . . . . . . . . . . . . . . . . . . . 225

é
23 .1

Corrigé : Pr. Boujaida, CPGE Rabat, Maroc . . . . . . . . . .


23 .2 . . . . . . . . . . . . . . . . . . . . . . . . . . . 229
24 Équations différentielles : Exemple d’étude. . . . . . . . . . . . . . . . . . . . . . . . . . . . . . . . . . . . . . 235

Po r
r
Énoncé : CNC 2004, MP . . . . . . . . . . . . . . . . . .
24 .1 . . . . . . . . . . . . . . . . . . . . . . . . . . . 235

u
Corrigé : Pr. Mamouni, CPGE Rabat, Maroc. . . . . . . . . .
24 .2 . . . . . . . . . . . . . . . . . . . . . . . . . . . 239
25 Fonctions holomorphes : Série de Fourier lacunaire quadratique . . . . . . . . . . . . . . . . . . . . . . . . . . . . 252

uor
Énoncé : CNC MP, 2011 . . . . . . . . . . . . . . . . . .
25 . 1 . . . . . . . . . . . . . . . . . . . . . . . . . . . 252
Corrigé : Pr. Hfa, CPGE Agadir, Maroc . . . . . . . . . . . .
25 . 2 . . . . . . . . . . . . . . . . . . . . . . . . . . . 257
26 Formule de Green-Rieman & Intégrale de Dirichlet . . . . . . . . . . . . . . . . . . . . . . . . . . . . . . . . . 269
Énoncé : e3a 2009, PSI . . . . . . . . . . . . . . . . . . . . . . . . . . . . . . . . . . . . . . . . . . . . . . 269

P
26 . 1

Corrigé : Pr. Devulder, CPGE France . . . . . . . . . . . . .


26 . 2 . . . . . . . . . . . . . . . . . . . . . . . . . . . 273

e r
er

m
m

or
or

F
✉: mamouni.myismail@gmail.com

4
i

r
s
✍ M AMOUNI M Y I SMAIL
MAMOUNI . NEW. FR
P ROBLÈMES C ORRIGÉS -MP

i
s

s
u

s
Devoir Libre
1 Autour de la Comatrice.

u
Jacques Salomon Hadamard (1865-1963)

r
Blague du jour

Mathématicien du jour
é
Mathématicien français, connu pour ses travaux en théorie des
• Pourquoi les vaches ne parlent pas ? - C’est marqué la nombres et en cryptologie. En 1884, il entra premier l’école nor-

Po r
r
ferme. x
• Deux vaches discutent dans un pré : male supérieure. Son résultat le plus célèbre est π ( x ) +∞ où
∼ ln x

u
La première : Dis, ça t’inquiète pas, ces histoires de π ( x ) désigne le nombre de nombres premiers inférieurs x. Il a

uor
vaches folles dont on parle en ce moment ? laissé son nom aux matrices de Hadamard utilisés en algorithmes
La deuxième : Non, moi, je m’en fou, je suis un lapin. quantiques, traitement du signal, compression de donnes, ...

P
b On suppose que A est non inversible.
Énoncé
i Montrer que ∃α 6= 0 tel que ∀0 < ε < α, on a A − εIn ,

r
① Soit A ∈ Mn (R ) triangulaire supérieure.
non inversible.
a On suppose que A est inversible. ii En déduire que com( A) est triangulaire inférieure.

e
Soit f ∈ L(R n ), associé A dans la base canonique B =
② Soit A ∈ Mn (R ).
er
(e1 , · · · , en ), on pose Fk = Vect(e1 , · · · , ek ) pour tout k ∈
Montrer que : si rg( A) = n alors rg(com( A)) = n

m
{1, . . . , n}.
si rg( A) = n − 1 alors rg(com( A)) = 1
i Montrer que f ( Fk ) = Fk . si rg( A) ≤ n − 2 alors com( A) = 0
ii En déduire que f −1 ( Fk ) = Fk .
m

☛ Indication : On pourra utiliser le résultat suivant, dit

r
théorème de Rouché-Fontené :
iii En déduire que A−1 est triangulaire supérieure. Si A ∈ Mn,p (R ) tel que rg( A) = r, alors il existe une matrice

o
iv En déduire que com( A) est triangulaire inférieure. carrée B ∈ Mr (R ) extraite de A qui soit inversible.
or

F
✉: mamouni.myismail@gmail.com

1
i

r
s
P ROBLÈMES C ORRIGÉS -MP ✍ M AMOUNI M Y I SMAIL
MAMOUNI . NEW. FR

i
s

s
u
③ Exprimer com (λA). en fonction de λ, n et com ( A). com ( AB) = (com A)(com B) et com ( A−1 ) = com ( A)−1 .

s
④ Calculer com (com A) dans le cas où A est inversible. c Démontrer le même résultat dans le cas général, en con-

é
sidérant des scalaires λ tels que A − λI et B − λI soient in-

u
⑤ Soit n ≥ 2 et A ∈ Mn (R ).
versibles.
a Calculer com ( In ).

r
d En déduire que si A et B sont semblables, alors com A

é
b Si A et B sont inversibles, démontrer que et com B le sont aussi.

Po r
r
u
uor
i
F

n
i

P
Á la prochaine

e r
er

m
m

or
or

F
✉: mamouni.myismail@gmail.com

2
i

r
s
✍ M AMOUNI M Y I SMAIL
MAMOUNI . NEW. FR
P ROBLÈMES C ORRIGÉS -MP

i
s

s
u

s
Devoir Libre
2 Endomorphismes nilpotents.

u
r

é
Blague du jour Pafnouti Lvovitch Tchebychev (1821-1894)

Po r
r
Un homme regarde un match de foot dans un café,

Mathématicien du jour
Mathématicien russe, connu pour ses travaux dans le domaine
lorsque son équipe nationale marque un but, le chien des probabilités et des statistiques. Tchebychev appartient à

u
se met à courir dans tout les sens. Le voisin demande l’école mathématique russe fondée par Daniel Bernoulli et Euler.

uor
à l’homme : Qu’est ce qui lui arrive votre chien ? Il démontra en 1850 une conjecture énoncée par Bertrand : Pour
- Il est supporter de l’équipe nationale, il est content. tout entier n au moins égal à 2, il existe un nombre premier
- Ben dites donc, juste pour un but ! Et qu’est-ce-qu’il fait entre n et 2n.
quand elle gagne un match ? ! !

P
- Je ne sais pas, je ne l’ai que depuis 5 ans...

r
b Montrer que la famille ( x0 , u( x0 ), · · · , u p−1 ( x0 )) est li-
Énoncé (cnc 2005, TSI)

e
bre.
er
E désigne un espace vectoriel réel de dimension finie n ≥ 2. L’iden- c En déduire que p ≤ n et que un = 0.

m
tité est notée IE . Pour u ∈ L(E), on pose u0 = IE .
On considère un endomorphisme nilpotent u de E, c’est à dire un ② On suppose qu’il existe v ∈ L(E) tel que v2 = u.
endomorphisme tel qu’il existe r ∈ N ∗ avec ur = 0 ; on pose alors n+1
n o a Calculer v2p et v2( p−1) , puis en déduire que p ≤ .
m

p = min k ∈ N ∗ /uk = 0 . 2

r
b Donner alors un exemple de matrice M ∈ M2 (R ) telle

o
p−1
① a Justifier qu’il existe x0 ∈ E tel que u ( x0 ) 6= 0. que l’équation X 2 = M n’ait pas de solution dans M2 (R ).
or

F
✉: mamouni.myismail@gmail.com

3
i

r
s
P ROBLÈMES C ORRIGÉS -MP ✍ M AMOUNI M Y I SMAIL
MAMOUNI . NEW. FR

i
s

s
u
q
Dans cette question, on suppose que p = n ; on a donc
∑ αk αq−k = 0 pour 2 ≤ q ≤ n − 1 (si n ≥ 3).

s
un−1 6= 0 et un = 0. On considère un endomorphisme g de E k=0
tel que g2 = IE + u.

u
d Montrer alors que α0 ∈ {−1, 1} et que, pour tout
a Soit x1 ∈ E tel que un−1 ( x1 ) 6= 0. Justifier que k ∈ {1, · · · , n − 1}, αk peut être exprimé de manière unique

r
( x1 , u( x1 ), · · · , un−1 ( x1 )) est une base de E et qu’il existe en fonction de α0 .

é
n −1
(α0 , · · · , αn−1 ) ∈ R n tel que g( x1 ) = e Conclure qu’il y a exactement deux endomorphismes de
∑ α k u k ( x 1 ).
E dont le carré est égal à IE + u.

Po r
r
k=0
n −1
③ ☛ :Application : Déterminer toutes les matrices X ∈ M4 (R )

u
b Vérifier que g ◦ u = u ◦ g et montrer que g = ∑ α k uk . 1 1 0 0

k=0

uor
 0 1 1 0 
c Justifier que la famille ( IE , u, · · · , un−1 ) est libre puis, en telles que X 2 = 
 0 0 1 1 

calculant g2 de 2 façons, montrer que α20 = 1, 2α0 α1 = 1 et 0 0 0 1

r P
i
F
n

e
i
n
er

m
Á la prochaine
m

or
or

F
✉: mamouni.myismail@gmail.com

4
i

r
s
✍ M AMOUNI M Y I SMAIL
MAMOUNI . NEW. FR
P ROBLÈMES C ORRIGÉS -MP

i
s

s
u
.

s
pour M ∈ L(R2 ), d’où suivant la question précédente si

é
Corrigé (Pr. Mamouni, CPGE My Youssef, Rabat)

u
3
X 2 = M, on devrait avoir p , ce qui n’est pas le cas, donc
2

r
l’équation X 2 = M, n’admet pas de solutions.

é
① a p = min{k ∈ N ∗ tel que uk = 0}, donc u p−1 6= 0, et par ③ a De la même façon que dans la question 1.2), on montre

Po r
r
suite ∃ x0 ∈ E tel que u p−1 ( x0 ) 6= 0. que la famille ( x1 , u( x1 ), . . . , un−1 ( x1 )) est libre, or son cardi-
nal est égal à n = dim(E), donc c’est une base, et pas suite

u
b Soit (λi )0≤i ≤ p tel que λ0 x0 + λ1 u( x0 ) + . . . + c’est une famille génératrice de E, or g( x1 ) ∈ E, d’où l’ex-

uor
λ p−1 u p−1 ( x0 ) = 0, on compose par u p−1 et comme uk = istence de nombres réels (αi )0≤i ≤n−1 tel que g( x1 ) = α0 x1 +
0, ∀k ≥ p, alors λ0 u p−1 ( x0 ) = 0, or u p−1 ( x0 ) 6= 0, d’où α 1 u ( x 1 ) + . . . + α n − 1 u n − 1 ( x 1 ).
λ0 = 0, ce qui donne λ1 u( x0 ) + . . . + λ p−1 u p−1 ( x0 ) = 0, on b g2 = u + IE , d’où u = g2 − IE et donc gu = g3 − g = ug.
compose cette fois par u p−2 , ce qui donne

P
Et par récurrence sur k ∈ N, on montre que guk = uk g.
λ1 u p−1 ( x0 ) = 0, d’où λ1 = 0 et on re-itère le même procédé
D’autre
 part on a les égalités suivantes :
jusqu’à montrer que tous les λi sont nuls. D’où la famille
 g ( x1 ) = α0 x1 + α1 u ( x1 ) + . . . + α n −1 u n −1 ( x1 )
C = ( x0 , u( x0 ), . . . , u p−1 ( x0 )) est libre. 

 gu( x ) =
1 u( g( x1 )) = α0 u( x1 ) + α1 u(u( x1 )) + . . . + αn−1 un−1 (

r
c C est libre, donc card(C) = p ≤ dim(E) = n, or u p = 0  ..

 .
et n ≥ p, d’où un = 0. 
gun−1 ( x1 ) = un−1 ( g( x1 )) = α0 un−1 ( x1 ) + . . . + αn−1 un−1 (un−1 ( x1

e
② a v2p = (v2 ) p = u p = 0 et v2( p−1) = u p−1 6= 0. Ainsi g et α0 IE + . . . + αn−1 un−1 coïncident sur la base
er
( x1 , u( x1 ), . . . , un−1 ( x1 )), et comme elles sont linéaires elles

m
Posons : q = min{k ∈ N ∗ tel que vk = 0}, donc 2( p − 1) <
coïncident sur E.
q ≤ 2p, et comme dans ce qui précède pour u, on peut aussi
affirmer pour v que q ≤ n, ainsi 2( p − 1) + 1 ≤ q ≤ n, d’où c Soit (λi )0≤i ≤n tel que λ0 IE + λ1 u + . . . + λ p−1 un−1 = 0,
n+1 on applique cette relation à x1 , on trouve λ0 ( x1 ) + λ1 u( x1 ) +
m

2p − 1 ≤ n, d’où p ≤ .
. . . + λ p−1 un−1 ( x1 ) = 0, or la famille ( x1 , u( x1 ), . . . , un−1 ( x1 ))

r
  2
0 1 est libre,
b Soit = . On a M2 = 0 et M = 0, donc p = 2,

o
0 0 d’où λi = 0, ∀1 ≤ i ≤ n, et donc ( IE , u, . . . , un−1 ) est libre.
or

F
✉: mamouni.myismail@gmail.com

5
i

r
s
P ROBLÈMES C ORRIGÉS -MP ✍ M AMOUNI M Y I SMAIL
MAMOUNI . NEW. FR

i
s

s
u
1 ére façon : g2 = IE + u. or 1 ≤ k ≤ q − 1 et 1 ≤ q − k ≤ q − 1, d’où les αk αq−k s’ex-

s
!2
n priment de façon unique en fonction de α0 , donc leur somme
2 ème façon : g2 = ∑ α k uk aussi, et par la suite 2αq α0 aussi et finalement αq aussi.

u
k=0 !
n q e Les solutions, g de l’équation g2 = IE + u, sont de la
q
∑ ∑ αk αq−k

r
= u n
forme g = ∑ αk uk , or ∀q ∈ {1, . . . , n}, αq s’exprime de façon

é
q =0 k=0
n q k=0
= ∑ β k uq Avec : β k = ∑ αk αq−k unique en fonction de α0 ∈ {−1, 1}. Donc deux possibilités

Po r
r
q =0 k=0 suivant la valeur prise par α0 .
Et par identification puisque la famille ( IE , u, . . . , un−1 ) est 2

u
④ L’équation
 peut s’écrire sous la forme X = I1 + A, avec :
libre, on a alors : β 0 = α20 = 1, β 1 = 2α0 α1 = 1 et β q =
0 1 0 0

uor
0, ∀q ≥ 2.  0 0 1 0  4 3
A =  
 0 0 0 1 , qui vérifie A = 0 et A 6= 0, donc
d α20 = 1, donc α0 ∈ {−1, 1}.
Montrons par récurrence sur q ∈ {1, . . . , n}, que αq s’exprime 0 0 0 0
de façon unique en fonction de α0 . X = α0 I4 + α1 A + α2 A2 + α3 A3 , avec les relations suivantes :

P
1 α0 ∈ {−1, 1} 2α0 α1 = 1
Pour q = 1, on a : α1 = , donc le résultat est vrai pour
2α0 2α0 α2 + α21 = 0 2α0 α3 + 2α1 α2 = 0
q = 1, supposons qu’il est vrai jusqu’à l’ordre q − 1, et mon- Les
 solutions possibles sont :
trons que c’est vrai pour q.  1 1 1

r
 α0 = 1 , α1 = , α2 = − , α3 =
q q −1 2 4 8
En effet ∑ αk αq−k = 0, donc 2αq α0 = − ∑ αk αq−k,  1 1 1

e
 α0 = −1 , α1 = − , α2 = , α3 = −
k=0 k=1 2 4 8
er

m
i
F
n
i
n
m

r
Á la prochaine

o
or

F
✉: mamouni.myismail@gmail.com

6
i

r
s
✍ M AMOUNI M Y I SMAIL
MAMOUNI . NEW. FR
P ROBLÈMES C ORRIGÉS -MP

i
s

s
u

s
Devoir Libre
3 Endomorphismes cycliques.

u
r
Blague du jour

é
Arthur Cayley (1821-1895)

Mathématicien du jour
Un vieux milliardaire téléphone à une conseillère : J’ai
Mathématicien et avocat britannique, l’un des fondateurs de l’é-

Po r
60 ans et je veux me marier avec une jeune fille de 20

r
ans. Pensez-vous que j’aie plus de chance de l’amener cole britannique moderne de mathématiques pures. Il est le pre-

u
à m’épouser si je lui dis il y a quelques année, j’avais mier à introduire la multiplication des matrices. Il a donné le
juste 50 ans ? La conseillère lui répond : A mon avis, vous premier, une définition qui s’approche de la notion moderne de

uor
feriez mieux de lui dire que quelques année, vous ap- groupe. Il a reçu la Médaille Copley en 1882. On lui doit aussi la
prochez des 80 ans ! découverte des nombres de Cayley, les octonions.

P
② Montrer que f k ( x0 ) est combinaison linaire de F pour tout
Énoncé (extrait cnc 99, MP) entier k ≥ p.

Dans tout le problème K est un sous-corps de C et E désigne un ③ En déduire que B = x0 , f ( x0 ), . . . , f n−1 ( x0 ) est une base de

r
K-espace vectoriel de dimension finie n ≥ 2. On dit que f ∈ L(E) E.
④ En déduire que deg π f = n, comparer π f et χ f .

e
est cyclique (ou monogène) s’il existe un vecteur x0 ∈ E tel que

la famille f k ( x0 ) k∈N engendre E. Dans tout le problème (sauf ⑤ On ne suppose plus f cyclique, mais que deg π f = n et on se
er
mention du contraire) f est un endomorphisme cyclique. propose de montrer que f est effectivement cyclique.

m
Partie I : Propriété caractéristique des endomorphismes cycliques a Pour tout x ∈ E, on note par I x l’ensemble des
polynômes P ∈ K [ X ] tels que P(u)( x ) = 0. Montrer que
m

I x est un idéal non nul de K[ X ], engendré par un unique

r
polynôme unitaire, qu’on notera π f ,x .
① Justifier l’existence  de l’entier p maximal tel que F =

o
x0 , . . . , f p−1 ( x0 ) soit libre. b Dire pourquoi π f ,x = π f .
or

F
✉: mamouni.myismail@gmail.com

7
i

r
s
P ROBLÈMES C ORRIGÉS -MP ✍ M AMOUNI M Y I SMAIL
MAMOUNI . NEW. FR

i
s

s
u
c En déduire que f est cyclique. Soit u ∈ L(()E) et x ∈ E avec x 6= 0. On pose Eu ( x ) =

s
{ P(u)( x ); P ∈ K[ X ]}.
⑥ Montrer que les assertions suivantes sont équivalents :

é
① Montrer que Eu ( x ) est un sous-espace vectoriel de E, stable

u
a f est cyclique.
par u et non réduit à {0}.
On note par ux l’endomorphisme induit par u sur Eu ( x ).

r
b deg πu = n.

é
② Si x ∈ ker u, donner la forme générale des éléments de Eu ( x ),
c χu = (−1)n πu . donner en particulier dim Eu ( x )

Po r
r
d ∃ x0 ∈ E tel que ( x0 , f ( x0 ), . . . , f (n−1) ( x0 )) soit une base ③ Même question si cette fois x est un vecteur propre de u asso-
de E. cié à une valeur propre λ de u.

u
e (id, f , . . . , f n−1 ) libre dans L(E). ④ On note par I x l’ensemble des polynômes P ∈ K [ X ] tels que

uor
P(u)( x ) = 0.
Partie II : Commutant d’un endomorphisme cyclique a Soit P ∈ K [ X ], montrer que Eu ( x ) est stable par P(u).

b Soit P ∈ I x , montrer que P(u) induit sur Eu ( x ) l’endo-

P
① Soit un endomorphisme g ∈ L(E) qui commute avec f .
morphisme nul.
a Dire pourquoi que ∃(ak )0≤n−1 ∈ R n tel que g( x0 ) =
n −1
c Montrer que I x est un idéal de K [ X ] non nul, engen-
∑ ak f k
( x0 ) dré par un polynôme unitaire, qu’on va noter πu,x et appelé

r
k=0 polynôme minimal de u en x.
n −1
⑤ a Dire pourquoi πu,x divise πu .

e
b Montrer que g( x ) = ∑ ak f k (x) pour tout x ∈ B.
k=0
er
n −1 b Donner un exemple où :

m
c En déduire que g = ∑ ak f k
.
i πu = πu,x .
k=0
② En déduire une base et la dimension du commutant de f , ii πu,x divise strictement πu .
m

défini par C( f ) = { g ∈ L(E) tel que f ◦ g = g ◦ f } c Donner π x,u quand x ∈ ker u.

r
⑥ Donner une condition nécessaire et suffisante portant sur x et
Partie III : Polynôme minimal en un vecteur.

o
u pour que deg(πu,x ) = 1.
or

F
✉: mamouni.myismail@gmail.com

8
i

r
s
✍ M AMOUNI M Y I SMAIL
MAMOUNI . NEW. FR
P ROBLÈMES C ORRIGÉS -MP

i
s

s
u
⑦ On suppose que u est un projecteur non nul, différent de l’i- ☞ On se propose de montrer que : u est irréductible si et seule-

s
dentité. Rappeler son polynôme minimal, ainsi que celui de ment si χu est un polynôme irréductible sur K.
x quand x ∈ Im u.

é
☞ On rappelle qu’un polynôme non constant P est dit ir-

u
⑧ On suppose dans cette question que deg(πu,x ) = k ≥ 2 avec réductible si et seulement si ses seuls diviseurs sont les
k−1 polynômes constants et ses polynômes associés de la forme

r
πu,x = X k − ∑ aj X j . λP où λ 6= 0

é
j =0
Pour cela pour tout x ∈ E, on pose K u [ x ] = { P(u)( x ) tel que P ∈
a Montrer que Bx = ( x, u( x ), . . . , uk−1 ( x )) est une base de

Po r
r
K [ X ]}, appelé sous-espace cyclique engendré par x
Eu ( x ) .
① On suppose que χu est irréductible.

u
b Que peut-on alors dire de l’espace Eu ( x ). a Si x 6= 0, montrer que χu = πu = π x,u .

uor
c En déduire dim Eu ( x ). En déduire que K u [ x ] = E, puis conclure.
b
b Donner la forme de MBx (ux ). ② Réciproquement, on suppose que u est irréductible.

P
c En déduire que πu x = πu,x . a Soit x 6= 0, montrer que K u [ X ] = E.

b Supposons qu’il existe P un diviseur non trivial de χu


Partie IV : Application χu
et soit y = P(u)( x ). Montrer que πy,u = , puis en déduire

r
P
☞ Un endomorphisme u est dit irréductible si et seulement si {0} une contradiction.

e
et E sont les seuls sous-espace vectoriel de E stables par u. c Conclure
er

m
i
F nn
i
m

r
Á la prochaine

o
or

F
✉: mamouni.myismail@gmail.com

9
i

r
s
P ROBLÈMES C ORRIGÉS -MP ✍ M AMOUNI M Y I SMAIL
MAMOUNI . NEW. FR

i
s

s
u

s
Contrôle
1 Endomorphismes nilpotents d’indice n et n-1

u
Conseils pour la rédaction et la présentation des copies.

é
✔ Chaque variable utilisée dans une démonstration doit être définie.
L’énoncé ne doit pas être recopié sur les copies.

Po r
r
✔ Chaque résultat énoncé doit être justifié en citant précisent le théorème du cours avec ses hypothèses exactes utilisé ou en citant le numéro
de la question précédente utilisée.

u
✔ Les résultats numériques importants doivent être simplifiés et encadrés.

uor
✔ Les calculs doivent être détailles et expliqués à l’aide de phrases simples.
✔ Laisser une marge à gauche de chaque feuille, en tirant un trait vertical, et un horizontal de la 1ère double feuille pour la note et les
remarques du correcteur.
✔ Numéroter les doubles feuilles de la façon suivante : 1/n,2/n,...,n/n où n est le nombre total des doubles feuilles.

P
✔ Les questions doivent être traités dans l’ordre de l’énoncé.
✔ Tirer deux traits diagonaux pour rayer une partie du raisonnement que vous considérez fausse.

r
Blague du jour
Marie Ennemond Camille Jordan (1838-1922)

Mathématicien du jour
e
- Quelle est la différence entre un prof à la retraite et le
Mathématicien français, connu à la fois pour son travail fonda-
er
sang ?
mental dans la théorie des groupes et pour son influent Cours

m
Y’en a pas, dans les deux cas il sort du corps enseignant
(en saignant). d’analyse. C’est un poyltechnicien (1855) fils de polytechnicie
(1818). Il enseigna à l’École polytechnique et succéda à Liouville
- Heureux l’étudiant qui, comme la rivière, arrive à au Collège de France, où il avait une réputation de choix de no-
m

suivre son cours sans sortir de son lit. tation excentriques.

or
or

F
✉: mamouni.myismail@gmail.com

10
i

r
s
✍ M AMOUNI M Y I SMAIL
MAMOUNI . NEW. FR
P ROBLÈMES C ORRIGÉS -MP

i
s

s
u
☛ Indication : Prendre G un supplémentaire de Im f k+1 dans
Énoncé

s
Im f k et montrer que Im f k+1 = Im f k+2 + f (G )

u
Partie I : Noyaux et images itérés. Partie II : Endomorphisme nilpotent.
Soit E un R-espace vectoriel de dimension finie et f ∈ L(E). On On considère dans cette partie f ∈ L(E) nilpotent d’indice de

r
pose dim E = n. nilpotence p, donc f p−1 6= 0 et f p = 0.

é
 
① a Montrer que la suite ker( f k ) est croissante (pour ① Soit x ∈ E \ ker f p−1 . Montrer que la famille
  k ∈N

Po r
r
l’inclusion) et que la suite Im ( f k ) est décroissante. x, f ( x ), . . . , f p−1 ( x ) est libre.
k ∈N
    ② En déduire que si p ≤ n, et que f n = 0.

u
k k
b Montrer que les suites ker( f ) et Im ( f )
③ Montrer que rg( f ) ≤ n − 1.

uor
k ∈N k ∈N
sont stationnaires à partir d’un même rang p.
④ On suppose dans cette question que p = n.
c Montrer que ker( f p ) ⊕ Im ( f p ) = E.
a Montrer que rg f = n − 1
② Soit k ∈ N, montrer que :

P
a ker f k = ker f k+1 =⇒ ker f k+1 = ker f k+2 b Montrer que dim ker f k = k, ∀k ∈ {0, . . . , n}.
☛ Indication : Utiliser la question ③ de la partie I.
b Im f k = Im f k+1 =⇒ Im f k+1 = Im f k+2
c En déduire que ker f k = Im f n−k , ∀k ∈ {0, . . . , n}.
③ a Soit F un sous-espace vectoriel de E, montrer que

r
☛ Indication : On rappelle le résultat suivant, si f , g ∈
ker f | F = ker f ∩ F et Im | F = f ( F). L(E) tel que f g = 0, alors Im g ⊂ ker f .

e
b Montrer que f (ker( f k+1 )) ⊂ ker( f k ). ⑤ On suppose dans cette question que p = n − 1.
er
c En déduire que : dim ker f k+1 ≤ dim ker f k + a Montrer que ∀k ∈ {1, . . . , n − 2}, on a :

m
dim ker f , ∀k ∈ N. i dim(ker f k ) ≥ k.
☛ Indication : Écrire la formule du rang pour f | F où F =
☛ Indication : On pourra raisonner par récurrence.
m

ker( f k+1 ).
  ii dim(ker f k ) ≤ k + 1.

r
④ Montrer que la suite dim(ker( f k+1 )) − dim(ker( f k ) ☛ Indication : On pourra raisonner par récurrence descen-
k ∈N

o
est décroissante. dante.
or

F
✉: mamouni.myismail@gmail.com

11
i

r
s
P ROBLÈMES C ORRIGÉS -MP ✍ M AMOUNI M Y I SMAIL
MAMOUNI . NEW. FR

i
s

s
u
b On suppose que ∃k ∈ {1, . . . , n − 2} tel que dim(ker f k ) = ④ On pose L0 (X ) = 1, L1 (X ) = X, Lk ( X ) =

s
X (X − 1) · · · (X − (k − 1))
k et dim(ker f k+1 ) = k + 2. pour tout 2 ≤ k ≤ n − 1.
k!

u
i Montrer que dim(ker f k ) ≥ dim(ker f k−1 ) + 2. a Montrer que ∆Lk = Lk−1 pour tout k ≥ 1.

r
ii En déduire une contradiction, puis conclure. b Que vaut ∆k Ln−1 ?

é
Partie III : Opérateur des différences finies. ⑤ En déduire que ∆ est l’indice de nilpotence de ∆ est égal à
n − 1.

Po r
r
Pour tout polynôme P ∈ R n−1 [ X ], on pose ∆( P)(X ) = P(X + 1) −
P ( X ). ⑥ En déduire que ker ∆n−k = Im ∆k = R n−k−1 [ X ].

u
m  
m− k m

uor
m
① Montrer que deg(∆P) ≤ deg( P), ⑦ ∆ P(X ) = ∑ (−1) P ( X + k ).
k
k=0
② En déduire que ∆ est un endomorphisme de R n−1 [ X ], Pré- ☛ Indication : Remarquer que ∆ = T − id
ciser son rang. où T ( P)(X ) = P(X + 1)
 

P
n
③ Montrer que deg(∆P) ≤ deg( P) − 1. En déduire que ∆ est k n
⑧ Simplifier la somme ∑ (−1) P( X + k)
nilpotent. k=0
k

r
i
F
n

e
i
n
er

m
Bonne Chance
m

or
or

F
✉: mamouni.myismail@gmail.com

12
i

r
s
✍ M AMOUNI M Y I SMAIL
MAMOUNI . NEW. FR
P ROBLÈMES C ORRIGÉS -MP

i
s

s
u
Soit k ∈ N, montrer que :
Corrigé, Pr. Mamouni, CPGE My Youssef, Rabat ③

s
a On a déjà ker f k+1 ⊂ ker f k+2 , Inversement x ∈

u
ker f k+2 =⇒ f k+1 ( f ( x )) = 0 =⇒ f ( x ) ∈ ker f k+1 =
Partie I : Noyaux et images itérés. ker f k =⇒ f k ( f ( x )) = f k+1 ( x ) = 0.

r
On a déjà Im f k+2 ⊂ Im f k+1 , Inversement x ∈

é
b
① a Classique et facile.
  Im f k+1 =⇒ ∃ x ∈ E; tel que x = f ( f k )( x ′ ), or f k ( x ′ ) ∈

Po r
r
b La suite ker( f k ) est croissante pour l’inclu- Im f k = Im f k+1 , donc ∃ x” ∈ E; tel que f k ( x ′ ) = f k+1 ( x”),
 k ∈N 
donc x = f k+2 ( x”) ∈ Im f k+2 .

u
sion, donc la suite dim ker( f k ) est croissante majorée
k ∈N
④ ker f | F = ker f ∩ F et Im | F = f ( F) (facile, utiliser juste les

uor
par dim E, donc converge, or elle est à valeur dans Z,
donc stationnaire (*), donc ∃ p ∈ N; tel que dim ker( f k ) = définition, cette propriété, bien que très simple, elle est très
dim ker( f p ) ∀k ≥ p, or ker( f k ) ⊂ ker( f p ) ∀k ≥ p, d’où utile)
l’égalité. D’après la formule du rang, on a dim Im ( f k ) = ⑤ La formule du rang pour f | F où F = ker( f k+1 ),
 

P
dim Im ( f p ) ∀k ≥ p, or Im ( f p ) ⊂ Im ( f p ) ∀k ≥ p, d’où s’écrit : dim ker( f k+1 ) = dim ker( f k+1 ) ∩ ker f +
l’égalité.  
(*) : Montrons le résultat suivant : Toute suite ( xn ) à valeurs dim f (ker( f k+1 )) ≤ dim ker f + dim ker f k car ker f ⊂
dans Z convergente, est stationnaire. En effet : pour ε = ker f k+1 et f (ker f k+1 ) ⊂ ker f k .

r
1 1
0, existsp ∈ N tel que ∀k ≥ p, on a | xk − x p | ≤ , or ⑥ Soit G tel que Im f k = Im f k+1 ⊕ G, donc Im f k+1 =
2 2

e
| xk − x p | ∈ N donc nul. f (Im f k ) = f (Im f k+1 ) + f (G ) = Im f k+2 + f (G ). En passant
er
c D’après la formule du rang, on a dim ker( f p ) + aux dimensions, on obtient : dim Im f k+1 = dim Im f k+2 +

m
dim Im ( f p ) = dim E, comme on est en dimension finie, il dim f (G ) − dim Im f k+1 ∩ f (G ) ≤ dim Im f k+2 + dim f (G ).
suffit de montrer que ker f p ∩ Im f p = 0. En effet, f p ( x ) = 0 Moyennant la formule du rang, on obtient : dim ker f k+2 −
et x = f p ( x ′ ) =⇒ f 2p ( x ′ ) = f p ( x ) = 0, d’où x ′ ∈ ker f 2p = dim ker f k+1 ≤ dim f (G ) ≤ dim G = dim Im f k −
m

ker f p , ce qui signifie x = f p ( x ′ ) = 0. dim Im f k+1 = dim ker f k+1 − dim ker f k .

r
② x ∈ f (ker( f k+1 )) =⇒ x = f ( x ′ ) tel que f k+1 ( x ′ ) = x, donc
Partie II : Endomorphisme nilpotent.

o
f k ( x ) = f k+1 ( x ′ ) = 0.
or

F
✉: mamouni.myismail@gmail.com

13
i

r
s
P ROBLÈMES C ORRIGÉS -MP ✍ M AMOUNI M Y I SMAIL
MAMOUNI . NEW. FR

i
s

s
u
① Supposons λ0 x + λ1 f ( x ) + · · · + λ p−1 f p−1 ( x ) = 0, on com- Supposons que dim ker f k ≥ k, on sait que la suite des noy-

s
pose une fois par f p−1 , puis par f p−2 et ainsi de suite pour aux itérés ne devienne stationnaire qu’à partir de k = p, donc
montrer que λ0 = 0, puis λ1 = 0 et ainsi de suite. dim ker f k+1 dim ker f k ≥ k, d’où dim ker f k+1 ≥ k + 1.

u
② La famille précédente est libre, donc de cardinal p ≤ n = ii Pour k = n − 2, on a f n−2 = f p−1 6= 0, donc
dim E. Or f p = 0, d’où f n = 0. ker f n−2 6= E, d’où dim ker f n−2 < n = dim E, autrement

é
③ On a f p ( x ) = 0, donc f p−1 ( x ) ∈ ker f or f p−1 ( x ) 6= 0, donc dit : dim ker f n−2 ≤ n − 1
dim ker f ≥ 1, d’où rg( f ) ≤ n − 1. Supposons que dim(ker f k+1 ) ≤ k + 2, toujours d’après le

Po r
r
a La famille ( f ( x ), · · · , f p−1 ( x )) de cardinal p − 1 = n − 1 même que la suite des noyaux itérés est strictement décrois-

sante pour k ≤ p, on a dim ker f k < dim(ker f k+1 ) ≤ k + 2,
est libre comme sous-famille d’une famille libre, or ses élé-

u
ments appartiennent tous à Im f , donc n − 1 ≤ dim Ima f = d’où dim ker f k ≤ k + 1.

uor
rg f ≤ n = dim E. Or f n’est pas bijective (car f p−1 ( x ) 6= 0 et b On suppose que ∃k ∈ {1, . . . , n − 2} tel que dim(ker f k ) =
f p−1 ( x ) ∈ ker f 6= 0), donc rg f 6= n, d’où rg f = n − 1. k et dim(ker f k+1 ) = k + 2.
b Montrer par récurrence sur k ∈ {0, . . . , n} que i D’après la question ④ de la partie I, on a dim(ker f k ) −

P
dim ker f k = k. dim(ker f k−1 ) ≥ dim(ker f k+1 ) − dim(ker f k ) = 2
En effet, pour k = 0 le résultat est évidement vérifié.
ii On a dim ker f k−1 ≥ k − 1, d’où k = dim ker f k ≥
Supposons maintenant que dim ker f k = k, d’après la ques- k + 1, absurde.
tion ③ de la partie I, on a dim ker f k = k ≤ dim ker f k+1 ≤ On montre de la même façon (par récurrence) que dans la

r
dim ker f k + dim ker f = k + 1. Or dim ker f k < dim ker f k+1 question ④ de la partie I, que dim ker f k = k, ∀0 ≤ k ≤ n − 2.
car la suite des noyaux itérés ne devienne stationnaire qu’à

e
partir de p = n, donc dim ker f k+1 = k + 1. Partie III : Opérateur des différences finies.
er

m
c On a f n = f k f n−k = 0, donc Im f n−k ⊂ ker f k , et sont de
① Utiliser la relation deg( P(X + 1) − P(X )) ≤ max(deg P(X +
même dimension d’après la formule du rang et la question
1), deg(X )) ≤ n car deg P(X + 1) ≤ n
précédente, d’où l’égalité.
m

② ∆ est linéaire, il est simple de vérifier que ∆( P + λQ) =


⑤ a

r
∆( P) + λ∆(Q). or ∆ : R n−1 [ X ] −→ R n−1 [ X ], donc ∆ est un
i Pour k = 1, on a déjà vu que ker f 6= 0, donc endomorphisme de R n−1 [ X ].

o
dim ker f ≥ 1. Avant de préciser son rang, chercher d’abord dim ker ∆. P ∈
or

F
✉: mamouni.myismail@gmail.com

14
i

r
s
✍ M AMOUNI M Y I SMAIL
MAMOUNI . NEW. FR
P ROBLÈMES C ORRIGÉS -MP

i
s

s
u
ker ∆ ⇐⇒ P(X + 1) = P(X ), Posons Q(X ) = P(X ) − P(0), b Par récurrence, on montre que ∆k Ln−1 = Ln−1−k

s
alors Q(0) = Q(1) = Q(2) = ..., ainsi Q admet une infinité
de racines, donc est nul, d’où P est constant, ker ∆ est l’ensem- ⑤ On a ∆n = 0 et ∆n−1 Ln−1 = L0 6= 0, donc ∆ est nilpotent

u
ble des polynômes constants, dont le dimension vaut 1. La d’indice égal à n − 1.
formule du rang donne rg∆ = dim R n−1 [ X ] − dim ker ∆ = ⑥ D’après la question ④-c de la partie II, appliquée à f = ∆, sur

r
n − 1. E = R n−1 [ X ], on a p = n − 1, donc ker ∆n−k = Im ∆k , d’autre

é
③ ∆P(X ) = P(X + 1) − P(X ) avec deg P(X + 1) = deg P(X ) part dim ker ∆n−k = n − k = R n−k [ X ], d’où l’égalité.
et coP(X + 1) = coP(X ), donc deg ∆P(X ) < deg P(X ),

Po r
r
i.e. deg(∆P) ≤ deg( P) − 1. Par récurrence, on montre que ⑦ On montre d’abord par récurrence que T k P(X ) =
P(X + k), d’où ∆m P(X ) = (T − id) P(X ) =
deg ∆k P ≤ deg P − k ≤ n − 1 − k. En particulier, deg ∆n P < 0,

u
m m
i.e., ∆n = 0. m m
∑ (−1)m−k k T k P(X) = ∑ (−1)n−k k P(X + k).

uor
④ a ∆Lk (X ) = Lk (X + 1) − Lk (X ) . k=0 k=0
n  
( X + 1) X · · · ( X − k ) − X ( X − 1) · · · ( X − k + 1) n
= ⑧ On remarque ∑ (−1)k P(X + k) = ∆n P(X ) = 0.
k! k=0
k
( X + 1) − ( X − k + 1)

P
= X · · · ( X − k)
k!
X · · · ( X − k)
= = Lk−1 ( X )
( k − 1) !

r
i
F

e
nn
er
i

m
Á la prochaine
m

or
or

F
✉: mamouni.myismail@gmail.com

15
✍ M AMOUNI M Y I SMAIL
MAMOUNI . NEW. FR

Devoir Libre
4 Arithmétique
Blague du jour

- Trois statisticiens vont la chasse au canard. Un canard décolle. Le premier


tire et passe dix centimètres au-dessus. Le second tire et passe dix cen-
timètres en-dessous. Le troisième, tout sourire : "c’est bon les gars, on l’a
eu !"
- La vie est complexe, elle a une partie réelle et une autre imaginaire.
- Qu’est-ce qu’un ours polaire ? C’est un ours cartésien qui a changé ses co-
ordonnés.

Sir William Rowan Hamilton (1805-1865)

Mathématicien du jour
Mathématicien, physicien et astronome irlandais. Il est connu pour sa découverte des
quaternions, mais il contribua aussi au développement de l’optique, de la dynamique et
de l’algèbre. Ses recherches se révélèrent importantes pour le développement de la mé-
canique quantique.
Enfant prodige ; et doué pour les langues à l’âge de 7 ans, il parlait déjà en hébreu et, à
l’âge de 13 ans, sous la direction de son oncle qui est linguiste, il parlait déjà 13 langues :
le persan, l’arabe, l’hindousthânî, le sanskrit, le malais,....

Matrices Stochastiques (extrait cnc 2002, TSI)

ÉJ« AÖÞ @ ø BñÓ úG ñÜØ ✉: mamouni.myismail@gmail.com

16
✍ M AMOUNI M Y I SMAIL
MAMOUNI . NEW. FR

✉: mamouni.myismail@gmail.com ÉJ« AÖÞ @ ø BñÓ úG ñÜØ


17
✍ M AMOUNI M Y I SMAIL
MAMOUNI . NEW. FR

i
F
nn

i
Á la prochaine

ÉJ« AÖÞ @ ø BñÓ úG ñÜØ ✉: mamouni.myismail@gmail.com

18
i

r
s
✍ M AMOUNI M Y I SMAIL
MAMOUNI . NEW. FR
P ROBLÈMES C ORRIGÉS -MP

i
s

s
u

s
Devoir Libre
5 Décomposition Spectrale et racine carrée d’un endomorphisme

u
r
Blague du jour Sophus Lie (1842-1899)

Mathématicien du jour
Êtes-vous accro l’Internet ? La réponse serait oui si : Mathématicien norvégien. Il a participé activement à la créa-

Po r
r
• A trois heures du matin, vous vous levez pour un be- tion de la théorie des symétries continues, et l’a appliquée à la
soin pressant et regardez en revenant si vous avez reçu géométrie et aux équations différentielles. On lui doit la création

u
des mails. de l’algèbre de Lie, ainsi que des groupes de Lie. Soupçonné

uor
• Vous inclinez la tête gauche quand vous souriez d’être un espion allemand, il profite de son incarcération pour
• Sur la porte de la cuisine est écrit : "upload" avancer sa thèse sur « une classe de transformation géométrique.
• Sur la porte des toilettes est écrit : "download" Il était marié à la petite fille de Niels Henrik Abel.

P
L’ensemble des valeurs propres de f sera noté Sp( f ) et on notera :
Énoncé (extrait CCP PC 2010) R( f ) = {h ∈ L(E)|h2 = f }.
R [ X ] désigne l’espace des polynômes à coefficients réels.

r

Notations et objectifs. Etant donné f ∈ L(E) et P ∈ R [ X ] donné par P(X ) = ∑ ak Xk , on

e
k=0
définit P( f ) ∈ L(E) par :
er
Dans tout ce problème, n est un entier naturel supérieur ou égal à 2

m
et E est un espace vectoriel de dimension fine n sur le corps R des
nombre réels.
P( f ) = ∑ ak f k
k=0
L(E) désigne l’algèbre des endomorphismes de E et GL(E) 0 ∗ k
où f = id et pour k ∈ N , f = f ◦ · · · ◦ f .
| {z }
m

l’ensemble des endomorphismes de E qui sont bijectifs.


k fois

r
On note 0 l’endomorphisme nul et id l’application identité. Si f 1 , . . . , f q désignent q endomorphismes de E (q ∈ N ∗ ) alors
Pour tout endomorphisme f , ker( f ) et Im ( f ) désigneront respec-

o
tivement le noyau et l’image de f .
or

F
✉: mamouni.myismail@gmail.com

19
i

r
s
P ROBLÈMES C ORRIGÉS -MP ✍ M AMOUNI M Y I SMAIL
MAMOUNI . NEW. FR

i
s

s
u
∏ f i désignera l’endomorphisme f 1 ◦ · · · ◦ f q . ⑦ Déduire de ce qui précède toutes les matrices H de M3 (R )

s
1≤ i ≤ q vérifiant H 2 = D, puis déterminer tous les endomorphismes
Pour tout entier p non nul, M p (R ) désigne l’espace des matrices h de R3 vérifiant h2 = f en donnant leur matrice dans la base

u
carrées à p lignes et p colonnes à coefficients dans R. canonique.
I p est la matrice identité de M p (R ).
B) Soient f et j les endomorphismes de R3 dont les matrices respec-

r
L’objectif du problème est d’étudier des conditions nécessaires ou

é
tives A et J dans la base
 canonique sontdonnéespar :
suffisantes à l’existence de racines carrées d’un endomorphisme f 
2 1 1 1 1 1
et de décrire dans certains cas l’ensemble R( f ).  
A = 1 2 1 et J = 1 1 1 .

Po r
r
Partie I 1 1 2 1 1 1
A) On désigne par f l’endomorphisme de R3 dont la matrice dans

u
① Calculer J m pour tout entier m ≥ 1.
la base canonique est donnée
 par :

uor
 1
8 4 −7 ② En déduire que pour tout m ∈ N ∗ , f m = id + (4m − 1) j.
A =  −8 −4 8  3
Cette relation est-elle encore valable pour m = 0 ?
0 0 1
③ Montrer que f admet deux valeurs propres distinctes λ et µ

P
① Montrer que f est diagonalisable. telles que λ < µ.
② Déterminer une base (v1 , v2 , v3 ) de R3 formée de vecteurs ④ Montrer qu’il existe un unique couple ( p, q) d’endomor-
propres de f et donner la matrice D de f dans cette nouvelle phismes de R3 tel que pour tout entier m ≥ 0, f m = λm p +
base. µm q et montrer que ces endomorphismes p et q sont linéaire-

r
ment indépendants.
③ Soit P la matrice de passage de la base canonique à la base
⑤ Après avoir calculé p2 , q2 , p ◦ q et q ◦ p, trouver tous les endo-

e
(v1 , v2 , v3 ). Soit un entier m ≥ 1. Sans calculer l’inverse de P,
exprimer Am en fonction de D, P et P−1 . morphismes h, combinaisons linéaires de p et q qui vérifient
er
h2 = f .

m
④ Calculer P−1 , puis déterminer la base de f m dans la base
⑥ Montrer que f est diagonalisable et trouver une base de
canonique.
vecteurs propres de f . Ecrire la matrice D de f , puis la ma-
⑤ Déterminer toutes les matrices de M3 (R ) qui commutent trice de p et de q dans cette nouvelle base.
m

avec la matrice D trouvée à la question 2).

r
⑦ Déterminer une matrice K de M2 (R ) non diagonale telle que
2
⑥ Montrer que si H ∈ M3 (R ) vérifie H = D, alors H et D K2 = I2 , puis une matrice Y de M3 (R ) non diagonale telle

o
commutent. que Y2 = D.
or

F
✉: mamouni.myismail@gmail.com

20
i

r
s
✍ M AMOUNI M Y I SMAIL
MAMOUNI . NEW. FR
P ROBLÈMES C ORRIGÉS -MP

i
s

s
u
⑧ En déduire qu’il existe un endomorphisme h de R3 vérifiant ⑨ Montrer que si l’ordre de multiplicité de la valeur propre λ

s
h2 = f qui n’est pas combinaison linéaire de p et q. est supérieur ou égal à 2, alors il existe un endomorphisme
2
⑨ Montrer que tous les endomorphismes h de R3 vérifiant h2 = p′ ∈ L(E) \ F tel que p′ = p et p′ ◦ q = q ◦ p′ = 0.

u
f sont diagonalisables. ⑩ En déduire que si dim(E) ≥ 3, alors R( f ) 6⊂ F.

r
Partie II
Partie III

é
Soit f un endomorphisme de E. On suppose qu’il existe (λ, µ) ∈ R2
et deux endomorphismes non Soient p1 , . . . , pm , m endomorphismes non nuls de E et λ1 , . . . , λm ,
 nuls p et q de E tels que :

Po r
m nombres réels distincts. Soit f un endomorphisme de E vérifiant

r
 id = p + q
λ 6= µ et f = λp + µq pour tout entier k ∈ N :

u
 2 m
f = λ2 p + µ2 q. fk = ∑ λik pi .

uor
① Calculer ( f − λid) ◦ ( f − µid). En déduire que f est diagonal- i =1
isable. ① Montrer que pour tout P ∈ R [ X ], on a :
m
② Montrer que λ et µ sont valeurs propres de f et qu’il n’y en a
pas d’autres.
P( f ) = ∑ P ( λi ) p i .

P
i =1
③ Déduire de la relation trouvée dans la question 1) que p ◦ q = m
q ◦ p = 0 puis montrer que p2 = p et q2 = q. ② En déduire que ∏( f − λi id) = 0, puis que f est diagonalis-
i =1
④ On suppose jusqu’à la fin de cette partie que λµ 6= 0. able.

r
Montrer que f est un isomorphisme et écrire f −1 comme com-
binaison linéaire de p et q. ③ Pour tout entier ℓ tel que 1 ≤ ℓ ≤ m, on considère le

e
polynôme :
⑤ Montrer que pour tout m ∈ Z :
( X − λi )
er
f m = λm p + µm q. Lℓ ( X ) = ∏ .

m
1≤ i ≤ m
( λ ℓ − λi )
⑥ Soit F le sous-espace de L(E) engendré par p et q. Déterminer i 6=ℓ
la dimension de F. Montrer que pour tout entier ℓ, tel que 1 ≤ ℓ ≤ m, on a
⑦ On suppose dans la suite de cette partie que λ et µ sont stricte- p ℓ = L ℓ ( f ).
m

ment positifs. Déterminer R( f ) ∩ F. En déduire que Im ( pℓ ) ⊂ ker( f − λℓ id), puis que le spectre

r
⑧ Soit k un entier supérieur ou égal à 2. Déterminer une matrice de f est :
Sp ( f ) = {λ1 , . . . , λm }.

o
K de Mk (R ) non diagonale et vérifiant K2 = Ik .
or

F
✉: mamouni.myismail@gmail.com

21
i

r
s
P ROBLÈMES C ORRIGÉS -MP ✍ M AMOUNI M Y I SMAIL
MAMOUNI . NEW. FR

i
s

s
u
④ Vérifier que pour tout couple d’entiers (i, j) tels que 1 ≤ i, j ≤ ① Montrer qu’il existe x ∈ E non nul tel que la famille

s
m, on a : ( ( x, f ( x ), f 2 ( x ), . . . , f p−1 ( x )) est libre. En déduire que p ≤ n
0 si i 6= j et que f n = 0.

u
pi ◦ p j =
pi si i = j. ② Montrer que si R( f ) 6= ∅, alors 2p − 1 ≤ n.

r
m √ n −1
∑ ak x k +

é
⑤ Justifier le fait que la somme ∑ ker( f − λi id) est directe et ③ Déterminer les réels a0 , . . . , an−1 tels que 1+x =
i =1 k=0
égale à E et que les projecteurs associés à cette décomposition O( x n ) au voisinage de 0. Dans la suite, Pn désigne le

Po r
r
de E sont les pi . n −1
polynôme défini par Pn (X ) = ∑ ak X k .

u
⑥ Soit F le sous-espace vectoriel de L(E) engendré par k=0
{ p1 , . . . , pm }. Déterminer la dimension de F.

uor
④ Montrer qu’il existe une fonction η bornée au voisinage de 0
⑦ Déterminer R( f ) ∩ F dans le cas où λ1 , . . . , λm sont des réels telle que l’on ait Pn2 ( x ) − x − 1 = x n η ( x ). En déduire que X n
positifs ou nuls. divise Pn2 − X − 1.
⑧ Dans cette question, on suppose de plus que m = n. ⑤ Montrer alors que R( f + id) 6= ∅.

P
Plus généralement, montrer que pour tout réel α réel, R(α f +
a Préciser alors la dimension des sous-espaces propres de
id) 6= ∅, puis que pour tout β réel strictement positif, R( f +
f. βid) 6= ∅.
b Montrer que si h ∈ R( f ), tout vecteur propre de f est B)

r
également vecteur propre de h. ① Soit T = (aij )1≤i,j≤n une matrice triangulaire supérieure de

e
c En déduire que R( f ) ⊂ F et donner une condition néces- Mn (R ) dont tous les coefficients diagonaux sont égaux à un
réel λ.
er
saire et suffisante sur les λi pour que R( f ) soit non vide.
Montrer que (T − λIn )n = 0.

m
⑨ Montrer que si m < n et si tous les λi sont positifs ou nuls,
alors R( f ) 6⊂ F. ② On suppose dans toute la suite que f est un endomorphisme
de E dont le polynôme caractéristique est scindé et qui n’ad-
Partie IV
m

met qu’une seule valeur propre λ. Déduire de la question

r
A) Soit f un endomorphisme non nul de E tel qu’il existe un entier précédente que E = ker( f − λid)n .
p > 1 tel que f p = 0 et f p−1 6= 0. ③ Montrer que si λ > 0 alors R( f ) 6= ∅.

o
or

F
✉: mamouni.myismail@gmail.com

22
i

r
s
✍ M AMOUNI M Y I SMAIL
MAMOUNI . NEW. FR
P ROBLÈMES C ORRIGÉS -MP

i
s

s
u
canoniqueest :
Corrigé (Pr. Blache, CPGE France)   

s
0 0 0 2 · 4m 4m 1 − 2 · 4m
A m = P · 0 1 0  · P − 1 =  − 2 · 4 m − 4 m 2 · 4m  .

u
Partie I 0 0 4 m
0 0 1
A) En particulier, on vérifie que cela redonne A pour m = 1.

r
⑤ Soit M = (mij )1≤i,j≤3 une matrice qui commute avec D. On

é
① Calculons le polynôme caractéristique de A (donc aussi de écrit que MD = DM, ce qui donne mij = 0 si i 6= j. Donc
f) : nécessairement, M est une matrice diagonale.

Po r
r
8−X 4 −7 Réciproquement, toute matrice diagonale commute avec D
Pf (X ) = PA (X ) = −8 −4 − X 8 = − X (X − 1)(X − 4). qui est elle-même diagonale.

u
0 0 1−X Finalement, les matrices qui commutent avec D sont les matrices dia

uor
Ainsi, Pf est scindé à racines simples, donc f est diagonalisable .
. ⑥ On a HD = DH = H 3 , donc H et D commutent .
② L’étude des sous-espaces propres donne : ⑦ D’après les questions 5) et 6), si H 2 = D, alors H est une ma-

P
2
E0 ( f ) = Vect (v1 ) avec v1 = (1, −2, 0), trice diagonale.
 La condition
 H = D donne également :
E1 ( f ) = Vect (v2 ) avec v2 = (1, 0, 1), 0 0 0
E4 ( f ) = Vect (v3 ) avec v3 = (1, −1, 0). H = 0 ±1 0  (ce qui fournit 4 solutions).

  0 0 ±2
0 0 0

r
Dans ce cas, la matrice D est donnée par : D = 0 1 0. Pour obtenir les matrices solutions dans la base canonique,
0 0 4 on effectue un changement de base : les matrices solutions

e
sont données par P · H · P−1 , où H est l’une des 4 solutions
③ La formule de changement de base donne : A = P · D · P−1 ,
er
précédentes. Après calculs, on obtient à nouveau 4 solutions,

m
et donc Am = P · D m · P−1 . qui sont :
   
4 2 −3 4 2 −5
④ En utilisant par
exemple la méthode
 du pivot de Gauss, on ± −4 −2 4  et ± −4 −2 4  .
m

−1 −1 1 0 0 1 0 0 −1
trouve : P−1 =  0 1 .

r
0
2 1 −2 B)

o
Après calculs, on trouve que la matrice de f m dans la base ① On trouve pour tout entier m ≥ 1 : J m = 3m−1 J .
or

F
✉: mamouni.myismail@gmail.com

23
i

r
s
P ROBLÈMES C ORRIGÉS -MP ✍ M AMOUNI M Y I SMAIL
MAMOUNI . NEW. FR

i
s

s
u
② Travaillons avec les matrices A et J. On a A = J + I3 . Comme Soit maintenant h = α · p + β · q tel que h2 = f . D’après les

s
J et I3 commutent, la formule du binôme donne : ! relations précédentes, on a
m   m   h2 = α2 · p + β2 · q = f = p + 4q.
1 m

é
m k m
· 3k−1 J = I3 + Comme

u
∀m ∈ N ∗ , Am = ( I3 + J )m = ∑ J = I3 + ∑ (4 − 1()p,
J. q) est une famille libre, cette égalité équivaut à
k=0
k k=1
k 32
Si on revient aux endomorphismes, cela donne : α = 1 et β2 = 4.

r
Donc il y a 4 endomorphismes h solutions, donnés par :

é
∗ m 1 m
pour tout m ∈ N , f = id + (4 − 1) j . h = ± p ± 2q .
3

Po r
Evidemment, cette relation est encore valable pour m = 0 ⑥ On détermine les sous-espaces propres de f :

r
(car dans ce cas, on a id =id ). E1 ( f ) = Vect (w1 , w2 ) avec w1 = (1, −1, 0) et w2 = (0, 1, −1),

u
③ Un calcul du polynôme caractéristique de A donne : E4 ( f ) = Vect (w3 ) avec w3 = (1, 1, 1).
dim(E1 ( f )) + dim(E4 ( f )) = 3 = dim(R3 ),

uor
Pf (X ) = PA (X ) = 2
−(X − 1) (X − 4). Donc Comme
f admet les deux valeurs propres λ = 1 et µ = 4 . f est diagonalisable .
Et (w1 , w2, w3 ) est une base de vecteurs propres pour f .
m m 1
④ D’après la question 2), on peut écrire f = 1 (id − j) + Notons w1 , w2 , w3 ). Alors
 Bd = (  :

P
3   
1 1 1 0 0 1 0 0 0 0 0
4m ( j) pour tout entier m ≥ 0. En posant p = id − j et D = 0 1 0 , MBd (q) = 0 1 0 et MBd (q) = 0 0 0 .
3 3
1 0 0 4 0 0 0 0 0 1
q = j, on obtient l’existence de la décomposition voulue .  
3 0 1

r
De plus, on a nécessairement id = p + q (pour m = 0) ⑦ On peut prendre par exemple : K = et Y =
1   1 0
et f = p + 4q (pour m = 1). Donc p = (4id − f ) et

e
3 0 1 0
1  1 0 0 .
er
q = ( f − id), d’où l’unicité de cette décomposition .
0 0 2

m
3
Enfin, comme id et j sont deux endomorphismes linéairement
indépendants (d’après leur écriture matricielle), il en est de ⑧ Soit h l’endomorphisme de R3 dont la matrice dans
même pour p et q. la base Bd est Y. Alors h2 = f car Y2 = D. Et
m

h n’est pas combinaison linéaire de p et q , car Y n’est pas

r
⑤ On obtient, en utilisant les expressions de p et q trouvées à la
question précédente : combinaison linéaire de leurs matrices (vues précédemment)

o
p2 = p, q2 = q, p ◦ q = q ◦ p = 0. dans la base Bd .
or

F
✉: mamouni.myismail@gmail.com

24
i

r
s
✍ M AMOUNI M Y I SMAIL
MAMOUNI . NEW. FR
P ROBLÈMES C ORRIGÉS -MP

i
s

s
u
⑨ Soit h tel que h2 = f . Comme f est diagonalisable et que Enfin, comme id = p + q, on obtient, en composant par p

s
ses valeurs propres sont 1 et 4, le polynôme Q1 (X ) = (X − (resp. par q) : p = p2 (resp. q = q2 ).
1)(X − 4) est un polynôme annulateur de f , donc de h2 .

u
Donc le polynôme Q2 (X ) = (X 2 − 1)(X 2 − 4) = (X − 1)(X + ④ Comme λµ 6= 0, f n’admet pas la valeur propre 0.
1)(X − 2)(X + 2) est un polynôme annulateur de h. Or ce Donc ker f = {0}, et comme E est de dimension finie,

r
polynôme est scindé à racines simples, donc d’après le cours, f est bijective .

é
h est diagonalisable . De plus, on a vu en 1) que f 2 − (λ + µ) f + (λµ) id = 0. D’où
−1

Po r
f −1 =

r
Partie II ( f − (λ + µ)id). On remplace f et id à l’aide de p
λµ

u
① On a, en utilisant les trois relations, ( f − λid) ◦ ( f − µid) = 1 1
et q, ce qui donne finalement : f −1 = p + q .
f 2 − (λ + µ) f + (λµ) id = 0. Donc (X − λ)(X − µ) est λ µ

uor
un polynôme annulateur de f , scindé à racines simples. Et
⑤ La relation f m = λm p + µm q est vérifiée pour m = 0, 1, 2
f est diagonalisable .
d’après l’énoncé, et pour m = −1 d’après la question précé-
② A la question précédente, on a trouvé un polynôme annula- dente.

P
teur de f qui n’a que λ et µ comme racines. Il en résulte que Une démonstration par récurrence sans difficulté, d’une part
Sp ( f ) ⊂ {λ; µ}. pour m ∈ N, d’autre part pour −m ∈ N, donne (en utilisant
Si µ n’est pas valeur propre de f , la seule valeur propre est le fait que p ◦ q = q ◦ p = 0) : ∀m ∈ Z, f m = λm p + µm q .
donc λ. Comme f est diagonalisable, on a donc f = λ id. En
⑥ Soient deux réels α et β tels que αp + βq = 0. En composant

r
utilisant les deux premières relations de l’énoncé, on a donc :
λid = λp + µq = λp + λq. par p, on a αp = 0 donc α = 0 puisque p 6= 0. De même, en

e
D’où (λ − µ)q = 0, et comme λ 6= µ, q = 0. Ceci est contraire composant par q, on obtient β = 0.
Donc ( p, q) est une famille libre et dim( F) = 2 .
er
aux hypothèses ; ainsi µ est valeur propre de f .

m
On montrerait de même que λ est aussi une valeur propre de
⑦ Soit h ∈ R( f ) ∩ F. Alors h = αp + βq et comme p ◦ q =
f . Donc Sp ( f ) = {λ; µ} . q ◦ p = 0, h2 = α2 p + β2 q = f = λp + µq. Comme ( p, q) est
③ D’après la question 1), on a : 0 = ( f − λid) ◦ ( f − µid) = (µ − une famille libre, on a α2 = λ √et β2 = µ, i.e. (puisque λ et µ
m


λ)q ◦ (λ − µ) p. Comme λ 6= µ, on en déduit que q ◦ p = 0 . sont supposés positifs) α = ± λ et β = ± µ. On obtient 4

r
De même, comme ( f − µid) ◦ ( f − λid) = 0, on trouve possibilités, qui réciproquement conviennent toutes. Par con-
√ √

o
p◦q = 0 . séquent, les 4 solutions sont h = ± λ p ± µ q .
or

F
✉: mamouni.myismail@gmail.com

25
i

r
s
P ROBLÈMES C ORRIGÉS -MP ✍ M AMOUNI M Y I SMAIL MAMOUNI . NEW. FR

i
s

s
u
⑧ Définissons la matrice K diagonale par blocs de la façon suiv- En résumé, l’endomorphisme p′ ainsi construit répond à la question .

s
ante :  
0 1

é
⑩ Si dim(E) ≥ 3, alors λ ou µ est d’ordre au √ moins 2. Sup-

u
K= 1 0 ,
′ √
Ik−2 posons par exemple que c’est λ. Posons h = λp + µq, où

r
où Ik−2 est la matrice identité de Mk−2 (R ) (bien définie car p′ est l’endomorphisme défini à la question précédente. On a

é
k ≥ 2). h2 = λp + µq = f par propriétés de p′ et q, et pourtant h ∈
/F
car p′ ∈/ F et λ 6= 0.
Alors un produit par blocs donne immédiatement K2 = Ik .

Po r
r
En conclusion, R( f ) 6⊂ F .
⑨ On va raisonner matriciellement. Appelons k l’ordre de mul-

u
tiplicité de la valeur propre λ (k ≥ 2) et considérons une

uor
base de diagonalisation Bd pour f ; c’est également une base Partie III
1
de diagonalisation pour p et q car p = ( f − µid)
λ−µ
1 ℓ
et q = ( f − λid). De plus, dans la base Bd , ces
① Pour tout P(X ) = ∑ ak Xk ∈ R[X], on a :

P
µ−λ
matrices sont définies par blocs comme suit : MBd ( f ) = k=0 ! !
    ℓ ℓ m m ℓ m
λIk 0 Ik 0 k
λik pi ak λik
0 µIn−k
, MBd ( p) =
0 0n − k
et MBd (q) = P( f ) = ∑ ak f = ∑ ak ∑ = ∑ ∑ pi = ∑ P ( λi ) p i .
  k=0 k=0 i =1 i =1 k=0 i =1

r
0k 0
. Soit alors p′ l’endomorphisme dont la matrice
0 In−k m
② Prenons P(X ) = ∏ ( X − λi ). Alors P(λi ) = 0 pour i =

e
dans la base Bd est :  
0 i =1
er
K
M= 1, ..., m, et d’après la question précédente P( f ) = 0. Le
0 0n − k

m
polynôme P est annulateur de f et il est scindé à racines sim-
où la matrice K ∈ Mk (R ) a été définie à la question précé-
dente. De plus, ples. Donc f est diagonalisable .
2
• un produit par blocs donne M2 = MBd ( p), donc p′ = p ;
m

r
• des produits par blocs donnent M · MBd (q) = MBd (q) · ③ D’après la question 1), Lℓ ( f ) =
M = 0n , donc p′ ◦ q = q ◦ p′ = 0n ;
∑ Lℓ (λi ) pi . Mais Lℓ (λi ) =
i =1

o
• comme M n’est pas diagonale, p′ ∈ / F = Vect ( p, q). δℓ,i (où δℓ,i = 1 si ℓ = i et 0 si ℓ 6= i).
or

F
✉: mamouni.myismail@gmail.com

26
i

r
s
✍ M AMOUNI M Y I SMAIL
MAMOUNI . NEW. FR
P ROBLÈMES C ORRIGÉS -MP

i
s

s
u
m
Donc Lℓ ( f ) = pℓ . De plus, à la question précédente. De plus, comme id = ∑ pi ,

s
∏im=1 ( f − λi id) k=1
( f − λℓ id) ◦ pℓ = ( f − λℓ id) ◦ Lℓ ( f ) = m

é
∏ 1≤ i ≤ m ( λ ℓ − λi )

u
on a E = ∑ Im ( pi ). Or on a vu que Im ( pi ) ⊂
i 6=ℓ
0 i =1

r
= . D’après
Eλi ( f )= 0. la somme directe précédente, on a donc
∏ 1≤ i ≤ m ( λ ℓ − λ i) M

é
m
i 6=ℓ E = Im ( pi ) et Im ( pi ) = Eλi ( f ) pour tout i. En-
Il en résulte que Im ( pℓ ) ⊂ ker( f − λℓ id) . i =1

Po r
r
En outre, le polynôme P(X ) de la question 2) est annulateur fin le fait que pi ◦ p j = 0 pour i 6= j montre que
de f et a pour racines λ1 , . . . , λm . Donc Sp ( f ) ⊂ {λ1 , . . . , λm }. les pi sont les projecteurs associés à cette somme directe .

u
Et par hypothèse, pour tout 1 ≤ ℓ ≤ m, pℓ 6= 0 donc

uor
⑥ Ecrivons une combinaison linéaire nulle des ( pi )1≤i ≤m :
Im ( pℓ ) 6= {0E } et ker( f − λℓ id) 6= {0E }. Ceci signifie que m
λℓ est effectivement une valeur propre de f . ∑ ai p i = 0. Soit ℓ ∈ [[ 1, m ]]. En composant par pℓ , on obtient
Finalement, on a bien Sp ( f ) = {λ1 , . . . , λm } . i =1
aℓ pℓ = 0, d’où aℓ = 0 car pℓ n’est pas nul d’après l’énoncé.

P
Ainsi tous les coefficients ai sont nuls et la famille ( p1 , ..., pm )
④ Comme pℓ ( f ) = Lℓ ( f ), pi ◦ p j = ( Li · L j )( j).
m est libre. Donc dim( F) = m .
• Si i 6= j, le polynôme P(X ) = ∏ (X − λi ) divise ( Li · L j )(X ). m m
2 2
i =1 ⑦ Soit h = ∑ αi pi ∈ F telle que h = f . Alors h = ∑ α2i pi =

r
Comme P( f ) = 0, on a donc ( Li · L j )( f ) = 0 et pi ◦ p j = 0 . i =1 i =1
m
m
∑ λi pi et comme la famille ( p1 , ..., pm ) est libre, α2i = λi pour

e
• Si i = j, comme id = ∑ pi (relation de l’énoncé pour k = 0), i =1
er
k=1
tout i. Réciproquement, tous les h vérifiant cette relation sont

m
en composant par pi on obtient p2i = pi . (
m √
)
solutions. En résumé, R( f ) ∩ F = ∑ ± λi pi .
⑤ L’endomorphisme f étant diagonalisable, d’après le cours on i =1
m

m
M
⑧ a Si m = n, il y a n sous-espaces propres dans l’espace E

r
a E= ker( f − λi id) .
i =1 de dimension n.

o
Le fait que chaque pi est un projecteur a été démontré Donc la dimension de chaque sous-espace propre de f est égale à 1 .
or

F
✉: mamouni.myismail@gmail.com

27
i

r
s
P ROBLÈMES C ORRIGÉS -MP ✍ M AMOUNI M Y I SMAIL
MAMOUNI . NEW. FR

i
s

s
u
p−1
b m Si h ∈ R( f ), h ◦ f = h3 = f ◦ h. Donc h et f

s
réels tels que ∑ ak f k (x) = 0.
commutent et d’après le cours, tout espace propre Eλi ( f ) est k=0

é
stable par h. Soit x un vecteur propre de f , par exemple En composant par f p−1 , comme f q = 0 pour tout q ≥ p, on

u
x ∈ Eλi ( f ) \ {0E }. Comme dim(Eλi ( f )) = 1, h( x ) = µi x et obtient a0 f p−1 ( x ) = 0 donc a0 = 0. On recommence en com-
posant par f p−2 , ..., f , ce qui donne au final a0 = · · · = a p−1 =

r
x est vecteur propre pour h .

é
0.
c Soit h ∈ R( f ). D’après la question précédente, pour tout Donc la famille ( x, f ( x ), f 2 ( x ), . . . , f p−1 ( x )) est libre . Cette

Po r
1 ≤ i ≤ m, il existe µi ∈ R tel que pour tout xi ∈ Eλi ( f ),

r
famille a p éléments dans un espace de dimension n, donc
h ( xi ) = µi xi .
p ≤ n et f n = f n− p ◦ f p = 0 .

u
n
M
Soit x ∈ E. Comme E = Eλi ( f ), x = x1 + · · · + xn avec
② Si R( f ) 6= ∅, soit h ∈ L(E) tel que h2 = f . Alors h2n = f n = 0

uor
i =1
xi ∈ Eλi ( f ) et donc h est nilpotent et d’après 1), hn = 0. De plus, h2p−2 =
n n f p−1 6= 0 donc 2p − 2 ≤ n − 1, i.e. 2p − 1 ≤ n .
h ( x ) = h ( x1 + · · · x n ) = ∑ µi xi = ∑ µi pi ( x )
i =1 i =1 ③ On sait d’après le cours que pour α ∈ / N, x ∈] − 1; 1[,

P
m n −1
soit h = α ( α − 1) · · · ( α − k + 1) k
∑ µi pi . Donc R( f ) ⊂ F . (1 + x ) α = ∑ x + O( xn )
i =1 k=0
k!
En reprenant la question III.7), on voit qu’une condition 1
nécessaire et suffisante sur les λi pour que R( f ) soit non vide au voisinage de 0. Ici, α = et pour tout k ∈ [[ 0, n − 1 ]],

r
2
est : ∀i ∈ [[ 1, n ]], λi ≥ 0 . 1 1
( − 1) · · · ( 21 − k + 1)
ak = 2 2

e
.
⑨ Si m < n, alors il existe i tel que dim(Eλi ( f )) ≥ 2. Si les λi k!
er
sont positifs ou nuls, on peut alors reprendre le même raison- ④ D’après la question précédente, pour −1 < x < 1,

m

nement qu’à la question II.10), qui montre que R( f ) 6⊂ F . 1 + x = Pn ( x ) + x n γ( x ) où γ est une fonction bornée au
voisinage de 0. En élevant au carré, cela donne 1 + x =
Partie IV ( Pn ( x ) + x n γ( x ))2 = Pn2 ( x ) + x n (2Pn ( x )γ( x ) + x n γ( x )2 ) =
m

Pn2 ( x ) + x n η ( x ) pour une fonction η bornée au voisinage de

r
A)
0.

o
① Soit x ∈ E tel que f p−1 ( x ) 6= 0E et (a1 , ..., a p ) une famille de Posons alors Qn ( x ) = Pn2 ( x ) − x − 1 ; c’est une fonction
or

F
✉: mamouni.myismail@gmail.com

28
i

r
s
P ROBLÈMES C ORRIGÉS -MP ✍ M AMOUNI M Y I SMAIL
MAMOUNI . NEW. FR

i
s

s
u
polynôme. D’après la relation précédente, x 7→ Qn ( x )/x n est B)

s
une fonction bornée au voisinage de 0. Ceci n’est possible que ① La matrice T − λIn est triangulaire supérieure avec des zéros
si Qn (X ) n’admet pas de terme en X k pour k ∈ [[ 0, n − 1 ]], ce

é
sur la diagonale ; il en résulte que rg(T − λIn ) ≤ n − 1. Un cal-

u
qui entraîne X n divise Qn (X ) . cul matriciel simple montre que rg((T − λIn )k ) ≤ n − k pour
On écrira dans la suite Qn (X ) = X n · Sn (X ) où Sn est une k ∈ [[ 2, n ]] et en particulier pour k = n : rg((T − λIn )n ) = 0,

é
fonction polynôme. i.e. (T − λIn )n = 0 .
⑤ • D’après les résultats des questions précédentes, ( Pn ( f ))2 − Remarquons que cette question peut se traiter directement

Po r
r
f − id = ( Pn2 )( f ) − f − id = f n ◦ Sn ( f ). Or f n = 0 d’après en utilisant le théorème de Cayley-Hamilton, mais que ce
1), donc ( Pn ( f ))2 = f + id, i.e. Pn ( f ) ∈ R( f + id). Donc théorème est hors programme en PC.

u
R( f + id) 6= ∅ . ② Comme f est un endomorphisme de E dont le polynôme car-

uor
actéristique est scindé, il est trigonalisable. De plus, comme f
• Plus généralement, ( Pn (α f ))2 − α f − id = ( Pn2 )(α f ) − α f −
n’admet qu’une seule valeur propre λ, il existe une base dans
id = (α f )n ◦ Sn (α f ). Comme f n = 0, ( Pn (α f ))2 = α f + id, i.e.
laquelle la matrice T de f est triangulaire supérieure, dont
Pn (α f ) ∈ R(α f + id). Donc R(α f + id) 6= ∅ . tous les coefficients diagonaux sont égaux à un réel λ. D’après

P
1 la question précédente, (T − λIn )n = 0n et ( f − λid)n = 0.
• Comme β 6= 0, soit h ∈ R( f + id) (c’est possible
β Donc E = ker( f − λid)n .
1
d’après ce qui précède). Alors h2 = f + id et comme
β ③ D’après la partie A), comme ( f − λid)n = 0, R ( f − λid) +
p p 

r
β > 0, ( β h)2 = f + βid. Donc β h ∈ R( f + βid) et λid 6= ∅ (question A)5) en prenant β = λ et en remplaçant
R( f + βid) 6= ∅ . f par f − λid). Donc si λ > 0 alors R( f ) 6= ∅ .

e
er

m
i
F
n
i
n
m

r
Á la prochaine

o
or

F
✉: mamouni.myismail@gmail.com

30
✍ M AMOUNI M Y I SMAIL
MAMOUNI . NEW. FR

Devoir Surveillé
1 Recherche de quelques polynômes minimaux
Énoncé : (extrait e3a 2008, MP)

✉: mamouni.myismail@gmail.com ÉJ« AÖÞ @ ø BñÓ úG ñÜØ


31
✍ M AMOUNI M Y. I SMAIL
.
MAMOUNI NEW FR

ÉJ« AÖÞ @ ø BñÓ úG ñÜØ ✉: mamouni.myismail@gmail.com

32
✍ M AMOUNI M Y I SMAIL
MAMOUNI . NEW. FR

✉: mamouni.myismail@gmail.com ÉJ« AÖÞ @ ø BñÓ úG ñÜØ


33
✍ M AMOUNI M Y. I SMAIL
.
MAMOUNI NEW FR

Facultative, ne sera pas comptée

ÉJ« AÖÞ @ ø BñÓ úG ñÜØ ✉: mamouni.myismail@gmail.com

34
i

r
s
✍ M AMOUNI M Y I SMAIL
MAMOUNI . NEW. FR
P ROBLÈMES C ORRIGÉS -MP

i
s

s
u
.

s
é
Corrigé Pr. Dufait, CPGE France

u
r

é
Blague du jour Marie Ennemond Camille Jordan (1838-1922)

Mathématicien du jour
Po r
Mathématicien français, connu à la fois pour son travail fonda-

r
- Quelle est la différence entre un prof à la retraite et le
sang ? mental dans la théorie des groupes et pour son influent Cours

u
Y’en a pas, dans les deux cas il sort du corps enseignant d’analyse. C’est un poyltechnicien (1855) fils de polytechnicie
(1818). Il enseigna à l’École polytechnique et succéda à Liouville

uor
(en saignant).
- Heureux l’étudiant qui, comme la rivière, arrive à suivre au Collège de France, où il avait une réputation de choix de no-
son cours sans sortir de son lit. tation excentriques.

P
Questions de cours et exemples 4. D’après le théorème de Cayley-Hamilton, J f 6= {0} puisque le
polynôme caractéristique de f , χ f appartient à J f .

r
 
1. Un polynôme annulateur de f est un polynôme P ∈ R [ X ] tel que 1/2 0 1/2 0

e
P( f ) = 0L(E) .  0 1/2 0 1/2 2
er
5. 1. On a M =  
1/2 0 1/2 0  donc M = M et donc, par

m
2. J f est un idéal de R [ X ] (et accessoirement un sous-espace vectoriel 0 1/2 0 1/2
de R [ X ]). récurrence, ∀k ∈ N ∗ , Mk = M .
m

2. Ainsi X 2 − X ∈ J f donc π f | X 2 − X = X (X − 1) et donc

r
3. Si J f 6= {0}, le polynôme minimal de f est l’unique polynôme uni- 
taire π f tel que J f = π f .R [ X ]. C’est aussi le polynôme unitaire ap- π f ∈ X, X − 1, X 2 − X (1 n’est pas annulateur si E 6= {0}). Or
M 6= 0 et M 6= I4 donc π f = X 2 − X .

o
partenant à J f de plus petit degré.
or

F
✉: mamouni.myismail@gmail.com

35
i

r
s
P ROBLÈMES C ORRIGÉS -MP ✍ M AMOUNI M Y I SMAIL
MAMOUNI . NEW. FR

i
s

s
u
6. 1. ⋄ L’équation homogène y′′ + y = 0 est à coefficients constants les solutions de ( H1 ) sont les y : x 7→ A cos( x ) + B sin( x ) + C ch ( x ) + D sh (

s
et a pour équation caractéristique X 2 + 1 = 0 donc les solutions 
sur R à valeurs dans R de y′′ + y = 0 sont les fonctions x 7→ 5. 1. Par définition de E, la famille cos, sin, ch, sh en est génératrice.

u
A cos( x ) + B sin( x ) pour ( A, B) ∈ R 2 . De plus, si A cos + B sin +C ch + D sh = 0 alors, en dérivant deux
Comme ch ′′
= ch, une solution particulière de y + ′′ fois, − A cos − B sin +C ch + D sh = 0 donc A cos + B sin = 0 et

r
1 C ch + D sh = 0. En prenant les valeurs en 0 (par exemple), on ob-

é
y = ch ( x ) est la fonction x 7→ ch ( x ) donc tient A = B = C = D = 0 donc cette famille est libre etc’est donc
2
1 une base de E. Ainsi dim(E) = 4 .
les solutions sur R de y′′ + y = ch ( x ) sont les y : x 7→ ch ( x ) + A cos( x ) + B sin( x ) pour ( A, B) ∈ R 2 .

Po r
r
2 ′
2. La dérivation est linéaire et A cos + B sin +C ch + D sh =
′′ 1

u
⋄ De même, les solutions sur R de y + y = sh ( x ) sont les fonctions x 7→ sh − A( xsin
) ++ABcoscos ( x+
)C+sh
B sin
+ (Dx )ch ∈ E donc E est stable. Donc
2
la dérivation induit bien un endomorphisme de E .

uor
pour ( A, B) ∈ R 2 .
3. D’après [6.4], E est l’ensemble des solutions de ( H1 ) donc ∀y ∈
2. Puisque f est supposée de classe C4 sur R alors g est de classe C2 E, y(4) = δ4 (y) = y. Ainsi X 4 − 1 ∈ Jδ et πδ | X 4 − 1. Si πδ 6= X 4 − 1
3
sur Ret donc :   

P
f solution de ( H1 ) ⇐⇒ ∀ x ∈ R , f (4) ( x ) = f ( x )
alors deg ( π δ ) ≤ 3 donc π δ = ∑ ai Xi . On alors
i =0 
  4
(4) ′′ ′′ ∀( A, B, C, D ) ∈ R , 0 = πδ (δ) A cos + B sin +C
⇐⇒ ∀ x ∈ R , f ( x ) + f ( x ) = f ( x ) + f ( x ) 3
  
′′
⇐⇒ ∀ x ∈ R , g ( x ) = g( x ) = ∑ a A cos + B sin +C

r
i
    i = 0 
donc f solution de ( H1 ) ⇐⇒ g = f ′′ + f solution de y′′ = y . = (a0 − a2 ) A + (a3 − a1 ) B cos + (a0

e
+ (a0 + a2 )C + (a1 + a3 ) D ch

er
donc, par liberté de cos, sin, ch, sh ,
( H2 ) : y′′ − y = 0 est linéaire, homogène, à coefficients constants 

m
3.
2 
 ( a0 − a2 ) A + ( a3 − a1 ) B = 0
et a pour équation caractéristique X − 1 = 0 donc a pour so- 

lutions les fonctions x 7→ A e x + B e− x avec ( A, B) ∈ R 2 , ou aussi, ( a0 − a2 ) B + ( a1 − a3 ) A = 0
∀( A, B, C, D ) ∈ R 4 ,
les solutions de ( H2 ) sont les y : x 7→ A ch ( x ) + B sh ( x ) pour ( A, B) ∈ R 2 .  ( a0 + a2 ) C + ( a1 + a3 ) D = 0
m



r
( a0 + a2 ) D + ( a1 + a3 ) C = 0
4. Le principe de superposition des solutions et les résultats de la soit a0 = a2 = − a0 et a1 = a3 = − a1 donc a0 = a1 = a2 = a3 = 0 ce

o
question [6.1] donnent : qui est absurde. Donc πδ = X 4 − 1 .
or

F
✉: mamouni.myismail@gmail.com

36
i

r
s
✍ M AMOUNI M Y I SMAIL
MAMOUNI . NEW. FR
P ROBLÈMES C ORRIGÉS -MP

i
s

s
u
k  
Problme k k k
∑ i Xi donc

s
⋄ ∀k ∈ N, v(X ) = ( X + 1) =
  i =0

é
1 1  1 · · · ··· 1

u
Partie I   
 2 .. .. n
0 1 . . 
 

r
  1 1
  
1. On a dim(En ) = n + 1 et une base est X k
. . 

é
 .. .. .. n 
k∈[[0,n ]]
 . 1 . 
Vn =  2  ∈ M n + 1 (R )
 .. .. .. .. .. 

Po r
r
2. ⋄ ∀ P ∈ E, u( P) = P′ ∈ E et ∀( P, Q) ∈ E2 , ∀λ ∈ R , u( P + λQ) = . . . .  . 


( P + λQ)′ = P′ + λQ′ = u( P) + λu(Q) donc u ∈ L(E) .  .. .. n 
. . 1 

u
De plus, si deg( P) 6= 0, deg( P′ ) = deg( P) − 1 et si deg( P) = 0,  n−1 

uor
deg( P′ ) = −∞ donc u(En ) ⊂ En . 0 ··· ··· ··· 0 1
d
⋄ Si P 6= 0 et deg( P) = d avec P = ∑ ak Xk avec ad 6= 0 alors
k=0 4. ⋄ P′ = 0 si et seulement si P est constant donc Ker(un ) = E0 et

P
d d   
v( P) = ∑ ak (X + 1)k = ∑ ak X k + kX k−1 + · · · = ad X d + dad + Im(un ) = Vect un (1), un (X ), . . . , un (X n ) = Vect 0, 1, . . . , nX n−1
 dk−=10 k=0 donc Im(un ) = En−1 .
ad−1 X + · · · donc v(En ) ⊂ En . ⋄ det(vn ) = det(Vn ) = 1 6= 0 donc vn est un automorphisme de En
On a donc ∀ P ∈ E, v( P) ∈ E et ∀( P, Q) ∈ E2 , ∀λ ∈ R , v( P +

r
et donc Ker(vn ) = {0} et Im(vn ) = En .
λQ) = ( P + λQ)(X + 1) = P(X + 1) + λQ(X + 1) = v( P) + λv(Q)

e
donc v ∈ L(E) .
′
er
∗ k k−1 5. On a ∀ P ∈ E, u ◦ v( P) = P(X + 1) = P′ (X + 1) = v ◦ u( P) donc

m
3. ⋄ u(1
) = 0 et ∀k ∈ N , u( X ) = kX donc
 un et vn commutent .
0 1 0 ··· 0
 .. .. 
0 0 2 . .
. 
m

.. ..
 ..
Un =  . . 0  ∈ M n + 1 (R ) 6. ⋄ χun = χUn donc χun = X n+1 .⋄ Donc 0 est valeur propre de mul-

r
. .. 
 .. . n tiplicité n + 1 et l’espace propre associé est E0 (un ) = Ker(un ) =

o
0 ··· ··· 0 0 E0 6= En car n ∈ N ∗ donc un n’est pas diagonalisable .
or

F
✉: mamouni.myismail@gmail.com

37
i

r
s
P ROBLÈMES C ORRIGÉS -MP ✍ M AMOUNI M Y I SMAIL MAMOUNI . NEW. FR

i
s

s
u
7. ⋄ χvn = χVn donc χvn = (X − 1)n+1 .⋄ Donc Sp(vn ) = {1} et 4. On lit sur la matrice Wn : rg(Wn ) = n, Q0 ∈ Ker(wn ),

s
donc si vn était diagonalisable sa matrice dans une base de di- ∀k ∈ [[0, n − 1]], Qk ∈ Im(wn ) donc Ker(wn ) = R .Q0 et

agonalisation serait In+1 et ceci serait vrai dans toute base donc Im(wn ) = Vect Qk 0≤k≤n−1 .

u
vn n’est pas diagonalisable .
Qk− j si j ≤ k

r
j
 5. On a, par récurrence sur j, wn (Qk ) =
0 si j k

é
8. 1. On a ∀k ∈ [[0, n]], deg(Qk ) = k donc la famille Qk 0≤k≤n est une

famille de degrés étagés donc Qk 0≤k≤n est une base de En . n

Po r


r
9. 1. Puisque B est une base de En , ∀ P ∈ En , ∃ ! β k 0≤ k ≤ n
, P= ∑ β k Qk .
2. ⋄ wn (Q0 ) = vn (Q0 ) − Q0 = Q0 − Q0 donc wn (Q0 ) = 0. k=0

u
k−1 k−1
1 1

uor
n n
j j
wn ( Qk ) = vn ( Qk ) − Qk =
k! ∏ (X + 1 − j) − k! ∏ (X − j) 2. On a wn ( P) = ∑ β k wn ( Qk ) = ∑ βk Qk− j sij ≤ n,
j =0 j =0 k=0 k= j
k−2 k−1
1 1 0sij n. De plus, Qi (0) = 1 si i = 0, 0si isup1.
=
k! ∏ (X − j) − k! ∏ (X − j) j
Donc wn ( P)(0) = β j si j ≤ n, 0 si j > n.

P
j=−1 j =0
k−2
et,
1  
= ∏ (X − j) ( X + 1) − ( X − k + 1) 3. Ainsi ∀k [[0, n]], β k = wkn ( P)(0). Or wkn = (vn −

k! j =0 k  
k− j k j
k k−2
1 k−2 e n )k = ∑ (−1) vn car vn et en commutent. Et, par

r
i
=
k! ∏ ( X − j ) =
( k − 1) ! ∏
(X − j) j =0
j =0 j =0 j
récurrence facile, vn ( P)(X ) = P(X + j) donc wkn ( P)(X ) =

e
pour k = 1, wn (Q1 ) = (X + 1) − X = 1 = Q0 donc k  
∀ksup1, wn (Qk ) = Qk−1 . k− j k
er
∑ (−1) j
P(X + j). En évaluant en 0, on obtient donc

m
j =0
   
0 1 0
··· 0 k
k− j k
 .. . ∀k ∈ [[0, n]], β k = ∑ (−1) P( j) .
0 0 1 . ..  j =0
i
. 
m

.. ..
3.  ..
Wn =  . . 0  ∈ M n + 1 (R )

r
. ..   k
 .. . 1 4. Et donc la base duale de B est B ∗ = Q∗k avec Q∗k ( P) = wkn ( P)(0) =
0≤ k ≤ n ∑(

o
0 ··· ··· 0 0 j =0
or

F
✉: mamouni.myismail@gmail.com

38
i

r
s
✍ M AMOUNI M Y I SMAIL
MAMOUNI . NEW. FR
P ROBLÈMES C ORRIGÉS -MP

i
s

s
u
10. ⋄ Selon la question [8.5], ∀k ∈ [[0, n]], wnn+1 (Qk ) = 0 donc 4. 1. Puisque deg( P) = m , on a am 6= 0 .

s
wnn+1 = θn .    
m m m
Xm Xm aj  aj ( j)

é
⋄ De la même question on tire wnn (Qn ) = Q0 . j
∑ m! u j Xm = Xm

u
2. r = ∑ aj u = ∑ m! =
m! j =0
m! j =0 j =0

r
m aj m aj m!
∑ m! m(m − 1) · · · (m − j + 1) Xm− j = ∑ m! (m − j)! Xm− j donc

é
j =0 j =0
Partie II  m m
X aj
X m− j .

Po r
=∑

r
r
m! j =0
( m − j ) !

u
1. Le théorème de Cayley-Hamilton donne χ f ∈ J f donc π f | χ f . 
Xm

uor
3. Ainsi r 6= 0 donc r 6= θ donc ∀ P ∈ R [ X ] \ {0}, P(u) 6= θ
m!
2. 1. On a, par récurrence sur k, ∀ P ∈ E, uk ( P) = P(k) . Or, si P ∈ En donc Ju = {0} .
alors deg( P) < n + 1 donc P(n+1) = 0. Ainsi unn+1 = θn .

P
 5. 1. Soit P ∈ Jv , on a P(v) = θ donc, par restriction à En stable par v,
2. De même, unn X n = n! (en dérivant n fois). P(vn ) = θn donc πvn | P. Ceci donne bien, avec le résultat de [3.2],
∀ n ∈ N ∗ , ( X − 1) n +1 | P .
3. Selon [1], unn+1 = θn donc X n+1 ∈ Jun donc πun | X n+1 et donc

r
∃ m ≤ n + 1, πun = X m . Si on avait m ≤ n alors πun | X n donc 2. Donc ∀n ∈ N ∗ , ∃ Qn ∈ E, P = (X − 1)n+1 Qn donc deg( P) = n +
unn = θn mais ceci est faux selon [2]. Finalement, πun = X n+1 . 1 + deg(Qn ). En prenant nsup deg( P), ceci donne deg(Qn ) = −∞

e
donc P = 0. Donc Jv = {0} .
er
4. Selon [I.10], wnn+1 = θn et wnn 6= θn donc, comme ci-dessus,

m
π w n = X n +1 .
6. 1. Soit Q = s( P) et R = s2 ( P), on a R(X ) = Q(1 − X ) = P(1 − (1 −
X )) = P(X ) donc s2 = e (s involution) .
3. 1. πvn | χvn = χVn = (X − 1)n+1 donc ∃ m ∈ [[1, n + 1]], πvn = (X − 1)m .
m

2. On a donc X 2 −1 ∈ Js et donc Js 6= {0}. Ainsi s a un polynôme min-

r
2. Donc (vn − en )m = θn soit wm m
n = θn donc π wn | X et donc, vu le
imal πs et πs ∈ X + 1, X − 1, X 2 − 1 . Or s 6= e car s(X ) = 1 − X 6=

o
résultat de [2.4], m = n + 1 . X et, de même, s 6= −e donc πs = X 2 − 1 et Js = (X 2 − 1).R [ X ] .
or

F
✉: mamouni.myismail@gmail.com

39
i

r
s
P ROBLÈMES C ORRIGÉS -MP ✍ M AMOUNI M Y I SMAIL
MAMOUNI . NEW. FR

i
s

s
u
Partie III La formule de Taylor en X appliquée à P ∈ E donne

s
+ ∞ (m)
P (X ) m
P(X + h) = ∑ h . En prenant h = 1, on obtient

é
m!

u
m=0
+∞ +∞
P(m) ( X ) um ( P)
+∞
um n
um P ( X + 1) = ∑ m! = ∑ nm! soit vn = exp(un ) .

r
n n
1. exp(un ) = ∑ = ∑ d’après [II.2.1]. On peut donner deux m=0 m=0

é
m!
m=0 m=0 m!
démonstrations de l’égalité demandée :
n

Po r
r
• Première démonstration : 2. 1. D’après [I.9.3], ∀k ∈ [[0, n]], un (Qk ) = ∑ wn (un (Qk )) (0) Q j .
j

Montrons que exp(un ) et vn coïncident sur la base canonique j =0




u
X k 0≤ k ≤ n : Or un et vn commutent ([I.5]) donc un et wn = vn − en
 

uor
k  
j j

 k également donc w n ( u n ( Q k )) = u n w n ( Q k ) = u n Q k − j
 k
vn (X )
 = ( X + 1) k = ∑ X k− j
j si j ≤ k. = un (0)i j > k d’après ([I.8.5]). Ceci
j =0
∀k ∈ [[0, n]], k 

 n
u m (X k ) k
k! donne 0, , un Qk− j (0) Q j soit
 k
exp(un )(X ) = ∑
 n
= ∑ X m − k ∀ k ∈
selon [II.4.2]
[[ n ]] u n ( Q k ) = ∑

P
m=0 m! (k − m )!
m! j =0
m=0
k
et donc vn = exp(un ) .
∀k ∈ [[0, n]], un (Qk ) = ∑ un ( Qm ) (0) Qk−m .
• Deuxième démonstration : m=0

e r
er

m
m

or
or

F
✉: mamouni.myismail@gmail.com

40
i

r
s
✍ M AMOUNI M Y I SMAIL
MAMOUNI . NEW. FR
P ROBLÈMES C ORRIGÉS -MP

i
s

s
u

s
Devoir libre
6 Commutant d’une matrice

u
r
Blague du jour Sir Andrew John Wiles (1953- )

é
• Quel est le genre d’humour que les dindes n’aiment Mathématicien britannique, connu pour sa démonstration du

Mathématicien du jour
dernier théorème de Fermat en 1994, résolvant ainsi l’un des

Po r
r
pas ?
☛ Réponse : les farces. problèmes les plus connus de l’histoire des mathématiques en

u
• Pourquoi les lapins jouent-ils avec 46 cartes au lieu de travaillant dans le plus grand secret pendant huit ans. Pour
dévoiler sa démonstration, Wiles s’y prend de manière quasi

uor
52 cartes ?
☛ Réponse : parce qu’ils ont mangent les trèfles ! théâtrale. Il annonce trois conférences (les 21, 22 et 23 juin 1993)
• C’est une bande de poissons en train de faire des bê- sans en donner l’objet, ce qu’il ne fait que lors de la dernière
tises, quand l’un voit une étoile de mer et dit : Attention en précisant que le grand théorème de Fermat est un corol-
laire de ses principaux résultats. Son travail met ainsi fin à une

P
voilà le shérif qui arrive !
recherche qui a duré plus de 300 ans.

r
α, β et γ.
Énoncé : (extrait e3a 2011, MP)
a Montrer que la matrice A est diagonalisable.

e
On note M3 (C ) l’ensemble des matrices carrées d’ordre 3 à coeffi-
er
cients complexes, le polynôme caractéristique d’une matrice A est b Soit B une matrice de M3 (C ) qui commute avec la ma-

m
noté χ A , le polynôme minimal de la matrice A est noté PA . trice A, montrer que la matrice B est diagonalisable.
On appelle commutant de la matrice A de M3 (C ) l’ensemble C( A) c Montrer qu’il existe un polynôme T de C [ X ] vérifiant
des matrices de M3 ( A) qui commutent avec la matrice A. 
m

 T (α ) = a
On suppose dans tout cet exercice PA = χ A pour une matrice A de

r
T ( β) = b ,
M 3 (C ). 
T (γ) = c

o
① On suppose dans cette question que PA est à racines simples où a, b et c sont les valeurs propres de la matrice B.
or

F
✉: mamouni.myismail@gmail.com

41
i

r
s
P ROBLÈMES C ORRIGÉS -MP ✍ M AMOUNI M Y I SMAIL
MAMOUNI . NEW. FR

i
s

s
u
d En déduire que le polynôme T de C [ X ] vérifie l’égalité b Montrer qu’il existe une base B ′ de C 3 telle que la ma-

s
 
B = T ( A ). ′ λ1 0
trice de f dans la base B soit de la forme , avec

é
0 U

u
e En déduire le commutant de la matrice A. 
λ1 0
② On suppose, dans cette question, qu’il existe un nombre com- U = λ2 I2 + N, I2 désigne la matrice et N est une

r
0 1
plexe λ tel que PA = (X − λ)3 . On note f l’endomorphisme

é
matrice appartenant à M2 (C ), nilpotente  d’indice 2, c’est-à-
de C 3 tel que la matrice dans la base canonique de C 3 soit la 2
matrice A. N =0
dire vérifiant les propriétés suivantes : .

Po r
r
a Montrer que l’endomorphisme g = f − λId est nilpo- N 6= 0
 
tent µ L
 3 d’indice 3, c’est-à-dire vérifie les relations suivantes : c On considère la matrice M =

u
de M3 (C ),
g =0  C V

uor
. µ∈C
g2 6 = 0 


L ∈ M1,2 (C )
b Montrer qu’il existe un vecteur u de C 3 tel que la famille où , on suppose que les matrices M et

 C ∈ M2,1 (C )

(u, g(u), g2 (u)) soit une base B de C 3 .  V∈  M 2 (C )

P
c Soit H une matrice de M3 (C ) qui commute avec la ma- λ1 0
commutent.
0 U
trice A. On appelle h l’endomorphisme de C 3 tel que la ma- 
trice de h dans la base canonique de C 3 soit la matrice H et on  L=0
note h(u) = x1 u + x2 g(u) + x3 g2 (u), où x1 , x2 , x3 sont trois i Montrer C=0 .

r

NV = V N
nombres complexes. Déterminer, en fonction de x1 , x2 , x3 , la

e
matrice de h dans la base B . ii Montrer qu’il existe un polynôme R de C [ X ] vérifiant
R(U ) = V.
er
d Montrer qu’il existe un polynôme Q de C [ X ] vérifiant

m
H = Q ( A ). iii Montrer qu’il existe un polynôme S de C [ X ] tel que

e X − λ1 divise S − µ
En déduire le commutant de la matrice A. .
(X − λ2 )2 divise S − R
③ On suppose, dans cette question, qu’il existe deux nombres  
m

λ1 0
complexes distincts λ1 et λ2 tels que iv En déduire S = M.

r
0 U
PA = (X − λ1 )(X − λ2 )2 .

o
a Montrer d Déterminer le commutant de la matrice A.
or

C3 = Ker( f − λ1 Id) ⊕ Ker( f − λ2 Id)2 .

F
✉: mamouni.myismail@gmail.com

42
i

r
s
✍ M AMOUNI M Y I SMAIL
MAMOUNI . NEW. FR
P ROBLÈMES C ORRIGÉS -MP

i
s

s
u
λI3 )2 6= 0, donc g2 6= 0 : g est nilpotent d’indice 2.
Corrigé, Pr. Stainer, Lycée Clemenceau, Nantes

s
b On vérifie facilement que, pour tout vecteur u ∈ C 3

u
tel que g2 (u) 6= 0, la famille B = (u, g(u), g2 (u)) est libre.
① a La matrice A a un polynôme caractéristique scindé, à Comme l’espace vectoriel est de dimension 3, cette famille

r
3
est une
 base deC . Dans une telle base, g a pour matrice

é
racines simples, donc est diagonalisable ; de plus ses sous-
0 0 0
espaces propres sont de dimension 1. 
N= 1 0 0 .

Po r
r
b Si B commute avec A, elle stabilise les trois sous-espaces 0 1 0
propres de A, qui sont des droites. Ces trois droites sont donc

u
c Tout d’abord, h commute avec f , donc avec g. On en dé-
dirigées par des vecteurs propres de B. Une base de vecteurs
duit h( g(u)) = g(h(u)) = x1 g(u) + x2 g2 (u) et h( g2 (u)) =

uor
propres de A est donc ausi une base de vecteurs propres de
g2 (h(u)) = x 2
1 g (u). La matrice de h dans B vaut donc
A : B et A sont simultanément diagonalisables. 
x1 0 0
c Interpolation de Lagrange (α, β, γ distincts). On peut im-  x2 x1 0  = x1 I3 + x2 N + x3 N 2 .
poser la condition supplémentaire deg T 6 2.

P
x3 x2 x1
d D’après la remarque faite en 1.b, il existe une ma- d On en déduit h = x1 Id + x2 g + x3 g2 , puis, en substitu-
trice P ∈ GL3 (C ) telle que A = P.diag(α, β, γ).P−1 et B = ant f − λId à g, l’existence d’un polynôme T de degré au plus
P.diag(a, b, c).P−1 . Alors diag(a, b, c) = T (diag(α, β, γ)) et 2 tel que h = T ( f ). Matriciellement, H = T ( A).

r
B = T ( A ). e On conclut comme au 1.e.

e
e Le commutant de A est donc inclus dans l’algèbre com-
a
er
mutative C [ A] des polynômes en A. L’inclusion inverse est ③ Le polynôme PA est annulateur de A, donc de f . Comme

m
vérifiée pour tout matrice. Donc C( A) = C [ A]. Avec la re- X − λ1 et (X − λ2 )2 sont premiers entre eux, d’après le lemme
marque du 1.c, C( A) = C 2 [ A] et, comme PA est de degré 3, des noyaux, C 3 = Ker( f − λ1 Id) ⊕ Ker( f − λ2 Id)2 .
( I3 , A, A2 ) est une base de C( A).
b Comme λ1 est une valeur propre simple de f (de mul-
m

② a Par définition, PA est un polynôme annulateur de A,

r
tiplicité 1 dans χ A ), le sous-espace propre associé Ker( f −
donc ( A − λI3 )3 = 0. En termes d’endomorphismes, g3 = 0. λ1 Id) est de dimension 1 ; donc Ker( f − λ2 Id)2 est de dimen-

o
De plus, comme PA est le polynôme minimal de A, ( A − sion 2. De plus, c’est le noyau d’un polynôme en f , donc il
or

F
✉: mamouni.myismail@gmail.com

43
i

r
s
P ROBLÈMES C ORRIGÉS -MP ✍ M AMOUNI M Y I SMAIL
MAMOUNI . NEW. FR

i
s

s

u
est stable par f . Dans une base B ′ de C 3 adaptée  à la décom-
  S ( λ1 ) = µ

s
λ1 0 S ( λ ) = R ( λ2 ) .
position du 3.a, la matrice de f est de la forme ,  ′ 2
0 U S ( λ2 ) = R ′ ( λ2 )

u
où U représente fe = f |Ker( f −λ2 Id)2 . Comme ( fe − λ2 Id)2 = 0, Or l’application ∆ définie de C 2 [ X ] dans C 3 par ∆( P) =
N = U − λ2 I2 vérifie N 2 = 0. ( P(λ1 ), P(λ2 ), P(λ2 )) est linéaire, injective (si ∆( P) = 0, alors

r
P est de degré au plus 2 et possède au moins 3 racines comp-

é
Si N = 0, alors la matrice de f dans B ′ est diagonale, f est di-
agonalisable et son polynôme minimal est à racines simples. tées avec leur multiplicité, donc vaut 0), entre deux espaces
Comme ce n’est pas le cas, N 6= 0 : N est nilpotente d’indice vectoriels de même dimension finie ; donc ∆ est un isomor-

Po r
r
2. phisme. D’où l’existence de S ∈ C 2 [ X ] satisfaisant les condi-
 S ( λ1 ) = µ

u
c
 tions S ( λ ) = R ( λ2 ) .
 ′ 2

uor
     L(U − λ1 I2 ) = 0
λ1 0 λ1 0 S ( λ2 ) = R ′ ( λ2 )
i M = M⇔ (U − λ1 I2 )C = 0 ⇔    
0 U 0 U  λ1 0 S ( λ1 ) 0
 UV = VU iv S = . Or S(λ1 ) = µ ;
0 U 0 S(U )
 L(U − λ1 I2 ) = 0
de plus,comme (X  − λ2 )2est annulateur  deU, S(U )= R(U ).

P
(U − λ1 I2 )C = 0

V N = NV λ1 0 µ 0 µ 0
Donc S = = = M.
0 U 0 R(U ) 0 V
Comme U admet comme polynôme annulateur (X − λ2 )2 , sa
seule valeur propre est λ2 , donc U − λ1 I2 estinversible. On d Soit B ∈ M3 (C ) et b l’endomorphisme de C 3

r
     L=0 canoniquement associé à B. Alors B commute avec A
λ1 0 λ1 0
en déduit M = M⇔ C=0 si et seulement
 si b commute avec f , i.e M = Mat B ′ (b) com-

e
0 U 0 U  
V N = NV λ1 0
mute avec . C’est encore équivalent (la réciproque
er
ii Il s’agit de reprendre avec U le raisonnement des ques- 0 U

m
tions 2.a, b, c et d. On peut imposer la condition supplémen- de 3.c.δ est
évidente) 
à l’existence d’un polynôme S ∈ C 2 [ X ]
taire deg R 6 1. λ1 0
tel que S = M, i.e S( f ) = b, i.e S( A) = B.
iii Les conditions imposées signifient que λ1 est racine 0 U
m

de S − µ et que λ2 est racine Comme dans les questions 1 et 2, C( A) = C 2 [ A] et C( A) est


 au moins double de S − R ;

r
 S ( λ1 ) = µ de dimension 3.
autrement dit, que S vérifie (S − R)(λ2 ) = 0 , ou encore

o

( S − R ) ′ ( λ2 ) = 0
or

F
✉: mamouni.myismail@gmail.com

44
i

r
s
✍ M AMOUNI M Y I SMAIL
MAMOUNI . NEW. FR
P ROBLÈMES C ORRIGÉS -MP

i
s

s
u

s
Devoir Surveillé
1 Diagonalisation du crochet de Lie

u
r
Blague du jour Otto Toeplitz (1881-1940)

Mathématicien du jour
é
☛ Quelle différence y a-t-il entre Windows et un clou ? Mathématicien allemand. Il était issu d’une famille de math-

Po r
ématiciens. Excellent pédagogue, Toeplitz s’intéressait aussi à

r
- Aucune : tous deux sont destins se planter.
☛ Quelle est la différence entre Windows XP et un virus l’histoire des mathématiques. On lui doit les matrices dites de

u
? Toeplitz, dont toutes les diagonales sont constantes, ces matri-
ces sont très utilises dans les théorie de complexité et d’analyse

uor
- Le virus il fonctionne !
de Fourrier.

P
Enoncé : CNC 2000, MP
Pour u ∈ L(E) on pose u0 = Id et si k ∈ N , k ≥ 2, uk = uuk−1 . On
Définitions et notations rappelle qu’un endomorphisme u est dit nilpotent s’il existe p ∈ N ∗
tel que u p = 0 (endomorphisme nul).

r
On considère un espace vectoriel E, de dimension finie n ≥ 2, sur
le corps K (K = R ou C ). L(E) désigne l’algèbre des endomor-
On définit l’application :

e
phismes de E ; si u, v ∈ L(E) , l’endomorphisme composé u ◦ v sera
Φ : L(E) × L(E) −→ L(E)
er
noté simplement uv, [u, v] désignera l’endomorphisme uv − vu et
(u, v) 7→ [u, v]

m
l’identité se notera Id.
et pour u ∈L(E) l’application :
Si u est un endomorphisme de E, on note Tr(u) la trace de u et Φu : L(E) −→ L(E)
v 7→ [u, v]
m

Sp(u) l’ensemble des valeurs propres de u. T désigne l’ensemble


∗2

r
des endomorphismes de E de trace nulle. Si λ est une valeur propre Pour (m, p) ∈ N , on note Mm,p (K ) l’ensemble des matrices é co-
de u, on note Eu (λ) le sous-espace propre de u associé é la valeur efficients dans K , é m lignes et p colonnes. Im est la matrice identité

o
propre λ. d’ordre m. Enfin, diag(α1 , α2 , . . . , αn ) désigne la matrice carrée d’or-
or

F
✉: mamouni.myismail@gmail.com

45
i

r
s
P ROBLÈMES C ORRIGÉS -MP ✍ M AMOUNI M Y I SMAIL
MAMOUNI . NEW. FR

i
s

s
u
dre n de terme général αi δij oé δij est le symbole de Kronecker ( on b En déduire que si u est nilpotent, alors Φu l’est aussi.

s
rappelle que δij = 1 si i = j et δij = 0 si i 6= j ).
B- Détermination de l’image de Φ

é
1ére Partie

u
A- Quelques propriétés de Φu Soit u un endomorphisme non nul de E de trace nulle.

r
① u peut-il étre une homothétie ?

é
① Montrer que T est un hyperplan de L(E) . ② Montrer qu’il existe e1 ∈ E tel que la famille (e1 , u(e1 )) soit
② Montrer que Φ est une application bilinéaire antisymétrique. libre.

Po r
r
③ Soit u ∈ L(E) un endomorphisme qui n’est pas une homoth- ③ En déduire l’existence d’une base (e1 , e2, . . . , en ) de E telle que
étie. la matrice A de u dans cette
 base soit
 de la forme :

u
0 tX
a Montrer que Vect({ Id, u, . . . , un−1 }) est inclus dans

uor
Y A1
Ker Φu et que dim (Ker Φu ) ≥ 2. 2
où (X, Y ) ∈ (Mn−1,1 (K )) et A1 ∈ Mn−1 (K ).
b Montrer que si v ∈ Ker Φu , alors v(Eu (λ)) ⊂ Eu (λ) pour ④ On suppose A1 = UV − VU avec (U, V ) ∈ (Mn−1 (K ))2
tout λ ∈ Sp(u). a Montrer qu’on peut trouver α ∈ K tel que la matrice

P
④ Montrer que l’image de Φ est incluse dans T et que pour U − αIn−1 soit inversible.
   t 
u ∈ L(E), Im Φu ⊂ T . ′ α 0 ′ 0 R
Existe-t-il u, v ∈ L(E) tels que [u, v] = Id ? Peut-on avoir b On pose U = et V = avec ( R, S) ∈
0 U S V
Im Φu = T ? (Mn−1,1 (K ))2 ; établir l’équivalence :

r
⑤ Soit u ∈ L(E). A = U ′ V ′ − V ′ U ′ ⇐⇒ [ t X = −t R(U − αIn−1 ) et Y =

e
a Montrer que u est une homothétie si et seulement si pour (U − αIn−1 )S ].
er
tout x ∈ E, la famille ( x, u( x )) est liée. ⑤ Montrer alors par récurrence sur n que l’image de Φ est égale

m
àT.
b En déduire que Ker Φu = L(E) si et seulement si u est
C- Détermination de Tr(Φu )
une homothétie.
Soit u un endomorphisme de E. Soient B = (e1 , e2, . . . , en ) une
m

⑥ a Soient u, v ∈ L(E) ; montrer par récurrence sur k que


base de E et A = (ai,j )1≤i,j≤n la matrice de u dans cette base. Pour

r
k
p (i, j) ∈ {1, 2, . . . , n}2 , ui,j désigne l’endomorphisme de E tel que :
(Φu )k (v) = ∑ (−1) p Ck uk− p vu p .

o
p=0 ∀k ∈ {1, 2, . . . , n}, ui,j (ek ) = δjk ei .
or

F
✉: mamouni.myismail@gmail.com

46
i

r
s
✍ M AMOUNI M Y I SMAIL
MAMOUNI . NEW. FR
P ROBLÈMES C ORRIGÉS -MP

i
s

s
u
① Rappeler pourquoi (ui,j )1≤i,j≤n est une base de L(E). imal de u ?

s
En déduire que Ker Φu = Vect( Id, u, . . . , un−1 ).
② Calculer, pour tout (i, j, k, l ) ∈ {1, 2, . . . , n}4 , le produit ui,j uk,l

é
et montrer que l’on a :

u
B- Cas où dim E = 2
n n
∀(i, j) ∈ {1, 2, . . . , n}2 , Φu (ui,j ) = ∑ ak,i uk,j − ∑ a j,k ui,k . Soit u un endomorphisme de E qui n’est pas une homothétie,

r
k=1 k=1 dim E = 2.

é
③ En déduire Tr(Φu ).
① Montrer que Ker Φu = Vect( Id, u) (on pourra utiliser une

Po r
éme
base de E de la forme (e, u(e)) dont on justifiera l’existence).

r
2 Partie
A- Cas où u est diagonalisable ② Montrer que le polynôme caractéristique de Φu est de la

u
Dans cette question on suppose que u est diagonalisable. forme X 2 (X 2 + β) avec β ∈ K .

uor
On pose Sp(u) = {λ1 , λ2 , . . . , λ p }. Pour tout i ∈ {1, . . . , p}, mi ③ Si β = 0, l’endomorphisme Φu est-il diagonalisable ?
désigne l’ordre de multiplicité de la valeur propre λi de u.
④ On suppose β 6= 0 ; étudier la diagonalisabilité de Φu selon
① Soit B = (e1 , e2, . . . , en ) une base de E formée de vecteurs pro- que K = R ou K = C .
pres de u. Pour simplifier les notations dans cette question,

P
on pose u(ei ) = µi ei ∀ i ∈ {1, .., n}. ⑤ On suppose Φu diagonalisable.

a Montrer que a Montrer que Sp(Φu ) = {0, λ, −λ} où λ est un scalaire


non nul .
∀(i, j) ∈ {1, 2, . . . , n}2 : Φu (ui,j ) = (µi − µ j )ui,j .

r
b En déduire que Φu est diagonalisable et préciser Dans la suite de la question, v (respectivement w) désigne un

e
Sp(Φu ). vecteur propre de Φu associé é la valeur propre λ (respective-
er
② Montrer que ment −λ).

m
Ker Φu = {v ∈ L(E) /∀i ∈ {1, .., p} v(Eu (λi )) ⊂ Eu (λi )}. b L’endomorphisme v peut-il être inversible ? Calculer
③ En déduire que Ker Φu est isomorphe é L(Eu (λ1 )) × Tr(v) puis v2 .
L(Eu (λ2 )) × . . . × L(Eu (λ p )).
m

Quel est le rang de Φu ? c Détermination de Sp(u) :

r
④ On suppose en plus que u a n valeurs propres distinctes. i Pour quelles valeurs du vecteur e la famille (e, v(e))

o
Quel est la dimension de Ker Φu ? Quel est le polynôme min- est-elle une base de E ?
or

F
✉: mamouni.myismail@gmail.com

47
i

r
s
P ROBLÈMES C ORRIGÉS -MP ✍ M AMOUNI M Y I SMAIL
MAMOUNI . NEW. FR

i
s

s
u
ii Vérifier que la matrice de u dans une telle base ③ Conclure que v est un endomorphisme nilpotent.

s
est triangulaire inférieure puis en déduire que Sp(u) = ☛ Dans la suite on suppose que dim Ker v = 1
Tr(u) − λ Tr(u) + λ

é
, }. Que peut-on alors dire de u ?

u
{ ④ a Montrer que pour tout p ∈ {1, 2, ..., n}, Im v p est stable
2 2
par les endomorphismes u et v.
d Montrer que E = Ker v ⊕ Ker w puis en déduire que u

r
b Soit p ∈ {1, 2, ..., n − 1} ; en considérant les endomor-

é
est diagonalisable.
phismes v1 et u1 induits par v et u sur Im v p , montrer que
C- Cas où Φu est diagonalisable dim (Im v p ) = 1 + dim (Im v p+1 ).

Po r
r
Soit u un endomorphisme de E tel que Φu soit diagonalisable d Déduire de ce qui précède que vn−1 6= 0 et vn = 0.

u
et Sp(u) 6= ∅. Soit (v1 , v2 , . . . , vn2 ) une base de L(E) formée de
vecteurs propres de Φu de sorte que Φu (vi ) = β i vi , ∀i ∈ {1, .., n2 }. ⑤ Soit e ∈ E tel que vn−1 (e) 6= 0 ; montrer que la famille

uor
Soit enfin λ ∈ Sp(u) et x ∈ E un vecteur propre associé. B = (e, v(e), . . . , vn−1 (e)) est une base de E et écrire la ma-
① Calculer u(vi ( x )) en fonction de λ, β i et vi ( x ). trice de l’endomorphisme v dans cette base.
② Montrer que l’application Ψ : L(E) −→ E, v 7→ v( x ) est ⑥ On pose A = {w ∈ L(E) / wv − vw = λv}.

P
linéaire surjective. a Montrer que A contient un endomorphisme w0 dont
③ Montrer alors que u est diagonalisable. la matrice relativement é la base B est diag(0, λ, 2λ, . . . , (n −
1 ) λ ).
3éme Partie

r
b Montrer que A est un sous-espace affine de L(E) dont
Soit λ une valeur propre non nulle de Φu et v un vecteur propre
on précisera la direction.
associé ; on désigne par Pu le polynôme caractéristique de u.

e
c Déterminer la dimension ainsi qu’une base de la direc-
er
① a Montrer que ∀ x ∈ K , v(u − xId) = (u − ( x + λ) Id)v. tion de A.

m
b Qu’en déduit-on sur Pu si det v 6= 0. ⑦ Quelle est alors la forme de la matrice dans la base B de l’en-
domorphisme u ?
c Montrer alors que l’endomorphisme v n’est pas in-
m

⑧ On suppose dans cette question que la matrice de u dans une


versible. base B ′ de E est de la forme diag(α, α + λ, α + 2λ, . . . , α + (n −

r
② Montrer que ∀k ∈ N ∗ , Φu (vk ) = kλvk ; qu’en déduit-on si 1)λ) ; décrire par leur matrice dans la base B ′ les éléments de

o
v p 6= 0 pour un certain p ∈ N ∗ ? l’espace EΦu (λ) ; quelle est sa dimension ?
or

F
✉: mamouni.myismail@gmail.com

48
i

r
s
✍ M AMOUNI M Y I SMAIL
MAMOUNI . NEW. FR
P ROBLÈMES C ORRIGÉS -MP

i
s

s
u
Corrigé, Pr. Mamouni, CPGE My Youssef, Rabat ⑤ a L’implication directe est évidente.

s
Réciproquement, supposons { x, u( x )} est liée, donc ∀ x ∈

u
E, ∃λx ∈ K tel que u( x ) = λx .x, pour montrer que u est une
homothétie il suffit de montrer que λx ne d"pond pas de x,
1ère Partie.

r
autrement dit λx = λy .

é
A-Quelques propriétés de Φu . Soit x, y ∈ E non nul.

Po r
r
☛ 1ér cas : { x, y} est liée, donc y = αx, d’où u(y) = αu( x ),
① T = Ker Tr est un hyperplan car Tr est une forme linéaire sur

u
ainsi λy .y = αλx .x = λx .y, d’où λy = λx .
E, non nulle, vu que Tr(IdE ) = n.

uor
② Vérifier rapidement que : ☛ 2ème cas { x, y} est libre.
☛ Φ(u, v) = −Φ(v, u), d’où l’antisymétrie. u( x + y) = u( x ) + u(y) =⇒ λx+y .( x + y) = λx .x + λy .y
☛ Φ(u + λv, w) = Φ(u, w) + λΦ(u, w), d’où la linéarité à =⇒ (λx+y − λx ).x + (λx+y − λy ).y = 0E

P
gauche, l’antisymétrie en plus implique la linéarité à droite, =⇒ λx+y = λx = λy
donc la bilinéarité.
③ a IdE , u, . . . , un−1 appartiennent à Ker Φu car commuttent
⑥ a Pour k = 0, vrai car (Φu )0 (v) = v.
avec u, donc Vect(Id E , u, . . . , un−1 ) ⊂ Ker Φu , d’autre part

r
{IdE , u} est libre dans Ker Φu , car u n’est pas une homothétie, Supposons vrai pour k, donc
donc dim Ker Φu ≥ 2.

e
(Φu )k+1 (v ) = Φu ◦ (Φu )k (v ) !
er
b v ∈ Ker Φu =⇒ v commute avec u, donc les sous- k  
p

m
espaces propres Eu (λ) de u sont stables par v. = Φu ∑ (−1) p uk− p vu p
p=0
k
④ Im Φ ⊂ T car Tr(uv) = Tr(vu), et aussi Im Φu ⊂ Im Φ ⊂ T . k    
p p k− p p
On ne peut pas avoir [u, v] = IdE , car Tr(IdE ) = n 6= 0 et = ∑ (−1) Φu u vu
m

p=0
k
Tr([u, v]) = 0.

r
k  
On ne peut pas avoir Im Φu = T , car dim T = n2 − 1, alors p p
uk+1− p vu p
= ∑ (−1)

o
que dim Im Φu = n2 − dim Ker Φu ≤ n2 − 2. p=0
k
or

F
✉: mamouni.myismail@gmail.com

49
i

r
s
P ROBLÈMES C ORRIGÉS -MP ✍ M AMOUNI M Y I SMAIL
MAMOUNI . NEW. FR

i
s

s
u
 
k
p E tel que {e1 , u(e1 )} soit libre.
p
uk− p vu p+1

s
− ∑ (−1) k ③ Prendre e2 = u(e1 ) puis utiliser le théorème de la base in-
p=0
  complète, car {e1 , e2 } libre, ainsi, u(e1 ) = e2 nepeut pass’ex-

u
k
p
= ∑ (−1) p
uk+1− p vu p 0 tX
p=0
k primer en fonction de e1 , d’où A = MB (u) = , où
Y A1

r
k+1  
p−1 B = ( e 1 , . . . , e n ).

é
+ ∑ (−1) p
uk+1− p vu p
k a
p=1 ④ Il suffit de prendre α qui n’est pas valeur propre de U.

Po r
On remplace p par p-1 dans la 2ème somme

r
k     b Un calcul trés simple à faire.
k+1 p p−1
= u v + ∑ (−1) p
kuk+1− p vu p

u
+ ⑤ On a déjà vu que Im Φ ⊂ T dans I.A.4), montrons l’autre in-
p=1
k k

uor
clusion réciproque par récurrence sur n = dim E. Pour n = 1,
+(−1)k+1 vuk+1
  dim L(E) = 1, donc tous les endomorphismes sont propor-
k
k+1 p tionnels à IdE , donc des homothéties, d’où Φ = 0, donc
=u v+ ∑ (−1) p
kuk+1− p vu p
p=1
k+1 dim Im Φ = 0 = dim T , d’où l’égalité.
Supposons vrai pour n − 1, donc Tr(u) = Tr( A) = Tr( A1 ) =

P
+(−1)k+1 vuk+1
k+1   0, appliquons l’hypothèse de récurrence pour l’endomor-
p phisme canoniquement associé à A1 , donc
= ∑ (−1) p
kuk+1− p vu p  A1 = UV −
p=0
k + 1 α 0
VU, d’où A = U ′ V ′ − V ′ U ′ , avec U ′ = , V′ =
0 U

r
b Supposons uk = 0, dans ce cas (Φu )2k (v) =  
  0 tR
2k où α, S, R vérifient la question précédente, choi-

e
p S V
∑ (−1) 2k ku2k− p vu p = 0, car u p = 0 si p ≥ k et
p
sis tels que U − αIn−1 inversible, S = (U − αIn−1 )−1Y et
er
p=0
R = −t ((U − αIn−1 )−1 )X.

m
u2k− p = 0 si p ≤ k. Donc u nilpotent =⇒ Φu nilpotent.
Soit u′ , v′ les endomorphismes canoniquement associés à U ′
B-Détermination de l’image de Φ. et V ′ , alors u = u′ v′ − v′ u′ ∈ Im Φ.
m

① Si u = λ IdE , alors Tr(u) = λn = 0, donc λ = 0, d’où u = 0, C-Détermination de Tr(Φu ).

r
contradiction, donc u ne peut pas être une homothétie. ① La famille (ui,j )1≤i,j≤n est de cardinal n2 = dim (L(E)) , il suf-

o
② Comme u n’est pas une homothétie d’aprés I.A.5.a) ∃e1 ∈ fit donc de montrer qu’elle est libre.
or

F
✉: mamouni.myismail@gmail.com

50
i

r
s
✍ M AMOUNI M Y I SMAIL MAMOUNI . NEW. FR
P ROBLÈMES C ORRIGÉS -MP

i
s

s
u
En effet supposons ∑ λi,j ui,j = 0, donc ∀1 ≤ k ≤ n, Tr(Φu ) = ∑ (ai,i − a j,j ) = ∑ ai,i − ∑ a j,j = 0 car

s
1≤i,j≤ n 1≤i,j≤ n 1≤i,j≤ n 1≤i,j≤ n
on a ∑ λi,j ui,j (ek ) = 0, d’où ∑ λi,j δj,k ei = 0, d’où i, j jouent des rôles symétriques.

u
1≤i,j≤ n 1≤i,j≤ n
∑ λi,k ei = 0, d’où λi,k = 0, ∀1 ≤ i, k ≤ n, car la famille 2ème Partie.

r
1≤ i ≤ n

é
(ei )1≤i ≤n est libre. A-Cas où u est diagonalisable.
 
② Pour tout 1 ≤ p ≤ n, on a : ui,j uk,l (e p ) = δl,p ui,j (ek ) = µ1

Po r
r
δl,p δj,k ei = δj,k ui,l (e p ), d’où ui,j uk,l = δj,k ui,l car ils coincident a  .. 
① A = MB (ui,j ) =  .  est diagonale,
sur la base (e p )1≤ p≤n .

u
µn
On a u = ∑ ak,l uk,l , d’où : d’aprés I.C.2), Φu (ui,j ) = ai,i ui,j − a j,i ui,j = (µi − µ j )ui,j .

uor
1≤ k,l ≤ n
Φu (ui,j ) = uui,j − ui,j u b D’aprés la question précédente, µi − µ j sont des
= ∑ ak,l uk,l ui,j − ∑ ak,l ui,j uk,l valeurs propres, dont les vecteurs propres associés sont les
1≤ k,l ≤ n 1≤ k,l ≤ n (ui,j )1≤i,j≤n qui forment une base de L(E), ainsi Φu admet

P
= ∑ ak,l δl,i uk,j − ∑ ak,l δj,k ui,l une base propre donc diagonalisable.
1≤ k,l ≤ n 1≤ k,l ≤ n
② v ∈ Ker Φu =⇒ v(Eu (λi )) ⊂ Eu (λi ), ∀1 ≤ i ≤ p, d’aprés
= ∑ ak,i uk,j − ∑ a j,l ui,l
I.A.3.a).
1≤ k ≤ n 1≤ l ≤ n
= ∑ ak,i uk,j − ∑ a j,k ui,k Inversement supposons v(Eu (λi )) ⊂ Eu (λi ), ∀1 ≤ i ≤ p, et

r
1≤ k ≤ n 1≤ k ≤ n montrons que vu = uv, il suffit alors de le montrer sur la base
On remplace l par k dans la 2ème somme ( e i )1 ≤ i ≤ n .

e
③ D’aprés la question précédente, on a : En effet ei ∈ Eu (µi ) =⇒ v(ei ) ∈ Eu (µi ) =⇒ uv(ei ) = µi v(ei ),
er
or vu(ei ) = v(µi ei ) = µi v(ei ), d’où l’égalité.

m
Φu (ui,j ) = ∑ ak,i uk,j + ai,i ui,j − ∑ a j,k ui,k − a j,j ui,j
1≤ k ≤ n 1≤ k ≤ n ③ Posons Ψ : Ker Φu −→ L(Eu (λ1 )) × · · · × L(Eu (λ p ))
k6=i k6= j
v 7−→ (v| Eu (λ1 ) , · · · , v| Eu (λ1 ) )
= ∑ ak,i uk,j − ∑ a j,k ui,k + (ai,i − a j,j )ui,j
Ψ est bien définie car les sous-espaces propres Eu (λi ) sont
m

1≤ k ≤ n 1≤ k ≤ n
stables par tout v ∈ Ker Φi qui y induit un endomorphisme.

r
k6=i k6= j

Ainsi les termes diagonaux de la matrice de Φu dans la Ψ est linéaire, car (v + λw)| Eu (λi ) = v| Eu (λi ) + λw| Eu (λi ) , pour

o
base (ui,j )1≤i,j≤n , sont les ai,i − a j,j tel que 1 ≤ i, j ≤ n, d’où tous v, w ∈ Ker Φi .
or

F
✉: mamouni.myismail@gmail.com

51
i

r
s
P ROBLÈMES C ORRIGÉS -MP ✍ M AMOUNI M Y I SMAIL
MAMOUNI . NEW. FR

i
s

s
u
Ψ est injective, car v ∈ Ker Φi =⇒ v = 0 sur Eu (λi )∀1 ≤ i ≤ teur de u, non nul de degré inférieur à n − 1, impossible car

s
p
M deg πu = n puisque u admet n valeurs propres.
p, donc v = 0 sur Eu (λi ) = E car u est diagonalisable.  

é
n −1
Donc dim Vect(Id E , u, · · · , u ) = n = dim (Ker Φu ) et

u
i =1
Enfin, Soit (v1 , · · · , v p ) ∈ L(Eu (λ1 )) × · · · × L(Eu (λ p )), cher- Vect(Id E , u, · · · , un−1 ) ⊂ Ker Φu , d’où l’égalité.

r
chons v ∈ Ker Φu tel que Ψ(v) = (v1 , · · · , v p ), pour cela, tout

é
x ∈ E s’écrit de façon unique sous la forme, x = x1 + · · · + B- Cas où dim E = 2.
x p tel que xi ∈ Eu (λi ), posons v( x ) = v1 ( x1 ) + · · · + v p ( x p ) il
① u n’est pas une homothétie, donc ∃e ∈ E tel que B = (e, u(e))

Po r
r
est clair que v| Eu (λi ) = vi et donc v(Eu (λi )) = vi (Eu (λi )) ⊂
libre dans E, d’aprés I.A.5.a), donc base de E car dim E = 2.
Eu (λi ), d’où v ∈ Ker Φu et Ψ(v) = (v1 , · · · , v p ). Donc Ψ est

u
surjective. Soit v ∈ Ker Φu , donc uv = vu, montrons que v ∈
Vect(IdE , u), c’est à dire v = λ IdE +µu

uor
Ainsi Ψ définit un isomorphisme de Ker Φu vers L(Eu (λ1 )) ×    
· · · × L(Eu (λ p )). 0 α a b
 Soit U = MB (u) = et V = MB (v) = ,
1 β c d
Donc dim (Ker Φu ) = dim L(Eu (λ1 )) × · · · × L(Eu (λ p )) =
p p p montrons alors que V = λIn + µU, il suffit de prendre λ = a
2 et µ = c en utilisant le fait que UV = VU.

P
∑ dim (L(Eu (λi ))) = ∑ dim (Eu (λi )) = ∑ m2i , car u
i =1 i =1 i =1 Ainsi Ker Φu ⊂ Vect(Id E , u), l’autre inclusion est évidente car
est diagonalisable, donc rg (Im Φu ) = dim (Im Φu ) = IdE et u commutent avec u.
p
dim (L(L(E))) − dim (Ker Φu ) = (n ) − ∑ m2i 2 2
② χΦ| Ker Φu divise χΦu , car Ker Φu stable par Φu , or dim Ker Φu =

r
i =1
2 et 0 est l’une valeur propre de Φ| Ker Φu , donc χΦ| Ker Φu = X 2 ,
④ Si u n’admet que des valeurs propres distinctes alors elles

e
d’où χΦu = X 2 (X 2 + β).
sont toutes simples, donc mi = 1, ∀1 ≤ i ≤ n, d’où
er
dim (Ker Φu ) = n. ③ Si β = 0, alors χΦu = X 4 , si de plus Φu est diagonalisable,

m
IdE , u, . . . , un−1 ∈ Ker Φu car commuttent avec u, donc alors πΦu = X, car ses racines simples, or πΦu (Φu ) = 0, d’où
Vect(IdE , u, · · · , un−1 ) ⊂ Ker Φu . Φu = 0, donc Ker Φu = L(E), donc u est une homothétie,
D’autre part, supposons (IdE , u, . . . , un−1 ) est liée, alors ils ex- contradiction.
m

isteraient des scalaires, (λk )0≤k≤n−1 non tous nuls, tels que ④ Supposons β 6= 0.

r
n −1 n −1
☛ K = C , soit λ, −λ les solutions dans C de l’équation :
∑ λk uk = 0, d’où P(X) = ∑ λk Xk est un polynôme annula-

o
k=0 k=0
X 2 + β = 0, ce sont des racines simples de χΦu et 0 est une
or

F
✉: mamouni.myismail@gmail.com

52
i

r
s
✍ M AMOUNI M Y I SMAIL
MAMOUNI . NEW. FR
P ROBLÈMES C ORRIGÉS -MP

i
s

s
u
valeur propre double, dont l’espace propre associé est de di- Tr(a) − λ
Ainsi Tr(u) = 2a + λ, d’où a = et

s
mension 2, donc Φu diagonalisable.  2 
☛ K = R et β < 0, pareil que le 1ér cas. Tr(a) − λ Tr(a) + λ

é
Sp(u) = a = ,a+λ = .

u
☛ K = R et β > 0, dans ce cas χΦu n’est pas scindé dans R , 2 2
car admet des racines complexes, non réelles, donc Φu n’est Donc u est diagonalisable car admet 2 valeurs propres dis-

r
pas diagonalisable. tinctes, et dim E = 2.

é
a Reprendre le raisonnement fait dans la question précé- ⑥ v non inversible, donc Ker v 6= {0E }, et v 6= 0, donc Ker v 6= E,

d’où dim Ker v = 1, de même dim Ker w = 1

Po r
r
dente.
Supposons Ker v ∩ Ker w 6= {0E }, alors Ker v = Ker w, vu
b Φu (v) = λv, donc uv − vu = λv, supposons v inversible, que dim Ker v = dim Ker w = 1.

u
h vi
donc u − vuv−1 = λid E , d’où uv−1 , = IdE , impossible car

uor
λ C- Cas où Φu est diagonalisable
IdE ∈
/ Im Φ.
λ λ ① uvi − vi u = βvi , donc uvi ( x ) = vi u( x ) + βvi ( x ) = (λi +
v = uv − vu), donc Trv = = 0. β)vi ( x ) car u( x ) = λi x.
( Tr(uv) − Tr(vu)
Donc vi ( x ) sont des vecteurs propres de u.

P
Puisque, dim E = 2, alors χv = v2 − Tr(v)v + det v, or
det v = 0 car v n’est pas inversible et Tr(v) = 0, d’où χv = v2 , ② Il est clair que Ψ est linéaire.
comme χv (v) = 0, alors v2 = 0. Surjection : Soit y ∈ E.
c Détermination de Sp(u). ☛ Si y = 0E , prendre v = 0.

r
☛ B = (e, v(e)) base de E ⇐⇒ (e, v(e)) libre dans E ☛ Si y 6= 0E , on complète x et y pour avoir deux bases B et B ′
car dim E = 2 qui commencent par x et y, et soit v l’application linéaire qui

e
⇐⇒ v(e) 6= λe tel que λ ∈ Sp(v) transforme B en B ′ , donc v( x ) = y.
er
⇐⇒ v(e) 6= 0E car Sp(v) = {0} ③ (v1 , . . . , vn2 ) est une base de L(E), donc son image par Ψ

m
puisque v2 = 0 est génératrice de Im Ψ = E car Ψ est surjective, ainsi
☛ Posons u(e) = ae + bv(e), et donc vu(e) = av(e) car v2 = 0 (v1 ( x ), . . . , vn2 ( x )) est une famille génératrice de E formée par
des vecteurs propres de u, de la quelle on peut extraire une
m

uv − vu = λv =⇒ uv(e) = vu(e) + λv(e)


base de E, donc u est diagonalisable.

r
=⇒ uv(e) = (a + λ)v(e)
 
a 0
D’où MB (u) = 3ème Partie.

o
b a+λ
or

F
✉: mamouni.myismail@gmail.com

53
i

r
s
P ROBLÈMES C ORRIGÉS -MP ✍ M AMOUNI M Y I SMAIL
MAMOUNI . NEW. FR

i
s

s
u
① ① Découle immédiatement de l’égalité uv − vu = λv. v(Im v p ) = Im v p+1 . D’aprés la formule du rang, on a :

s
② Supposons det v 6= 0, la question précédente implique dim Im v p = dim Ker v1 + dim Im v p+1 .
que Supposons Ker v1 = {0E }, donc v p+1 ( x ) = 0 =⇒

u
det(u − x IdE ) = det(u − ( x + λ) IdE ), donc v p ( x ) ∈ Ker v1 =⇒ v p ( x ) = 0
χ u ( x ) = χ u ( x + λ ), ∀ x ∈ K .

r
③ dim Ker v = 1 =⇒ dim Im v = n − 1

é
Supposons χu n’est pas constant, soit x ∈ C racine de χu ,
=⇒ dim Im v2 = dim Im v − 1 = n − 2
alors x + λ, x + 2λ, ... sont des racines de χu qui est donc ..
nul car admet une infinité de racines, ce qui est impossi- .

Po r
r
ble, donc χu est constant. =⇒ dim Im vn−1 = 1
=⇒ dim Im vn = 0

u
③ Si v est inversible, alors det v 6= 0, donc χu est constant,
d’où deg u = 0, ce qui est impossible car dim E = deg χu Donc vn−1 6= 0 et vn = 0.

uor
② Raisonnons par récurrence sur k ∈ N ∗
⑤ cardB = n = dim E, il suffit donc de montrer que B est libre.
Pour k = 1, c’est vrai car v vecteur propre de Φu associé à la
En effet : Soit λ0 , . . . , λn−1 ∈ K tel que λ0 e + . . . +
valeur propre à la valeur propre λ. n −1
(e) = 0, on compose par un−1 , donc λ0 un−1 (e) = 0,

P
λn −1 u
Supposons vrai pour k, dans ce cas Φu (vk+1 ) = uvk+1 −
d’où λ0 = 0, puis on compose par un−2 pour montrer que
vk+1 u = (uv)vk − vk+1 u = (vu + λv)vk − vk+1 u = v(uvk − λ1 = 0 et ainsi de suite.
vk u) + λvk+1 = vΦu (vk ) + λvk+1 = (k + 1)λvk+1 .  
0 ... 0
Si v p 6= 0, alors c’est un vecteur propre de Φu associé à la  . .. 

r
valeur propre pλ.  1 .. . 
 . 
③ Si v p 6= 0, ∀ p ∈ N ∗ , alors Φu aurait une infinité de valeurs pro- M B (v) =  0 . .
 

e

 . . 
pres distinctes, les pλ, absurde, donc ∃ p ∈ N ∗ tel que v p = 0.  .. . . 
er
④ ① Im v p est stable par v, car v p commute avec v. 0 ... 0 1 0

m
D’autre part, soit y = v p ( x ) ∈ Im v p , on a uv p = v p u +
⑥ ① Il suffit de définir w0 sur la base B , pour cela posons
pλv p , d’aprés III.2, donc u(y) = uv p ( x ) = v p (u( x ) +
pλx ) ∈ Im v p , d’où Im v p est aussi stable par u. w0 (vk (e)) = Φu (vk )(e), d’aprés III.2, on a w0 (vk (e)) =
m

kλvk (e), donc MB (w) = Diag(0, λ, . . . , (n − 1)λ)


② On a : v1 : Im v p −→ Im v p avec Ker v1 = Ker v ∩

r
x 7−→ v( x ) ② ∀w ∈ A, on a w − w0 ∈ Ker Φv , d’où A est un espace

o
Im v p ⊂ Ker v, donc dim Ker v1 ≤ 1 et Im v1 = affine de direction Ker Φv et d’origine w0 .
or

F
✉: mamouni.myismail@gmail.com

54
i

r
s
✍ M AMOUNI M Y I SMAIL
MAMOUNI . NEW. FR
P ROBLÈMES C ORRIGÉS -MP

i
s

s
u
③ Montrons que (IdE , v, . . . , vn−1 ) base de Ker Φv . n −1 n −1 n −1

s
∑ λk ek , donc uv(e0 ) = ∑ λk u(ek ) = ∑ λk αk ek et vu(e0 ) =
Soit λ0 , . . . , λn−1 ∈ K tel que λ0 IdE + . . . + λn−1 vn−1 = k=0 k=0 k=0

é
0, on applique l’égalié à e, donc λ0 e + . . . + n −1 n −1

u
λn−1 vn−1 (e) = 0, or B libre, donc λ0 = . . . = λn−1 = 0,
α0 v ( e0 ) = ∑ λk α0 e k , or λv(e0 ) = ∑ λk λek , d’où λk αk −
k=0 k=0

r
donc la famille est libre. λk α0 = λk λ, donc λk (αk − α0 − λ) = 0, donc (k − 1)λλk = 0,

é
D’autre part, soit w ∈ Ker Φv , donc commute avec v d’où λk = 0 si k 6= 1, ainsi 
la 1èrecolonne de la matrice de u
mais aussi avec vk pour 0 ≤ k ≤ n − 1, or B base 0

Po r
r
 a 
de E, donc w(e) = λ0 e + . . . + λn−1 vn−1 (e) = P(v)(e),  1 
 
et wvk (e) = vk w(e) = vk P(v)(e) = P(v)(vk (e)), d’où sera de la forme suivante :  0 .

u
 .. 
w = P(v) car égaux sur la base B , donc notre famille est  . 

uor
génératrice pour Ker Φv , donc base et par suite sa dimen- 0
sion vaut n. En adoptant le même raisonnement pour calculer v (e1 ), on

⑦ Posons u(e) = λ0 e + . . . + λn−1 vn−1 (e) = P(v)(e), on a 0
 0 

P
Φu (vk ) = kλvk , d’où uvk = vk u + kλvk , d’où uvk (e) = 
 a2 

vk P(v)(e) + kλvk (e), or vn = 0, donc vP(v) = λ0 v(e) +  
trouve que la 2éme colonne est de la forme suivante  0 .
. . . + λn−2 vn−1 (e), v2 P(v) = λ0 v2 (e) + . . . + λn−3 vn−1 (e), ..., 
 .. 

vn−1 P(v)(e) = λ0 vn−1 (e),  . 

r
  0
λ0 0 ... 0
 . ..  Et
 ainsi de suite laforme finale de la matrice sera
λ0 + λ . . . 

e
 λ1 0 ... 0
 . . . 
d’où MB (u) =  .. .. ..   .. .. 
er
   a1 . . 
 0   

m
 0 ... 
λn −1 λn −2 . . . λ1 λ0 + ( n − 1 ) λ  
 . . 
 . . . . 
⑧ Posons B ′ = (e0 , . . . , en−1), donc u(ek ) = (α + kλ)ek = αk ek . 0 . . . 0 an −1 0
m

Soit v ∈ EΦu (λ), donc uv − vu = λv, posons v(e0 ) = Ces matrices forment un espace vectoriel de dimension n − 1.

or
or

F
✉: mamouni.myismail@gmail.com

55
i

r
s
P ROBLÈMES C ORRIGÉS -MP ✍ M AMOUNI M Y I SMAIL
MAMOUNI . NEW. FR

i
s

s
u

s
Devoir Libre
7 Topologie des matrices

u
Ernesto Cesàro (1859-1906)

r
Blague du jour

Mathématicien du jour
☛ Un petit garçon rentre de l’école avec son bulletin de Mathématicien italien, connu pour ses contributions à la
notes et va voir son père : géométrie différentielle et à la théorie des séries infinies. En

Po r
r
- Papa c’est vrai que tes lunettes grossisse tout ? lui théorie des nombres, il est l’origine du résultat suivant : La prob-
demande-t-il. abilité pour que deux nombres entiers, choisis aléatoirement,

u
- Bien-sûr pourquoi ? soient premiers entre eux est égale à 0,6.Sa mort fût survenue

uor
- Alors mets les avant de regarder mon bulletin de notes ! alors qu’il tenta de sauver son plus jeune fils, en train de se
noyer.

P
converge vers A.
Énoncé (extrait CNC)
d Soit A ∈ Mn (R ) telle que sp( A) 6= {0} et soit ε =
Partie I : Densité Min{|λ|, tel que λ ∈ sp( A) \ {0}}.

r
① On se propose de montrer que l’ensemble GLn (R ) des matri- i Justifier que ε existe.
1

e
ces inversibles est un ouvert dense dans Mn (R ). ii Montrer que la suite ( A − In ) 1 est une suite à
k k≥ ε +1
er
a Dire pourquoi l’application det : Mn (R ) −→ R
valeurs dans GLn (R ) qui converge vers A.

m
A 7−→ det( A)
est continue. ② Application
b En déduire que GLn (R ) est un ouvert de Mn (R ). a Montrer que l’application χ : Mn (R ) −→ R n [ X ]
m

A 7−→ χ A (X )

r
c Soit A ∈ Mn (R ) telle que sp( A) = {0}, montrer que la
est continue.
1

o
suite ( A − In )k≥1 est une suite à valeurs dans GLn (R ) qui b Soient A et B deux matrices réelles d’ordre n.
k
or

F
✉: mamouni.myismail@gmail.com

56
i

r
s
✍ M AMOUNI M Y I SMAIL
MAMOUNI . NEW. FR
P ROBLÈMES C ORRIGÉS -MP

i
s

s
u
i On suppose A inversible. Montrer que χ AB = χ BA . ① Montrer que l’ensemble des matrices orthogonales On (R )

s
(celles qui vérifient t MM = In ) est un fermé borné, en dé-
ii Montrer que ce résultat subsiste si on se suppose plus duire qu’il est compact.

u
A inversible.
② Soit P = X p + a p−1 X p−1 + · · · + a1 X + a0 ∈ C [ X ] un
c Montrer que si un matrice commute avec toutes les ma-
polynôme unitaire de degré p. Montrer que les racines de P

r
trices inversibles, alors elle commute avec toute matrice car- sont toutes dans le disque fermé D de centre 0 et de rayon

é
rée. R = Max{1, pM}, avec M = Max | ai |.
③ Pour A ∈ Mn (R ), on note par sp( A) le n-uplet formé par les 0≤ i ≤ p − 1

Po r
r
valeurs propres de A comptées avec leurs multiplicités. ③ On se propose de montrer dans cette question que l’ensem-
ble des polynômes de degré p unitaires et scindés sur R est

u
④ Soit ( An ) une suite de matrices carrés convergente vers A et
soit P une matrice carrée inversible, montrer que PAn P−1 con- un fermé de R p [ X ].

uor
(n ) (n ) (n )
verge vers A. Soit Pn = X p + a p−1 X p−1 + · · · + a1 X + a0 une suite de
⑤ . a Soit T une matrice triangulaire dont les termes diago- polynômes unitaires de degré p scindés sur R qui converge
vers un certain polynôme P
naux sont λ1 , 
. . . , λn et 

P
p
| λi − λ j |
soit ε = Min tel que λi 6= λ j . Montrer que pour a Dire pourquoi P est de la forme ∑ ai X i .
2 i =0
1 (n )
tout entier k tel que ≤ ε, le polynôme caractéristique de b Montrer que : lim ai = ai pour tout i ∈ [[ 0, p ]].
 k  n→∞

r
1 1 1
T − diag , ,..., est à racines simples. c Dire pourquoi a p = 1.
k k+1 k+n

e
(n ) (n )
b En déduire que l’ensemble des matrices à valeurs pro- d Pour tout entier naturel n, notons Zn = (z1 , · · · , z p )
er
pres simples est dense dans Mn (R ). une liste des zéros (supposés réels) du polynôme Pn pris dans

m
un ordre arbitraire, mais bien sûr comptés avec leurs multi-
⑥ Montrer que dans M p (R ), de l’ensemble des matrices diago- plicités.
nalisables est dense dans celui des matrices trigonalisables.
Montrer que la suite (Zn ) admet une suite extraite (Z ϕ(n) )
m

⑦ Montrer que l’ensemble des matrices diagonalisables est convergente, de limite Z = (z1 , · · · , z p ).

r
dense dans Mn (C ). p

Partie II : Parties fermés e En déduire que ∏(X − zi ).

o
i =1
or

F
✉: mamouni.myismail@gmail.com

57
i

r
s
P ROBLÈMES C ORRIGÉS -MP ✍ M AMOUNI M Y I SMAIL
MAMOUNI . NEW. FR

i
s

s
u
f Conclure ③ Soit ( A, H ) ∈ Gln K × Mn (K ) avec k H k < k A−1 k−1 . Mon-

s
trer que A + H est une matrice inversible et exprimer ( A +
Partie III : Connexité par arc H )−1 − A−1 comme limite d’une suite convergente.

u
Dans toute la suite Mn (K ) est muni de la norme de Schur- (On pourra écrire A + H = A( In + A−1 H .)
!1

r
2 ④ En déduire que l’application I : G(K ) → Mn (K ), A 7→ A−1

é
Frobenius k.k : A = (ai,j ) 7→ k Ak = ∑ |ai,j |2 ; G (K ) est continue.
1≤i,j≤ n
⑤ Soient A et B deux éléments de Gln (C ). Montrer que T ( x ) =

Po r
désigne Gln (C ) si K = C et Glpn (R ) = { M ∈ Gln (R ), det M > 0} si

r
det( xB + (1 − x ) A), x ∈ C , est un polynôme en x, coefficients
K = R . On se propose de montrer la connexité par arcs de l’ensem-
complexes, et que T n’est pas le polynôme nul.

u
ble
C(K ) = { A ∈ Mn (K ), deg(π A ) = n − 1}. ⑥ Soient z1 , . . . , z p les racines de T et soit r > 0,

uor
On admet le résultat suivant : Soit A ∈ Mn (K ), π A son polynôme soit φ :[0, 1] → Mn (C ), φ(t) = γ(t) B + (1 − γ(t)) A avec
minimal. deg π A = n − 1 si et seulement si existe une matrice P 
 t(1 + 2ir ) 1
si 0 ≤ t ≤ ;
n −2
γ(t) = 2
dans Gln K et a0 , . . . , an−2 , α, éléments de K , avec αn−1 = ∑ ak α k  1
 t + 2ir (1 − t) si ≤ t ≤ 1.

P
k=0 2
tels que P−1 AP soit de la forme ⑦ Montrer que φ est continue et calculer φ(0) et φ(1).
  ⑧ Montrer que l’on peut choisir r tel que φ soit valeurs dans
0 0 ··· 0 0
a0

r
1 0 · · · 0 0
a1 Gln (C ) et conclure.
 .. .. 
..
 . ..  (Si I = {i ∈ {1, . . . , p}, Im zi > 0} n’est pas vide, choisir
0 . . . . .
.

e
M = . .  (1) r < min{Im zi , i ∈ I } .)
 .. .. 1 0 an −3 0 
er
  ⑨ On admet que Glpn (R ) est connexe par arcs. J étant la matrice
0 · · · 0 1 an −2 0 

m
0 0 ··· 0 0 α de la forme (1), montrer que l’ensemble { PJP−1 , P ∈ G(K )}
Lorsque K = R on peut choisir P dans Glpn (R ). est connexe par arcs.
① Montrer que l’application det : Mn (K ) → K , A 7→ det( A) ⑩ Soit M une matrice de la forme (1) où a0 , . . . , an−2 et α
m

n −2
est continue et que G(K ) est un ouvert.

r
sont des éléments de K tels que αn−1 = ∑ ak αk . En rem-
② Montrer que si A et B sont des éléments de Mn (K ), alors k=0

o
k ABk ≤ k Akk Bk. plaçant dans M les éléments a1 , . . . , an−2 respectivement par
or

F
✉: mamouni.myismail@gmail.com

58
i

r
s
✍ M AMOUNI M Y I SMAIL
MAMOUNI . NEW. FR
P ROBLÈMES C ORRIGÉS -MP

i
s

s
u
ta1 , . . . , tan−2 , α par tα et a0 par ε(t) + a0 , où ε(t) = (tα)n−1 − que l’application ψ : [0, 1] → Mn (K ), t 7→ M(t) est continue.

s
n −2
∑ tak (tα)k − a0 , b Calculer ψ(0) et ψ(1).

u
k=1
a Montrer que l’on obtient une matrice M(t) ∈ C(K ) et c Déduire de ce qui précède que C(K ) est connexe par arcs.

é
Po r
r
i
F

nn
u
i

uor
Á la prochaine

r P
e
er

m
m

or
or

F
✉: mamouni.myismail@gmail.com

59
i

r
s
P ROBLÈMES C ORRIGÉS -MP ✍ M AMOUNI M Y I SMAIL
MAMOUNI . NEW. FR

i
s

s
u

s
Devoir Libre
8 Problème du point fixe.

u
Blague du jour Guillaume François Antoine de l’Hôpital (1661-1704)

é
☛ Une souris rencontre sa copine : J’ai décidé de me met- Marquis de Sainte-Mesme, comte d’Entremont, seigneur d’Ouc-

Mathématicien du jour
tre au régime, lui dit-elle. ques, de La Chaise, de Le Brau et d’autres lieux, mathématicien

Po r
r
- Tu ne manges plus ton fromage gruyère alors ? français. Il est plus connu pour la règle qui porte son nom. Il
- Si, mais je ne mange plus que les trous ! est aussi l’auteur du premier livre connu sur le calcul infinitési-

u
☛ Comment appelle t-on un chien sans pattes ? mal différentiel, publié en 1696, ses textes comportent des con-

uor
Réponse : On ne l’appelle pas, on va le chercher ! férences de son professeur Jean Bernoulli. C’est d’ailleurs ce
☛ Comment appelle-t-on une chauve-souris qui a des dernier qui a donné au marquis toutes les donnes nécessaires
cheveux ? pour réaliser le livre au nom de l’Hôpital, dans un le but de le
Réponse : Une souris. publier sous un nom français, car Bernoulli est Suisse.

P
Soit E = R d muni d’une norme k · k. On rappelle qu’une applica- ② Soit f une application de E dans E telle qu’il existe un entier
tion f de E dans E est dite contractante s’il existe K ∈]0, 1[ tel que n tel que f n soit contractante. On note x0 le point fixe de f n .

r
k f ( x ) − f (y)k ≤ kk x − yk ∀ x, y ∈ E.
a Montrer que tout point fixe de f est un point fixe de f n .

e
① On se propose de montrer que toute application contractante
admet un unique point fixe. Soit a ∈ R d , on pose x0 = a et
er
b Montrer que si x est un point fixe de f n , il en est de

m
x n + 1 = f ( x n ).
même pour f ( x ).
a Montrer que k xn − xn−1 k ≤ kn−1 k x1 − x0 k pour tout
n ≥ 1. c En déduire que x0 est l’unique point fixe de f .
m

b En déduire que ( xn ) est de Cauchy, puis qu’elle con-

r
③ Soit X et F deux parties d’un espace vectoriel normé, F étant
verge. une partie complète. On considère une application F : X ×

o
c Montrer que x = lim xn est un point fixe de f . E → E, (λ, x ) 7→ F(λ, x ) continue, et k-contractante en la sec-
or

F
✉: mamouni.myismail@gmail.com

60
i

r
s
✍ M AMOUNI M Y I SMAIL
MAMOUNI . NEW. FR
P ROBLÈMES C ORRIGÉS -MP

i
s

s
u
onde variable, c’est–dire qu’elle existe k ∈]0, 1[ tel que : b Ces résultats subsistent-ils si on suppose simplement E

s
∀λ ∈ X, ∀( x, y) ∈ E2 , k F(λ, x ) − F(λ, y)k ≤ kk x − yk. complet ?

é
a Montrer que, pour tout λ ∈ X, il existe un unique xλ ∈ E

u
⑤ R2 est muni d’une norme quelconque. Soit f : R2 → R2 telle
tel que F(λ, xλ ) = xλ .
1
que ∃α ∈]0; [, ∀( x, y) ∈ R2 , on a :

r
b Montrer ensuite que l’application X → E, λ 7→ xλ est 2

é
continue. k f ( x ) − f (y)k 6 α(k f ( x ) − x k + k f (y) − yk)

 1 a Montrer que f admet au plus un point fixe.
 x1 = (2 sin x1 + cos x2 )

Po r
r
c Montrer que le système 5
 x2 = 1 (cos x1 + 3 sin x2 )
 b On considère la suite définie par un+1 = f (un ) et

u
5
u0 ∈ R 2 .
admet une solution unique ( x1 , x2 ) ∈ R2 .

uor
④ Soit E une partie compacte d’un espace vectoriel normé, et i Montrer que ∀n ≥ 0,
α
f : E → E une fonction continue vérifiant : k un +2 − un +1 k ≤ k un +1 − un k .
∀( x, y) ∈ E2 , x 6= y =⇒ k f ( x ) − f (y)k < k x − yk. 1−α
ii Montrer que la suite u est de Cauchy.

P
a Montrer que f admet un unique point fixe (que l’on
notera α). iii Conclure.

e r
er
i
F

m
nn
i
m

r
Á la prochaine

o
or

F
✉: mamouni.myismail@gmail.com

61
i

r
s
P ROBLÈMES C ORRIGÉS -MP ✍ M AMOUNI M Y I SMAIL
MAMOUNI . NEW. FR

i
s

s
u

s
Devoir Surveillé
2 Topologie des racines carrés d’une matrice

u
r
Blague du jour Bernard Placidus Johann Nepomuk Bolzano (1781-1848)

Mathématicien du jour
☛ Il ne faut jamais traiter quelqu’un de compact, c’est Mathématicien, logicien, philosophe, théologien bohémien alle-

Po r
une insulte. Parce qu’un compact est un fermé borné ! mand. Il enseigna d’abord les sciences de la religion avant de

r
☛ L’injectivité implique la bijectivité mm̂e en dimension consacrer le reste de son temps aux mathématiques. Dans sa

u
infinie, en effet : Si f est injective tout élément possède au philosophie, Bolzano critique l’idéalisme d’Hegel et de Kant en
plus un antécédent ; mais qui peut le plus peut le moins, affirmant que les nombres, les idées, et les vérités existent in-

uor
donc tout élément possède au moins un antécédent, donc dépendamment des personnes qui les pensent. Ainsi l’acte men-
f est surjective puis bijective. tal se distingue de la signification de l’acte.

P
Enoncé : CCP 2005, MP de Mn (R ), c’est à dire des matrices A de Sn (R ) vérifiant : pour
toute matrice X ∈ Mn,1 (R ), t XAX ≥ 0.

Notations. Si x1 , . . . , xn sont des réels, on note diag( x1 , . . . , xn ) la matrice di-

r
agonale de Mn (R ) qui admet pour coefficients diagonaux les réels
Dans ce sujet, n est un entier naturel non nul et on note : Mn (R ) la
x1 , . . . , xn dans cet ordre.

e
R -algèbre des matrices carrées réelles de taille n.
Si p est un entier naturel non nul, on notera k.k∞ la norme infinie
er
Mn,1 (R ) le R -espace vectoriel des matrices à n lignes et une
sur R p :

m
colonne.
GLn (R ) le groupe des matrices inversibles de Mn (R ). si x = ( x1 , . . . , x p ), k x k∞ = Max | xi |.
1≤ i ≤ p
In la matrice unité de Mn (R ). p
Si a ∈ R et r > 0, on note B∞ (a, r ) la boule ouverte de centre a de
Id l’application identité de R n .
m

rayon r pour la norme k.k∞ .


Pour une matrice A ∈ Mn (R ), t A est sa matrice transposée.

r
Sn (R ) le sous-espace vectoriel des matrices symétriques de Mn (R ).
Objectifs.

o
Sn+ (R ) le sous-espace vectoriel des matrices symétriques positives
or

F
✉: mamouni.myismail@gmail.com

62
i

r
s
✍ M AMOUNI M Y I SMAIL
MAMOUNI . NEW. FR
P ROBLÈMES C ORRIGÉS -MP

i
s

s
u
Soit A une matrice de Mn (R ), on dit qu’une matrice R de Mn (R ) d Que peut-on dire de Rac( A) si A admet une valeur pro-

s
est une racine carrées de A si R2 = A. pre strictement négative ?
On note Rac( A) l’ensemble des racine carrées de A, c’est à dire

u
Rac( A) = { R ∈ Mn (R )/ R2 = A} e Si on suppose toutes les valeurs propres de A positives
Le problème propose de déterminer les racines carrées de A dans ou nulles, déterminer les racines carrées de la matrice D. On

r
différents exemples, (on pourra constater qu’une matrice peut par- pourra poser ε i ∈ {−1, +1} pour i ∈ {1, . . . , n}.

é
fois admettre une infinité de racines) et étudier quelques propriétés ③ Écrire toutes les racines carrées de A à l’aide de la matrice
topologiques de Rac( A). P. Combien de racines carrées A admet-elle ? (On discutera

Po r
r
Les trois parties du problème sont indépendantes. selon le signe des valeurs propres de A).
④ Application : Écrire toutes les racines carrées de A =

u
Les trois premiers exemples de la partie I sont tous indépendants.  
11 −5 5

uor
 −5 3 −3  à l’aide de la matrice P que l’on détermin-
5 −3 3
Partie I. Exemples de Rac( A ). era.
Exemple 2 : cas où A est la matrice nulle de Mn (R ).

P
Exemple 1 : cas où A possède n valeurs propres distinctes. Dans cet exemple, on cherche à déterminer les racines carrées
On suppose que la matrice A ∈ Mn (R )admet n valeurs propres de la matrice nulle.
réelles λ1 < λ2 < · · · < λn . Soit R ∈ Mn (R ), une matrice carrée de la matrice nulle.
① Justifier l’existence d’une matrice P ∈ Mn (R ) inversible telle

r
⑤ Soit f l’endomorphisme de R n dont R est la matrice dans la
que A = PDP−1 où D = diag(λ1 , λ2 , . . . , λn ), puis montrer base canonique de R n . On note r le rang de f .
que R est une racine carrée de A, si et seulement si la matrice

e
n
a Comparer Im( f ) et Ker ( f ) puis montrer que r ≤ .
S = P−1 RP est une racine carrée de D. 2
er
② Racines carrées de D. b On suppose f non nul, donc r ≥ 1. Soit (e1 , . . . , er ) une

m
Soit S une racine carrées de D. base de Im( f ) que l’on complète avec (er+1 , . . . , en−r ) pour
a Montrer que DS = SD. former une base de Ker ( f ). Pour i ∈ {1, . . . , r }, on note ui le
m

b En déduire que la matrice S est diagonale. vecteur tel que f (ui ) = ei .

r
Montrer que la famille B = (e1 , . . . , en−r , u1 , . . . , ur ) est une
c On note alors S = diag(s1 , . . . , sn ). Que vaut s2i lorsque base de R n puis écrire la matrice de f dans la base B . On

o
i ∈ {1, . . . , n} ? notera Mr cette matrice.
or

F
✉: mamouni.myismail@gmail.com

63
i

r
s
P ROBLÈMES C ORRIGÉS -MP ✍ M AMOUNI M Y I SMAIL
MAMOUNI . NEW. FR

i
s

s
u
⑥ . Si A est une matrice de Mn (R ) qui a pour coefficients (ai,j )1≤i,j≤n ,

s
a Déterminer les racines carrées dans Mn (R ) de la matrice on définit une norme en posant N ( A) = Max | ai,j |. On munit
1≤i,j≤ n
nulle.

u
Mn (R ) de cette norme N.
b Application : déterminer dans M4 (R ), les racines car-
rées de la matrice nulle. ① Fermeture de Rac( A).

r
Soit A une matrice de Mn (R ). Montrer que Rac( A) est une

é
Exemple 3 : cas où A = In . partie fermée de Mn (R ).
⑦ Soit R une racine carrée de l’unité In .

Po r
② Étude du caractère borné de Rac( In ).

r
a Vérifier que R est une matrice inversible.
a Un exemple instructif.

u
 
b Montrer que R est semblable à une matrice diagonale 1 0
Pour tout entier naturel q, on pose Sq = . Calculer

uor
que l’on décrira. q −1
2
⑧ Déterminer Rac( In ). On pourra poser ε i ∈ {+1, −1} pour Sq . Rac( I2 ) est-elle une partie bornée de M2 (R ) ?
i ∈ {1, . . . , n}.
b Rac( In ) est-elle une partie bornée de Mn (R ) pour n ≥
Exemple 4 : cas où A est une matrice symétrique réelle.

P
3?
Dans cet exemple, toutes les matrices que l’on considérera ap- c Application : pour cette question, n ≥ 2.
partiennent à Mn (R ). Montrer qu’il n’existe pas de norme k.k “surmultiplicative"
⑨ Une matrice symétrique admet-elle nécessairement une sur GLn (R ), c’est à dire vérifiant pour tous A et B dans

r
racine carrée ? GLn (R ), k ABk ≥ k Ak.k Bk.
⑩ Montrer qu’une matrice symétrique positive admet au moins

e
une racine carrée qui est elle même symétrique et positive.
er
Remarque : On peut montrer l’unicité de cette racine carrée

m
dans Sn+ (R ) mais ce ne sera pas utile pour la suite du prob- Partie III. Zéros de polynômes et intérieur de Ra
lème.

Soit p un entier naturel non nul. On munit R p de la norme infinie


m

r
k.k ∞ .
Partie II. Étude topologique de Rac( A ). On note Γ p l’ensemble des fonctions polynomiales sur R p c’est à

o
dire : si P ∈ Γ p , il existe N entier naturel et une famille de réels
or

F
✉: mamouni.myismail@gmail.com

64
i

r
s
✍ M AMOUNI M Y I SMAIL MAMOUNI . NEW. FR
P ROBLÈMES C ORRIGÉS -MP

i
s

s
u
{ai1 ,...,i p , 0 ≤ i1 , . . . , i p ≤ N } tels que 2x1 − x2 − 1 et Q( x1 , x2 ) = x12 − x2 . Représenter graphique-

s
∀( x1 , . . . , x p ) ∈ R p , P( x1 , . . . , x p ) = ∑ ai1 ,...,i p xi1 . . . xi p ment dans le plan R 2 les ensembles Z( P) et Z(Q). Z( P) et
Z(Q) sont-il infinis ?

é
0≤i1 ,...,i p ≤ N

u
Par exemple si p = 3, P( x1 , x2 , x3 ) = 5x12 + 3x1 x2 x3
+ 4x25 est une ③ Intérieur de l’ensemble des zéros d’une fonction polynomi-
fonction polynomiale sur R 3 . ale.

r
Si p = 1, Γ1 est l’ensemble des fonctions polynômes se R . Soit P ∈ Γ p .

é

Enfin, si p ∈ Γ p , on pose Z( P) = ( x1 , . . . , x p ) ∈ R p / P( x1 , . . . , x p ) = 0 a Soient I1 , . . . , I p des parties infinies de R . Montrer par
(Z( P) est l’ensemble des zéros de la fonction polynomiale P).

Po r
r
récurrence que si la fonction polynomiale P s’annule sur
L’objectif de cette partie est d’étudier l’intérieur de Z( P), afin de I1 × · · · × I p , alors P est la fonction nulle.

u
déterminer l’intérieur de Rac( A). b En déduire que si P s’annule sur une partie d’intérieur

uor
On rappelle que si Ω est une partie de R p , un vecteur a de R p est un non vide, P est la fonction nulle.
point intérieur à Ω s’il existe un nombre réel r strictement positif c Si l’on suppose que P n’est pas la fonction nulle, que
tel que B∞ (a, r ) ⊂ Ω et que l’intérieur d’une partie est l’ensemble vaut l’intérieur de Z( P) ?
de ses points intérieurs.

P
④ Application à l’étude de l’intérieur de Rac( A).
① Questions préliminaires : Dans cette question, on confondra les espaces vectoriels
2
a Soit a = (a1 , . . . , a p ) ∈ R p et r > 0. Montrer que B∞ (a, r ) Mn (R ) et R n . Par exemple, on prendra la liberté d’écrire que
peut s’écrire comme produit de p intervalles. 2
pour M ∈ Mn (R ), M = (mi,j )1≤i,j≤n ∈ R n , sans se soucier de

r
b Soient F et G deux parties de R p . On suppose que F et G l’ordre des termes.
Soit A une matrice de Mn (R ).

e
sont d’intérieur vide, montrer que F ∩ G est encore d’intérieur 2
vide. a Ecrire Rac( A) sous forme d’un ensemble de R n puis
er
montrer qu’il existe des éléments P1 , . . . , Pn2 de Γn2 tels que

m
② Exemples d’ensembles de zéros de fonctions polynomiales. 2
n
\
a Dans cette question, p = 1. Soit P une fonction polynôme Rac( A) = Z( Pl ).
sur R . Dans quel cas Z( P) est-il infini ? Justifier votre réponse. l =1
m

b Dans cette question, p = 2. On considère P( x1 , x2 ) = b Déterminer l’intérieur de Rac( A).

or
or

F
✉: mamouni.myismail@gmail.com

65
i

r
s
P ROBLÈMES C ORRIGÉS -MP ✍ M AMOUNI M Y I SMAIL
MAMOUNI . NEW. FR

i
s

s
u
Blague du jour Karl Theodor Wilhelm Weierstrass (1815-1897)

s
Mathématicien du jour
Comparaison entre Internet Explorer et la drogue : Mathématicien allemand, lauréat de la médaille Copley en 1895.

u
- La première dose est gratuite, mais quand vous serez Il est souvent cité comme le « père de l’analyse moderne ». Ses
accrocs ils augmenteront les prix. travaux les plus connus portent sur les fonctions elliptiques.

r
- Microsoft, comme les dealers, savent que, faute d’autre C’est lui qui le premier rendit public un exemple de fonction

é
came, vous reviendrez. continue nulle part dérivable. Weierstrass étudia la fiabilité de l’-
- Les deux vous explosent le système de temps en temps. analyse, dont il propose une construction logique rigoureuse. À

Po r
r
- Bill, comme les dealers, voudrait bien que tu revendes cette époque, les démonstrations de l’analyse s’appuyaient sur
ses produits aux autres. des définitions ambiguës

u
uor
multipliant par P à gauche et P−1 à droite) c’est à dire S2 = D.
Corrigé, Pr. Devulder, CPGE France On peut donc écrire
Rac( A) = P.Rac( D ).P−1

P
Partie I. Exemples de Rac( A ).
② .
① Les sous espaces propres Eλi ( A) sont de dimension ≥ 1 et en a On a SD = S3 = DS.

r
somme directe. Leur somme a donc une dimension au moins b On fait le produit matriciel pour obtenir
égale à n. Comme elle est incluse dans R n , sa dimension est n n

e
en réalité égale à n et chaque Eλi ( A) a une dimension égale à ∀i, j, Si,j λ j = ∑ Si,k Dk,j = ∑ Di,k Sk,j = λi Si,j
1. Notons ( f i ) une base de Eλi ( A). La famille ( f 1 , . . . , f n ) est
er
k=1 k=1
une base de R n . Si P est la matrice de la base canonique de Les λk étant deux à deux distincts, on a donc

m
R n aux f i alors P−1 AP est la matrice dans la base ( f i ) de l’en- ∀i 6= j, Si,j = 0
domorphisme canoniquement associé à A. Par choix des f i , et S est diagonale.
cette matrice est diag(λ1 , . . . , λn ) et on a donc c On a alors S2 = diag(s21 , . . . , s2n ). Comme S2 = D, on a
m

A = PDP−1 avec D = diag(λ1 , . . . , λn )

r
donc
Soit R ∈ Mn (R ) et S = P−1 RP. On a R2 = A si et seulement ∀i, s2i = λi

o
si P−1 R2 P = D (il y a équivalence car on revient en arrière en
or

F
✉: mamouni.myismail@gmail.com

66
i

r
s
✍ M AMOUNI M Y I SMAIL
MAMOUNI . NEW. FR
P ROBLÈMES C ORRIGÉS -MP

i
s

s
u
d Si il existe un i tel que λi < 0, les relations précédentes On a alors P−1 AP = diag(0, 1, 16). A admet quatre racines

s
carrées qui sont
sont impossible et donc
P.diag(0, 1, 4).P−1 , P.diag(0, −1, 4).P−1 , P.diag(0, 1, −4).P−1 , P.diag(0

é
Rac( A) = ∅

u
ou
 encore    
3 −1 1 7/3 −5/3 5/3 −7/3 5/3 −5
qst e Si tous les λi sont positifs, on vient
p de voir que

r
p  −1 1 −1  ,  −5/3 1/3 −1/3  ,  5/3 −1/3 1/

é
Rac( D ) ⊂ {diag(ε 1 λ1 , . . . , p
ε n λn )/ ∀i,pε i = ±1} 1 −1 1 5/3 −1/3 1/3 −5/3 1/3 −1
Réciproquement, si S = diag(ε 1 λ1 , . . . , ε n λn ) (où ε i = On remarque bien sûr que les matrices sont deux à deux op-

Po r
±1) alors S2 = D. L’inclusion ci-dessus est une égalité.

r
posées.

③ L’application M 7→ P−1 MP est une bijection de Rac( A) dans

u
⑤ .
Rac( D ). a R2 = 0 se traduit par f ◦ f = 0 et donc par

uor
a Si λ1 < 0, on a vu en 2.d que Rac( A) = ∅. Il n’y a donc Im( f ) ⊂ Ker ( f )
pas de racine carrée pour A. Or, le théorème du rang indique que r + dim(Ker ( f )) = n.
Comme dim(Ker ( f )) ≥ r, on a donc
b Si λ1 ≥ 0 alors une racine carrée de D est connue par le n

P
r≤
choix des ε i et p p 2
Rac( A) = { P.diag(ε 1 λ1 , . . . , ε n λn ).P−1 / ∀i, ε i = ±1}
b La famille B ayant n éléments, il suffit de montrer
Deux choix différents des ε i donneront deux racines carrées
distinctes de D sauf dans le cas où λ1 = 0. On a donc qu’elle est libre ou génératrice pour conclure que c’est une

r
Card( Rac( A)) = 2n−1 si λ1 = 0 base de R n . Supposons donc que
n −r r
Card( Rac( A)) = 2n si λ1 > 0

e
(∗) : ∑ α i e i + ∑ β i ui = 0
i =1 i =1
er
④ (0, 1, 1) est vecteur propre associé à la valeur propre 0. Avec les notations de l’énoncé, ceci s’écrit

m
(1, 1, −1) est vecteur propre associé à la valeur propre 1. Avec r n −r r
la trace, on voit que la dernière valeur propre est 16. Une ré- ∑ α i f ( ui ) + ∑ e i + ∑ β i ui = 0
solution de système montre que (2, −1, 1) est vecteur propre i =1 i =r +1 i =1
En composant par f , on obtient (avec f 2 = 0 et f (ei ) = 0 si
m

associé. On pose donc  


i ∈ {r + 1, . . . , n − r })

r
0 1 2
r r
P =  1 1 −1 
∑ β i ei = ∑ β i f ( ui ) = 0

o
1 −1 1 i =1 i =1
or

F
✉: mamouni.myismail@gmail.com

67
i

r
s
P ROBLÈMES C ORRIGÉS -MP ✍ M AMOUNI M Y I SMAIL
MAMOUNI . NEW. FR

i
s

s
u
Comme (e1 , . . . , er ) est libre, les β i sont nuls. En reportant ⑧ Ce qui précède
n montre que o

s
dans (∗) et comme (e1 , . . . , en−r ) est libre, les αi sont aussi nul. Rac( In ) ⊂ P.diag(ε 1 , . . . , ε n ).P−1 / P ∈ GLn (R ), ∀i, ε i ∈ {−1, +1}
Ainsi, B est libre et c’est une base de R n .

é
Réciproquement D = diag(ε 1 , . . . , ε n ) verifie D2 = In quand

u
Par choix des vecteurs de B , on a (définition
 par blocs)
0 Ir les ε k valent 1 ou −1 et ( PDP−1 )2 = PD2 P−1 = In . L’inclu-

r
Mr = Mat( f , B) = sion précédente est donc une égalité.
0 0

é
⑨ diag(−1, −2, . . . , −n) est une matrice symétrique réelle qui,
⑥ . d’après l’exemple 1, n’admet pas de racine carrée.
a

Po r
Si R ∈ Rac( A) alors soit R = 0 soit il existe une matrice

r
inversible P et un entier r ∈ [1..n/2] telle que R = PMr P−1 .

u
Réciproquement, la matrice nulle est une racine carrée de 0 et Partie II. Étude topologique de Rac( A ).

uor
si r ≤ n/2, un produit par blocs montre que Mr2 = 0 et donc
( PMr P−1 )2 = PMr2 P−1 = 0. Ainsi,
① Soit A ∈ Sn+ (R ). Le théorème spectrale donne l’existence
Rac(0) = { PMr P−1 / P ∈ GLn (R ), r ∈ [1, n/2]} ∪ {0}
d’une matrice orthogonale P et d’une matrice diagonale D
telles que P−1 AP = D. Les coefficients diagonaux di de D

P
b Dans le cas n = 4, les racines carrées de 0 sont 0 et les
sont valeurs propres pour A. Si Xi est vecteur propre associé
matrices semblables
 à l’une
 des deux
 matrices  alors
0 0 0 1 0 0 1 0
 0 0 0 0   0 0 0 1  0 ≤ t Xi AXi = t Xi (di Xi ) = di k Xi k2
  
 0 0 0 0  ou  0 0 0 0 
 où k.k est le norme euclidienne. Comme k Xi k2 > 0 (Xi est

r
non nul puisque c’est un vecteur propre), on a di ≥ 0. On
0 0 0 0 0 0 0 0
peut alors poser

e
p p
⑦ . R = P.diag( d1 , . . . , dn ).P−1 = P∆P−1
er
a R2 = In donne det( R)2 = 1 et donc det( R) 6= 0. R est P étant orthogonale on a P−1 = t P et R est symétrique. Par

m
donc inversible. ailleurs, R2 = PDP−1 = A et R est racine carrée de A. Enfin,
b X 2 − 1 est un polynôme qui annule R. Comme il est R est positive :
n p
m

scindé à racines simples, R est diagonalisable. En outre, les ∀ X t XRX = t XP∆t PX = ∑ di y2i avec Y = t PX
valeurs propres de R sont racines de X 2 − 1 et ne peuvent

r
i =1
valoir que 1 ou −1. Ainsi, R est semblable à une matrice diag- et cette quantité est bien positive. On a finalement montré que

o
onale où les coefficients diagonaux valent 1 ou −1. Rac( A) ∩ Sn+ (R ) 6= ∅
or

F
✉: mamouni.myismail@gmail.com

68
i

r
s
✍ M AMOUNI M Y I SMAIL
MAMOUNI . NEW. FR
P ROBLÈMES C ORRIGÉS -MP

i
s

s
u
Mn (R ) étant de dimension finie, toutes les normes y sont
Partie III. Zéros de polynômes et intérieur de

s
équivalentes. On pourra choisir la norme N de l’énoncé ou
toute autre norme. Cela ne change rien du point de vue

u
topologique. ① .
② Par théorèmes généraux, R 7→ R2 est continue sur Mn (R ) a On a

é
p
(chaque fonction coordonnée l’est comme fonction polynomi-
ale des coefficients de R). Ainsi, si ( Rk ) est une suite conver- B∞ (a, r ) = ∏ ] ai − r, ai + r[
i =1
gente d’éléments de limite R alors R2k → R2 .

Po r
r
Ainsi, si ( Rk ) est une suite convergente d’éléments de Rac( A), b Soit a ∈ F ∩ G. Si, par l’absurde, il existait r > 0 tel

u
la limite est dans Rac( A). Ce dernier ensemble est donc que B∞ (a, r ) ⊂ F ∩ G alors on aurait a fortiori B∞ (a, r ) ⊂ F
fermé. et donc a serait intérieur à F ce qui est exclus (et donne une

uor
③ . contradiction). F ∩ G n’a donc pas de point dintérieur.
a On a S2q = I2 . Comme N (Sq ) = Max(|q|, 1) → +∞ Remarque : pour arriver à cette conclusion, il suffit que F OU G
soit d’intrieur vide.
quand q → +∞, Rac( I2 ) n’est pas borné.  

P
Sq 0 ② .
b Définissons par blocs la matrice Mq = . a Le seul polynôme admettant une infinité de racines est
0 In−q
On a alors Mq2 = In (calcul par blocs) et N ( Mq ) → +∞ quand le polynôme nul. Pour le voir, on peut, par exemple, prouver
q → +∞. Ainsi, Rac( In ) n’est pas borné pour n ≥ 3. par récurrence q’un polynôme non nul de degré n admet au

r
plus n racines.
c On vient de voir que l’on peut trouver une suite ( Rk ) de
i Si P est constant non nul il n’admet pas de racine.

e
racines carrées de In telles que ( Rk ) n’est pas bornée. Si, par
l’absurde, il existait une norme surmultiplicative k.k alors on ii Supposons le résultat vrai jusqu’au rang n. Soit P de
er
aurait degré n + 1. Entre deux racines de P, il y a une racine de

m
∀k, k Rk k2 ≤ k R2k k = k I2 k P′ (théorème de Rolle). Comme P′ admet au plus n racines
Le membre de droite est constant et celui de gauche de lim- (deg( P′ ) = deg( P) − 1 = n), P en admet au plus n + 1.
ite infinie (voir remarque préliminaire en début de partie). On peut aussi prouver le résultat (et on n’utilise alors plus la
m

On obtient une contradiction ce qui prouve la non existence structure ordonnée de R) en montrant que si P(a) = 0 alors

r
d’une norme surmultiplicative. (X − a) divise P et en raisonnant par degré.

o
b Dans le plan (0, x1 , x2 ), 2x1 − x2 = 1 est l’équation d’une
or

F
✉: mamouni.myismail@gmail.com

69
i

r
s
P ROBLÈMES C ORRIGÉS -MP ✍ M AMOUNI M Y I SMAIL
MAMOUNI . NEW. FR

i
s

s
u
droite. Z( P) est donc infini. b D’après la question 13.a, toute partie d’intérieur non

s
x12 − x2 = 0 est l’équation d’une parabole et Z(Q) est aussi
vide contient une sous-partie ∏ Ik où chaque Ik est infini (in-
infini.

u
tervalle de longueur 2r > 0). Si P s’annule sur une partie d’in-
térieur non vide, P est alrs nul avec la question précédente.
③ .

r
c En contraposant le résultat de la question précédente, si
a Le résultat pour p = 1 a été justifié en question 14.a.

é
P 6= 0 alors Z( P) est d’intérieur vide.
Supposons le résultat vrai jusqu’à un rang p ≥ 1. Soient alors
P une fonction polynomiale qui s’annule sur I1 × · · · × I p+1 ④ .

Po r
r
où chaque Ik est une partie infinie de R. En ordonnant les a R2 est une matrice dont le coefficient générique est

u
puissances de x p+1 , on peut écrire n
N ∑ Ri,k Rk,j

uor
P ( x1 , . . . , x p +1 ) = ∑ Pi ( x1 , . . . , x p ) xip+1 k=1
i =0
Considérons alors !
n
où chaque Pi est dans Γ p .
Fixons x1 , . . . , x p avec xi ∈ Ii et considérons l’expression
Qi,j = ∑ xi,k xk,j − ai,j ∈ Γn2
k=1

P
précédente comme fonction de x p+1 . C’est une fonction poly- Par définition de Rac( A), on a \
nomiale qui s’annule en une infinité de points. D’après l’ini- Rac( A) = Z(Qi,j )
tialisation, c’est le polynôme nul. On a donc 1≤i,j≤ n
∀i ∈ {1, .., N }, ∀( x1 , . . . , x p ) ∈ I1 × · · · × I p , Pi ( x1 , . . . , x p ) = 0 2
ce qui fait apparaître Rac( A) comme sous-ensemble de R n .

r
L’hypothèse de récurrence donne la nullité des Pi et donc celle
de P. On a ainsi prouvé le résultat au rang p + 1 et complété b Comme intersection de parties d’intérieur vide, Rac( A)

e
la récurrence. est d’intérieur vide avec la question ①.
er
i
F

m
n
i
n
m

r
Á la prochaine

o
or

F
✉: mamouni.myismail@gmail.com

70
SESSION 2003

✍ M AMOUNI M Y I SMAIL MAMOUNI . NEW. FR

Devoir libre Epreuve spécifique - filière PC

10 matrices positives et décomposition de Cholesky


Blague du jour MATHEMATIQUE 1

• Quel sont les deux animaux les plus intelligents ? - Le Cerf et le Veau
(cerveau)
• C’est un chien qui rencontre un crocodile :
Durée : 4heures
- Le crocodile dit au chien : Salut, sac à puces !
- Et le chien lui répond : Salut, sac à main !

Mathématicien du jour
John Wallis (1616-1703)

Mathématicien anglais. Ses travaux sont précurseurs de ceux de Newton. Il est également
précurseur de la phonétique, de l’éducation des sourds et de l’orthophonie. Il a été l’un des
fondateurs de la Royal Society. Ses travaux concernent principalement le calcul différentiel
et intégral. On lui doit le symbole de l’infini ∞.

Énoncé : CCP 2003, PSI


Notations
 
      
   
     
Soit et des entiers supérieurs ou égaux à 1. On note le -espace vectoriel des

 
            
matrices à coefficients dans ayant lignes et colonnes. Lorsque , est noté plus
simplement et est muni de sa structure d’algèbre, représentant la matrice identité.
désigne l’ensemble des matrices inversibles de et l’ensemble des ma-
trices symétriques de

  .
               
de la         
Tout vecteur
ème
de est identifié à un élément

de
ligne de soit . Dans toute la suite, nous noterons indifféremment
tel que l’élément
un        
                            
élément de aussi bien que le vecteur de qui lui est associé.
Pour dans et dans , on note le coefficient de
la  ligne de   .
Selon le contexte,  désigne soit le réel nul, soit la matrice nulle de     , soit encore la matrice
ème

nulle de       .
  est muni de son produit# scalaire canonique noté   !  " et de la norme $ associée notée ! !  ! ! .
Une matrice symétrique de     est $ dite positive si et seulement si : %      & '  # ( 
et définie positive si et seulement si : %       ) *  + & '  # "

ÉJ« AÖÞ @ ø BñÓ úG ñÜØ ✉: mamouni.myismail@gmail.com

80
✍ M AMOUNI M Y I SMAIL
MAMOUNI . NEW. FR

On note , .- / 0 1
l’ensemble des matrices symétriques réelles positives et , .- - / 0 1 l’ensemble des
matrices symétriques réelles définies positives.

Partie I
/2 2 4=3 4 < 1 5 4 /2 6 . 7 8 / 0 1 1 9 : 5 , . / 0 1
I.1 Soit
a) ; / ; 2 4 1 9 ; <; 2 / 4 ; 4 1 2<; 4 / 2 ; 2 1 4
.
et . Etablir les égalités :

b)
c) ; 2 : 4< >2? : 4=@ < > : 2? 4@.
.

A // : 8 8 33 : 9 1 1 5 5 / , . . - / /0 0 1 1 C 1 9 3 .: 8 / B 0 1 : 39 5 8 , B . - / 0 1 5 ./ 0 1
I.2 Démontrer les propriétés suivantes :

AA D : 5 :6 9 . / 0 , 1 - 3 ; D D 5 , , - .-- / 0 1 : : 9 , - -
a) .
b) .
c)
: 5 , . /0 1 .
A E
2 
5 6 . 7 8 / 0 1 3 ; 2 : 2F<=G
:
I.3 a) Soit vérifiant :
: <G
. Montrer que toute valeur
propre de est nulle et en déduire .
b) Donner un exemple de matrice carrée d’ordre 3, non nulle et vérifiant : H
A 25 6I 7 8 / 0 1 3 ; 2 H 2<G
I.4 a) Soit : 5 , . / 0 1 . Montrer que : appartient à , . - / 0 1 si et seulement si toutes ses valeurs
propres sont positives.
b) Que peut-on dire d’une matrice symétrique réelle semblable à une matrice symétrique
réelle positive ?
I.5 On munit , . /0 1 J @
des relations notées et , définies respectivement par :

A / : 8 3 : 9 1 5 / , . / 0 1 1 9 3 / : 8 J: 9 K L : 8 M : 9 5 , . - / 0 1 1
A / : 8 3 : 9 1 5 / , . / 0 1 1 9 3 / : 8 @ : 9 K L : 8 M : 9 5 , .- - / 0 1 1
et

a) Montrer que la relation J est une relation d’ordre sur , . / 0 1 .


b) Montrer que pour N JO , cet ordre n’est pas total sur , . / 0 1 .
c) La relation @ est-elle une relation d’ordre ?
d) Trouver un exemple dans , 9 / 0 1 montrant que : 8 J: 9 et : 8 < P : 9 n’implique pas
nécessairement : 8 @ : 9 .
.
I.6 Soit Q et R deux endomorphismes de 0 diagonalisables et vérifiant Q S R < R S Q .
a) Démontrer que tout sous-espace propre de Q est stable par R .
b) Soit T 8 3 T 9 3 U U U 3 T V les valeurs propres distinctes de Q et W X Y 3 W X Z 3 U U U 3 W X [ les sous-
espaces propres de Q respectivement associés. Pour tout \ 5] ^ 3 O 3 U U U 3 _ ` , on note R a l’endomor-
phisme de W X b induit par R . Montrer que pour tout \ 5] ^ 3 O 3 U U U 3 _ ` il existe une base c a de W X b
formée de vecteurs propres de R . En déduire qu’il existe une base c de 0
. telle que les matrices de
Q et R dans cette base soient toutes deux diagonales.

✉: mamouni.myismail@gmail.com ÉJ« AÖÞ @ ø BñÓ úG ñÜØ


81
✍ M AMOUNI M Y I SMAIL
MAMOUNI . NEW. FR

d e
I.7 a) Soit et deux matrices diagonalisables de fg h i j
. Montrer que les matrices et d e
d e
commutent si et seulement si elles sont diagonalisables au moyen d’une même matrice de passage.

lmonp E
n q n v
t s u o
m l p
w E
n q n t s
b) On donne les matrices et suivantes :

d k q np nq nE nrq nn e k qq wr yrxF


wrq wn
Montrer que d et e sont diagonalisables au moyen d’une même matrice de passage et déterminer

I.8 Soit h z { | z } j ~ h  g € h i j j } tel que z { z } k z } z { . Montrer que z { z } ~  g € h i j .


explicitement une telle matrice de passage.

I.9 a) Soit h z { | z } j ~ h  g h i j j } tel que z { z } k z } z { . Montrer que :

z }  z { ‚ k ƒz } }  z {}
nE nn ‚
b) Montrer que les matrices z { ko„ E
n et z } ko„ }† vérifient z }  z { ‚ .
Vérifient-elles z } }  z }{ ?
E
‚ ‡
Partie II
On se propose dans cette partie de caractériser de diverses manières la définie positivité d’une

z ~  g hi j
matrice symétrique réelle.

z
II.1 Soit . Montrer que les quatre propositions suivantes sont équivalentes :
a) est définie positive.
z z k‹ ˆ ˆ

ˆ ~ ‰ Š g h i j
b) Toutes les valeurs propres de sont strictement positives.

zd g e g
c) Il existe telle que .
d) est positive et inversible.
 g hi j
lŒŒŒ ‚
n ‚Ž    ‚ s 
II.2 Soit et les matrices de données par :

ŒŒŒ n
‚n .n .. . . . .... 
e g k ŒŒm ‚ ... Œ . . .. 
| d g k w ‘g q e g
.. ..
n ‚ t 
n
.. .. ..
. . .
. ‚n
.. .. ..
‚F    ‚ ‚
. .

a) Montrer que pour tout vecteur ’k h “ ” j { • ” • g de i g :

} g˜ — { q } }
‹ ’ d g ’k“ { – ” ™ { h “ ” “ ” € { j – “ g

ÉJ« AÖÞ @ ø BñÓ úG ñÜØ ✉: mamouni.myismail@gmail.com

82
✍ M AMOUNI M Y I SMAIL MAMOUNI . NEW. FR

š› œ›
b) En déduire que est définie positive.

žŸŸŸ ® ¯¯¯
c) En cherchant une matrice de la forme :

ŸŸŸ ¡ ¢F£ ¢¥¤r¦ . ¦. .¦§¤ ... ¯¯¯


œ ›  ¤¨ .. . .
¡ 
© £
..
© ..

› ª › ¤ ª› °± ¡ ² ± £ ² ³ ´
. . . .
.. . . ..
. . .

¡ 
¢ £ ¢
¤«¦ œ ¦ ¦› inversible ¦ ¦ ¦¬¤­ telle¡ que š › µ œ › œ › .
II.3 Soit ¶ 
› ¹ œ › ¹ ¶  µ œ œ . On note ½  ¹ ¾ ¾ ¾ ›
déterminer explicitement une matrice

famille des vecteurs colonnes de œ . Pour ¿


et telles que
et Å
› , on note Æ ¹ ± Å la± projection ±
la

³
orthogonale de Å sur Vect ¹ ¾ · ¸ ¸ ´ º ¾ ³ » ¼ ¾ ´ º › ±  ± ± Ã Ä ¢
² º © ¦ ¦ ¦ º
a) Justifier que ½ est ± une± base± de ³ . À Á ¦¦¦ ³´
.

b) On définit la famille de vecteurs Ç  ¹ È È ¢ © ¦ ¦ ¦ ² º È › par les relations :


´ ¢ ± © ± È ¦ ¦ ¦ ± ¾ º Æ ª ¹ ¾
È  ¾ et É ¿
Montrer que la famille¢ Ç est¢ orthogonale ³ À  et± ¦ ¦que¦ ± à c’est Ä ± une ² base² Ê de² ¢ › . ² º
c) Soit Ë  ¹ Ì Ì Ì › la famille de vecteurs définie par Ì
´  Í Í È Í Í È pour tout
¿la base Ë à la base. Ë ½ estest ¢alors ± © ± une¦ ¦ ¦ base ± ºorthonormale de › . Montrer que la matrice ² Á de passage
²
³ À Á ± Â d)± ¦ ¦ Soit
¦ ± Ã Ä Î la matrice ´ de › à la la base Ë .² Montrer que
de
triangulaire supérieure.

œqu’alors
peut s’écrire sous la forme œ  Î Ï où Ï est une matrice triangulaire supérieure inversible et
de passage de la base canonique


¶ µ Ï Ï . ž ® ´
e) Montrer que la matrice ¶    ° admet une décomposition de la forme
r
 Ê ʤ
¶ µ Ï Ï où Ï est une matrice triangulaire supérieure Ê Â inversible et en déduire que ¶ est symétrique
définie positive. Ö Ê r
¤ Ñ
II.4 a) Soit š Ò Ó ¹ . Déterminer × ¹ tel que µ × š Ò ×  .
Ö
¤Ó Û ÔÕ ³ · © ´ º ¹ . Montrer que³ Øš © estÙ ¢ ´ définie º Ú À ¤ Ä positive si et seulement ¤ si
b) Soit š oÔ
¹ Tr š Ü c) Soit et det šÜ Ô ³. On· © décompose ´ encore º à ¹ Û ¶ Ü sous etla forme Û ©Ü .
¶ › Ô¹ , Õ équivaut
ce qui

¤ ¤ º Ã 
Ý Â Ö ¤ Õ Ê Ô ¤ º
³ ¶ ·  ´ Ó º Û µ¶ È Þ Û È › ª ¹ ¶ Þ › ª ¹
È ± ³ ´ ± ³ Ø ¢ Ù¢ ´ º ± ³ · ¢ ´ º

✉: mamouni.myismail@gmail.com ÉJ« AÖÞ @ ø BñÓ úG ñÜØ


83
✍ M AMOUNI M Y I SMAIL MAMOUNI . NEW. FR

ßà áâ ã ä å æ ç sous la forme è ß é ê ëì é à æ ì ß ê à áâ í ä ã ä å æ ç , montrer que pour


î ðï ñ :
En écrivant
ò ò÷ ò ÷ ò÷
ß ó ß ð î ô è é õîö ß ê ë ø õî ö ß ê å î ó ê ù ç ÷ ß ò ê÷ ú åç
ø î û ñ î ö
et en déduire que ó est définie positive si et seulement si å et ó ê ù est définie positive ç .

récurrence une suite de nombres réels å î÷ ü ç ä ý ü ÷ ý â et une suite de matrices å ÷ ó ü ò ç ÷ ä ý ü ý â comme suit. On
d) En gardant les notations de la question II.4 c) précédente, on peut alors construire par

ó ä ð ó ì î ä ð î ì ä ð ì ó êä ð ó ê ì ó ð î ä ó äê ù ä ä
pose d’abord :

Si þ ÿ , on décompose ó ò ÷ sous la forme ø


î ÷ ÷
ó è ø ó ê ëì ÷ î ò ÷ à æ ì à áâ í ã ä å æ ç ì ó ê à  â í å æ ç
ð
ø øðø î ø ø
On pose à nouveau ó  øó ê ù ü etü on ø itèreü le processus ø précédent. ø ø On obtient ainsi une
suite de matrices symétriques réelles å ó ç ä ý ý â où ó est d’ordre þ ù
îå ü ç ä ý ü ý â liés par les relations :ø ø ø ø ò ÷ õ ö ÷ ò÷
et une suites de réels

à  ü ð î ÷ü ü ü ð îü ü ü ü
ìö  ì ì þ ù ö ì ó è ü ó ü ê ëì ó ä ó ê ù
Le processus s’arrête pour  ð þ car ó â est alors d’ordre et on note ó â ð å î â ç .
Montrer que ó est définie positive si et seulement si tous ö les réels de la suite å î ü ç ä ý ü ý â sont
strictement positifs.
î  

e) Soit ó ð  
 à   å æ ç . Selon les notations précédentes, déterminer explici-
tement les réels î ä î î  associés à cette matrice ó et en déduire que ó est définie positive si et
ì ì
seulement si :
ø îûñ  î û ñ et  î 
   û ñ

ì  
  i    
F
nn

i
ÁFin de l’énoncé
la prochaine

ÉJ« AÖÞ @ ø BñÓ úG ñÜØ ✉: mamouni.myismail@gmail.com

84
i

r
s
P ROBLÈMES C ORRIGÉS -MP ✍ M AMOUNI M Y I SMAIL
MAMOUNI . NEW. FR

i
s

s
u
.

s
Blague du jour Hermann Minkowski (1864 - 1909)

Mathématicien du jour
u
Les types d’ingénieurs en informatique sont : Mathématicien et un physicien théoricien allemand. On lui
• L’ingénieur DISQUE DUR : il se rappelle tout, POUR doit surtout l’espace à 4 dimension (espace-temps) appelé de

r
TOUJOURS. Minkowski, considéré comme la base de tous les travaux sur la

é
• L’ingénieur CD-ROM : il va toujours plus vite avec le théorie de la relativité. Son travail le plus original est sans aucun
temps. doute sa géométrie des nombres. Ces travaux posent de nom-

Po r
r
• L’ingénieur RAM : il oublie tout de vous, dès le mo- breuses questions sur le gain de place, ou comment faire rentrer
ment où vous lui tournez le dos. une forme donnée à l’intérieur d’une autre forme donnée.

u
.

uor
Corrigé, Pr. J. DEBARBIEUX, Lycée Faidherbe, n’est
Lillepas un formule
t
classique on refait le calcul et
X (SY ) = ∑ xi si,j y j = h X, SY i
(i,j)

P
puis comme S est symétrique t X (SY ) =t X t SY =t (SX )Y =
hSX, Y i
PARTIE 1 2a) On a : ∀ X ∈ Mn,1 (R ) ,t XS1 X ≥ 0 et t XS2 X ≥ 0 et donc en
ajoutant t X (S1 + S2 ) X ≥ 0

r
1a) Si X = ( xi )in=1 et Y = (yi )in=1 sont des éléments de Mn,1 (R ) on 2
(S1 , S2 ) ∈ Sn+ (R ) ⇒ S1 + S2 ∈ Sn+ (R )
a

e
n 2b) idem car la somme d’un réel positif et d’un réel strictement
t t
XY = YX = ∑ xi yi positif est un réel strictement positif.
er

i =1 2c) On a : ∀ X ∈ Mn,1 (R ) , t X t AA X =t ( AX )( AX ) =

m
1b) Développement sans problème :
2 h AX, AX i = k AX k2 ≥ 0 . Et donc
t
XY = (t XY )(t XY ) = (t XY )(t YX ) calcul précédent ∀ A ∈ Mn (R ) , t AA ∈ Sn+ (R )

3a) Si SX = λX on a t XSX = λt XX = λ k X k2 . Donc si t XSX = 0
m

= (t X ) Y t Y (X ) par associtivité

r
on a λ = 0 (car X est non nul donc k X k 6= 0 )
= (t YX )(t XY ) =t Y (X t X )Y symétriquement
S est donc une matrice diagonalisable (car symétrique réelle) ayant

o
1c) Si on considère que ” t XY = h x, yi dans une base orthonormée” une unique valeur propre 0 . S est donc la matrice nulle : S =
or

F
✉: mamouni.myismail@gmail.com

88
i

r
s
✍ M AMOUNI M Y I SMAIL
MAMOUNI . NEW. FR
P ROBLÈMES C ORRIGÉS -MP

i
s

s
u
P.0.P−1 = (0) 0 donc c’est la matrice nulle. . S2 = S1

s
3b) On veut que MX soit orthogonal à X pour tout X . c’est une ☛ transitive : Si S1 ≥ S2 et S2 ≥ S3 on a S1 − S2 ∈ Sn+ (R ) et
propriété classique du produit vectoriel . Il suffit de prendre pour S2 − S3 ∈ Sn+ (R ) donc d’après 2a la somme S1 − S3 ∈ Sn+ (R ) et

u
M la matrice de x − > i ∧ x   donc S1 ≥ S3 .
0 0 0 5b) il suffit de prendre S1 = 0 et pour S2 une matrice symétrique

r
M =  0 0 −1  ayant une valeur propre positive et unenégative . Exemple

é

0 1 0 0 0 0
t  0 1 0 
On vérifie alors bien XMX = 0

Po r
r
4a) S étant symétrique réelle est diagonalisable dans une base or- 0 0 −1
thonormée (Vi )in=1 : il existe D = diag(λk ) telle que ∀k , SVk = λk Vk / Sn++ (R )
5c) la relation > n’est pas réflexive car (0)n ∈

u
Si toutes les valeurs propres sont positives on a alors pour toute 5d) On peut se douter (ou montrer) qu’une matrice de Sn++ (R ) a

uor
n des valeurs propres strictement positives.
matrice colonne X = ∑ yk Vk On prend donc S2 = 0 et S1 symétrique ayant des valeurs pro-
k=1* +
n n n pres
 positives  et ayant la valeur propre 0 . Par exemple  =
S1 
t
XSX = h X, SX i = ∑ yk Vk , ∑ λk yk Vk = ∑ λk y2k ≥ 0 0 0 0 x 6= 0

P
k=1 k=1 k=1  0 0 0  On a S1 6= (0) t XS1 X = z2 ≥ 0 et si X =  0 
Réciproquement si λ est valeur propre de S et X un vecteur propre 0 0 1 0
asocié.le calcul du 3a donne t XSX = λ k X k 2 . comme on suppose t
XS1 X = 0


t
XSX ≥ 0 et que X 6= 0 on a bien λ ≥ 0 6a) question de cours .On doit montrer x ∈ Eλ (u) ⇒ v( x ) ∈ Eλ (u)

r
4b) deux matrices semblables ont même spectre . Donc si S′ est . Donc u( x ) = λx ⇒ u(v( x )) = λv( x ). Or
symétrique réelle semblable à S symétrique positive les valeurs u(v( x )) = (u ◦ v)( x )

e
propres de S (donc de S′ ) sont toutes positives donc S′ est posi- = (v ◦ u) ( x ) par hypothèse sur u et v
er
tive.

m
= v(u( x )) = v(λ( x ))
5a) Sur Sn (R ) la relation ≥ est bien : = λv( x ) par linéarité de v
☛ binaire 6b) L’endomorphisme induit par v diagonalisable sur un sous es-
☛ réflexive : (0)n est bien positive donc S1 ≥ S1
m

pace stable est lui même diagonalisable. Donc l’endomorphisme vi

r
☛ antisymétrique : si S1 ≥ S2 et si S2 ≥ S1 les valeurs propres de est diagonalisable et il existe une base de Eλi (u) qui est une base de
S2 − S1 sont toutes à la fois positives et négatives. S2 − S1 est donc vecteurs propres de vi . u étant diagonalisable E est somme directe

o
diagonalisable ( car symétrique réel) ayant une seul valeur propre des sous espaces propres .L’union des bases précédente est donc
or

F
✉: mamouni.myismail@gmail.com

89
i

r
s
P ROBLÈMES C ORRIGÉS -MP ✍ M AMOUNI M Y I SMAIL
MAMOUNI . NEW. FR

i
s

s
u
une base de E . Par construction ces vecteurs sont des vecteurs pro- sage est :  

s
pres de v et de u (car éléments des sous espaces propres) . Dans 1 1 0
cette base u et v sont donc simultanément diagonalisables. P= 1 1 1 

u
−1 2 1
7a) Si A et B commutent A et B sont diagonalisables au moyen remarque : je ne pense pas que le passage par l’endomorphisme induit par

r
d’une même matrice de passage . On prend la question précédente v sur E0 ( A) soit plus simple

é
avec A = MatC (u) et B = MatC (v) . P est alors la matrice de 8) S1 et S2 sont diagonalisables (symétriques réels) , et commutent
passage de C à B . . S1 et S2 sont donc diagonalisables avec une même matrice de pas-

Po r
r
Réciproquement si A et B son diagonalisables au moyen d’une sage (S1 = PDP−1 , S2 = P∆P−1 ). Cette matrice de passage diag-
onalise aussi S1 S2 = S2 S1 = PD∆P−1 , la matrice diagonale sem-

u
même matrice de passage . On a A = PDP−1 , B = P∆P−1
et comme deux matrices diagonales commutent AB = BA = blable à S1 S2 étant le produit des deux matrices semblables à S1

uor
et S2 . S1 et S2 étant positives ont toutes leurs valeurs propres posi-
P ( D∆) P−1
tives. Les valeurs propres de S1 S2 sont donc aussi toutes positives
7b) et S1 S2 est symétrique positive.(toujours 4a)
9a) Avec les notations précédentes (S1 = PDP−1 , S2 = P∆P−1 ). On

P
A est de rang 1 et E0 ( A) est le plan ( x + y − z = 0) . Par la trace donc ∆ − D positives . Donc pour les termes diagonaux δi − di ≥ 0
on en déduit que
 la troisième
 valeur propre est 3 puis on trouve et di ≥ 0 . La fonction carrée est croissante sur R + donc ∀i , δi2 ≥ d2i
1 . ∆2 − D2 est donc positive et S22 − S12 est une matrice symétrique
E3 ( A) = Vect  1  semblable à une matrice symétrique positive donc est aussi posi-

r
−1 tive. S22 ≥ S12 ( cf 4b) 
1/2 −1

e
Pour B le calcul du polynôme caractéristique en commençant par 9b) On a S2 − S1 = de valeurs propres 0 et 5/2 réels
−1 2
er
exemple par faire C2 + C3 − > C3 donne deux valeurs propres
positifs. La mtrice est positive est S2 ≥ S1

m
4(double) et 1(simple) . Puis le calcul des sous espaces propres   S1 de valeurs  propres 0 et1 donc S1 ≥ 0
1
1/4 −2
donne : E4 ( B) est le plan −2x + y − z = 0 et E1 ( B) = Vect  1  et S22 − S12 = de déterminant −9/4 . Le produit des
−2 7
2
m

valeurs est négatif. L’une des valeurs propres es négatives.S22 − S12

r
. On vérifie alors que E1 ( B) ⊂ E0 ( A) , E3 ( A) ⊂ E4 ( B) . Les trois
droites E1 ( B), E3 ( A) , E0 ( A) ∩E4 ( B) sont trois droites de vecteurs n’est pas positive.
Partie II

o
propres communs qui engendrent l’espace . Une matrice de pas-
or

F
✉: mamouni.myismail@gmail.com

90
i

r
s
✍ M AMOUNI M Y I SMAIL
MAMOUNI . NEW. FR
P ROBLÈMES C ORRIGÉS -MP

i
s

s
u
n
1) Si X = ( xi )in=1 et Y = AX = y j j=1 on a :

s

a ⇔ b : idem I4a  y1 = 2x1 − x2
S étant symétrique réelle est diagonalisable dans une base or-

é
∀ j ∈ [[2, n − 1]] , y j = − xi −1 + 2xi − xi +1

u
thonormée (Vi )in=1 : il existe D = diag(λk ) telle que ∀k , SVk = λk Vk 
yn = − xn−1 + 2xn
Si toutes les valeurs propres sont strictement positives on a alors

r
n On a donc

é
n n n n −1
pour toute matrice colonne non nulle X = ∑ yk Vk
* k=1+
t
XAX = ∑ xi yi = 2 ∑ xi2 − ∑ xi−1 xi − ∑ xi xi+1
i =1 i =1 i =2
! i =1

Po r
r
n n n !
t
XSX = h X, SX i = ∑ yk Vk , ∑ λk yk Vk = ∑ λk y2k >0 n n −1 n −1 n −1
= ∑ xi2 + x12 + ∑ xi2 + x2n − ∑ x j x j +1 − ∑ xi xi +1

u
k=1 k=1 k=1
En effet on a une somme de termes positifs , un au moins étant i =2
!
i =1
!
i =1 i =1

uor
strictement positif. n −1 n −1 n −1
Réciproquement si λ est valeur propre de S et X vecteur propre = ∑ x2j+1 + x12 + ∑ xi2 + x2n −2 ∑ xi xi +1
j =1 i =1 i =1
asocié on a t XSX = λ k X k 2 . comme on suppose t XSX > 0 et que

→ n −1  
X 6= 0 on a bien λ > 0 x12 x2n ∑ xi2+1 − 2xi xi +1 + xi2

P
= + +
b ⇒ c . S étant diagonalisable dans une base orthonormée i =1
(symétrique réelle) on peut écrire S = PD t P . avec D = diag(di ) n −1
. Par hypothèses p les di sont strictement positifs . On peut donc = x12 + x2n + ∑ ( x i − x i + 1 )2
i =1
définir ∆ = diag( di ) qui est inversible car les termes diagonaux

r
sont non nuls. M = ∆t P est alors une solution du problème. 2b pour toute colonne X on constate que t XAX est une somme

e
t
MM = P∆∆t P = PD t P = S de carré donc est un réel positif. De plus la somme est nulle
c ⇒ d si S =t MM avec M inversible , S est inversible (comme si
 et seulement si chaque terme est nul donc si et seulement si
er
produit de matrices inversibles) et S est positive d’après I2c  x1 = 0

m
d ⇒ b : S est positive donc toutes les valeurs propres de S sont ∀i ∈ [[1..n−]] , xi +1 − xi = 0

positives et S est inversible donc 0 n’est pas valeur propre de S . xn = 0
Les valeurs propres de S sont donc strictement positives.
m

Tous les xi sont donc nuls . Donc si X 6= (0) t XAX est strictement
On a la chaînes b ⇒ c ⇒ d ⇒ b et a ⇔ b donc l’équivalence des 4

r
positif.
propositions. 

o
2a) A est bien une matrice symétrique. 2c) Avec la matrice M du sujet notons S =t MM = si,j on a en
or

F
✉: mamouni.myismail@gmail.com

91
i

r
s
P ROBLÈMES C ORRIGÉS -MP ✍ M AMOUNI M Y I SMAIL
MAMOUNI . NEW. FR

i
s

s
u
faisant le produit
 : ☛ (V1 , V2 ) est une famille orthogonale de vecteurs non nuls et

s

 s1 = u21 Vect(V1 , V2 ) = Vect(U1 , U2 ). En effet

i > 1 ⇒ si = u2i + v2i −1 ☛ p1 est la projection orthogonale sur Vect(U1 ) = Vect(V1 ) donc

u

 1≤i ≤ n − 1 ⇒ si,i +1 = si +1,i = ui v i V2 = U2 − p1 (U2 ) est orthogonal à V1

| j − i | > 1 ⇒ si,j = 0 ☛ si V2 était nul , on aurait U2 = p1 (U2 ) ∈ Vect(U1 ) . Absurde car

r
On doit donc résoudre le système non linéaire (U1 , U2 ) est libre

é

 u21 = 2 ☛ (V1 , V2 ) est une famille orthogonale de vecteurs non nuls , c’est
i > 1 ⇒ u2i + v2i −1 = 2 donc une famille libre.

Po r
r
 ☛ Enfin Vect(V1 , V2 ) ⊂ Vect(U1 , U2 ) par construction,et comme les
1 ≤ i ≤ n − 1 ⇒ ui v i = − 1
1 1 deux familles de deux vecteurs sont libres il y a égalité.

u
On a donc vi = − et en reportant u2i = 2 − 2 . Soit en posant
ui ui − 1 ☛ On suppose que (Vi )ik=−11 est une famille orthogonale de vecteurs

uor
2
ai = ui la suite homographique : non nuls tels que Vect(Vi )ik=−11 = Vect(Ui )ik=−11 . Montrons que

 a1 = 2 (Vi )ik=1 est une famille orthogonale de vecteurs non nuls tels que
1 Vect(Vi )ik=1 = Vect(Ui )ik=1 .
 ai = 2 −

P
ai − 1 ☛ par hypothèse de récurrence on doit seulement montrer que Vk
l’équation l = 2 − 1/l donne un point fixe double l = 1 . La suite est un vecteur non nul orthogonal à Vect(Vi )ik=−11 puis Vect(Vi )ik=1 =
1
est donc arithmétique . Or Vect(Ui )ik=1 .
ai − 1
1 1 ai − 1 1 ☛ Par construction pk−1 est la projection orthogonale sur

r
= = = 1+ Vect(Vi )ik=−11 = Vect(Ui )ik=−11 donc Vk = Uk − pk−1 (Uk ) est orthog-
ai − 1 1− a1 ai − 1 − 1 ai − 1 − 1
i −1 onal à Vect(Vi )ik=−11

e
1
d’où = i et ☛ Si Vk est nul alors Uk = pk−1 (Uk ) ∈ Vect(Ui )ik=−11 et la famille
er
ai − 1 r r (Ui )in=1 est lié . Absurde

m
i+1 i
ui = , vi = − ☛ Enfin par construction Vk ∈ Vec(Uk ) ⊕ Vect(Ui )ik=−11 =
i i+1
3a) U est une base car S est une matrice inversible d’après II1c Vect (Ui )ik=1 et Vect(Vi )ik=−11 = Vect(Ui )ik=−11 ⊂ Vect(Ui )ik=1 . Donc
3b) C’est la méthode d’orthogonalisation de Schmidt .Démonstra- Vect(Vi )ik=1 ⊂ Vect(Ui )ik=1 . Les deux familles étant libres de même
m

cardinal , les deux sous espaces sont égaux.

r
tion par récurrence
☛ (V1 ) est réduit à un seul vecteur non nul donc est une famille pour k = n on obtient que V est une base orthogonale de R n .

o
orthogonale de vecteurs non nuls 3c) La base est orthonormale car on norme une base orthonormée (
or

F
✉: mamouni.myismail@gmail.com

92
i

r
s
✍ M AMOUNI M Y I SMAIL
MAMOUNI . NEW. FR
P ROBLÈMES C ORRIGÉS -MP

i
s

s
u
et les dénominateurs sont non nuls car les Vi sont des vecteurs non by + 2cx = 0

s
nuls) 4b)
Par construction de V on a vu que Vk ∈ Vect(Ui )ik=1 . Les coordon- ☛ si A est définie positive les valeurs propres de A sont strictement

u
nées de Vk sur Uk+1 , · · · Un sont donc nuls . positives (cf II 1). Leur somme (la trace) et leur produit (le détermi-
MatU (V ) est triangulaire supérieure. Diviser chaque colonne par nant) le sont aussi. On a donc a + b > 0 et ab − c2 > 0. On en déduit

r
sa norme ne change pas les coefficients nuls . MatU (W ) est trian- que a + b et ab sont strictement positifs donc a et b le sont.

é
gulaire supérieure. 2 c2
☛ Si a > 0 et ab − c > 0 on a b > > 0 donc Tr ( A) > 0
3d) notons B la base canonique . On a

Po r
a

r
M = MatB (U ) = MatB (W ) MatW (U ) = PT et det( A) > 0 . La somme et le produit des valeurs propres sont
strictement positifs donc les valeurs propres sont strictement posi-

u
où T est l’inverse de la matrice triangulaire supérieure construite à
la question précédente. tives . D’après II 1 A est définie positive.

uor
On a alors S =t MM =t T t PPT . Mais P est la matrice de passage 4c) calcul par
 bloct :     
 a V x  x
de B à W toutes deux bases orthonormées . Donc P est orthogonale x Xt ′
= t ′ t
xa + X V x V + X S t ′ ′
et t PP = In . Il reste donc S =t TT V S′ X′ X′

= xax + x t X ′ V ′ + x t VX ′ +t X ′ S′ S′

P
a b c
3e) Si on pose à priori T =  0 d e  le calcul donne : = ax2 + x (t VX ′ +t X ′ V ) +t X ′ S′ X ′
0 0 f
 2    = ax2 + 2x t VX ′ +t X ′ S′ X ′ car t VX ′ =t X ′
a ab ac 4 −2 −2  t VX ′ 2
t
TT =  ab b2 + d2 bc + de  =  0 2 0  1 t 2

r
= a x+ − VX ′ +t X ′ S′ X ′
2 2 2 −2 0 3 a a
ac bc + de c + e + f
 

e
en résolvant le système ligne par ligne et en choisissant pour a, d, f t VX ′ 2 1 t ′ t 
= a x+ − X V VX ′ +t X ′ S
er
les racines carrées positiveson obtient  a a
"

m
2 −1 −1 
t VX ′ 2 1t 
T =  0 1 −1  = a x+ + 2 X ′ −V t V + aS′
0 0 1 a a
On vérifie que tous les produits matriciels ont un sens les matrices
m

On constate que T est inversible et donc d’après II 1 S est définie


étant de tailles compatibles.

r
positive.
 
x On en déduit donc :
4a) Si X = on a t XA0 X = by2 + 2cxy donc y = 0 ou

o
y ☛ si a > 0 et aS′ − V t V définie positive , pour toute ma-
or

F
✉: mamouni.myismail@gmail.com

93
i

r
s
P ROBLÈMES C ORRIGÉS -MP ✍ M AMOUNI M Y I SMAIL
MAMOUNI . NEW. FR

i
s

s
u
 t VX ′ 2 matrice Sn définie positive.

s
trice colonne X ∈ Mn,1 (R ) on a x + ≥ 0 et
 a ☛ Réciproquement si les ( ai ) sont tous strictement positifs Sn =
t ′
X aS′ − V t V X ′ ≥ 0 donc t XSX ≥ 0 . De plus si t XSX = 0 (an ) est définie positive . Sn est définie positives et an−1 > 0 donc

u
on a une somme nulle de réelles positives
 ′ donc chaque ′terme est Sn−1 est définie positive et par récurrence si Si −1 est définie posi-
t ′ ′ t
nulle . En particulier X aS − V V X = 0 et donc X = 0 car tive Si est définie positive car ai −1 > 0 .

r
 
aS′ − V t V est définie positive on trouve alors x = 0 en reportant    

é
a d e
 t VX ′ 2   d ′ b f
4e) Si S = d b f on a a1 = a, V1 = , S1 =
dans x + = 0 . Donc X 6= 0 ⇒t XSX > 0 et S est définie e f c

Po r
e f c

r
a
positive. d’où  
  ab − d2 a f − de

u
1 S2 =
☛ Si S est définie positive alors a > 0 car pour X = , a f − de ac − e2
(0 )

uor
t
XSX = a d’après le calcul précédent (avant la division par a) S est donc définie positive si et seulement si a > 0 et S2 définie pos-
et aS′ − V t V est définie positive car pour toute matrice non nul itive . Donc en utilisant II 4b si et seulement si a > 0, ab − d2 > 0
X ′ ∈ Mn−1,1 (R ) et det (S2 ) > 0 or det (S2 ) = (ab − d2 )(ac − e2 ) − (a f − de)2 =
 
 ′ 0 a det(S)

P
t ′ ′ t 2t
X aS − V V X = a XSX > 0 en prenant X =
X′ a d
S est définie positive si seulement si a> 0 , > 0 ,
4d) d b
☛ Si S est définie positive la question précédente donne par une a d e
récurrence évidente que toutes les Si sont définies positives et tous d b f >0

r
les ai positifs pour i < n . Enfin an > 0 comme valeur propre de la e f c

e
i
er
F

m
n
ni
m

r
Á la prochaine

o
or

F
✉: mamouni.myismail@gmail.com

94
✍ M AMOUNI M Y I SMAIL
MAMOUNI . NEW. FR

Devoir Surveillé
3 Espaces d’Artin et applications
Blague du jour

Salon de l’auto : Comment reconnaître les nationalités des visiteurs du Mon-


dial de l’Automobile ?
- L’Allemand examine le moteur
- L’Anglais examine le cuir
- Le Grec examine l’échappement
- L’Italien examine le Klaxon

Jacques Salomon Hadamard (1865-1963)

Mathématicien du jour
Mathématicien français, connu pour ses travaux en théorie des nombres et en cryptologie.
Il entra premier à l’école normale supérieure. C’est Émile Picard qui dirigea ses travaux
de recherches.
Son
 nom  est lié à la 
suite de matrices( H2k ) définie de la façon suivante : H1 = (1), H2 =
1 1 H2k−1 H2k−1
et H2k = . Elles sont utilisées dans les codes correcteurs, ou
1 −1 H2k−1 − H2k−1
encore pour réaliser les plans d’analyse sensorielle et les plans d’expériences factoriels.

Énoncé : Extrait Centrale 2010, MP


Les calculatrices sont autorisées
Notations :
• IK désigne le corps IR ou le corps C
I .
• On fixe un IK -espace vectoriel E de dimension n ≥ 1 .

Partie I -
2
I.A - On fixe une application ϕ de E dans IK . On suppose que ϕ est une forme
3
bilinéaire symétrique sur E , c’est-à-dire que, pour tout ( x, y, z ) ∈ E et pour tout
α ∈ IK , ϕ( x + αy, z) = ϕ( x, z) + αϕ( y, z) et ϕ( x, z) = ϕ( z, x) .
I.A.1) Pour tout élément x de E , on note h(x) l’application de E dans E telle
que ∀ y ∈ E , h ( x ) ( y ) = ϕ ( x, y ) .
a) Montrer que, pour tout x de E , h(x) est élément du dual de E , noté E* .
b) Montrer que h est une application linéaire de E dans E* .
⊥ϕ
I.A.2) Si A est une partie de E , on note A = { x ∈ E ⁄ ∀a ∈ A ϕ(x, a) = 0 } .
⊥ϕ
Montrer que A est un sous-espace vectoriel de E .
⊥ ⊥ϕ
Par la suite, lorsqu’il n’y aura pas d’ambiguïté, on notera A au lieu de A .
⊥ϕ
I.A.3) On dit que ϕ est non dégénérée si et seulement si E = { 0 } .
Montrer que ϕ est non dégénérée si et seulement si h est un isomorphisme.

✉: mamouni.myismail@gmail.com ÉJ« AÖÞ @ ø BñÓ úG ñÜØ


✍ M AMOUNI M Y I SMAIL MAMOUNI . NEW. FR

. I.A.4) Soit e = ( e 1, …, e n ) une base de E .


On note e∗ = ( e∗1 , …, e∗n ) la base duale de e .
a) Montrer que la matrice de h dans les bases e et e∗ est :
mat ( h, e, e∗ ) = ( ϕ(e i, e j) ) 1 ≤ i ≤ n
1≤ j≤n

Cette dernière matrice sera également appelée la matrice de ϕ dans la base e


et notée mat(ϕ, e)
2
b) Soit ( x, y ) ∈ E . On note X et Y les matrices colonnes dont les coefficients
sont les composantes de x et y dans la base e .
t t
Montrer que ϕ(x, y) = XΩY où Ω = mat(ϕ, e) et où X désigne la matrice ligne
obtenue en transposant X .
I.B - Si ϕ est une forme bilinéaire symétrique sur E , on note q ϕ l’application de
E dans IK définie par : ∀ x ∈ E , q ϕ( x) = ϕ( x, x) . On dit que q ϕ est la forme qua-
dratique associée à ϕ . On note Q ( E ) l’ensemble des q ϕ où ϕ est une forme bili-
néaire symétrique sur E .
I.B.1) Soit q ∈ Q ( E ) .
Montrer qu’il existe une unique forme bilinéaire symétrique sur E , notée ϕ ,
telle que q = q ϕ . On dira que ϕ est la forme bilinéaire symétrique associée à la
forme quadratique q . On dira que q est non dégénérée si et seulement si ϕ est
non dégénérée. Si e est une base de E , on notera mat ( q, e ) = mat ( ϕ, e ) . On
l’appellera la matrice de q dans la base e .
I.B.2) Soit q une forme quadratique sur E . Soit E′ un second IK -espace vec-
toriel de dimension n , et soit q′ une forme quadratique sur E′ .
On appelle isométrie de ( E, q ) dans ( E′, q′ ) tout isomorphisme f de E dans E′
vérifiant : pour tout x ∈ E , q′ ( f (x) ) = q ( x ) . On dira que ( E, q ) et ( E′, q′ ) sont iso-
métriques si et seulement si il existe une isométrie de ( E, q ) dans ( E′, q′ ) .
Montrer que ( E, q ) et ( E′, q′ ) sont isométriques si et seulement si il existe une
base e de E et une base e′ de E′ telles que mat ( q, e ) = mat ( q′, e′ ) .
*˙ 2p
I.B.3) Soit p ∈ IN . Notons c = ( c 1, …, c 2 p ) la base canonique de IK .
2p p

∑ , on pose q p(x) = 2 ∑ x i x i + p .
2p
Pour tout x = x i c i ∈ IK
i=1 i=1
2p
a) Montrer que q p est une forme quadratique sur IK et calculer mat(q p, c) .
b) On appelle espace de Artin (ou espace artinien) de dimension 2 p tout couple
( F , q ) , où F est un IK -espace vectoriel de dimension 2 p , et où q est une forme
2p
quadratique sur F telle que ( F, q ) et ( IK , q p ) sont isométriques.
Montrer que dans ce cas, q est non dégénérée.
Lorsque p = 1 , on dit que ( F, q ) est un plan artinien.
c) On suppose que IK = C I et pour tout
2p 2p

∑ ∑
2p 2
x = xk ck ∈ C
I , on pose q(x) = xk
k=1 k=1
2p
Montrer que ( C
I , q ) est un espace de Artin.
d) On suppose que IK = IR et pour tout
2p p 2p

∑ xi ci ∈ IR ∑ xi – ∑
2p 2 2
x = , : on pose q′(x) = xi .
i=1 i=1 i = p+1
2p
Montrer que ( IR , q′ ) est un espace de Artin.
e) Si ( F, q ) est un espace de Artin de dimension 2 p , montrer qu’il existe un
sous-espace vectoriel G de F de dimension p tel que la restriction de q à G est
identiquement nulle.

ÉJ« AÖÞ @ ø BñÓ úG ñÜØ ✉: mamouni.myismail@gmail.com


✍ M AMOUNI M Y I SMAIL
MAMOUNI . NEW. FR

Partie II -
.
Pour toute la suite de ce problème, on suppose que ϕ est une forme bilinéaire
symétrique non dégénérée sur E , et on note q sa forme quadratique.
II.A -
II.A.1) Soit e = ( e 1, …, e n ) une base de E . On note encore e∗ = ( e∗1 , …, e∗n ) la
base duale de e . Soit p ∈ { 1, …, n } . On note F l’espace engendré par e 1, …, e p .

a) Montrer que F est l’image réciproque par h de Vect ( e∗p + 1 , …, e∗n ) , où h est
définie au I.A.1.

b) Montrer que dim ( F ) + dim ( F ) = n .
⊥ ⊥
c) Montrer que ( F ) = F .
II.A.2) Soient F et G deux sous-espaces vectoriels de E .
⊥ ⊥ ⊥
a) Montrer que ( F + G ) = F ∩ G .
⊥ ⊥ ⊥
b) Montrer que ( F ∩ G ) = F + G .
II.A.3) Soit F un sous-espace vectoriel de E . On note ϕ F la restriction de ϕ à
2
F . On dira que F est singulier si et seulement si ϕ F est dégénérée.
Montrer que F est non singulier si et seulement si l’une des propriétés suivan-
tes est vérifiée :

• F ∩ F = {0} ;

• E = F⊕F ;

• F est non singulier.
II.A.4) On dit que deux sous-espaces vectoriels F et G de E sont orthogonaux
si et seulement si pour tout (x, y) ∈ F × G , ϕ ( x, y ) = 0 .
Si F et G sont deux sous-espaces vectoriels de E orthogonaux et non singuliers,
montrer que F ⊕ G est non singulier.
II.B - Soit q′ une seconde forme quadratique sur E dont la forme bilinéaire
2
symétrique associée est notée ϕ′ . Comme au I.A.1, on note, pour tout (x, y) ∈ E ,
h ( x ) ( y ) = ϕ ( x, y) et h ′( x ) ( y ) = ϕ′ ( x, y) .
Soit e = ( e 1, …, e n ) une base de E . On dit que e est q -orthogonale si et seule-
2
ment si, pour tout (i, j) ∈ { 1, …, n } , avec i ≠ j , ϕ ( e i, e j ) = 0 .
2 2 2 2
II.B.1) On suppose que E = IR et pour tout (x, y) ∈ IR , q (x, y) = x – y et
q′ ( x, y) = 2xy .
Déterminer une base q -orthogonale et une base q′ -orthogonale.
2
II.B.2) Existe-t-il une base de IR orthogonale pour q et pour q′ définies à la
question II.B.1 ?
II.B.3) Supposons que e est à la fois q -orthogonale et q′ -orthogonale.
–1
Montrer que, pour tout i ∈ { 1, …, n } , e i est un vecteur propre de h o h ′ .
–1
II.B.4) On suppose que h o h ′ admet n valeurs propres distinctes.
Montrer qu’il existe une base de E orthogonale à la fois pour q et pour q′ .
II.C -
II.C.1) Soit x ∈ E tel que q ( x ) = 0 et tel que x ≠ 0 .
On se propose de démontrer qu’il existe un plan Π ⊂ E contenant x et tel que
(Π,q /Π) soit un plan artinien (où q /Π désigne la restriction de l’application q au
plan Π ).
a) Démontrer qu’il existe z ∈ E tel que ϕ ( x, z ) = 1
q( z)
b) On pose y = z – ----------- x . Calculer q ( y ) .
2
c) Conclure.

✉: mamouni.myismail@gmail.com ÉJ« AÖÞ @ ø BñÓ úG ñÜØ


✍ M AMOUNI M Y I SMAIL MAMOUNI . NEW. FR

. II.C.2) Soit F un sous-espace vectoriel singulier de E . On suppose que


⊥ ⊥
( e 1, …, e s ) est une base de F ∩ F . On note G un supplémentaire de F ∩ F dans
F.
a) Montrer que G est non singulier.

b) Démontrer par récurrence sur la dimension de F ∩ F (en commençant par
⊥ ⊥
dim ( F ∩ F ) = 1 , puis dim ( F ∩ F ) > 1 ) qu’il existe s plans P 1, …, P s de E tels
que les trois propriétés suivantes soient vérifiées :
1) Pour tout i ∈ { 1, …, s } , ( P i, q ⁄ Pi ) est un plan artinien contenant e i
2
2) Pour tout (i, j) ∈ { 1, …, s } avec i ≠ j , P i est orthogonal à P j .
3) Pour tout i ∈ { 1, …, s } , P i est orthogonal à G .
II.C.3) Montrer que F = G ⊕ P 1 ⊕ … ⊕ P s est non singulier.
On dira que F est un complété non singulier de F .
n
II.C.4) Montrer que si q ⁄ F = 0 , alors dim ( F ) ≤ --- .
2
II.C.5) On suppose que n = 2 p . Montrer que (E,q) est un espace de Artin si et
seulement si il existe un sous-espace vectoriel F de E de dimension p tel que
q⁄F = 0.
Partie III -

On note O ( E, q ) l’ensemble des isométries de ( E, q ) dans lui-même, c’est-à-dire


l’ensemble des automorphismes f de E vérifiant :
pour tout x ∈ E , q ( f ( x ) ) = q ( x ) .
III.A -
III.A.1) Soit f un endomorphisme de E .
2
a) Montrer que f ∈ O ( E, q ) si et seulement si, pour tout (x, y) ∈ E :
ϕ ( f ( x ), f ( y ) ) = ϕ ( x, y ) .
Montrer que si F est un sous-espace vectoriel de E et si f ∈ O ( E, q ) , alors
⊥ ⊥
f ( F ) = ( f ( F )) .
b) Soit e une base de E . Calculer la matrice de la forme bilinéaire :
( x, y) a ϕ ( f ( x ), f ( y ) ) en fonction de mat ( f , e ) et de mat ( ϕ, e ) .
c) Posons M = mat ( f , e ) et Ω = mat ( ϕ, e ) .
t
Montrer que f ∈ O ( E, q ) si et seulement si Ω = MΩM .
d) Montrer que si f ∈ O ( E, q ) , alors det ( f ) ∈ {1,– 1} . On notera :
+ –
O ( E, q ) = { f ∈ O ( E, q ) ⁄ det ( f ) = 1 } et O ( E, q ) = { f ∈ O ( E, q ) ⁄ det ( f ) = – 1 } .
III.A.2) Soient F et G deux sous-espaces vectoriels de E tels que E = F ⊕ G .
On note s la symétrie par rapport à F parallèlement à G .
a) Montrer que s ∈ O ( E, q ) si et seulement si F et G sont orthogonaux (pour ϕ ).
b) En déduire que les symétries de O ( E, q ) sont les symétries par rapport à F

parallèlement à F , où F est un sous-espace non singulier de E .
c) Lorsque H est un hyperplan non singulier, on appellera réflexion selon H la

symétrie par rapport à H parallèlement à H . Montrer que toute réflexion de

E est un élément de O ( E, q ) .
2
d) Soit (x, y) ∈ E tel que q ( x ) = q ( y ) et q ( x – y ) ≠ 0 .

On note s la réflexion selon H = { x – y } . Montrer que s ( x ) = y .
III.B.3) Soit f ∈ O ( E, q ) . On suppose que pour tout x ∈ E tel que q ( x ) ≠ 0 , on a
f ( x ) – x ≠ 0 et q ( f ( x ) – x ) = 0 .
+
On se propose de démontrer que f ∈ O ( E, q ) et que E est un espace de Artin.
a) Montrer que dim ( E ) ≥ 3 .
b) On note V = Ker ( f – Id E ) . Montrer que q ⁄ V = 0 .

c) Soit x ∈ E tel que q ( x ) = 0 . Notons H = { x } . Montrer que q ⁄ H n’est pas
identiquement nulle.
En déduire qu’il existe y ∈ E tel que q ( x + y ) = q ( x – y ) = q ( y ) ≠ 0 .
ÉJ« AÖÞ @ ø BñÓ úG ñÜØ ✉: mamouni.myismail@gmail.com
✍ M AMOUNI M Y I SMAIL MAMOUNI . NEW. FR

d) On note U = Im ( f – Id E ) . Montrer que q ⁄ U = 0 .


. e) Montrer que U = V = U .

+
f) En déduire que E est un espace de Artin et que f ∈ O ( E, q ) .

Partie IV -

IV.A - On souhaite démontrer le théorème de Cartan-Dieudonné, dont voici


l’énoncé : « si f ∈ O ( E, q ) , f est la composée d’au plus n réflexions, où
n = dim ( E ) , en convenant que Id E est la composée de 0 réflexion.»
IV.A.1) Montrer le théorème de Cartan-Dieudonné lorsque n = 1 . On veut
ensuite raisonner par récurrence. On suppose donc que n > 1 et que le théorème
de Cartan-Dieudonné est démontré en remplaçant E par tout espace vectoriel
de dimension n – 1 .
IV.A.2) Conclure lorsqu’il existe x ∈ E tel que f ( x ) = x avec q ( x ) ≠ 0 .
IV.A.3) Conclure lorsqu’il existe x ∈ E tel que q ( x ) ≠ 0 et q ( f ( x ) – x ) ≠ 0 .
IV.A.4) Conclure dans les autres cas.
IV.B - On se propose de démontrer le théorème de Witt, dont voici l’énoncé :
« soient F et F′ deux sous-espaces vectoriels de E tels qu’il existe une isométrie
f de ( F , q ⁄ F ) dans ( F′, q ⁄ F ′ ) (la définition d’une isométrie a été donnée au I.B.2).
Alors il existe g ∈ O ( E, q ) telle que g ⁄ F = f .»
IV.B.1) Montrer qu’on peut se ramener au cas où F et F′ sont non singuliers.
IV.B.2) On suppose que F et F′ sont non singuliers, avec
dim ( F ) = dim ( F′ ) = 1 . Soit x ∈ F avec x ≠ 0 . Posons y = f ( x ) .
a) Montrer que q ( x + y ) ou q ( x – y ) est non nul.
b) Montrer le théorème de Witt dans ce cas, en utilisant la question III.A.2-d).
IV.B.3) On suppose maintenant que F et F′ sont non singuliers, avec
dim ( F ) = dim ( F′ ) > 1 .
a) Montrer qu’il existe F 1 et F 2 non singuliers, tels que F 1 ⊥F 2 et F = F 1 ⊕ F 2 ,
avec dim ( F 1 ) = dim ( F ) – 1 .
b) Supposons qu’il existe g ∈ O ( E, q ) telle que g ⁄ F = f ⁄ F . Notons
⊥ ⊥ 1 1
F′1 = f ( F 1 ) . Montrer que f ( F 2 ) ⊂ F′1 et que g ( F 2 ) ⊂ F′1 .
c) Montrer qu’il existe
⊥ –1
h ∈ O ( F′1 , q ⁄ ⊥) telle que h ⁄ g ( F ) = ( f o g ) ⁄ g ( F ) .
F′1 2 2

d) Montrer qu’il existe k ∈ O ( E, q ) telle que k ⁄ F = f .


IV.B.4) Démontrer le théorème de Witt.

i
F
nn

i
Á la prochaine

✉: mamouni.myismail@gmail.com ÉJ« AÖÞ @ ø BñÓ úG ñÜØ


i

r
s
P ROBLÈMES C ORRIGÉS -MP ✍ M AMOUNI M Y I SMAIL
MAMOUNI . NEW. FR

i
s

s
u
Blague du jour George Boole (1815-1864)

s
3 amis sont au restaurant. Après leur repas, l’addition Logicien, mathématicien et philosophe britannique. Il est le créa-

u
s’élève a 30 dhs. Ils donnent chacun 10 dhs. Le serveur teur de la logique moderne, que l’on appelle algèbre de Boole en

Mathématicien du jour
s’aperçoit qu’il s ’est trompé dans les comptes et qu’en son honneur. Autodidacte, il publia ses premiers travaux tout

r
fait ils ne doivent payer que 25 dhs. en exerçant son métier d’instituteur et de directeur d’école pri-

é
Comme 5 dhs n’est pas divisible en 3, il décide de garder maire. En effet, issu d’une famille pauvre, George Boole n’a
2 dhs de pourboire et de leur rendre le reste. Ainsi les pas les moyens financiers d’aller l’université, il fût obligé de

Po r
r
amis se partagent les 3 dhs et ils ont donc payé le repas 3 travailler pour soutenir sa famille, il devient enseignant à 16
fois 9dhs = 27 dhs plus les 2 dhs du serveur, cela donne ans. Quatre ans plus tard, il fonde et dirige sa propre école. Ses

u
29 dhs ! travaux lui valurent la Royal Medal de la Royal Society.

uor
Où est passé le dernier dh ? Il épouse Mary Everest, nièce de Sir George Everest, le responsable
de la mission cartographique qui baptisa le mont Everest.

P
④ ① Sachant que pour tout forme linéaire f de E, f =
Corrigé : Pr. Boujaida, CPGE Rabat n
∑ f (ei )ei∗ , Il suffit d’écrire pour tout (i, j) ∈ [[ 1, n ]]2
Partie I i =1
n n

r
① ① Élémentaire mon cher Watson !
h(e j ) = ∑ h(e j )(ei )ei∗ = ∑ ϕ(ei , e j )ei∗
i =1 i =1

e
② Si x1 , x2 ∈ E et λ ∈ K alors pour tout y ∈ E N.B : du fait que ϕ est symétrique, sa matrice

er
h( x1 + λx2 )(y) = ϕ( x1 + λx2 , y) = ϕ( x1 , y) + λϕ( x2 , y) = h( x1 ) + λh( x2 ) (y). M( ( ϕ), e) = ϕ(ei , e j ) ij est symétrique.

m
et donc h( x1 + λx2 ) = h( x1 ) + λh( x2 ). ② La matrice de h( x ) en tant qu’élément de E∗ dans la base
\
② A =⊥ϕ
Ker h(a), c’est un sous espace vectoriel de E. e∗ est le vecteur colonne [h( x )]e = M( (h), e, e∗ )X = ΩX,
a∈ A sa matrice en tant que forme linéaire de E dans la base
m

  e est le vecteur ligne M( (h)( x ), e) = t [h( x )]e = t X t Ω =


③ Ker h = x ∈ E ∀y ∈ E, ϕ( x, y) = 0 = E⊥ ϕ , dim E =

r
t
dim E∗ = n donc h est un isomorphisme si et seulement si XΩ

o
⊥ϕ
E = {0}. Et ainsi ϕ( x, y) = h( x )(y) = M( (h)( x ), e)Y = t XΩY.
or

F
✉: mamouni.myismail@gmail.com

102
i

r
s
✍ M AMOUNI M Y I SMAIL
MAMOUNI . NEW. FR
P ROBLÈMES C ORRIGÉS -MP

i
s

s
u
    
N.B : si on note X = ( xi )i , Y = (yi )i et Ω = (aij )ij alors x e alors f ( x ) e′ = X et par suite q′ f ′ ( x ) = t XΩX = q( x ).

s
ϕ( x, y) = t XΩY = ∑ aij xi y j (∗) f est donc une isométrie de E sur E′ .
1≤i,j≤ n

é
2p p

u

En outre, puisque M( (h), e, e ) = M( ( ϕ), e) et que ϕ est ③ Pour tout x = ∑ xi ci ∈ K 2p , q p (x) = 2 ∑ xi xi+ p .
non dégénérée si seulement si h est un isomorphisme i =1 i =1

r
2p 2p
alors

é
2p
ϕ est non dégénérée ⇐⇒ M( ( ϕ), e) est inversible
① On pose pour tout x = ∑ xi ci ∈ K et y = ∑ yi ci ∈
i =1 i =1
K 2p

Po r
r
① Soit q ∈ Q(E), par définition de Q(E), il existe une forme p
bilinéaire symétrique ϕ de E telle que q = q ϕ . Si maintenant 1
ϕ p ( x, y) = (q( x + y) − q( x ) − q(y)) = ∑ ( xi yi + p + xi + p yi )

u
ψ est une forme bilinéaire symétrique vérifiant la même con- 2 i =1

uor
dition q = qψ , l’identité de polarisation donne : On vérifie que ϕ p est une forme bilinéaire symétrique, et
1  que q p est la forme quadratique qui lui est associée.
∀( x, y) ∈ E2 , ϕ( x, y) = q( x + y) − q( x ) − q(y) = ψ( x, y)
2 La lecture de la matrice de q p peut être faite directement
et donc ψ = ϕ. à partir de l’expression de ϕ p ( x, y) selon la formule (∗),

P
② On va noter respectivement ϕ et ϕ′ les formes polaires des question (I.A.4.b).
formes quadratiques q et q′ .  
0 Ip
Supposons qu’il existe une isométrie f de (E, q) dans (E′ , q′ ). M ( (q) p , c ) =
Ip 0
Soit e une base quelconque de E et soit e′ son image par l’iso-
On notera dans la suite de ce corrigé J p cette dernière

r
morphisme f .
matrice.
Pour tout vecteur ei de e et son image ei′ par f , q′ (ei′ ) =

e
② q p est non dégénérée puisque elle représentée par la ma-
q′ ( f (ei )) = q(ei ) et donc par polarisation
trice inversible J p (det( J p ) = (−1) p ) dans la base canon-
er
∀(i, j) ∈ [[ 1, n ]]2 , ϕ′ (ei′ , e′j ) = ϕ(ei , e j ). ique de K 2p .

m
ce qui signifie que M( (q)′ , e′ ) = M( (q), e). Si maintenant ( F, q) est un espace de Artin, il est
Réciproquement, supposons qu’il existe des bases e de E et isométrique à (K 2p , q p ). q et q p seront donc représentées
e′ de E′ telles que M( (q)′ , e′ ) = M( (q), e) et notons Ω cette
m

par une même matrice Ω dans des bases bien choisies de


matrice. F et de K 2p . q p est non dégénérée donc toutes ses matri-

r
Soit f l’unique application linéaire de E dans E′ qui trans- ces sont inversibles. Ω est donc inversible et par suite q

o
forme e en e′ . M( ( f ), e, e′ ) = In donc pour tout x ∈ E, si X = est non dégénérée.
or

F
✉: mamouni.myismail@gmail.com

103
i

r
s
P ROBLÈMES C ORRIGÉS -MP ✍ M AMOUNI M Y I SMAIL
MAMOUNI . NEW. FR

i
s

s
u
2p
③ En remarquant que M( (q), c) = I2p , il suffit de trouver

s
est sa base anté-duale, alors pour tout x = ∑ yk ek =
une base e de C 2p telle que M( (q) p , e) = I2p . Ce qui peut k=1
nous faire penser à une certaine méthode de Gauss (vue

é
2p

u
pour les formes quadratiques réelles ... quand même). ∑ ψk ( x ) e k ∈ E
2p k=1

r
Soit x = ∑ xk ck ∈ C 2p , 2p

é
k=1p 1
p
 q p (x) = ∑ y2k .
q p ( x ) = 2 ∑ xk xk+ p = ∑ ( x k + x k + p )2 − ( x k − x k + p )2 k=1
2 k=1 et donc M( (q) p , e) = I2p

Po r
r
k=1 CQFD.
!
p p
1
∑ (xk + xk+ p )2 + ∑ (ixk − ixk+ p )2

u
= N.B : Rien n’empêche d’utiliser la méthode de Gauss
2 k=1 k=1 pour une forme quadratique q d’un C-ev E. Elle per-

uor
p 2p
1 1 mettrait de prouver l’existence d’une base de formes
∑ ( x k + x k + p )2 + (ixk− p − ixk )2
2 k=∑
= linéaires (ψ1 , ψ2 , . . . , ψn ) et de scalaires complexes
2 k=1 p+1
α1 , α2 , . . . , αn tels que
On considère alors la famille de formes linéaires Ψ = n

P
(ψ1 , ψ2, · · · , ψ2p ) définies par ∀ x ∈ E, q( x ) = ∑ αk ψk ( x )2
1 1 k=1
ψk ( x ) = √ ( xk + xk+ p ) si k ∈ [[ 1, p ]] et ψk ( x ) = √ (ixk− p − ixk ) si k ∈ [[ p +q 1,
est2palors
]]. non dégénérées si et seulement si tous les co-
2 2
2p efficient αk sont non nuls, et dans ce cas en considérant
pour tout k ∈ [[ 1, n ]], un complexe β k tel que β2k = αk ,

r
de telle sorte que q p ( x ) = ∑ ψk ( x )2 . Définitions qu’on
k=1 ψk′ = 1 ψk et e′ la base anté-duale de (ψ1′ , ψ2′ , . . . , ψn′ ),

e
peut résumer matriciellement par  βk
 nous aurions
1
er
I p iI p n n
matc∗ (Ψ) = √
∑ kk ′
, ∑ y2k

m
2 p I − iI p ∀ x = y e q ( x ) =
On note P cette dernière matrice. k=1 k=1
Ainsi par “transitivité”, si K = C et dim E est paire alors
Procédure de routine (opération par blocs, L2 ← L2 −
(E, q) est un espace de Artin ⇐⇒ q est non dégénérée
L1) :
m

I p 0 I p iI p ④ Là on se retrouve en terrain connu, celui des formes

r
I iI p I p iI p
= p soit = (−2i ) p
− I p I p I p −iI p 0 −2iI p I p −iI p quadratique réelles.

o
Donc det P 6= 0 et par suite Ψ est une base de E . Si e ∗
Il suffit de trouver une base e′ de R 2p telle que
or

F
✉: mamouni.myismail@gmail.com

104
i

r
s
✍ M AMOUNI M Y I SMAIL MAMOUNI . NEW. FR
P ROBLÈMES C ORRIGÉS -MP

i
s

s
 

u
′ ′ Ip 0 mension p et il vérifie : ∀ x ∈ V, q p ( x ) = 0.
M ( (q) p , e ) = M ( (q) , c ) =

s
0 − Ip Son image f (V ) est un sous espace vectoriel de F de
2p
dimension p ( f est un isomorphisme) et elle vérifie :

u

ou encore une base e telle que pour tout x = ∑ yi ei′ , ∀ x ∈ V, q( f ( x )) = q p ( x ) = 0.
i =1

r
p 2p
Partie IV
∑ y2i − ∑ y2i . On reconnait là l’écriture

é
q p (x) =
i =1 i = p+1 ① ① Soit ψ ∈ E∗ . h est un isomorphisme puisque q est non
canonique d’une forme quadratique de signature ( p, p). dégénérée, soit donc l’unique vecteur x ∈ E tel que

Po r
r
il suffit donc de montrer que la signature de q p est ( p, p). ψ = h ( x ).

u
2p n n
Soit x = ∑ xi ci ∈ R 2p On peut alors écrire ψ = ∑ h(x)(ei )ei∗ = ∑ ϕ(x, ei )ei∗ et

uor
i =1 i =1 i =1
p 2p par suite
1 1 
q p (x) = ∑ ( x i + x i + p )2 − ∑ ( x i − p − x i )2 ψ ∈ Vect e∗p+1 , e∗p+2, · · · , e∗n ⇐⇒ ∀i ∈ [[ 1, p ]], ϕ( x, ei ) = 0 ⇐⇒ x
2 i =1 2 i = p+1 
et donc : ψ ∈ Vect e∗p+1 , e∗p+2, · · · , e∗n ⇐⇒ ψ ∈ h( F⊥ )
il reste à justifier que la famille de formes linéaires

P

(ψ1 , ψ2, · · · , ψ2p ) définies par Ainsi h( F⊥ ) = Vect e∗p+1 , e∗p+2, · · · , e∗n
ψ( x ) = xi + xi +k si i ∈ [[ 1, p ]] et ψi ( x ) = xi − p − xi si i ∈ [[ p + 1, 2p ]] ② h est un isomorphisme donc dim F⊥ = dim h( F⊥ ) =
est libre. Pour cela, il suffit de  justifier que la matrice n − p. soit
I Ip

r
P= p dim F + dim F⊥ = n
Ip − Ip
est inversible. ③ Soit x ∈ F alors pour tout y ∈ F⊥ , ϕ( x, y) = 0 et donc

e
x ∈ ( F⊥ )⊥ . Ce qui montre que F ⊂ ( F⊥ )⊥ . Ensuite les
er
N.B : Là encore, en déployant un raisonnement simi- relations dim F + dim F⊥ = dim F⊥ + dim( F⊥ )⊥ = n

m
laires à celui fait à la fin de la question précédente, on donnent dim F = dim( F⊥ )⊥ et ainsi ( F⊥ )⊥ = F.
prouve que si K = R et dim E = 2p alors ② ① Un vecteur de ( F + G )⊥ est orthogonal à tout vecteur de
(E, q) est un espace de Artin ⇐⇒ la signature de q est ( p, p).
F et à tout vecteur de G donc ( F + G )⊥ ⊂ F⊥ ∩ G ⊥ et in-
m

⑤ Soit ( F, q) un espace de Artin de dimension 2p, et soit versement un vecteur qui est à la fois orthogonal à tout

r
donc f une isométrie de (K 2p , q p ) sur ( F, q). vecteur de F et à tout vecteur de G va être orthogonal à

o
Le sous espace vectoriel V = Vect cp de K 2p est de di- tout vecteur de F + G, soit F⊥ ∩ G ⊥ ⊂ ( F + G )⊥ .
or

F
✉: mamouni.myismail@gmail.com

105
i

r
s
P ROBLÈMES C ORRIGÉS -MP ✍ M AMOUNI M Y I SMAIL
MAMOUNI . NEW. FR

i
s

s
u
Alors ( F + G )⊥ = F⊥ ∩ G ⊥ ① Avec les formes quadratiques de R 2 , q( x, y) = 2xy et

s
② Il suffit d’appliquer le résultat du a) à F⊥ et G ⊥ et de q′ ( x, y) = x2 − y2 , nous avons 
ϕ ( x, y), ( x ′ , y′ ) = xy′ + yx ′ et ϕ′ ( x, y), ( x ′ , y′ ) = xx ′ − yy′

é
passer ensuite à l’orthogonal.

u

  (1, 1), (1, −1) est une base q-orthogonale de R 2 ,
③ Notons que F⊥ ϕF = x ∈ F ∀y ∈ F, ϕ( x, y) = 0 = F ∩ F⊥ . 
(1, 0), (0, 1) en est une base q′ -orthogonale.

r
- [1 ⇒ 2)] Supposons que F est non singulier. ϕ F est donc non

é
dégénérée et donc d’après (I.A.3), F⊥ ϕF = {0} soit F ∩ F⊥ = ② Soit e1 = (a, b) un vecteur non nul de R 2 et voyons s’il peut
{0}. exister un vecteur e2 = ( x, y) tel que (e1 , e2 ) soit une base de

Po r
R 2 à la fois q-orthogonale et(q′ -orthogonale. On devrait avoir

r
-[2 ⇒ 3)] Supposons que F ∩ F⊥ = {0}. Nous avons dim F +
dim F⊥ = dim E donc F ⊕ F⊥ = E. bx + ay = 0

u
(S)
- [3 ⇒ 4)] Supposons que F ⊕ F⊥ = E. Alors F⊥ ∩ ( F⊥ )⊥ = ax − by = 0

uor

{0} et donc ( F⊥ ) ϕF⊥ = {0}. Alors ϕ F ⊥ est non dégénérée et ce système linéaire a pour déterminant δ = −b2 − a2 . e1 est
donc F⊥ est non singulier. non nul donc δ 6= 0. (S) a donc pour unique solution la solu-
- [4 ⇒ 1)] Supposons que F⊥ est non singulier. L’implication tion nulle.
1 ⇒ 4) ayant été justifiée, il suffit de l’appliquer à F⊥ pour Ainsi il n’existe aucune base de R 2 qui soit à la fois q-

P
s’assurer que F est non singulier. orthogonale et q′ -orthogonale.
④ Supposons que F et G sont orthogonaux et non singuliers. ③ D’après des résultats démontrés dans la premières par- 
Notons que le fait que F et G soient orthogonaux signifie que ties pour toute base e de E, M( (h), e, e∗ ) = ϕ(ei , e j ) ij .

r
F ⊂ G ⊥ . Puisque G ∩ G ⊥ = {0} alors F ∩ G = {0} et donc la On voit ainsi que e est une base q-orthogonale si et seule-
somme F + G est bien directe. ment si M( (h), e, e∗ ) est diagonale. Plus exactement e est q-

e
Soit maintenant x ∈ ( F ⊕ G ) ∩ ( F ⊕ G )⊥ , et posons x = orthogonale si et seulement si 
M( (h), e, e∗ ) = diag q(e1 ), q(e2 ), · · · , q(en ) .
er
x F + x G où x F ∈ F et x G ∈ G.

m
( F ⊕ G)⊥ = F⊥ ∩ G⊥ donc x ∈ F⊥ ∩ G⊥ . Notons aussi que puisque q est non dégénérée M( (h), e, e∗ )
est inversible et donc dans le cas où e est une base q-
x ∈ F⊥ donc pour tout u ∈ F, ϕ( x, u) = ϕ( x F , u) = 0, et donc orthogonale alors q(ei ) 6= 0 pour tout i ∈ [[ 1, n ]].
x F ∈ F ∩ F⊥ . F est non singulier donc x F = 0.
m

Supposons que e est une base à la fois q-orthogonale et q′ -


De même x ∈ C⊥ donne x G = 0.

r
orthogonale de E alors  ′
Alors x = 0 et donc ( F ⊕ G ) ∩ ( F ⊕ G )⊥ = {0}. F ⊕ G est −1 ′ −1 ∗ ′ ∗ q ( e1 ) q ′ ( e2 ) q′ (e n
M( (h) ◦ h , e) = M( (h) , e , e)M( (h) , e, e ) = diag , ,··· ,

o
donc non singulier. q ( e1 ) q ( e2 ) q(e n
or

F
✉: mamouni.myismail@gmail.com

106
i

r
s
✍ M AMOUNI M Y I SMAIL
MAMOUNI . NEW. FR
P ROBLÈMES C ORRIGÉS -MP

i
s

s
u
M( (h)−1 ◦ h′ , e) est diagonale donc la base e est formée de ② Avec y = z − (q(z)/2) x nous avons

s
vecteurs propres de h−1 ◦ h′ . q ( z )2 q(z)
q(y) = q(z) + q( x ) − 2. .ϕ(z, x ) = 0
4 2

u
④ Supposons que h−1 ◦ h′ admet n valeurs propres deux à deux
③ ϕ( x, y) = ϕ( x, z) − (q(z)/2)q( x ) = ϕ( x, z) = 1. Donc y
distinctes, il est donc diagonalisable. Soit e une base de diag-
est non colinéaire à x car sinon nous aurions ϕ( x, y) = 0.

r
onalisation et posons pour tout i ∈ [[ 1, n ]] h−1 ◦ h′ (ei ) = λi ei

é
ou encore h′ (ei ) = λi h(ei ). Vu la définition des applications h Soit Π le plan engendré par x et y. Pour tout vecteur
et h′ ceci signifie que v = αx + βy ∈ Π
q(v) = α2 q( x ) + β2 q(y) + 2αβϕ( x, y) = 2αβ

Po r
r
∀i ∈ [[ 1, n ]], ∀ x ∈ E, ϕ′ ( x, ei ) = λi ϕ( x, ei )
Alors Π, q/Π est un espace de Artin.
Ainsi pour tout p ∈ [[ 1, n ]], un vecteur x qui serait orthogonal

u
aux vecteurs e1 , e2 , · · · , e p pour q le serait aussi pour q′ . ② F un sous-espace vectoriel singulier de E et (e1 , e2 , · · · , es )

uor
Posons alors pour tout p ∈ [[ 1, n ]], Vp = Vect ep. Si p ≥ 2 une base de F ∩ F⊥ . G un supplémentaire de F ∩ F⊥ dans F :

F = F ∩ F⊥ ⊕ G.
alors Vp ∩ Vp⊥−1 est non nul, car sinon nous aurions dim Vp +
  
dim Vp⊥−1 ≤ n soit p + (n − p + 1) = n + 1 ≤ n. Considérons ① F⊥ = F⊥ + F ∩ G ⊥ donc F ∩ F⊥ = F ∩ ( F⊥ + F) ∩

P
alors une famille(de vecteurs (u1 , u2 , · · · , un ) telle que G⊥ = F ∩ G⊥ .
u1 = e 1 Comme G ⊂ F alors G ∩ G ⊥ ⊂ F ∩ F⊥ , mais G ∩ G ⊥ ⊂ G

u p ∈ Vp ∩ Vp⊥−1 \ 0 si p ≥ 2 et G ∩ ( F ∩ F⊥ ) = {0} donc G ∩ G ⊥ = {0}.
Pour tout p ≥ 2, u p est orthogonal aux vecteurs D’après la question (II.A.3), G est non singulier.

r
e1 , e2, · · · , e p−1 pour q donc aussi pour q′ et donc il est ort ② Si s = 1, F ∩ F⊥ = Ke1 . e1 ∈ F⊥ donc ϕ(e1 , e1 ) = 0 et
hogonal aux vecteurs u1 , u2 , · · · , u p−1 (car ils sont tous dans

e
pour tout v ∈ G, ϕ(e1 , v) = 0. Donc q(e1 ) = 0 et e1 ∈ G ⊥ .
Vp−1 = Vect ep − 1) pour q et pour q′ . Au final la famille
er
(u1 , u2 , · · · , un ) est à la fois q-orthogonale et q′ -orthogonale. Considérons la restriction q1 de q sur G ⊥

m
D’après la question (II.C.1) il existe un plan artinien P1
① x ∈ E tel que x 6= 0 et q( x ) = 0. (on dit que x est un vecteur pour q1 dans G ⊥ contenant e1 . P1 est naturellement or-
q-isotrope). thogonal à G pour q. Ce qu’on nous voulions démontrer.
m

/ E⊥ , il existe donc au Si s ≥ 2, supposons que la propriété est vraie pour tout

r
① q est non dégénérée, donc x ∈
moins un vecteur v ∈ E tel que ϕ( x, v) 6= 0. En posant sous-espace F′ tel que dim F′ ∩ F′⊥ = s − 1.

o
z = (1/ϕ( x, v))v nous avons ϕ( x, z) = 1. Posons F′ = Vect es − 1 et E′ = (G ⊕ Kes )⊥ .
or

F
✉: mamouni.myismail@gmail.com

107
i

r
s
P ROBLÈMES C ORRIGÉS -MP ✍ M AMOUNI M Y I SMAIL
MAMOUNI . NEW. FR

i
s

s
u
G ⊕ Kes ⊂ F donc F⊥ ⊂ E′ . F′ ⊂ F ∩ F⊥ ⊂ F⊥ donc On en déduit que ϕ(y, v) = 0 et ceci pour tout v ∈ G. Alors

s
F′ ⊂ E′ . y ∈ G ∩ G ⊥ et donc y = 0.
De plus F′ ⊂ F⊥ et F⊥ ⊂ F′⊥ donc F′ ⊂ F′⊥ soit Ensuite, soit pour k ∈ [[ 1, s ]], un vecteur quelconque vk de Pk ,

u
F′ ∩ F′⊥ = F′ . {0} est le seul supplémentaire de F′ ∩ F′⊥ ⊥
alors ϕ( x, vk ) = 0 puisque F ⊂ Pk⊥ , et ϕ( xi , vk ) = 0 si i 6= k
dans F′ . puisque dans ce cas Pk est orthogonal à Pi .

r
dim F′ ∩ F′⊥ = s − 1 donc par hypothèse de récurrence

é
D’où ϕ( xk , vk ) = 0, et ceci pour tout vk ∈ Pk . Mais ( Pk , q/Pk )
il existe des plans artiniens P1 , P2 , · · · , Ps−1 dans E′ qui est artinien donc q/Pk est non dégénérée, par suite xk = 0.
sont deux à deux orthogonaux et qui pour chaque k ∈

Po r
r

Finalement x = 0, et donc F ∩ F = {0}.
[[ 1, s − 1 ]], Pk contient ek .
④ Notons qu’en général :

u
Ses plans, puisqu’ils sont dans E′ , sont tous orthogonaux
à G et au vecteur es . - [•] F = G ⊕ P1 ⊕ · · · ⊕ Ps donc dim F = dim G + 2s =

uor
dim F + dim F ∩ F⊥ .
Considérons maintenant le sous-espace E′′ = (G ⊕ P1 ⊕
- [•] q/F = 0 ⇐⇒ ϕ/F × F = 0 ⇐⇒ ∀(u, v) ∈ F2 , ϕ(u, v) =
· · · ⊕ Ps−1 )⊥ . es ∈ E′′ et q(es ) = 0 donc il existe un plan
0 ⇐⇒ F ⊂ F⊥ .
artinien Ps de E′′ contenant es
Ainsi si q/F = 0, alors dim F ≤ n − dim F et donc dim F ≤

P
Finalement
n/2.
- [•] Pour tout k ∈ [[ 1, s ]], Pk est un plan artinien contenant
⑤ Supposons que n = 2p.
ek .
- [•] Les plans Pk sont deux à deux orthogonaux Supposons que (E, q) est un espace de Artin, il existe donc

r
une base e de E telle que
- [•] Ils sont tous orthogonaux à G. 2p p

e
CQFD ∀x = ∑ xi ei ∈ E, q(x) = ∑ xi xi + p
③ F = G ⊕ P1 ⊕ · · · ⊕ Ps , les plans Pk étant deux à deux orthog- i =1 i =1
er
Si on pose F = Vect ep, on voit alors que pour tout x ∈ F,
onaux et tous orthogonaux à G.

m
q( x ) = 0. dim F = p et nous avons bien q/F = 0.

Soit x ∈ F ∩ F et posons
s Réciproquement s’il existe un sous-espace vectoriel F de E
x = y+ ∑ xk , où y ∈ G et xk ∈ Pk pour tout k ∈ [[ 1, s ]]. de dimension p et tel que q/F = 0.
m

k=1 F ⊂ F⊥ donc F ∩ F⊥ = F et donc F = P1 ⊕ P2 ⊕ · · · ⊕ Pp où

r

Soit v ∈ G. x ∈ F ⊂ G ⊥ , donc ϕ( x, v) = 0. Les plans Pk sont P1 , P2 , · · · , Pp sont des plans artiniens deux à deux orthogo-

o
orthogonaux à G donc pour tout k ∈ [[ 1, s ]] , ϕ( xk , v) = 0. naux. Nous avons alors dim F = 2p et donc F = E.
or

F
✉: mamouni.myismail@gmail.com

108
i

r
s
✍ M AMOUNI M Y I SMAIL
MAMOUNI . NEW. FR
P ROBLÈMES C ORRIGÉS -MP

i
s

s
u
Soit pour tout k une base (ek , uk ) de Pk telle que q(ek ) = alors

s
q(uk ) = 0 et ϕ(ek , uk ) = 1. ϕ ( y1 , y2 ) = ϕ ( x1 , x2 ) = 0
⊥ ⊥
b = (e1 , · · · , e p , u1 , · · · , u p ) est une base de E, puisque E = donc f ( F ) ⊂ f ( F) . Comme f est par définition un au-

u
P1 ⊕ P2 ⊕ · · · ⊕ Pp . tomorphisme alors dim f ( F⊥ ) = dim F⊥ = n − dim F et
p dim f ( F)⊥ = n − dim f ( F) = n − dim F

r
Maintenant pour tout x = ∑ ( xk ek + xk+ p uk ) ∈ E, les et ainsi dim f ( F⊥ ) = dim f ( F)⊥ . Alors f ( F⊥ ) = f ( F)⊥

é
k=1
vecteurs xk ek + xk+ p uk étant deux à deux orthogonaux (car N.B : Si jamais F est stable par f (et donc f ( F) = F), ce
les plans Pk le sont) on aura

Po r
qui précède démontre que f ( F⊥ ) = F⊥ .

r
p p

q( x ) = ∑ q x k e k + x k+ p uk = 2 ∑ x k x k+ p ② Posons pour tout ( x, y) ∈ E, ψ( x, y) = ϕ( f ( x ), f (y)),

u
k=1 k=1 M = M( ( f ), e) et Ω = M( (q), e) .

uor
Ainsi E est un espace de Artin.
Soient x, y ∈ E et soient X = [ x ]e et Y = [y]e , nous avons
N.B : avec dim E = 2p et dim F = p et donc aussi
alors
dim F⊥ = p ψ( x, y) = ϕ( f ( x ), f (y)) = t ( MX )Ω( MY ) = t X t MΩMY
q/F = 0 ⇐⇒ F ⊂ F⊥ ⇐⇒ F = F⊥ donc M( (ψ), e) = t MΩM.

P
Résumons : Si E est un K-ev de dimension 2p muni d’une ③ f ∈ O(E, q) ⇐⇒ ψ = ϕ ⇐⇒ M ( ( ψ ), e ) =
forme quadratique q, alors ( M( ( ϕ), e) ⇐⇒ Ω = t MΩM.
q est non dégénérée
④ Si f ∈ O(E, q) alors t MΩM = Ω et en passant au déter-

r
(E, q) est un espace de Artin ⇐⇒
il existe un sev F de E tel que F⊥ = F minant on aura det( M)2 det(Ω) = det(Ω). q est non
dégénérée donc Ω est inversible et donc det( M)2 = 1.

e
Partie II
Ainsi det( f ) ∈ {−1, 1}.
er

m
① ① L’identité de polarisation est votre amie ② F et G deux sous-espaces vectoriels supplémentaires de E. s
1 
∀( x, y) ∈ E2 , ϕ( x, y) = q( x + y) − q( x ) − q(y) la symétrie de E d’axe F et de direction G.
2
Soient ensuite f ∈ O(E, q) et un sous-espace vectoriel F ① Supposons que s ∈ O(E, q).
m

de E.

r
Pour tout x ∈ F et y ∈ G, ϕ(s( x ), s(y)) = ϕ( x, −y) =
Soient y1 ∈ f ( F⊥ ) et y2 ∈ f ( F), et soient x1 ∈ F⊥ et − ϕ( x, y) et donc ϕ( x, y) = − ϕ( x, y) soit ϕ( x, y) = 0. F

o
x2 ∈ F tel que y1 = f ( x1 ) et y2 = f ( x2 ). Nous avons et G sont donc orthogonaux.
or

F
✉: mamouni.myismail@gmail.com

109
i

r
s
P ROBLÈMES C ORRIGÉS -MP ✍ M AMOUNI M Y I SMAIL
MAMOUNI . NEW. FR

i
s

s
u
Réciproquement, supposons que F et G sont orthogo- N.B : si x 6= y, q( x ) = q(y) mais q( x − y) = 0, il ne peut

s
naux. exister de réflexions s de E telle que s( x ) = y.
Soit x ∈ E. x + s( x ) ∈ F et x − s( x ) ∈ G donc En effet, si une telle réflexion existait et H désignait

u
ϕ( x + s( x ), x − s( x )) = 0, ce qui donne ϕ( x, x ) − son axe, alors s( x − y) = y − s2 ( x ) = y − x et donc
ϕ(s( x ), s( x )) = 0 ou encore q(s( x )) = q( x ). x − y ∈ H ⊥ ou encore H ⊥ = K.( x − y). Mais comme

r
Alors f ∈ O(E, q). q( x − y) = 0, alors x − y ∈ { x − y}⊥ = H. H est donc

é
② Une symétrie est dans O(E, q) si et seulement si son non singulier, il ne peur être l’axe d’une symétrie orthog-
axe F et sa direction G son orthogonaux, F et G étant onale.

Po r
r
naturellement supplémentaires nous aurions forcément ① E est un espace de Artin de dimension 2p et F un sous-espace

u
G = F⊥ (car G ⊂ F⊥ et dim G = dim F⊥ = n − dim F). de E tel que q/F = 0.

uor
③ Soit s la reflexion de E d’axe H. Soit e un vecteur di- Soit f ∈ O(E, q) tel que f ( F) = F.
recteur de la droite H ⊥ et (e2 , e3 , · · · , en ) une base de H. On reprend la base b = (e1 , · · · , e p , u1 , · · · , u p ) construite
H est non singulier donc H et H ⊥ sont supplémen- dans ( I I.C.5) et qui vérifie pour rappel les conditions suiv-
taires et donc b = (e, e2 , · · · , en ) est une base de E et antes 

P
M( (s), b) = diag(−1, 1, · · · , 1). (e1 , e2, · · · , e p ) est une base de F

Alors det(s) = −1 et donc s ∈ O− (E, q). Les plans Pk = Vect{ek , uk } sont 2 à 2 orthogonaux


④ x, y ∈ E tels que q( x ) = q(y) et q( x − y) 6= 0. Soit ∀k ∈ [[ 1, p ]], q(ek ) = q(uk ) = 0 et ϕ(ek , uk ) = 1.
H = { x − y}⊥ = (K.( x − y))⊥ . Dans cette base, les matrices Ω et M de ϕ et de f sont de la

r
forme (F étant
 stable  par f ) 
Notons que q( x − y) 6= 0 implique que H ⊥ ∩ H = {0} et 
0 Ip A B

e
donc que H est non singulier. Ω= et M = où A, B, C ∈ M p (K ).
Ip 0 0 C
er
Soit maintenant s la réflexion d’axe H.
1 1 D’après (III.A.1.c),  nous avons Ω =t MΩM,
 ce qui  donne

m
x = ( x + y) + ( x − y) avec 0 t
CA 0 Ip
2 2 =
 1 t
AC t BC + t CB Ip 0
 ( x − y) ∈ H ⊥
2 On en déduit que t CA = I p et donc
m

 1
 ( x + y) ∈ H car ϕ( x − y, x + y) = q( x ) − q(y) = 0 det( f ) = det( A) det(C) = det(t C) det( A) = det(t CA) = 1.

r
2 Alors f ∈ O + (E, q).
1 1

o
donc s( x ) = ( x + y) − ( x + y) = y ② F un sous-espace de E tel que F = E et f ∈ O(E, q) telle que
2 2
or

F
✉: mamouni.myismail@gmail.com

110
i

r
s
✍ M AMOUNI M Y I SMAIL
MAMOUNI . NEW. FR
P ROBLÈMES C ORRIGÉS -MP

i
s

s
u
f /F = id F . La matrice ∆ étant celle de ϕ/G × G , la forme de M est due au

s
Soit s = dim F ∩ F . ⊥ fait que (ε 1 , · · · , ε n−2s , e1 · · · , es ) est une base de F et que
f /F = id F .

é
Il existe par définition du complété F des plans artiniens

u
deux à deux orthogonaux et tous orthogonaux à G tels que La relation Ω= t MΩM donne ici   
∆ 0 0 ∆ 0 0

r
E = G ⊕ P1 ⊕ · · · ⊕ Ps .  0  =  0 0 Is 
0 C

é
G est non singulier donc G ⊕ G ⊥ = E. Le sous-espace H = t t
A∆ C BC + CB t t
0 Is 0
P1 ⊕ P2 · · · ⊕ Ps et inclu dans G ⊥ , car les plan Pk le sont tous. En particulier C = Is et donc det f = det C = 1.

Po r
r
De plus dim G ⊥ = dim H = n − dim G, donc G ⊥ = H = ③ f ∈ O(E, q) tel que : ∀ x ∈ E, q( x ) 6= 0 =⇒ f ( x ) − x 6=
P1 ⊕ P2 · · · ⊕ Ps . 
0 et q( f ( x ) − x ) = 0 .

u
Maintenant, f /G = id G , en particulier f (G ) = G et donc F un sous-espace de E stable par f .

uor
f ( H ) = H. G et H sont ainsi des sous-espaces supplémen-
taires dans E, tous les deux stables par f donc det( f ) = ① Puisque q est non dégénérée, il existe au moins un
det( f /G ) det( f /H ) = det( f /H ). vecteur x ∈ E tel que q( x ) 6= 0.

q( x − f ( x )) = 0 donc forcément la famille x, x − f ( x )
H est un espace de Artin puisque F ∩ F⊥ ⊂ H,

P
1 est libre (et donc déjà dim E ≥ 2).
dim F ∩ F⊥ = dim H et q F ∩ F ⊥ = 0. De plus f /F = id F donc Ensuite q( x − f ( x )) = 0 donc x − f ( x ) est orthogonal à
2
f ( F ∩ F⊥ ) = F ∩ F⊥ . D’après la question précédente nous lui même. De plus
avons donc det( f /H ) = 1. 0 = q( x − f ( x )) = q( x ) + q( f ( x )) − 2ϕ( x, f ( x )) =

r
Par suite det( f ) = 1. 2 q( x ) − ϕ( x, f ( x )) donc ϕ( x, f ( x )) = q( x ). On en dé-
duit que ϕ( x, x − f ( x )) = 0, et donc que x − f ( x ) est

e
Une deuxième façon : sans utiliser la question précédente.
aussi orthogonal à x.
Il existe une base b = (ε 1 , · · · , ε n−2s , e1 · · · , es , u1 , · · · , us ) de
er
 ⊥
E telle que (ε 1 , · · · , ε n−2s ) soit une base de G, (e1 , e2, · · · , es ) Alors x − f ( x ) ∈ Vect x, f ( x ) − x avec x − f ( x ) 6= 0.

m

une base de F ∩ F⊥ et pour tout k ∈ [[ 1, p ]] , (ek , uk ) une base q est non dégénérée donc forcément Vect x, f ( x ) − x
de Pk avec q(ek ) = q(uk ) = 0 et ϕ(ek , uk ) = 1. est inclu strictement dans E.
Alors dim E ≥ 3.
m

Dans cette base, les  matricesde ϕ et def sont de la forme 


② V = Ker( f − id E ).

r
∆ 0 0 In−2s 0 A
Ω =  0 0 Is  et M =  0 Is B  Soit x ∈ V, alors f ( x ) − x = 0 et donc par contre–

o
0 Is 0 0 0 C apposée de l’hypothèse faite sur f , q( x ) = 0. Ainsi
or

F
✉: mamouni.myismail@gmail.com

111
i

r
s
P ROBLÈMES C ORRIGÉS -MP ✍ M AMOUNI M Y I SMAIL
MAMOUNI . NEW. FR

i
s

s
u
q/V = 0. ϕ(z, x ) = ϕ( f (y) − y, x ) = ϕ( f (y), f ( x )) − ϕ(y, x ) = 0.

s

③ Soit x ∈ E tel que q( x ) = 0 et soit H = { x } . Donc V ⊂ U ⊥ . L’égalité des dimensions donne alors
V = U⊥.

é
dim H ≥ n − 1 selon que x = 0 ou non. Comme n ≥ 3

u
alors dim H > n/2. On ne peur donc avoir q/H = 0, ⑥ E est de dimension paire et V et un sev de E de dimen-
selon (II.4.C).

r
sion n/2 tel que q/V = 0. D’après (II.C.5), E est un espace

é
Soit maintenant y ∈ H tel que q(y) 6= 0. x ∈ H ⊥ donc de Artin.
ϕ( x, y) = 0 et donc q( x + y) = q( x ) + q(y) = q(y). De De plus V = Ker( f − id E ) donc f (V ) = V et donc
même q( x − y) = q(y).

Po r
r
d’après (III.B.1) f ∈ O + (E, q).
④ U = Im( f − id E ).

u
Soit z ∈ U et soit donc x ∈ E tel que z = f ( x ) − x. Partie III 
N.B : Il est aisé de vérifier que O(E, q), ◦ est un

uor
Si q( x ) 6= 0 alors q(z) = q( f ( x ) − x ) = 0. groupe.
Si q( x ) = 0 alors il existe y ∈ E tel que q( x + y) = Si F est un sous-espace non singulier de E, on notera dans la suite
q( x − y) = q(y) 6= 0. s F la symétrie orthogonale de E d’axe F.
q(y) 6= 0 donc q( f (y) − y) = 0.

P
q( x + y) 6= 0 donc q( f ( x + y) − ( x + y)) = q(z + f (y) − ① Si n = 1, posons E = K.e. Soit f un automorphisme de E et
y) = 0 et donc q(z) + 2ϕ(z, f (y) − y) = 0. posons f (e) = λe. q( f (e)) = q(e) si seulement si λ2 = 1, soit
De même q( x − y) 6= 0 donne q(z) − 2ϕ(z, f (y) − y) = 0. λ = ±1. Dans ces cas f = ±id E . Les deux endomorphismes
La somme de ces deux dernières relations donne q(z) = id E et −id E sont effectivement des isométries de E, ce sont

r
0. donc les seules. −id E est la réflexion de E d’axe {0}.
N.B : Nous avons ainsi démontré qu’en fait : ∀ x ∈ ② Supposons qu’il existe x ∈ E tel que f ( x ) = x et q( x ) 6= 0.

e
E, q( f ( x ) − x ) = 0. q( x ) 6= 0 donc H = (Kx )⊥ est un hyperplan non singulier,
er
⑤ q/U = q/V = 0 donc dim U ≤ n/2 et dim V ≤ n/2. la forme quadratique q/H est donc non dégénérée. Kx est sta-

m
D’après la formule du rang dim U + dim V = n donc n ble par f donc H est stable par f . Soit f ′ l’endomorphisme
est forcément paire et dim U = dim V = n/2. induit par f sur H. f ′ est un automorphisme de H et pour
q/U = 0 donc U ⊂ U ⊥ , dim U ⊥ = n − dim U = n/2 = tout x ∈ H, q/H ( f ′ ( x )) = q( f ( x )) = q( x ) = q/H ( x ). Donc
m

dim U donc U ⊥ = U. f ′ ∈ O( H, q/H ).

r
Soient ensuite z ∈ U et x ∈ V. f ( x ) = x et il existe y ∈ E Par hypothèse de récurrence, f ′ est la composée d’au plus

o
tel que z = f (y) − y. n − 1 réflexions s′H ′ , s′H ′ , · · · , s′Hr′ de H.
1 2
or

F
✉: mamouni.myismail@gmail.com

112
i

r
s
✍ M AMOUNI M Y I SMAIL
MAMOUNI . NEW. FR
P ROBLÈMES C ORRIGÉS -MP

i
s

s
u
Posons pour tout k ∈ [[ 1, r ]], Hk = Kx ⊕ Hk′ . Kx et Hk′ sont Soit (e1 , e2 ) une famille libre de U. U = Im( f − id E ) donc il

s
orthogonaux et tous les deux non singuliers donc Hk est un existe des vecteurs v1 et v2 dans E tel que ( f − id E )(v1 ) = e1
hyperplan non singulier de E (d’après (II.A.4)). et ( f − id E )(v2 ) = e2 .

u
Nous
( avons ensuite (v1 , v2 ) est une famille libre car sinon son image par f − id E ,
s H1 ◦ · · · ◦ s Hr ( x ) = x = f ( x ) ie (e1 , e2 ) serait liée.

r
∀y ∈ H, s H1 ◦ · · · ◦ s Hr (y) = s′H ′ ◦ · · · ◦ s′Hr′ (y) = f ′ (y) = f (y) Soient α1 , α2 , β 1 , β 2 des scalaires tels que

é
1
donc f = s H1 ◦ · · · ◦ s Hr . f est ici la composée d’au plus n − 1 α1 e1 + α2 e2 + β 1 v1 + β 2 v2 = 0
réflexions de E. Puisque e1 , e2 ∈ Ker( f − id E ) en appliquant f − id E à cette re-

Po r
r
lation nous obtenons β 1 e1 + β 2 e2 = 0 et donc β 1 = β 2 = 0.
③ Supposons qu’il existe x ∈ E tel que q( x ) 6= 0 et q( x −
Par suite nous avons aussi α1 = α2 = 0.

u
f ( x )) 6= 0.
La famille v = (e1 , e2 , v1 , v2 ) est libre. Considérons alors le

uor
Nous avons q( f ( x )) = q( x ) et q( f ( x ) − x ) 6= 0 donc
sous-espace F = Vect(v). Nous avons
d’après (III.A.2.d), la réflexion s H où H = { f ( x ) − x }⊥ vérifie
f (e1 ) = e1 , f (e2 ) = e2 , f (v1 ) = e1 + v1 et f (v2 ) = e2 + v2
s H ( f ( x )) = x.
donc F est stable par f . Soit f ′ l’endomorphisme induit par f
Posons g = s H ◦ f . Alors g ∈ O(E, q) et g( x ) = x avec sur F. La matrice de f ′ dans la  base v de Fest

P
q( x ) 6= 0. Nous retrouvons les hypothèses du cas précédent. 1 0 1 0
Il existe donc au plus n − 1 réflexions s H1 , s H2 , · · · , s Hr de E 0 1 0 1 
telle que g = s H1 ◦ s H2 ◦ · · · ◦ s Hr . A= 0 0 1 0 

Sachant que s2H = id E , nous avons donc f = s H ◦ s H1 ◦ s H2 ◦ 0 0 0 1

r
· · · ◦ s Hr . Ce qui permet de rapidement voir que Ker( f ′ − id F ) =

Alors f est la composée d’au plus n réflexions. Im( f ′ − id F ) = Vect e1 , e2 . Notons U ′ ce sous-espace.

e
④ Le cas restant : il n’existe aucun vecteur x ∈ E tel que U ′ ⊂ U donc q/U ′ = 0. Ce qui permet d’affirmer que
er
∀ x ∈ F, q( x ) 6= 0 =⇒ x ∈ / U ′ =⇒ f ′ ( x ) − x 6= 0 et q( x − f ′ ( x )) = 0

m
q( x ) 6= 0 et ( f ( x ) = x ou q( x − f ( x )) 6= 0).
Autrement dit : ∀ x ∈ E, q( x ) 6= 0 =⇒ f ( x ) − x 6= À son tour f ′ vérifie les mêmes hypothèses que vérifiait l’i-
0 et q( f ( x ) − x ) = 0 sométrie f de E donc ( F, q/F ) est un espace de Artin. q/F est
non dégénérée donc F est non singulier et par suite F ⊕ F⊥ =
m

Nous retrouvons ainsi les hypothèses de la question (III.B.3).

r
Alors E est un espace de Artin et le sous-espace U = Ker( f − E.
id E ) = Im(u − id E ) est de dimension p = n/2 et vérifie Les vecteurs v1 et v2 ayant rempli leurs mission, oublions

o
q/U = 0. De plus n ≥ 3 donc en fait n ≥ 4 et p ≥ 2. les et considérons plutôt deux vecteurs u1 et u2 tels que
or

F
✉: mamouni.myismail@gmail.com

113
i

r
s
P ROBLÈMES C ORRIGÉS -MP ✍ M AMOUNI M Y I SMAIL
MAMOUNI . NEW. FR

i
s

s
u
 ⊥
(e1 , e2 , u1 , u2 ) soit une base “artinienne” de F, ie telle que  Si nous posons G2 = Vect ε 3 , ε 4 alors (s G2 ◦ g)/F = id F .

s
q(u1 ) = q(u2 ) = 0, ϕ(e1 , u1 ) = ϕ(e2 , u2 ) = 1 et Vect e1 , u1 ⊥Vect e2 , u2 Passons à F⊥ maintenant. F est stable par g donc F⊥ aussi.
f (u1 ) − u1 et f (u2 ) − u2 sont des vecteurs de U ′ donc on peut

é
Par hypothèse de récurrence l’isométrie g′′ induite par g sur

u
écrire ( F⊥ est la composée d’au plus n − 4 réflexions s H ′ , s H ′ , . . . , s Hr′
f ( u1 ) = u1 + α 1 e 1 + β 1 e 2 1 2

r
f ( u2 ) = u2 + α 2 e 1 + β 2 e 2 de F⊥ . Posons alors pour tout k ∈ [[ 1, r ]] Hk = Hk′ + F. Hk′

é
ϕ( f (u1 ), u1 ) = q(u1 ) = 0 donne α1 = 0 et de même β 2 = 0. et F sont non singuliers et orthogonaux (Hk′ ⊂ F⊥ ), donc Hk
est non singulier. C’est aussi un hyperplan de E, soit donc la

Po r
r
ϕ( f (u1 ), f (u2 )) = ϕ(u1 , u2 ) = 0 donne 0 = ϕ(u1 + β 1 e2 , u2 + refléxion s Hk de E d’axe Hk . et posons h = s H1 ◦ s H2 ◦ · · · ◦ s Hr .
α2 e1 ) = α2 + β 1 . Nous avons alors (

u
Notre écriture
( devient alors ∀ x ∈ F⊥ , s G2 ◦ g( x ) = g( x ) = h( x )
!

uor
f (u1 ) = u1 + αe2 ∀ x ∈ F, s G2 ◦ g( x ) = x = h( x )
/ U′
où α 6= 0 car u1 ∈
f (u2 ) = u2 − αe1 car si x ∈ F⊥ , nous avons F⊥ ⊂ G2 et g( x ) ∈ F⊥ donc
 ⊥ s G2 ◦ g( x ) = g( x ).
Soit maintenant le sous-espace G1 = Vect e2 , u2 de E
Et si x ∈ F, alors pour tout k ∈ [[ 1, r ]], x ∈ Hk donc s Hk ( x ) = x

P
(noter qu’alors e1 , u1 ∈ G1 ) et posons g = s G1 ◦ f .
et donc h( x ) = x.
F est stable par f et par s G1 donc stable par g et la ma- Ainsi s G2 ◦ g = h, soit s G2 ◦ s G1 ◦ f = s H1 ◦ s H2 ◦ · · · ◦ s Hr , ou
trice de l’isométrie g′ induite par g sur F dans la base encore
u = (e1, e2, u1 , u2 ) est  

r
   f = s G1 ◦ s G2 ◦ s H1 ◦ s H2 ◦ · · · ◦ s Hr
1 0 0 0 1 0 0 −α 1 0 0 −α
0 − 1 0 0   0 1 α 0   0 − 1 − α 0  Il suffit ensuite de voir que les symétries s G1 et s G2 sont cha-

e
B= 
0 0 1 0   0 0 1 0  =  0 0
   cune la composée de deux réflexions (par exemple s G2 =
1 0 
s{ε 1 }⊥ ◦ s{ε 1 }⊥ ) pour conclure que f est la composée d’au plus
er
0 0 0 −1 0 0 0 1 0 0 0 −1


m
′ n réflexions.
g est la symétrie orthogonale de F d’axe Vect ε 1 , ε 2 et de di-

rection Vect ε 3 , ε 4 (diagonaliser B pour le voir) avec
(
ε 1 = e1 − αe2 + 2u1 et ε 2 = e1 + αe2 − 2u1 M AKING O F : Il s’agissait d’imiter la démarche suivie dans les
cas précédents, donc de trouver une symétrie orthogonale
m

ε 3 = αe1 + e2 + 2u2 et ε 4 = αe1 − e2 + 2u2 s G telle que H = Ker(s G ◦ f − id E ) soit non nul et d’appliquer l’hy-

r
Les vecteurs ε k sont choisis non isotropes et formant une base pothèse de récurrence à l’endomorphisme induit par s G ◦ f sur H ⊥

o
orthogonale de F. (forcément stable par s G ◦ f ).
or

F
✉: mamouni.myismail@gmail.com

114
i

r
s
✍ M AMOUNI M Y I SMAIL
MAMOUNI . NEW. FR
P ROBLÈMES C ORRIGÉS -MP

i
s

s
u
En aucun cas SG ne devrait être une réflexion car dans ce cas un cas général un sous-espace de E de dimension 4 stable par f et on

s
vecteur non nul x de H, va vérifier s G ◦ f ( x ) = x donc f ( x ) = s G ( x ) injecte.
et donc f ( x ) − x ∈ G ⊥ , mais f ( x ) − x est isotrope et donc G ⊥ ⊂ G.

u
G est donc non singulier et ne peut donc être l’axe d’une réflexion. N.B : la réponse est trop longue, mais après plusieurs essais in-
Noter que de toute façon, on aurait toujours f ( x ) − x ∈ G ⊥ si fructueux, c’est le mieux que j’ai pu faire. Je suis preneur pour

r
s G ◦ f ( x ) = x, et donc G ⊥ doit contenir au moins une vecteur non toute idée alternative. Mon adresse eMail se trouve dans les mé-

é
nul de U. tadonnées du fichier PDF (menu fichier –> propriétés dans Adobe
On commence par essayer d’appréhender f quand dim E = 4 (la ReaderTM ) ou au début du fichier source LaTeX.

Po r
r
dimension minimale que peut prendre E), on confectionne dans le Le corrigé est inachevé.

u
uor
i
F

nn
i

P
Á la prochaine

e r
er

m
m

or
or

F
✉: mamouni.myismail@gmail.com

115
✍ M AMOUNI M Y I SMAIL
MAMOUNI . NEW. FR

Devoir Libre
11 Coniques-Quadriques
Blague du jour

Salon de l’auto : Comment reconnaître les nationalités des visiteurs du Mon-


dial de l’Automobile ?
- L’Allemand examine le moteur
- L’Anglais examine le cuir
- Le Grec examine l’échappement
- L’Italien examine le Klaxon

Jacques Salomon Hadamard (1865-1963)

Mathématicien du jour
Mathématicien français, connu pour ses travaux en théorie des nombres et en cryptologie.
Il entra premier à l’école normale supérieure. C’est Émile Picard qui dirigea ses travaux
de recherches.
Son
 nom  est lié à la 
suite de matrices( H2k ) définie de la façon suivante : H1 = (1), H2 =
1 1 H2k−1 H2k−1
et H2k = . Elles sont utilisées dans les codes correcteurs, ou
1 −1 H2k−1 − H2k−1
encore pour réaliser les plans d’analyse sensorielle et les plans d’expériences factoriels.

Exercice 1 : Extrait e3a 2009, MP

On considère trois réels α, β et γ non nuls et la quadrique (Σ) dont une équation est :

α2 (x + y + z)2 + β2 (– x + y)2 + γ 2 (2y + z)2 = 1


r r r
dans un repère orthonormé R = (O, i , j , k ) d'un espace affine de dimension trois.

1) a) Montrer que les plans (P) et (Q) d'équations respectives x + y + z = 0 et – x + y = 0 sont orthogonaux.
r r
b) On note I et J les deux vecteurs unitaires, d'abscisses positives, normaux respectivement aux plans
r r r r r r
(P) et (Q). Déterminer les vecteurs I et J ainsi que le vecteur K tel que le repère R' = (O, I , J , K ) soit
orthonormé direct.

c) Déterminer une équation de la quadrique (Σ) dans le repère R'.


On notera X, Y et Z les coordonnées d'un point M dans le repère R'.

d) Que peut-on dire de la nature de la quadrique (Σ) ?

✉: mamouni.myismail@gmail.com ÉJ« AÖÞ @ ø BñÓ úG ñÜØ


✍ M AMOUNI M Y I SMAIL
MAMOUNI . NEW. FR

.
2) On note (E) la conique obtenue comme intersection de la quadrique (Σ) avec le plan d'équation Z = 0
dans le repère R'.
r r r
Réduire l'équation de la conique (E). En déduire qu'il existe un repère orthonormé R" = (O, I ' , J' , K' )
tel que l'équation dans le repère R" de la quadrique (Σ) soit de la forme :

X' 2 Y' 2
+ 2 =1
a2 b

avec 0 < a < b. Les coordonnées d'un point M dans le repère R" sont notées X', Y' et Z'.

3) Soit k un réel quelconque ; on appelle (Pk) le plan dont une équation dans le repère R" est :

X' b 2 − a 2 Z'
– = k.
ab b

a) En considérant l'expression :
X' 2 Y' 2 1
2
+ 2 – 2 (X'2 + Y'2 + Z'2)
a b b

montrer que l'intersection (Ck) du plan (Pk) et de la quadrique (Σ) est l'intersection du plan (Pk) avec une
sphère (Sk) dont on précisera le centre Ωk et le rayon Rk.

b) Montrer que l'ensemble (Ck) est un cercle dont on précisera le rayon.

c) Que peut-on dire du rayon du cercle (Ck) ? Pouvait-on le prévoir ?

ÉJ« AÖÞ @ ø BñÓ úG ñÜØ ✉: mamouni.myismail@gmail.com


✍ M AMOUNI M Y. I SMAIL
.
MAMOUNI NEW FR

. Exercice 2 : Extrait e3a 2004, MP

i
F
nn

i
✉: mamouni.myismail@gmail.com Á la prochaine ÉJ« AÖÞ @ ø BñÓ úG ñÜØ
MAMOUNI . NEW. FR
✍ M AMOUNI M Y I SMAIL
Corrigé Exericie 1, Pr. Dufait
 
−5 sin t
1. (i) 1
t 7→ M (t) est C sur [−π, π] et ∀t ∈ [−π, π], M (t) = 0
6= ~0 donc la tangente à E en M (t) est
3 cos t
x − 5 cos t −5 sin t
la droite d’équation = 0 soit (Tt ) : 3 cos t x + 5 sin t y = 15 .
y − 3 sin t 3 cos t


−→
(ii) On remarque que O ∈ / Tt donc, pour tout P ∈ Tt , la droite (OP ) existe dirigée par le vecteur OP 6= ~0. Le

−→ → −
point P est solution si et seulement si P ∈ Tt et OP ⊥ Tt c’est à dire si et seulement si P est le projeté
orthogonal de O sur Tt . D’où l’existence et l’unicité de P .

(iii) À un sommet S de E, la tangente à E est orthogonale à l’axe (OS). Comme S appartient à cette tangente,
si S est un sommet alors P = S .
     
−−→ X(t) 3 cos t X(t)
(iv) OP = est orthogonal à Tt donc colinéaire à donc il existe λt ∈ R tel que =
 Y (t)
 5 sin t Y (t)
3 cos t
λt et, d’autre part, P ∈ Tt donc 3 cos t X(t) + 5 sin t Y (t) = 15. On obtient λt 9 cos2 t +
5 sin t
 X(t) = 45 cos t 2

25 sin2 t = 15 soit λt =
 15 = 15 . Donc 9 + 16 sin t
9 cos2 t + 25 sin2 t 9 + 16 sin2 t  Y (t) = 75 sin t 2
9 + 16 sin t

2. (i) On a immédiatement :
π
t 0 2

X 0 (t) 0 −
X(t) 1 & 0

   
  3 donc ∃ !α ∈ 0, π  , Y 0(α) = 0 et on a α = arcsin 3 .
(ii)  t ∈ 0, π2 et Y 0 (t) = 0 sin t = 4 2 4
9 2 π 3 12 2 π 1 8 2 π 2 π
 sin α = 16 , sin 3 = 4 = 16 , sin 4 = 2 = 16 donc sin 4 < sin α < sin 3 et sin2 est croissante
2 2

sur 0, π2 donc π π
 
4 <α< 3 .
 X(α) = 5 √7

q √
3  π
 9
 On a sin α = 4 et α ∈ 0, 2 donc cos α = 1 − 16 = 4 d’où 7 8
 Y (α) = 25
8

(iii) On a donc :
π
t 0 α 2

Y 0 (t) + 0 − 0
25
Y (t) 0 % 8 & 3

( (
X(t + π) = −X(t) X(−t) = X(t)
donc il suffit d’étudier l’arc t 7−→ Pt sur 0, π2 , puis de
 
3. On a et
Y (t + π) = −Y (t) Y (−t) = −Y (t)

le compléter par une symétrie par rapport à O et une une symétrie orthogonale par rapport à O,~ı .

✉: mamouni.myismail@gmail.com ÉJ« AÖÞ @ ø BñÓ úG ñÜØ


1
MAMOUNI . NEW. FR
✍ M AMOUNI M Y I SMAIL
.
3

-4 -2 0 2 4

-1

-2

-3

4. Question Hors Programme


Notons S le sommet S = O +~ı de E. À l’instant θ = 0, il coı̈ncide avec le sommet S 0 de E 0. La condition
de roulement sans glissement est qu’à l’instant θ les deux ellipses sont en contact en M ∈ E et M 0 ∈ E 0
_ _
tels que la longueur de l’arc SM soit égale à celle de l’arc S 0 M 0 . Si on prend comme paramétrage de E 0
celui obtenu par la translation τ , c’est à dire, par exemple, que S 0 = M 0(π), on a donc que, si à l’instant
θ le contact a lieu en M (t) sur E alors
 il a lieu en M 0 (π − t) sur E 0. Le point M (π − t) étant symétrique
du point M (t) pra rapport à O, ~ , l’angle entre les droites (OM (π − t)) et Tπ−t est l’opposé, modulo π,
de l’angle entre les droites (OM (t)) et Tt . D’autre part, c’est aussi l’angle entre les droites (O 0 M 0(π − t))
0 0
et Tπ−t . Comme Tt = Tπ−t à l’instant θ, le droites (OM (t)) et (O 0 M 0(π − t)) sont symétriques l’une de
l’autre par rapport à Tt , donc en particulier, O 0 est, à l’instant θ, le symétrique de O par rapport à Tt . On
−−→ −
−→
a donc OO0 = 2 OP et donc le lieu de O 0 est l’image de F par l’homothétie de centre O et rapport 2 .

illustration :
10

-10 -5 0 5 10

-2

-4

-6

ÉJ« AÖÞ @ ø BñÓ úG ñÜØ ✉: mamouni.myismail@gmail.com

2
MAMOUNI . NEW. FR
✍ M AMOUNI M Y I SMAIL
Corrigé Exericie 2, Mma Gayout
r r
1) a) Un vecteur normal au plan (P) est n (1, 1, 1) et un vecteur normal au plan (Q) est m (– 1, 1, 0).
r r
On a : n . m = 0. Donc les plans (P) et (Q) sont orthogonaux.

r 3 3 3 r 2 2 r r r r 6 6 6
b) On prend I ( , , ), J ( ,– , 0) et K = I ∧ J , soit K ( , ,– ).
3 3 3 2 2 6 6 3
r r r
c) D'après les formules de changement de bases, en notant P la matrice de passage de la base ( i , j , k )
 3 2 6
x = X+ Y+ Z
 x  
X  3 2 6
r r r      3 2 6
à la base ( I , J , K ), on a :  y  = P×  Y  , soit  y = X− Y+ Z et on a aussi :
z  Z  3 2 6
     z 3 6
 = X − 2× Z
 3 6
 3
X= ( x + y + z)
 
X  
x  3
  t    2
 Y  = P×  y  , soit  Y = ( x − y) .
 Z z  2
    Z = 6 ( x + y − 2 z )
 6

Donc (x + y + z) 2 = 3 X 2, (– x + y) 2 = 2 Y2 et (2 y + z) 2 = ( 3 X – 2 Y) 2.
Une équation de (Σ) dans le repère R' est donc : 3 α 2 X 2 + 2 β 2 Y2 + γ 2( 3 X – 2 Y) 2 = 1.
r
d) On en déduit que (Σ) est un cylindre d'axe dirigé par le vecteur K .

2) (E) a pour équation : 3 α 2 X 2 + 2 β 2 Y2 + γ 2( 3 X – 2 Y) 2 = 1, soit


3(α2 + γ 2) X 2 – 2 γ2 6 XY + 2(β2 + γ2) Y 2 = 1.
La matrice associée à la forme quadratique q(X, Y) = 3(α2 + γ 2) X 2 – 2 γ2 6 XY + 2(β2 + γ2) Y 2
 3(α 2 + γ 2 ) − 6γ 2 
est : M =  2 2 2 
.
 − 6γ 2 ( β + γ ) 
Par le théorème spectral, on sait que M admet deux valeurs propres réelles, que les sous-espaces propres
associés à ces valeurs propres sont orthogonaux et que M est diagonalisable dans une base orthonormale
r r
( I ' , J' ) constituée de vecteurs propres.
1
Après calculs, on trouve comme valeurs propres : a' = (3α 2 + 2 β 2 + 5γ 2 + θ )
2
1
et b' = (3α 2 + 2 β 2 + 5γ 2 − θ ) où θ = 9α4 + 4β4 + 25 γ4 – 12 α2β2 – 4 β2γ2 + 6 α2γ2.
2
r
Un vecteur propre associé à a' est U ( 6 γ2, 3(α2 + γ 2) – a') et un vecteur propre associé à b' est
r r r
V ( 6 γ2, 3(α2 + γ 2) – b') dans la base ( I , J ).
r 1 r r 1 r
On pose : I ' = r U et J' = r V . On sait de plus que : a' + b' = Tr(M) = 3α2 + 2β2 + 5 γ 2 > 0
U V
et que a'b' = det(M) = 6(α2β2 + α2γ2 + β2γ2) > 0. Donc on a : 0 < b' < a'.
r r ✉ : mamouni.myismail@gmail.com ÉJ« AÖÞ @ ø BñÓ úG ñÜØ
Dans la repère orthonormal (O, I ' , J' ), (E) a pour équation réduite : a' X'2 + b' Y'2 = 1.
(E) est donc une ellipse et (Σ) un cylindre elliptique.3
MAMOUNI . NEW. FR
✍ M AMOUNI M Y I SMAIL
r r
.
1 1
On pose K' = K , a = et b = . On a : 0 < a < b et l'équation de (Σ) dans le repère orthonormal
a' b'
r r r X' 2 Y' 2
R" = (O, I ' , J' , K' ) est alors : 2 + 2 = 1.
a b

3) a) On a :
X' 2 Y' 2 1 2 2 2 b2 − a2 2 1 2 X' b 2 − a 2 Z' X' b 2 − a 2 Z'
(*) + – (X' + Y' + Z' ) = X' – Z' = ( – )( + ).
a2 b2 b2 a 2b 2 b2 ab b ab b
Soit M∈(Ck) ; alors ses coordonnées (X', Y', Z') dans le repère R" vérifient :
X' 2 Y' 2 X' b 2 − a 2 Z'
+ = 1 et – = k.
a2 b2 ab b
1 2 2 2 X' b 2 − a 2 Z'
Donc, en remplaçant dans (*), on obtient (**) : 1 – (X' + Y' + Z' ) = k ( + ), soit
b2 ab b
kb b 2 − a 2 kb b 2 − a 2 2 kb 2 b 2 ( k 2 b 2 + 4a 2 )
X'2 + Y'2 + Z'2 + X' + kb Z' = b2, soit (X' + ) + Y'2 + (Z' + ) = .
a 2a 2 4a 2
kb b 2 − a 2 kb
Donc M appartient aussi à la sphère (Sk) de centre Ωk de coordonnées (– , 0, – ) dans le repère
2a 2
b k 2 b 2 + 4a 2
R" et de rayon Rk = .
2a

X' b 2 − a 2 Z'
Réciproquement, si M∈(Sk) ∩ (Pk), on a la relation (**) et – = k, d'où :
ab b
1 2 2 2 X' b 2 − a 2 Z' X' b 2 − a 2 Z' X' 2 Y' 2 1
1– 2
(X' + Y' + Z' ) = ( – )( + ) = 2
+ 2 – 2 (X'2 + Y'2 + Z'2), donc
b ab b ab b a b b
2 2
X' Y'
+ 2 = 1. Donc M∈(Σ). D'où l'égalité : (Ck) = (Pk) ∩ (Σ) = (Pk) ∩ (Sk).
a2 b

b) En tant qu'intersection d'un plan et d'une sphère, (Ck) est soit vide, soit un point, soit un cercle.
C'est un cercle ssi la distance d de Ωk au plan (Pk) est strictement inférieure à Rk.
Dans ce cas, d'après le théorème de Pythagore, le rayon de (Ck), noté rk,est égal à : R 2k − d 2 .
k (b 2 − a 2 ) k
− + −k
b k 2 b 2 + 4a 2 2a 2 2 kb 2
On a : Rk = et d = d(Ωk, (Pk)) = = .
2a b2 − a2 1 2a
+ 2
a 2b 2 b
D'où Rk – d =
b
2a
( )
k 2 b 2 + 4a 2 − kb > 0 car a > 0. Donc (Ck) est un cercle de rayon rk = b.

c) Le rayon du cercle (Ck) est égal à la demi-longueur du grand axe de l'ellipse (E).

On pouvait le prévoir puisque l'ellipse (E) est l'image du cercle (Ck) par la projection orthogonale sur le plan
d'équation Z' = 0.
r
Or, (Ck) est inclus dans le plan (Pk) dont un vecteur directeur est le vecteur J' aussi vecteur directeur du plan
r
Z' = 0. Donc un diamètre de (Ck) dirigé selon J' a sa longueur inchangée par la projection orthogonale
ÉJ« AÖÞ @ ø BñÓ úG ñÜØ
considérée.
✉ : mamouni.myismail@gmail.com
4
✍ M AMOUNI M Y I SMAIL
MAMOUNI . NEW. FR

DL
12 Fonctions harmoniques
Blague du jour

Pour une question bien précise :


• Un ingénieur pense que ses équations sont une approximations de la réal-
ité.
• Un physiciens pense que la réalité est une approximation de ses équations.
• Un mathématicien s’en moque.

John Machin (1680 - 1751)

Mathématicien du jour
Mathématicien anglais, connu principalement pour avoir calculé en 1706, les 100 déci-
π 1
males de π grâce à la formule qui porte son nom : = 4 arctan − arctan 1239. Malgré
4 5
un nom de famille très drôle, John Machin a enseigné les mathématiques à Brook Tay-
lor. Il était aussi professeur d’astronomie, secrétaire de la Royal Society et membre de la
commission qui décida de la priorité de Calcul entre Leibniz et Newton en 1712.

Énoncé : Mines-Ponts 2004, MP

Soit f une fonction à valeurs réelles ou complexes, définie dans un ouvert U du plan R2 , deux fois
continûment dérivable ; le laplacien de la fonction f est, par définition, la fonction, notée ∆f , définie
dans l’ouvert U par la relation suivante :

∂2f ∂2f
∆f (x, y) = 2
(x, y) + 2 (x, y) .
∂x ∂y

Une fonction f à valeurs réelles ou complexes, définie dans un ouvert U du plan R2 , deux fois con-
tinûment dérivable, est harmonique dans U si et seulement si son laplacien est nul dans U :

∂2f ∂2f
∆f (x, y) = (x, y) + (x, y) = 0.
∂x2 ∂y 2
Exemple : en électrostatique, le potentiel électrique dans le vide est harmonique.

✉: mamouni.myismail@gmail.com ÉJ« AÖÞ @ ø BñÓ úG ñÜØ


1
✍ M AMOUNI M Y I SMAIL MAMOUNI . NEW. FR

.
Le but du problème est de donner des exemples de telles fonctions puis de démontrer certaines pro-
priétés de ces fonctions : le principe du maximum, la propriété de moyenne, le fait que les fonctions
bornées harmoniques dans tout le plan sont constantes.
Le plan R2 est supposé muni de la norme euclidienne.
Quelques exemples de fonctions harmoniques :
1. Démontrer que les fonctions complexes f et gn , n ∈ N, définies dans le plan R2 par les relations
ci-dessous, sont harmoniques :

n
f (x, y) = ex + i y
, gn (x, y) = (x + i y) .

2. Déterminer les fonctions u réelles, de classe C2 , définies sur la demi-droite


 ouverte ]0, ∞[ , telles
que chaque fonction h, définie dans le plan R2 privé du point O R2 \ {O} par la relation ci-dessous,
soit harmonique

p 
h (x, y) = u x2 + y 2 .
p
Poser si nécessaire : r = x2 + y 2 .

3. Déterminer les fonctions v réelles, de classe C2 , définies sur la droite réelle R, telles que chaque
fonction k, définie dans le plan R2 privé de l’axe y´Oy R2 \ y´Oy par la relation ci-dessous, soit har-
monique.

y
k (x, y) = v .
x

Soit la suite (un )n∈N de fonctions définies dans tout le plan R2 par les relations suivantes :

n
n (x + iy)
un (x, y) = (−1) .
(2n)!

4. Soit K un ensemble fermé borné quelconque du plan R2 ; démontrer que la restriction un|K de la
fonction un au fermé K est le terme général d’une série de fonctions uniformément convergente.
En déduire que la série de fonctions de terme général un converge en tout point du plan et que sa
somme, la fonction ϕ, définie par la relation suivante


X
ϕ (x, y) = un (x, y) ,
n=0

est continue dans le plan.

5. Démontrer que cette fonction ϕ est harmonique dans tout le plan R2 .

ÉJ« AÖÞ @ ø BñÓ úG ñÜØ ✉: mamouni.myismail@gmail.com

2
✍ M AMOUNI M Y I SMAIL
MAMOUNI . NEW. FR

.
Principe du maximum :
Soit f une fonction réelle harmonique définie dans tout le plan R2 . Soit D le disque fermé de centre
O et de rayon strictement positif r (r > 0) ; soit C le cercle de centre O et de rayon r :

(x, y) | x2 + y 2 ≤ r2 ,

D =
(x, y) | x2 + y 2 = r2 .

C =

Étant donné un entier strictement positif p (p > 0) , soit fp la fonction définie dans R2 par la relation
suivante :

x2 + y 2
fp (x, y) = f (x, y) + .
p
6. Démontrer l’existence d’un point Mp de coordonnées ap et bp , appartenant au disque fermé D en
lequel la fonction fp atteint son maximum :

fp (ap , bp ) = max fp (x, y) .


(x,y)∈D

7. Démontrer que, si le point Mp appartient à l’intérieur du disque D, les deux dérivées secondes de
la fonction fp , obtenues en dérivant deux fois par rapport à x ou deux fois par rapport à y, sont, en ce
point Mp , négatives ou nulles :

∂ 2 fp ∂ 2 fp
(ap , b p ) ≤ 0 ; (ap , bp ) ≤ 0.
∂x2 ∂y 2
8. En déduire, en calculant par exemple le laplacien de la fonction fp , que le point Mp est situé sur
le cercle C.

9. Démontrer qu’il existe un point P de coordonnées a et b du cercle C en lequel la fonction f atteint


son maximum sur D :

f (a, b) = max f (x, y) .


(x,y)∈D

10. En déduire que deux fonctions harmoniques dans le plan R2 égales le long d’un cercle C du plan
(de rayon strictement positif), sont égales dans tout le disque D de frontière C.

Propriété de la moyenne

Soit f une fonction réelle harmonique définie dans le plan R2 . Étant donnés un point M0 de coor-
données x0 et y0 et un réel ρ positif ou nul, soit F la fonction définie sur la demi-droite fermée [0, ∞[ par
la relation suivante :

Z 2π
F (ρ) = f (x0 + ρ cos θ, y0 + ρ sin θ) dθ.
0

✉: mamouni.myismail@gmail.com ÉJ« AÖÞ @ ø BñÓ úG ñÜØ


3
✍ M AMOUNI M Y I SMAIL MAMOUNI . NEW. FR

.
11. Démontrer que la fonction F est définie et continue sur la demi-droite fermée [0, ∞[.

12. Démontrer que la fonction F est continûment dérivable. Préciser sa dérivée F ´(ρ).

13. Démontrer que le produit ρ.F ´(ρ) est égal à la valeur d’une intégrale curviligne d’une forme
différentielle α = A (x, y) dx + B (x, y) dy le long d’un arc orienté Γ :

Z
ρ.F ´(ρ) = (A (x, y) dx + B (x, y) dy) .
Γ

Préciser la forme différentielle α et l’arc orienté Γ.

14. Démontrer que la fonction F est une fonction constante ; préciser sa valeur.

15. Soit D le disque fermé de centre le point M0 de coordonnées (x0 , y0 ) et de rayon r (r > 0)
; démontrer que l’intégrale double I de la fonction f étendue au disque D se calcule simplement en
fonction de f (x0 , y0 ) suivant la relation :

ZZ
I= f (x, y) dx dy = π r2 f (x0 , y0 ) .
D

Fonctions harmoniques bornées dans le plan :


Soit f une fonction définie dans tout le plan, réelle, harmonique et bornée : il existe donc une constante
C telle qu’en tout point (x, y) du plan :

|f (x, y)| ≤ C.

16. Soient deux disques fermés D1 et D2 de centres, distincts l’un de l’autre, O et M0 , de coordonnées
respectives
p (0, 0) et (x0 , y0 ). Soit r le rayon commun de ces disques. La distance d des centres O et M0
(égale à x20 + y02 ) est supposée strictement inférieure au rayon r (0 < d < r). Soit L2 l’ensemble des
points du disque D2 qui ne sont pas dans le disque D1 .

En considérant par exemple un disque contenu dans l’intersection des disques D1 et D2 , démontrer
que l’aire de L2 est majorée par l’expression π r d.

17. À l’aide par exemple de la question 15, donner un majorant de la valeur absolue de la différence
f (x0 , y0 ) − f (0, 0) au moyen de la constante C, du rayon r et de d.
En déduire que la fonction f est constante.

i
F
nn

i
Á la prochaine

ÉJ« AÖÞ @ ø BñÓ úG ñÜØ ✉: mamouni.myismail@gmail.com

4
✍ M AMOUNI M Y I SMAIL
MAMOUNI . NEW. FR

DL
12 Fonctions harmoniques
Blague du jour

Pour une question bien précise :


• Un ingénieur pense que ses équations sont une approximations de la réal-
ité.
• Un physiciens pense que la réalité est une approximation de ses équations.
• Un mathématicien s’en moque.

John Machin (1680 - 1751)

Mathématicien du jour
Mathématicien anglais, connu principalement pour avoir calculé en 1706, les 100 déci-
π 1
males de π grâce à la formule qui porte son nom : = 4 arctan − arctan 1239. Malgré
4 5
un nom de famille très drôle, John Machin a enseigné les mathématiques à Brook Tay-
lor. Il était aussi professeur d’astronomie, secrétaire de la Royal Society et membre de la
commission qui décida de la priorité de Calcul entre Leibniz et Newton en 1712.

Énoncé : Mines-Ponts 2004, MP

Soit f une fonction à valeurs réelles ou complexes, définie dans un ouvert U du plan R2 , deux fois
continûment dérivable ; le laplacien de la fonction f est, par définition, la fonction, notée ∆f , définie
dans l’ouvert U par la relation suivante :

∂2f ∂2f
∆f (x, y) = 2
(x, y) + 2 (x, y) .
∂x ∂y

Une fonction f à valeurs réelles ou complexes, définie dans un ouvert U du plan R2 , deux fois con-
tinûment dérivable, est harmonique dans U si et seulement si son laplacien est nul dans U :

∂2f ∂2f
∆f (x, y) = (x, y) + (x, y) = 0.
∂x2 ∂y 2
Exemple : en électrostatique, le potentiel électrique dans le vide est harmonique.

✉: mamouni.myismail@gmail.com ÉJ« AÖÞ @ ø BñÓ úG ñÜØ


1
✍ M AMOUNI M Y I SMAIL MAMOUNI . NEW. FR

.
Le but du problème est de donner des exemples de telles fonctions puis de démontrer certaines pro-
priétés de ces fonctions : le principe du maximum, la propriété de moyenne, le fait que les fonctions
bornées harmoniques dans tout le plan sont constantes.
Le plan R2 est supposé muni de la norme euclidienne.
Quelques exemples de fonctions harmoniques :
1. Démontrer que les fonctions complexes f et gn , n ∈ N, définies dans le plan R2 par les relations
ci-dessous, sont harmoniques :

n
f (x, y) = ex + i y
, gn (x, y) = (x + i y) .

2. Déterminer les fonctions u réelles, de classe C2 , définies sur la demi-droite


 ouverte ]0, ∞[ , telles
que chaque fonction h, définie dans le plan R2 privé du point O R2 \ {O} par la relation ci-dessous,
soit harmonique

p 
h (x, y) = u x2 + y 2 .
p
Poser si nécessaire : r = x2 + y 2 .

3. Déterminer les fonctions v réelles, de classe C2 , définies sur la droite réelle R, telles que chaque
fonction k, définie dans le plan R2 privé de l’axe y´Oy R2 \ y´Oy par la relation ci-dessous, soit har-
monique.

y
k (x, y) = v .
x

Soit la suite (un )n∈N de fonctions définies dans tout le plan R2 par les relations suivantes :

n
n (x + iy)
un (x, y) = (−1) .
(2n)!

4. Soit K un ensemble fermé borné quelconque du plan R2 ; démontrer que la restriction un|K de la
fonction un au fermé K est le terme général d’une série de fonctions uniformément convergente.
En déduire que la série de fonctions de terme général un converge en tout point du plan et que sa
somme, la fonction ϕ, définie par la relation suivante


X
ϕ (x, y) = un (x, y) ,
n=0

est continue dans le plan.

5. Démontrer que cette fonction ϕ est harmonique dans tout le plan R2 .

ÉJ« AÖÞ @ ø BñÓ úG ñÜØ ✉: mamouni.myismail@gmail.com

2
✍ M AMOUNI M Y I SMAIL
MAMOUNI . NEW. FR

.
Principe du maximum :
Soit f une fonction réelle harmonique définie dans tout le plan R2 . Soit D le disque fermé de centre
O et de rayon strictement positif r (r > 0) ; soit C le cercle de centre O et de rayon r :

(x, y) | x2 + y 2 ≤ r2 ,

D =
(x, y) | x2 + y 2 = r2 .

C =

Étant donné un entier strictement positif p (p > 0) , soit fp la fonction définie dans R2 par la relation
suivante :

x2 + y 2
fp (x, y) = f (x, y) + .
p
6. Démontrer l’existence d’un point Mp de coordonnées ap et bp , appartenant au disque fermé D en
lequel la fonction fp atteint son maximum :

fp (ap , bp ) = max fp (x, y) .


(x,y)∈D

7. Démontrer que, si le point Mp appartient à l’intérieur du disque D, les deux dérivées secondes de
la fonction fp , obtenues en dérivant deux fois par rapport à x ou deux fois par rapport à y, sont, en ce
point Mp , négatives ou nulles :

∂ 2 fp ∂ 2 fp
(ap , b p ) ≤ 0 ; (ap , bp ) ≤ 0.
∂x2 ∂y 2
8. En déduire, en calculant par exemple le laplacien de la fonction fp , que le point Mp est situé sur
le cercle C.

9. Démontrer qu’il existe un point P de coordonnées a et b du cercle C en lequel la fonction f atteint


son maximum sur D :

f (a, b) = max f (x, y) .


(x,y)∈D

10. En déduire que deux fonctions harmoniques dans le plan R2 égales le long d’un cercle C du plan
(de rayon strictement positif), sont égales dans tout le disque D de frontière C.

Propriété de la moyenne

Soit f une fonction réelle harmonique définie dans le plan R2 . Étant donnés un point M0 de coor-
données x0 et y0 et un réel ρ positif ou nul, soit F la fonction définie sur la demi-droite fermée [0, ∞[ par
la relation suivante :

Z 2π
F (ρ) = f (x0 + ρ cos θ, y0 + ρ sin θ) dθ.
0

✉: mamouni.myismail@gmail.com ÉJ« AÖÞ @ ø BñÓ úG ñÜØ


3
✍ M AMOUNI M Y I SMAIL MAMOUNI . NEW. FR

.
11. Démontrer que la fonction F est définie et continue sur la demi-droite fermée [0, ∞[.

12. Démontrer que la fonction F est continûment dérivable. Préciser sa dérivée F ´(ρ).

13. Démontrer que le produit ρ.F ´(ρ) est égal à la valeur d’une intégrale curviligne d’une forme
différentielle α = A (x, y) dx + B (x, y) dy le long d’un arc orienté Γ :

Z
ρ.F ´(ρ) = (A (x, y) dx + B (x, y) dy) .
Γ

Préciser la forme différentielle α et l’arc orienté Γ.

14. Démontrer que la fonction F est une fonction constante ; préciser sa valeur.

15. Soit D le disque fermé de centre le point M0 de coordonnées (x0 , y0 ) et de rayon r (r > 0)
; démontrer que l’intégrale double I de la fonction f étendue au disque D se calcule simplement en
fonction de f (x0 , y0 ) suivant la relation :

ZZ
I= f (x, y) dx dy = π r2 f (x0 , y0 ) .
D

Fonctions harmoniques bornées dans le plan :


Soit f une fonction définie dans tout le plan, réelle, harmonique et bornée : il existe donc une constante
C telle qu’en tout point (x, y) du plan :

|f (x, y)| ≤ C.

16. Soient deux disques fermés D1 et D2 de centres, distincts l’un de l’autre, O et M0 , de coordonnées
respectives
p (0, 0) et (x0 , y0 ). Soit r le rayon commun de ces disques. La distance d des centres O et M0
(égale à x20 + y02 ) est supposée strictement inférieure au rayon r (0 < d < r). Soit L2 l’ensemble des
points du disque D2 qui ne sont pas dans le disque D1 .

En considérant par exemple un disque contenu dans l’intersection des disques D1 et D2 , démontrer
que l’aire de L2 est majorée par l’expression π r d.

17. À l’aide par exemple de la question 15, donner un majorant de la valeur absolue de la différence
f (x0 , y0 ) − f (0, 0) au moyen de la constante C, du rayon r et de d.
En déduire que la fonction f est constante.

i
F
nn

i
Á la prochaine

ÉJ« AÖÞ @ ø BñÓ úG ñÜØ ✉: mamouni.myismail@gmail.com

4
i

r
s
P ROBLÈMES C ORRIGÉS -MP ✍ M AMOUNI M Y I SMAIL
MAMOUNI . NEW. FR

i
s

s
u

s
Devoir Libre
13 Courbes & Surfaces

u
Joseph Liouville (1809-1882)

r
Blague du jour

Mathématicien du jour
☛ Qu’est ce qu’un taureau avec un sac à main ? Mathématicien français. Diplômé de l’École Polytechnique et
- Un vache folle de l’École des ponts et chaussées, il préfère suivre une carrière

Po r
r
☛ Qu’ce qu’est un oiseau migrateur ? académique plutôt qu’une carrière d’ingénieur. Liouville pub-
- C’est un oiseau qui se gratte que d’un côté. lia dans divers domaines des mathématiques, dont la théorie

u
☛ Qu’est-ce qu’on obtient si on croise un pitbull et un des nombres, l’analyse complexe, la géométrie différentielle et

uor
yorkshire ? la topologie différentielle, mais aussi la physique mathématique
- Un yorkshire mort... et même l’astronomie. Il fut le premier à reconnaitre les travaux
inédits d’Évariste Galois

P
et préciser la nature de cette courbe.
Énoncé : e3a 2009, PSI
ii Comparer C1 avec la courbe paramétrée par h, c’est-à-

r
On rappelle les deux formules usuelles de trigonométrie, pour t ∈ R : dire :
{(cos2 (t), cos(t) sin(t)), t ∈ R }.

e
cos(2t) = 2 cos2 (t) − 1 = 1 − 2 sin2 (t) sin(2t) = 2 sin(t) cos (t).
b Soit f 2 : R −→ R 2 avec f 2 (t) = (cos2 (t), sin (t)).
er
① On considère l’espace vectoriel réel usuel R 2 muni de son

m
produit scalaire canonique tel que la base canonique B soit Étudier et représenter la courbe C2 paramétrée par f 2 , c’est-à-
orthonormale. dire :
a Soit h : R −→ R 2 définie par : h(t) = C2 = {(cos2 (t), sin (t)), t ∈ R }.
m

Pour cela, on commencera par comparer C2 avec la courbe


(cos2 (t), cos (t) sin(t)).

r
d’équation dans B :
i Représenter la courbe C1 d’équation dans B : x + y2 = 1, avec − 1 ≤ y ≤ 1,

o
(2x − 1)2 + (2y)2 = 1, et préciser la nature de cette courbe.
or

F
✉: mamouni.myismail@gmail.com

130
i

r
s
✍ M AMOUNI M Y I SMAIL
MAMOUNI . NEW. FR
P ROBLÈMES C ORRIGÉS -MP

i
s

s
u
c Soit f 3 : R −→ R 2 avec f 3 (t) = (cos(t) sin (t), sin (t)). tout point régulier M0 = ( x0 , y0 , z0 ) de S2 .

s
Étudier et représenter la courbe C3 paramétrée par f 3 , c’est-à- En déduire la tangente en tout point M0 = ( x0 , y0 , z0 ) régulier

é
dire : de Γ.

u
C3 = {(cos(t) sin (t), sin (t)), t ∈ R }. d Déterminer un paramétrage de Γ, en utilisant les coor-
Montrer que C3 est la courbe d’équation dans B : (2x )2 + (1 −

r
données cylindriques : c’est-à-dire que l’on exprimera pour
2y2 )2 = 1.

é
M = ( x, y, z) = (r cos(θ ), r sin(θ ), z) les conditions sur r, θ, z
② On considère l’espace vectoriel réel usuel R 3 orienté, muni pour que M soit sur Γ.

Po r
r
de son produit scalaire canonique tel que la base canonique C En déduire une représentation paramétrique du cône de som-
soit orthonormale directe. 1

u
a Soit S1 = {( x, y, z) ∈ R 3 /x2 − x + y2 = 0} et S2 = met S = ( , 0, 0), engendré par les droites passant par S et un
2

uor
point variable sur Γ.
{( x, y, z) ∈ R 3 /x2 + y2 + z2 = 1}. Préciser la nature des deux
surfaces S1 et S2 . e Pour t ∈ R, on pose : F(t) = (cos2 (t), cos(t) sin (t), sin (t)).
  
3 x = x 2 + y2 Soit γ = { F(t), t ∈ R }. Montrer que γ ⊂ Γ. Y-a-t-il égalité
b Soit Γ = ( x, y, z) ∈ R / .
x 2 + y 2 + z2 = 1 γ = Γ?

P
Que représente Γ vis-à-vis de S1 et S2 ? f Préciser comment on obtient les trois courbes planes qui
c Déterminer l’équation dans C du plan tangent en tout sont les projections orthogonales de Γ sur les plans xOy, xOz
point régulier M0 = ( x0 , y0 , z0 ) de S1 . et yOz, en faisant le lien avec les courbes étudiées dans la pre-

r
De même déterminer l’équation dans C du plan tangent en mière question.

e
er

m
i
F nn
i
m

r
Á la prochaine

o
or

F
✉: mamouni.myismail@gmail.com

131
i

r
s
P ROBLÈMES C ORRIGÉS -MP ✍ M AMOUNI M Y I SMAIL
MAMOUNI . NEW. FR

i
s

s
u
Corrigé : Pr Skler, CPGE France

s
é

u
① On considère l’espace vectoriel réel usuel R2 muni de son θ
on pose t = , on a alors 2x − 1 = cos (2t) et 2y = sin (2t) ,
produit scalaire canonique tel que la base canonique B soit 2
d’où

r
orthonormale.
x = cos2 (t) et y = cos (t) sin (t)

é
a Soit h : R −→ R2 définie par : h(t) = Tout point du cercle C1 est aussi un point de la courbe
2
(cos (t), cos (t) sin(t)). paramétrée par h.

Po r
r
Conclusion : C1 est le support de la courbe paramétrée par h
i Représenter la courbe C1 d’équation dans B :

u
(2x − 1)2 + (2y)2 = 1

uor
et préciser la nature de cette courbe.
Correction : on a b Soit f 2 : R −→ R2 avec f 2 (t) = (cos2 (t), sin (t)).
 
2 2 1 2 1 Étudier et représenter la courbe C2 paramétrée par f 2 , c’est-à-
(2x − 1) + (2y) = 1 ⇔ x − + y2 =
2 4 dire :

P
c’est
 l’équation
 du cercle de centre le point de coordonnées C2 = {(cos2 (t), sin (t)), t ∈ R }.
1 1 Pour cela, on commencera par comparer C2 avec la courbe
, 0 et de rayon .
2 2 d’équation dans B :
ii Comparer C1 avec la courbe paramétrée par h, c’est-à- x + y2 = 1, avec − 1 ≤ y ≤ 1,

r
dire : et préciser la nature de cette courbe.
{(cos2 (t), cos (t) sin(t)), t ∈ R }. Correction : soit t ∈ IR on a

e
cos2 (t) + (sin (t))2 = 1 et − 1 ≤ sin (t) ≤ 1
er
Correction : on a pour tout t ∈ IR donc la courbe C2 est incluse dans la courbe définie par
x + y2 = 1, avec−1 ≤ y ≤ 1.

m
 2
2 cos (t) − 1 + (2 sin (t) cos (t))2 = (cos (2t))2 + (sin (2t))2 = 1
2
Réciproquement : soit un point de coordonnées ( x, y) tel que
On en déduit que la courbe paramétrée par h est incluse dans x + y2 = 1, avec−1 ≤ y ≤ 1. Comme y ∈ [−1, 1] il existe
t ∈ IR tel que y = sin (t) . On a alors x = 1 − y2 = cos2 (t) . La
m

C1 .
courbe C2 est donc définie par x + y2 = 1, avec−1 ≤ y ≤ 1.

r
Réciproque : soit M un point de coordonnées ( x, y) appar-
tenant à C1 , on a C’est une portion de parabole.

o
(2x − 1)2 + (2y)2 = 1 ⇔ θ ∈ IR / 2x − 1 = cos (θ ) et 2y = sin (θ ) Etude de la courbe paramétrée : f 2 est 2π périodique. On a
or

F
✉: mamouni.myismail@gmail.com

132
i

r
s
✍ M AMOUNI M Y I SMAIL
MAMOUNI . NEW. FR
P ROBLÈMES C ORRIGÉS -MP

i
s

s
u
de plus   Correction : la fonction f 3 est 2π périodique. On a

s
2 ∀t ∈ [−π, π ] f 3 (t) = − f 3 (t)
∀t ∈ IR f 2 (−t) = cos (t) , − sin (t)
la courbe admet donc O comme centre de symétrie. On peut

é
la courbe admet l’axe des abscisses comme axe de symétrie.

u
restraindre l’étude à [0, π ]
On peut donc rerstraindre l’étude à [0, π ] , cependant on peut
De plus

r
remarquer que
∀t ∈ [0, π ] f 3 (π − t) = (− cos (t) sin (t) , sin (t))

é
∀t ∈ [0, π ] f 2 (π − t) = f 2 (t)
la courbe admet donc l’axe des ordonnées comme h π iaxe de
La courbe définie pour t ∈ [0, π ]hest donc parcourue 2 fois on
πi symétrie. On peut donc restraindre l’étude à 0, . Avec

Po r
2

r
peut donc restraindre l’étude à 0, avant la symétrie par  
2 1
rapport à l’axe des abscisses. f ( t) = sin (2t) , sin (t) on a

u
2 h πi
La fonction hf 2 estide classe C∞ sur IR et on a
f 3′ (t) = (cos (2t) , cos (t))

uor
π ∀t ∈ 0,
∀t ∈ 0, f 2′ (t) = (−2 sin (t) cos (t) , cos (t)) 2
2 d’où le tableau de variations
d’où le tableau de variation : π π
π t 0
t 0 4 2
2 x3′ (t) + 0 −

P
x2′ (t) 0 − 0 1
1
2
x2 ( t ) ց x3 ( t ) ր ց
0 0 0

r
1 1

y2 ( t ) ր 2

e
0 y3 ( t ) ր ր
2
y2′ (t)
er
+ 0 0

m

y3 ( t ) + +
c Soit f 3 : R −→ R2 avec f 3 (t) = (cos(t) sin (t), sin (t)).  2
∀t ∈ IR, (2x3 (t)) 2 + 1 − 2y3 (t)2 = sin2 (2t) +
Étudier et représenter la courbe C3 paramétrée par f 3 , c’est-à-  2
m

dire : 1 − 2 sin2 (t) = sin2 (2t) + cos2 (2t) = 1. La courbe C3 est

r
C3 = {(cos(t) sin (t), sin(t)), t ∈ R }.
Montrer que C3 est la courbe d’équation dans B : (2x )2 + (1 − bien incluse dans la courbe définie par (2x )2 + (1 − 2y2 )2 = 1.
Soit un point de coordonnées ( x, y) appartenant à cette

o
2y2 )2 = 1.
or

F
✉: mamouni.myismail@gmail.com

133
i

r
s
P ROBLÈMES C ORRIGÉS -MP ✍ M AMOUNI M Y I SMAIL
MAMOUNI . NEW. FR

i
s

s
u
courbe, f ( x, y, z) =0 est
−−−→ 

s
∃θ ∈ IR / 2x = sin(θ ) et 1 − 2y2 = cos(θ ) Vect grad ( f ) ( x0 , y0 , z0 ) | M0 M = 0
1

é
On pose t = θ et on obtient x = cos(t) sin (t) et y2 = sin2 (t). ou encore

u
2 ∂f ∂f ∂f
Si y = sin (t) alors on obtient bien un point de C3 . Si ( x 0 , y 0 , z0 ) ( x − x 0 ) + ( x 0 , y 0 , z0 ) ( y − y 0 ) + ( x 0 , y 0 , z0 ) ( z − z0 ) =
∂x ∂y ∂z

r
y = − sin (t) on a y = sin (t + π ) et x = cos(t) sin (t) = On trouve donc : pour S1

é
cos(t + π ) sin (t + π ). c’est encore un point de C3 . Donc C3 est (2x0 − 1) .( x − x0 ) + 2y0 .(y − y0 ) = 0
bien la courbe définie par l’équation (2x )2 + (1 − 2y2 )2 = 1 et donc, avec M0 = ( x0 , y0 , z0 ) ∈ S1

Po r
r
② On considère l’espace vectoriel réel usuel R3 orienté, muni (2x0 − 1) .x + 2y0 .y + 2x0 = 0
de son produit scalaire canonique tel que la base canonique C Pour S2

u
soit orthonormale directe. 2x0 .( x − x0 ) + 2y0 .(y − y0 ) + 2z0 .(z − z0 ) = 0

uor
a Soit S1 = {( x, y, z) ∈ R3 /x2 − x + y2 = 0} et S2 = et donc, avec M0 ∈ S2
{( x, y, z) ∈ R3 /x2 + y2 + z2 = 1}. Préciser la nature des deux x0 .x + y0 .y + z0 .z = 1
surfaces S1 et S2 . Lorsque les plans tangents aux surfaces ne sont pas confon-
Correction : S1 est le cylindre d’axe parallèle à Oz et de di- dus, la tangente au point d’intersection des deux surface est

P
rectrice le cercle C1 , et S2 est la sphère de centreO et de rayon alors l’intersection des deux plans tangents.
1.    d Déterminer un paramétrage de Γ, en utilisant les coor-
3 x = x 2 + y2
b Soit Γ = ( x, y, z) ∈ R / . données cylindriques : c’est-à-dire que l’on exprimera pour
x 2 + y 2 + z2 = 1

r
M = ( x, y, z) = (r cos(θ ), r sin(θ ), z) les conditions sur r, θ, z
Que représente Γ vis-à-vis de S1 et S2 ?
pour que M soit sur Γ.
Correction : Γ est l’intersection des deux surfaces S1 et S2

e
c Déterminer l’équation dans C du plan tangent en tout En déduire une représentation paramétrique du cône de som-
er
1
met S = ( , 0, 0), engendré par les droites passant par S et un

m
point régulier M0 = ( x0 , y0 , z0 ) de S1 .
2
De même déterminer l’équation dans C du plan tangent en point variable sur Γ.
tout point régulier M0 = ( x0 , y0 , z0 ) de S2 . Correction : Soit M un point de coordonnées ( x, y, z) =
m

En déduire la tangente en tout point M0 = ( x0 , y0 , z0 ) régulier (r cos (θ ) , r sin (θ ) , z) .On a


M ∈ Γ ⇔ r cos(θ ) = r2 et r2 + z2 = 1

r
de Γ.
Correcton : en un point régulier M0 = ( x0 , y0 , z0 ) ⇔ (r = 0 ou r = cos(θ )) et z2 = 1 − r2 = 1 − cos2 (θ ) = sin2 (θ ).

o
une él’équation du plan tangent à la surface d’équation
or

F
✉: mamouni.myismail@gmail.com

134
i

r
s
✍ M AMOUNI M Y I SMAIL
MAMOUNI . NEW. FR
P ROBLÈMES C ORRIGÉS -MP

i
s

s
u
On obtient alors r = cos(θ ) et z = sin(θ ) avec θ ∈ IR, ou f Préciser comment on obtient les trois courbes planes qui

s
r = cos(θ ) et z = − sin(θ ) avec θ ∈ IR. Ces deux paramé- sont les projections orthogonales de Γ sur les plans xOy, xOz
trages donnent la même courbe.

é
et yOz, en faisant le lien avec les courbes étudiées dans la pre-

u
On obtient alors pour paramétrage de Γ : x = cos2 (θ ), y = mière question.
cos(θ ) sin (θ ), z = sin(θ ), θ ∈ IR.  Correction : Les trois courbes planes projections orthogo-

r
1 nales de Γ sur les plans xOy, xOz et yOz sont obtenues en

é
Soit C le cône de sommet S = , 0, 0 s’appuyant sur Γ on
2 annulant une coordonnée.
a.
Sur xOy : z = 0, x = cos2 (t), y = cos(t) sin (t), t ∈ IR : courbe

Po r
r
−→ −−→ C1 ;
M ∈ C ⇔ ∃λ ∈ IR ∃ M0 ∈ Γ SM = λSM0
   sur xOz : y = 0, x = cos2 (t), z = sin(t), t ∈ IR : analogue à la

u

 1 2 1
 x = + λ cos (θ ) − courbe C2 ;

uor
2 2 2 sur yOz : x = 0, y = cos(t) sin (t), z = sin(t), t ∈ IR : analogue
⇔ ∃(λ, θ ) ∈ R :

 y = λ cos(θ ) sin (θ )
 à la courbe C3
z = λ sin(θ )
e Pour t ∈ R, on pose : F(t) = (cos2 (t), cos(t) sin (t), sin (t)).

P
Soit γ = { F(t), t ∈ R }. Montrer que γ ⊂ Γ. Y-a-t-il égalité
γ = Γ? i
Correction : soit t ∈ IR on a
F

nn
cos(t)4 + sin(t)2 cos(t)2 = cos(t)2 et

r
i

e
cos(t)4 + sin(t)2 cos(t)2 + sin(t)2 = cos(t)2 + sin(t)2 = 1
er
donc γ ⊂ Γ. La réciproque a été étudiée dans la question d)
Á la prochaine

m
donc γ = Γ
m

or
or

F
✉: mamouni.myismail@gmail.com

135
✍ M AMOUNI M Y I SMAIL MAMOUNI . NEW. FR

Devoir Surveillé
4 Calcul Différentiel, Courbes & Surfaces
ri

ir
s
Blague du jour
s
Lors d’un discours prononcé devant une assemble de professeurs de math-

s
ématiques, George W.Bush les met en garde contre le mauvais usage des
u
mathématiques pour inculquer aux jeunes américains des visions politiques

s
extrémistes.
Si j’ai bien compris, dit le président, dans vos cours d’algèbre vous apprenez
é

u
à vos étudiants la résolution des problèmes et d’équations avec l’aide des
radicaux. Je ne peux pas dire que j’approuve ceci...
r

é
Joseph Louis, comte de Lagrange (1736-1813)

Mathématicien du jour
Po r
r
En italien Giuseppe Lodovico Lagrangia , est un mathématicien, mécanicien et astronome
franco-italien. N en Italie, mais de famille française par son arrière-grand-père.
u
Fondateur du calcul des variations avec Euler et de la théorie des formes quadratiques,
uor
son nom figure partout en mathématiques. Il développe la mécanique analytique, pour
laquelle il introduit les multiplicateurs de Lagrange. Il entreprend aussi des recherches
importantes sur le problème des trois corps en astronomie. Il élabore le système métrique
avec Lavoisier et enseigne les mathématiques à l’école normale et à l’école polytechnique.
P
Problème : extrait X 2004, MP
e r
Courbures des surfaces dans l’espace R3
er

Ce problème propose une étude des surfaces de l’espace R3 et de leurs courbures totale et moyenne. Pour tout
m
entier n > 0, l’espace Rn sera muni de son produit scalaire et de sa norme usuels notés respectivement (· | ·) et
k·k. La première partie est consacrée à des préliminaires algébriques.
rm

Première partie
r
 
(i)
1. Soient x(1) , . . ., x(n) des éléments de Rn+1 , xj les composantes de x(i) dans la base canonique
o j=1,...,n+1  
(i)
de Rn+1 . Pour tout k = 1, · · · , n + 1 on note Vk le produit par (−1)k+1 du déterminant de la matrice xj
où i = 1, · · · , n et j = 1, · · · , k − 1, k + 1, · · · , n + 1. On note V le vecteur de Rn+1 de composantes Vk .
Fo

a) Montrer que V est orthogonal à tous les x(i) .


b) Comparer les conditions suivantes :
i) V = 0
ii) la famille x(i)

i=1,··· ,n
est liée.
c) Exprimer en fonction de kV k le déterminant des n + 1 vecteurs V , x(1) , . . ., x(n) dans la base canonique
de Rn+1 .

✉: mamouni.myismail@gmail.com
✍ M AMOUNI M Y I SMAIL
MAMOUNI . NEW. FR

.
2. a) Montrer que, pour tout n-uple de vecteurs x(1) , · · · , x(n) linéairement indépendants, il existe un unique


vecteur W x(1) , · · · , x(n) ayant les propriétés suivantes



ri
i) W x(1) , · · · , x(n) est de norme 1 et orthogonal à tous les x(i)


ir
s
ii) le déterminant des n + 1 vecteurs W x(1) , · · · , x(n) , x(1) , . . ., x(n) dans la base canonique de Rn+1


est strictement positif.


s
b) Vérifier que, pour toute rotation R de Rn+1, on a

s
W R(x(1) ), . . ., R(x(n) ) = R W x(1) , · · · , x(n) .

u
3. Soit (e1 , . . ., en ) une base de Rn , Q la matrice de coefficients qi,j = (ei | ej ).

s
a) Montrer que Q est inversible et diagonalisable. Que peut-on dire de ses valeurs propres ?
b) Soit v un vecteur de Rn , de coordonnées vi dans la base (e1 , . . ., en ). Exprimer le vecteur ligne (v1 , . . ., vn )
é

u
en fonction de Q et du vecteur ligne ((v | e1 ) , . . ., (v | en )).
Dans la suite du problème, on désigne par U une partie ouverte de Rn , par u = (u1 , . . ., un ) un élément
r
quelconque de U , par F une application de classe C 2 de U dans Rn+1 , par ∂i F (resp. ∂i ∂j F ) ses dérivées

é
partielles d’ordre 1 (resp. 2). On suppose que les n vecteurs (∂i F )(u) sont linéairement indépendants pour
tout u, et on pose W (u) = W ((∂1 F )(u), . . ., (∂n F )(u)).
Po r
r
4. a) Vérifier que l’application u 7→ W (u) est de classe C 1 .
u
b) Comparer ((∂k W )(u) | (∂i F )(u)) et (W (u) | (∂i ∂k F )(u)).
uor
c) Démontrer l’existence et l’unicité de nombres réels ai,j (u) tels que l’on ait
P
(∂i W )(u) = ai,j (u)(∂j F )(u).
j
d) On note respectivement A(u), S(u), Q(u) les matrices de coefficients respectifs ai,j (u), (W (u) | ∂i ∂j F )(u)),
((∂i F )(u) | (∂j (F )(u))). Vérifier que A(u) = −S(u)Q(u)−1 .
P
Deuxième partie
Dans toute la suite du problème, on suppose n = 2 ; on a donc un ouvert U de R2 et une application F de classe
C 2 de U dans R3 telle que les vecteurs (∂1 F )(u) et (∂2 F )(u) soient linéairement indépendants pour tout u de U .
r
On a en outre
(∂1 F )(u) ∧ (∂2 F )(u)
W (u) =
e
k(∂1 F )(u) ∧ (∂2 F )(u)k
où · ∧ · désigne le produit vectoriel dans R3 . On pose
er

1
m K(u) = det A(u), H(u) = tr (A(u))
2
où A(u) est la matrice définie à la question 4.d). On note Fi (u), i = 1, 2, 3, les composantes de F (u) ; on suppose
que U contient le point 0 et on fait l’étude de la surface F (U ) au voisinage du point F (0).
rm

5. Soit R une rotation de R3 . Montrer que les objets K̂(0) et Ĥ(0) associés à l’application F̂ = R ◦ F sont
r
égaux respectivement à K(0) et H(0).
6. On suppose que, pour u suffisamment voisin de 0, F (u) est de la forme
o
F (u) = (u1 , u2 , f (u1 , u2 ))
avec f (0) = (∂1 f )(0) = (∂2 f )(0) = 0.
a) Calculer K(0) et H(0) en fonction des nombres
Fo

r = (∂1 ∂1 f )(0), s = (∂1 ∂2 f )(0), t = (∂2 ∂2 f )(0).


b) (Cas d’un cylindre) On suppose que f (u1 , u2 ) est fonction de u1 seul, soit f (u1 , u2 ) = g(u1 ). Exprimer
H(0) en fonction de la courbure de la courbe Γ, intersection du cylindre avec le plan x2 = 0.

✉: mamouni.myismail@gmail.com
✍ M AMOUNI M Y I SMAIL
MAMOUNI . NEW. FR

7. .Dans cette question, on considère le cas d’une surface de révolution :


F (u) = (f (u1 ) cos u2 , f (u1 ) sin u2 , u1 )
où f est une fonction strictement positive de classe C 2 définie sur un intervalle I.
ri
a) Dire pour quelles valeurs de u les vecteurs (∂1 F )(u) et (∂2 F )(u) sont linéairement indépendants.

ir
s
b) Vérifier que
−3/2 f (u1 )f 00 (u1 )
 
−1 0 2 0
A(u) = f (u1 ) 1 + f (u1 ) .
s
0 − (1 + f 0 (u1 ))2

s
c) Donner une fonction f élémentaire pour laquelle H(u) est nul pour tout u.
u
d) Montrer que, pour tous nombres réels α et β, α > 0, il existe f satisfaisant

s
H(u) = 0 pour tout u, f (0) = α, f 0 (0) = β.
Interpréter géométriquement le résultat obtenu.
é

u
e) Calculer K(u) pour une telle fonction f .
8. Indiquer, sans aucun calcul, des surfaces pour lesquelles K(u) et H(u) sont des constantes.
r

é
Troisième partie
Dans cette partie, on se propose d’étudier l’effet d’un changement de paramétrage sur les fonctions Het K.
Po r
r

∂F ∂F
Dans la situation du début de la deuxième partie on note la matrice (jacobienne) de coefficients =
∂u ∂u i,j
u
∂W
∂j Fi . Notation analogue pour .
uor ∂u
∂W ∂F t
9. Vérifier que = A(u).
∂u ∂u
On se donne maintenant un difféomorphisme Φ de U sur un autre ouvert Ũ de R2 et on pose Ψ = Φ−1 .
Pour tout u ∈ U on écrira aussi ũ = Φ(u) ; on pose F̃ (ũ) = F (u), c’est-à-dire F̃ = F ◦ Ψ, et on note W̃ (ũ),
P
Ã(ũ), K̃(ũ), H̃(ũ) les objets définis à partir de F̃ et ũ comme W (u), A(u), K(u), H(u) l’ont été à partir de
F et u. On suppose U connexe par arcs.
∂ F̃ ∂F ∂Ψ
10. a) Exprimer en fonction de et , puis (∂1 F̃ )(ũ) ∧ (∂2 F̃ )(ũ) en fonction de (∂1 F )(u) ∧ (∂2 F )(u)
∂ ũ ∂u ∂ ũ
r
∂Ψ
et det .
∂ ũ
e
b) Montrer qu’il existe ε ∈ {1, −1} tel que l’on ait W̃ (ũ) = εW (u) pour tout u ∈ U .
er

∂Ψ
c) Exprimer Ã(ũ) en fonction de ε, A(u) et .
m d) Comparer H̃(ũ) et H(u), K̃(ũ) et K(u).
∂ ũ
rm

Problème : Mines-Ponts 2004, MP


or
Soit ϕ la fonction définie sur la droite réelle par la relation suivante :
 
si |t| < 1, ϕ (t) = exp 2 1 ; si |t| ≥ 1, ϕ (t) = 0.
t −1
. Un difféomorphisme f de la droite réelle R sur elle-même de classe C1 est dit difféomorphisme de
Fo

classe C∞ si la fonction f est indéfiniment dérivable.


Un difféomorphisme de R de classe C∞ :
18. Démontrer que la restriction ϕI de la fonction ϕ à l’intervalle ouvert I = ]−1, 1[ est indéfiniment
(n)
dérivable et que, pour tout entier n, il existe un polynôme Pn tel que la dérivée ϕI de ϕI d’ordre n
s’écrive sous la forme suivante :

✉: mamouni.myismail@gmail.com
✍ M AMOUNI M Y I SMAIL
MAMOUNI . NEW. FR

 
(n) Pn (t) 1
ϕI (t) = 2 n exp 2
.
(t2 − 1) t −1
19. En déduire que la fonction ϕ est indéfiniment dérivable sur la droite réelle R. Justifier, sans calcul,
l’existence d’un majorant M de la valeur absolue de la dérivée première ϕ´sur la droite réelle :

ir
M = sup |ϕ´(t)| .
t ∈ R
ri
Étant donné un réel λ (λ ∈ R) , soit ψλ la fonction définie sur la droite réelle par la relation suivante :
s

s
ψλ (x) = x + λ ϕ (x) .
20. Démontrer que, si la valeur absolue du réel λ est strictement majorée par 1/M, (|λ| < 1/M ), la
fonction ψλ est une bijection de la droite réelle R sur elle-même et un difféomorphisme de classe C∞ de
s

s
R.
Quelle est, dans ces conditions (|λ| < 1/M ) , l’image du segment I = [−1, 1] par l’application x 7−→
u

u
ψλ (x) ? Que dire de la restriction de l’application x 7−→ ψλ (x) aux demi-droites fermées ]−∞, −1] et
[1, ∞[ ?
Un difféomorphisme de classe C1 du plan R2 , défini par des fonctions indéfiniment dérivables est
é

é
appelé difféomorphisme de classe C∞ .
Difféomorphismes du plan R2 de classe C∞ :
Le plan R2 est supposé muni de la norme euclidienne et rapporté à un repére orthonormé Oxy.
r

r
Étant donnés un réel λ (λ ∈ R) , un réel strictement positif r (r > 0) et un point P du plan R2 de
P 2
coordonnées (p, q) , soit θλ, r l’application de R dans lui-même définie par la relation suivante :
Po r
   !
ur P
θλ,

x

7−→
x + λ ϕ (x − p)
2
+ (y − q)
2
/r 2
r : y
.
y
u
P
L’image du point de coordonnées (x, y) par l’application θλ, r est le point de coordonnées :
! !
2 2
(x − p) + (y − q)
x+λ ϕ , y .
r2
o
P P
21. Quelle est l’image par cette application θλ, r du point P ? du cercle Cr de centre le point P et de
P
rayon égal à r ? de l’ouvert Ωr des points du plan situé à une distance du point P strictement supérieure
à r ?
P
Existence de difféomorphismes du plan de classe C∞ :
22. Démontrer qu’il existe un réel m strictement positif tel que, si le réel λ a une valeur absolue
P 2
strictement inférieure à m (|λ| < m) , l’application θλ, r est une bijection du plan R sur lui-même et un
∞ 2
difféomorphisme de classe C de R .
Soient n points A1 , A2 , . . . , An du plan R2 , deux à deux distincts, et deux points B et B´, distincts
r
er

entre eux et distincts des points Ai , 1≤ i ≤ n. Les coordonnées des points Ai , 1≤ i ≤ n, sont (xi , yi ) ,
1≤ i ≤ n ; celles de B et B´respectivement (b, c) et (b´, c´).
e
Le but des questions 23 à 26 est de montrer qu’il existe un difféomorphisme de classe C∞ du plan
2
R transformant B en B´et laissant les points A1 , A2 , . . . , An invariants. Un difféomorphisme de classe
C∞ du plan R2 laissant les points A1 , A2 , . . . , An invariants est dit avoir la propriété A.
m
rm

Il est admis que l’ensemble des difféomorphismes de classe C∞ du plan R2 est un groupe pour la loi
de composition des applications. Trois cas sont envisagés :
1er cas : Les points B et B´ont même ordonnée ; les ordonnées des points Ai , 1≤ i ≤ n, sont toutes
différentes de celle de B (yi 6= c = c´).
23. Démontrer, dans ce cas, que, si les points B et B´ sont suffisamment proches, il existe une appli-
r
P
cation θλ, r transformant B en B´et laissant les Ai , 1≤ i ≤ n invariants.
24. Démontrer, toujours dans ce cas, que, quelle que soit la position des points B et B´, il existe une
o
Pi
Fo

suite finie de bijections θλ, r , 0 ≤ i ≤ k, telle que la composée F de ces applications transforme B en B´
et ait la propriété A.
Pk Pk−1 P0
F = θλ, r ◦ θλ, r ◦ . . . ◦ θλ, r .

2ième cas : Les points B et B´ont même abscisse ; les abscisses des points Ai , 1≤ i ≤ n, sont toutes
différentes de celle de B (xi 6= b = b´) .
P P
25. Indiquer comment modifier l’application θλ, r en ηλ, r pour construire un endomorphisme G
transformant B en B´et ayant la propriété A.

✉: mamouni.myismail@gmail.com
MAMOUNI . NEW. FR
✍ M AMOUNI M Y I SMAIL
Corrige Problème I : Pr. Duval, CPGE France
ri
Première partie

ir
s
P
1a) (V | x(`) ) = nk=1+1
(−1)k x(k`) det(Xk ) ou X est la matrice de terme general xj(i) et Xk la matrice deduite de X en
s
supprimant la k-eme ligne. La somme precedente est egale a l'oppose au determinant de la matrice X completee

s
par la colonne x(`) sur sa gauche. Cette matrice a deux colonnes egales donc son determinant vaut zero.
u
= 0 si et seulement si tous les mineurs d'ordre n extraits de X sont nuls, ce qui equivaut au fait que rg(X) < n,

s
1b) V
ou aussi a la liaison de la famille (x(i) ).
é

u
1c) En reprenant le raisonnement de 1a, pour tout vecteur u ∈ Rn+1 on a (u | V ) = det(u, x(1) , ... , x(n) ), determinant
calcule dans la base canonique de Rn+1 . En particulier kVk2 = det(V, x(1) , ... , x(n) ).
r
Si (x(1) , ... , x(n) ) est libre alors le vecteur W = V/kVk veri e i et ii. Par ailleurs, {x(1) , ... , x(n) }⊥ est une

é
2a)
droite vectorielle (orthogonal d'un hyperplan de Rn+1 ) donc les seuls vecteurs orthogonaux a tous les x(i) sont les
multiples de W et les conditions i et ii imposent que le coecient de proportionnalite vaut 1. Ceci prouve l'unicite
Po r
r
de W .
u
2b) La notion de rotation n'est pas claire. S'agit-il de la composee de deux re exions ou d'un endomorphisme orthogonal
positif quelconque (les deux notions concident en dimension 2 et 3, la deuxieme est plus generale dans le cas n
uor
quelconque) ? On considere ici qu'il s'agit d'un endomorphisme orthogonal positif. Soit donc R ∈ O+ (Rn+1 )
et (x(1) , ... , x(n) ) une famille libre de Rn+1 . La famille image est aussi libre par injectivite de R. De plus,
comme R conserve la norme et le produit scalaire, on a R(W ) ⊥ R(x(i) ) pour tout i et kR(W )k = 1. En n,
det(R(W ), R(x(1) ), ... , R(x(n) )) = det(R) det(W, x(1) , ... , x(n) ) = det(W, x(1) , ... , x(n) ) > 0 car det(R) = 1.
P
3a) Q est diagonalisable car symetrique reelle. La forme quadratique q associee a Q est de nie par q(a1 , ... , an ) =
ka1 e1 + ... + an en k2 , donc elle est de nie positive puisque (e1 , ... , en ) est libre. On en deduit Spec(Q) ⊂ R+∗ et
en particulier Q est inversible.
0
..
r
0
3b) (v | ei ) = (v1 ... vn )Q  
 10  ou le (( 1 )) est en i-eme ligne. On en deduit : ((v | e1 ) ... (v | en )) = (v1 ... vn )Q.
e
..
0
er

Chaque vecteur x(i) = ∂i F(u) est une fonction de classe C 1 de u. Le vecteur V de ni en 1 est une fonction
m
4a)
polynomiale des x(ji) , donc est aussi de classe C 1 et V 6= 0 pour tout u par hypothese. On en deduit que kVk et
W = V/kVk sont aussi des fonctions de u de classe C 1 .
rm

4b) En derivant la relation (W (u) | ∂i F(u)) = 0 par rapport a la k-eme coordonnee de u et a l'aide du theoreme de
r
Schwarz on obtient : (∂k W (u) | ∂i F(u)) = −(W (u) | ∂k ∂i F(u)) = −(W (u) | ∂i ∂k F(u)).
De m^eme, la relation (W (u) | W (u)) = 1 donne par derivation : (W (u) | ∂i W (u)) = 0. Donc le vecteur ∂i W (u)
o
4c)
appartient a W (u)⊥ = < ∂1 F(u), ... , ∂n F(u) >. Les coecients aij (u) sont les composantes de ∂i W (u) dans la
base (∂1 F(u), ... , ∂n F(u)) de W (u)⊥ , d'ou leur existence et leur unicite.
Fo

4d) Consequence immediate des resultats obtenus en 3b et 4b.

Deuxième partie

5) R etant lineaire, ∂i F^ = R ◦ (∂i F) donc W


^ = R ◦ W d'apres 2b. Comme R conserve le produit scalaire et commute
avec les derivations, on en deduit S^ = S et Q
^ = Q, d'ou A^ = A d'apres 4d, et en n K^ = K, H
^ = H.

✉: mamouni.myismail@gmail.com
MAMOUNI . NEW. FR
✍ M AMOUNI M Y I SMAIL
p
. ∂1 F(u) = (1, 0, ∂1 f(u)), ∂2 F(u) = (0, 1, ∂2 f(u)) donc 2 2
6a) µ ¶ µ ¶ W (u) = (−∂1 f(u), −∂2 f(u), 1)/ (∂1 f(u)) + (∂2 f(u)) + 1 et
W (0) = (0, 0, 1). S(0) =
r s
, Q(0) =
1 0 , A(0) = −S(0) d'ou K(0) = rt − s2 , H(0) = − r + t .
s t 0 1 2
ri
6b) Ici s = t = 0 donc H(0) = − 21 r. Pour de nir la courbure d'une courbe dans le plan d'equation x2 = 0, il est

ir
s
necessaire d'orienter ce plan. On convient de l'orienter de sorte que la base (e3 , e1 ) soit directe (ce qui revient a

→0 − → −

orienter la normale < e2 > dans le sens de e2 ). D'apres la relation generale c = det(M , M00 )/kM0 k3 donnant la
s
1
courbure d'une courbe plane parametree, on a ici c(0) = −r donc H(0) = 2 c(0).

s
7a) ∂1 F(u) = (f0 (u1 ) cos u2 , f0 (u1 ) sin u2 , 1) et ∂2 F(u) = (−f(u1 ) sin u2 , f(u1 ) cos u2 , 0) sont lies si et seulement si
u
∂2 F(u) = 0 vu la troisieme composante, soit si et seulement si f(u1 ) = 0, ce qui est exclu dans l'enonce.

s
7b) Calcul sans diculte.
é

u
7c) On veut ff00 = 1 + f02 , ce qui est realise pour f(t) = ch t par exemple. La surface obtenue est appelee catenode,
voir http://www.mathcurve.com/surfaces/catenoid/catenoid.shtml pour une description de cette surface et de ses
r
proprietes.

é
7d) En essayant f(t) = λpch(µt + ν) on obtient les equations : λ2 µ2 = 1, λ ch ν = α, λµ sh ν = β qui ont pour solution
ν = argsh β, λ = α/ 1 + β2 et µ = 1/λ.
Po r
r
Interpretation geometrique : f(0) = α est la distance du point F(0) a l'axe de revolution et f0 (0) = β est la pente
u
de la meridienne passant par F(0). On vient donc de constater que par tout point du plan < e1 , e2 > autre
que l'origine et pour toute droite secante a < e3 > passant par ce point, il existe une surface de revolution
uor
autour de < e3 > a courbure moyenne nulle, passant par ce point et tangente a cette droite ...

7e) K(u) =
−f00 (u1 )
= −µ2 .
f(u1 )(1 + f02 (u1 ))2 4
ch (µu1 + ν)
P
8) En prenant f constante non nulle, la matrice A est constante et donc K et H le sont. Les surfaces correspon-
dantes √sont les cylindres de revolution autour de < e3 >. On peut aussi penser aux spheres d'axe < e3 >, soit
f(t) = R2 − t2 , qui conviennent (avec un peu de calcul) : A(u) = I/R o
u I est la matrice identite.
r
Troisième partie
e
er

Consequence immediate de 4c.


m 9)

10a) Remarque
³
: la ´classe de  n'est pas precisee. On suppose ici que  est de classe C 2 .
~
∂F = ∂F ◦ × ∂ (produit de matrices jacobiennes).
~ ~ ¶
rm

∂u ∂u µ ∂u
r
En notant ∂ = ac db on a donc ∂1 F~ = (a∂1 F + c∂2 F) ◦ et ∂2 ~F = (b∂1 F + d∂2 F) ◦ ,
~
∂u ³ ´
d'ou ∂1 F ∧ ∂2 ~F = (ad − bc)((∂1 F ∧ ∂2 F) ◦ ) = det ∂ ((∂1 F ∧ ∂2 F) ◦ ).
~
o
~
∂u
³ ³ ´´ ³ ´ ³ ´
10b) ~ = sgn det ∂
D'apres la relation precednte, W ~ (det ∂
(W ◦ ), et le signe de det ∂ est constant sur U
∂u~ ~
∂u ∂u~
Fo

est une fonction continue jamais nulle et U~ = (U) est connexe par arcs).
~ ³ ´ ³³ ´ ´ ³ ´ ³ ´ ³ ~´ ³ ´−1³ ´
10c) ∂W = ε ∂W ◦ × ∂ = ε ∂F × tA ◦ × ∂ = ε ∂F ◦ × tA◦ × ∂ = ε ∂F × ∂ × tA◦ × ∂ .
~
∂u ∂u ~
∂u ∂u ~
∂u ∂u ~
∂u ~
∂u ~
∂u ~
∂u
~ ³ ´ ³ ´−1
~ (~u) = ε ∂ × A(u) × ∂
La matrice ∂F est inversible puisque ∂F l'est. On en deduit : A t t
.
∂u~ ∂u ~
∂u ~
∂u

10d) ~ (~u) = εH(u) et K~ (~u) = K(u).


En prenant la demi-trace et le determinant : H

✉: mamouni.myismail@gmail.com
MAMOUNI . NEW. FR
✍ M AMOUNI M Y I SMAIL
Corrigé Problème II : Pr. Patte, CPGE France
Un difféomorphisme de R de classe C ∞ :
ri
18. La fonction ϕI est la composée de l’exponentielle et d’une fonction rationnelle sans pôle dans I, donc est de

ir
s
classe C ∞ sur I.
Soit n ∈ N. ³ ´
s
(n)
S’il existe un polynôme Pn tel que ∀t ∈ I, ϕI (t) = (t2P−1)
n (t)
2n exp
1
t2 −1 , alors ∀t ∈ I,

s
µ ¶ µ ¶
u
(n+1) Pn0 (t) −2n.2t Pn (t) −2t 1
ϕI (t) = + Pn (t) + . exp
(t2 − 1)2n (t2 − 1)2n+1 (t2 − 1)2n (t2 − 1)2 t2 − 1

s
µ ¶ (1)
(t2 − 1)2 Pn0 (t) − 4nt(t2 − 1)Pn (t) − 2tPn (t) 1
= exp 2 .
é
(t2 − 1)2(n+1) t −1

u
Donc, avec le polynôme Pn+1 (t) = (t2 − 1)2 Pn0 (t) − 4nt(t2 − 1)Pn (t) − 2tPn (t), on obtient :
r
µ ¶

é
(n+1) Pn+1 (t) 1
∀t ∈ I, ϕI (t) = 2 exp .
(t − 1)2(n+1) t2 − 1
³ ´
Po r
r
(0) P0 (t) 1
Comme ∀t ∈ I, ϕI (t) = ϕI (t) = (t2 −1)2.0 exp t2 −1 avec P0 = 1 :
u µ ¶
(n) Pn (t) 1
∀n ∈ N, ∃Pn ∈ R[X], ∀t ∈ I, ϕI (t) = exp .
uor (t2 − 1)2n t2 − 1

19. La fonction ϕ est continue sur R. Ses restrictions à ] − ∞, −1[, ] − 1, 1[ et ]1, +∞[ sont de classe C ∞ et, pour tout
n ∈ N∗ , la fonction ϕ(n) a pour limite 0 en 1 et en -1 (en 1 à gauche et en -1 à droite par croissances comparées).
Donc ϕ est de classe C ∞ sur R.
P
Comme ϕ0 est continue sur R et nulle en dehors du compact [−1, 1], elle est bornée sur R. D’où l’existence de
M = sup |ϕ0 |.
R
20. Pour tout λ ∈ R, la fonction ψλ est de classe C ∞ sur R et ∀x ∈ R, ψλ0 (x) = 1 + λϕ0 (x). Comme |λϕ0 (x)| ≤ |λ|M ,
si |λ| < 1/M , alors ψλ0 (x) ≥ 1 − |λ|M > 0 et la fonction ψλ est strictement croissante sur R. Mieux, c’est un
r
C ∞ -difféomorphisme de R sur ψλ (R) =] lim ψλ , lim ψλ [.
−∞ +∞
De plus, si x 6∈ [−1, 1], ψλ (x) = x. Donc lim ψλ = +∞. De même que lim ψλ = −∞. Donc la fonction ψλ est un
e
+∞ −∞
C ∞ -difféomorphisme strictement croissant de R sur R.
er

m Difféomorphismes de R2 de classe C ∞ :
P
21. Par la fonction θλ,r , le point P est envoyé sur le point de coordonnées (p + λe , q). En notant M = (x, y) et P M
2 2 2 2 2
la distance usuelle de P à M , on a (x−p) r+(y−q)
2 = PrM2 et ϕ( (x−p) r+(y−q)
2 ) = 0 dès que P M ≥ r. Donc la
rm

P
fonction θλ,r fixe tout point hors du disque ouvert de centre P et de rayon r ; en particulier les points du cercle
r
CrP et de l’ouvert ΩPr .
o
Existence de difféomorphisme de R2 de classe C ∞ :
22. Comme ϕ est de classe C ∞ sur R, la fonction θλ,r P
est de classe C ∞ sur le plan R2 . D’après le théorème d’inversion
globale, il suffit que la fonction θλ,r soit bijective de R2 dans R2 et que son jacobien ne s’annule pas sur R2 pour
P

que ce soit un C ∞ -difféomorphisme de R2 sur R2 .


Fo

Étude du jacobien : au point (x, y), il vaut :


à ³ ´ !
2 2
³ ´
1 + λ 2(x−p) ϕ 0 (x−p) +(y−q)
∗ ∗ ∗ 2 2
J(x, y) = det r2 r2 = 1 + λ 2(x−p)
r 2 ϕ0 (x−p) +(y−q)
r 2 .
0 1
¯ ³ ´¯
¯ 2 2 ¯ 2|λ|M
Si (x − p)2 + (y − q)2 ≥ r2 , alors J(x, y) = 1 ; sinon |x − p| ≤ r, donc ¯λ 2(x−p)
r2 ϕ
0 (x−p) +(y−q)
r2 ¯ ≤ r , donc
2|λ|M
J(x, y) ≥ 1 − r .

✉: mamouni.myismail@gmail.com
MAMOUNI . NEW. FR
✍ M AMOUNI M Y I SMAIL
. Finalement, sur tout le plan, J(x, y) ≥ 1 − 2|λ|M
r .
Donc, si on choisit λ et r tels que 1 − 2|λ|M
r > 0, i.e. |λ| < r
2M , le jacobien J ne s’annule pas sur le plan.
P
Bijectivité de θλ,r sous cette condition en λ :
ri
Soit (X, Y ) ∈ R2 . On cherche (x, y) ∈ R2 tel que (X, Y ) = θλ,r
P
(x, y). Or

ir
s
µ ¶
P (x − p)2 + (y − q)2
(X, Y ) = θλ,r (x, y) ⇔ X = x + λϕ et Y = y
s
r2 (2)

s
⇔ y = Y et X = f (x)
u
³ ´
(x−p)2 +(Y −q)2
où f (x) = x + λϕ . Il suffit donc de montrer que f est une bijection de R sur R pour obtenir

s
r2
P
l’existence et l’unicité de (x, y) tel que (X, Y ) = θλ,r (x, y).
³ ´
é

u
2 2
La fonction f est de classe C ∞ et f 0 (x) = 1 + λ 2(x−p)
r2 ϕ
0 (x−p) +(y−q)
r2 ≥ 1 − 2|λ|M
r . Comme 1 −
2|λ|M
r est

une constante strictement positive, f est strictement croissante sur R ; mieux, f est un C -difféomorphisme
r
2
−q)2
strictement croissant de R sur f (R). Au voisinage de l’infini, (x−p) r+(Y
2 > 1, donc f (x) = x et, comme à la

é
question 20, f (R) = R et f est un C ∞ -difféomorphisme strictement croissant de R sur R.
r P
Finalement, sous la condition |λ| < 2M , la fonction θλ,r est bijective du plan sur lui-même. Avec l’étude du
P ∞ 2 2
Po r
jacobien, θλ,r est un C -difféomorphisme de R sur R .

r
B
23. L’application θλ,r transforme B en B 0 dès que b0 = b + λe , i.e. λ = e(b0 − b). C’est un C ∞ -difféomorphisme du
u r
plan si |λ| < 2M . Elle laisse les points Ai invariants dès que ∀i, BAi ≥ r, i.e. r ≤ min BAi .
1≤i≤n
uor
Il suffit donc de trouver un couple (λ, r) tel que λ = e(b0 − b) et 2M |λ| < r ≤ min BAi , i.e. λ = e(b0 − b)
1≤i≤n
min BAi
et 2M e|b − b| < r ≤ min BAi . C’est possible dès que 2M e|b − b| < min BAi , i.e. |b0 − b| <
0 0 1≤i≤n
2M e .
1≤i≤n 1≤i≤n
On pose alors λ = e(b − b) ; on choisit r dans l’intervalle ]2M e|b0 − b|, min BAi ] : l’application θλ,r
0 B
est un
1≤i≤n
P
C ∞ -difféomorphisme du plan transformant B en B 0 et conservant les points Ai .
Remarque : on peut remplacer min BAi par min |yi − c| sans changement.
1≤i≤n 1≤i≤n
min |yi −c|
|b0 −b|
24. On partage le segment [B, B 0 ] en k + 1 sous-segments de sorte que
1≤i≤n
k+1 < 2M e . On note Bi le point de
r
0 |b0 −b|
coordonnées (b + i bk+1−b
, 0). Les couples (Bi , Bi+1 ) vérifient les hypothèses de la question 23 : Bi Bi+1 = k+1 <
min |yi −c| 0 0
. On pose alors λ = e (bk+1
−b)
; on choisit r dans l’intervalle ]2M e |bk+1
−b|
1≤i≤n
, min |yi − c|] : l’application
e
2M e 1≤i≤n
Bi
θλ,r est un C ∞ -difféomorphisme du plan transformant Bi en Bi+1 et conservant les points Aj . L’application
er

Bk B1 B0
F = θλ,r ◦ . . . ◦ θλ,r ◦ θλ,r est alors un C ∞ -difféomorphisme du plan transformant B en B 0 et conservant les points
m Aj .
³ ³ ´´
2 2
P
25. On échange le rôle des coordonnées : on utilise l’application ηλ,r P
définie par ηλ,r (x, y) = x, y + λϕ (x−p) r+(y−q)
2 .
rm

26. Deux cas :


Cas particulier : aucun point Ai n’appartient à la droite (BB 0 ).
r
On se ramène au cas étudié dans la question 24 par rotation. On note θ l’angle entre l’axe y = 0 et la droite (BB 0 )
o
et, pour τ ∈ R, on note Rτ la rotation d’angle τ autour de l’origine. Les points B̂ = R−θ (B) et B̂ 0 = R−θ (B 0 )
et la suite finie constituée des points Âi = R−θ (Ai ) pour i ∈ {1, . . . , n} sont dans la configuration de la question
24. On considère un C ∞ -difféomorphisme du plan : F , transformant B̂ en B̂ 0 et conservant les points Âi . Soit
G = Rθ ◦ F ◦ R−θ . Comme les rotations sont des C ∞ -difféomorphismes du plan, G aussi et G transforme B en
Fo

B 0 et conserve les points Ai .


Cas général : on utilise un point A tel qu’aucun point Ai n’appartienne aux droites (BA) et (AB 0 ).
D’après le cas particulier, il existe des C ∞ -difféomorphismes du plan F1 et F2 conservant les points Ai et
transformant respectivement B en A et A en B 0 . L’application F2 ◦ F1 est un C ∞ -difféomorphisme du plan
conservant les points Ai et transformant B en B 0 .
Existence de A : il suffit de choisir A sur le cercle (C) de diamètre [B, B 0 ] autre que B, B 0 , et les points (en
nombre fini) d’intersection du cercle (C) et des droites (BAi ) et (B 0 Ai ).
27. On procède par récurrence sur n.

✉: mamouni.myismail@gmail.com
i

r
s
✍ M AMOUNI M Y I SMAIL
MAMOUNI . NEW. FR
P ROBLÈMES C ORRIGÉS -MP

i
s

s
u

s
Devoir Surveillé
15 Calcul Différentiel, Courbes & Surfaces

u
Joseph Louis, comte de Lagrange (1736-1813)

é
Blague du jour En italien Giuseppe Lodovico Lagrangia , est un mathémati-
cien, mécanicien et astronome franco-italien. N en Italie, mais

Po r
r

Mathématicien du jour
Lors d’un discours prononcé devant une assemble de de famille française par son arrière-grand-père.
professeurs de mathématiques, George W.Bush les met Fondateur du calcul des variations avec Euler et de la théorie

u
en garde contre le mauvais usage des mathématiques des formes quadratiques, son nom figure partout en mathéma-

uor
pour inculquer aux jeunes américains des visions poli- tiques. Il développe la mécanique analytique, pour laquelle il
tiques extrémistes. introduit les multiplicateurs de Lagrange. Il entreprend aussi
Si j’ai bien compris, dit le président, dans vos cours d’al- des recherches importantes sur le problème des trois corps en
gèbre vous apprenez à vos étudiants la résolution des astronomie. Il élabore le système métrique avec Lavoisier et en-

P
problèmes et d’équations avec l’aide des radicaux. Je ne seigne les mathématiques à l’école normale et à l’école polytech-
peux pas dire que j’approuve ceci... nique.

r
Problème : extrait cnc 2007, PSI

e
er
−→

m
Dans ce problème, E désigne un plan affine euclidien orienté de direction E , et (O,~i,~j) un repère orthonormée direct de E ; le produit scalaire


de deux vecteurs ~ e1 et ~
e2 de E se notera (~ e 2 ).
e1 |~
Un point M de E peut être repéré par ses coordonnées cartésiennes x et y dans le repère (O,~i,~j), ou par ses coordonnées polaires ρ et θ (rayon
m

et angle polaires).

r
Étant donné dans E un arc γ birégulier et un point M de γ, on note :

o
• s l’abscisse curviligne de M sur γ,
or

F
✉: mamouni.myismail@gmail.com

145
i

r
s
P ROBLÈMES C ORRIGÉS -MP ✍ M AMOUNI M Y I SMAIL
MAMOUNI . NEW. FR

i
s

s
u

→ −
→ \

→ −
→ π
• T le vecteur unitaire tangent à γ en M et N le vecteur unitaire vérifiant ( T , N ) = ,

s
2
• R le rayon de courbure algébrique de γ en M et I le centre de courbure de γ en M,

é

→ π

u
• ~u(θ ) et ~v(θ ) les vecteurs de E défini par : ~u(θ ) = cos θ ~i + sin θ ~j et ~v(θ ) = ~u(θ + ),
2
\ −→ \ −→

r
• V l’angle (~u(θ ), T ) et α l’angle (~i, T ).

é
Première partie

Po r
r
On considère l’arc γ1 de E d’équation polaire ρ = 1 + cos θ et on note ϕ l’application de R vers E définie par

u
θ 7−→ O + (1 + cos θ )~u (θ ).
① ① Déterminer le domaine de définition de la fonction ρ et en préciser une période.

uor
② Étudier la parité de ρ et en déduire que le support de l’arc γ1 possède un axe de symétrie à préciser.
③ Comment peut-on obtenir le support de l’arc γ1 à partir de celui de l’arc γ2 = ([0, π ], ψ) où ψ désigne la restriction de ϕ au segment
[0, π ].

P
② Préciser la nature du pôle O, point du support de γ1 de paramètre π.
③ Soit M0 = ϕ(θ0 ) un point de γ1 distinct du pôle O. Montrer que M0 est un point birégulier et préciser la concavité de γ1 en ce point.
④ Étudier la fonction ρ : θ 7−→ 1 + cos θ sur le segment [0, π ] et dresser son tableau de variations.

r
⑤ Tracer soigneusement le support de l’arc γ1 en précisant les tangentes aux points d’intersection de son support avec les axes des coor-
données (unité : 3cm).

e
⑥ Calculer la longueur de l’arc γ2 .
er
⑦ Calculer l’aire de la portion du plan délimité par le support de l’arc γ1 .

m
Deuxième partie

A- Questions de cours
m

r
Soit γ un arc birégulier de E d’équation polaire ρ = f (θ ) ; on note s une abscisse curviligne sur γ orienté dans le sens des θ croissants.
−→ −
→ ds f
On rappelle que MI = R N , R = et tan V = ′ .

o
dα f
or

F
✉: mamouni.myismail@gmail.com

146
i

r
s
✍ M AMOUNI M Y I SMAIL
MAMOUNI . NEW. FR
P ROBLÈMES C ORRIGÉS -MP

i
s

s
u
① Faire un croquis propre et lisible en traçant une portion de l’arc γ et en plaçant en un point M de paramètre θ, distinct du pôle O, les

s

→ − →
vecteurs ~u(θ ), T , N et les angles θ, V et α.

u
ds
② Rappeler la définition de s et exprimer à l’aide de f et f ′ .

r
dV
③ Calculer et en déduire l’expression du rayon de courbure R.

é

④ Exprimer les coordonnées de I, centre de courbure de γ en M, dans le repère ( M, ~u(θ ), ~v (θ )).

Po r
r
B- Retour à l’arc γ1

u
Soit s une abscisse curviligne sur l’arc γ1 orientée dans le sens des θ croissants. À tout point M(θ ) de l’arc γ1 , distinct du pôle O, on associe le
centre de courbure noté I (θ ).

uor
① Préciser les coordonnée de I (θ ) d’abord dans le repère (O, ~u(θ ), ~v (θ )) puis dans le repère (O,~i,~j).
② Montrer que le point I (θ ) est l’image du point M(θ + π ) de γ1 par une homothétie dont on précisera le centre Ω et le rapport λ .


P
③ On note H (θ ) le projeté orthogonal du point I (θ ) sur la droite OM(θ ) joignant les points O et M(θ ). Montrer que le point H (θ ) est
l’image du point M(θ ) par une homothétie de centre O dont on précisera le rapport µ.
④ On note γ I et γ H les courbes décrites respectivement par le centre de courbure I (θ ) et son projeté orthogonal H (θ ). Tracer les supports
de γ1 , γ I et γ H sur le même graphique, et placer un point M(θ ) de γ1 et les points I (θ ) et H (θ ) correspondant.

r
⑤ Donner la longueur de la courbe γ H décrite par le point H (θ ) ainsi que l’aire de la portion du plan qu’elle délimite.

e
Exercice I : extrait e3a 2007, MP
er

m
m

or
or

F
✉: mamouni.myismail@gmail.com

147
✍ M AMOUNI M Y. I SMAIL
.
MAMOUNI NEW FR

Exercice I : extrait e3a 2008, MP

i
F
nn

i
Á la prochaine

✉: mamouni.myismail@gmail.com
✍ M AMOUNI M Y I SMAIL MAMOUNI . NEW. FR

Corrigé Problème : Pr. Chabchi, CPGE Marrakech


ri
PARTIE I

ir
s
1. .
s
(a) Le domaine de dé…nition de est R et est 2 périodique.

s
(b) Par parité de la fonction cosinus, est aussi paire, donc l’arc 1 est symétrique par rapport à l’axe
u
O ~i :

s
(c) Le support de 1 est obtenu en prenant le support de 2 union son symétrique par rapport à l’axe
O ~i :
é

u
2. Le pôle O est paramétré par = ; de plus ( ) = 0 ( ) = 0 et 00 ( ) = cos ( ) = 1 6= 0: Puisque 2 est
r
pair alors le pôle O est un point de rebroussement du premier espèce et la tangente est porté par ~u ( ) ,

é
càd horizontale.
3. Soit M0 = ( 0 ) un point de 1 autre que le pôle O; donc 0 2 [0; 2 ] avec 0 6= : On a alors
Po r
2

r
2 0 00
( 0 ) + 2 ( ( 0 )) ( 0 ) ( 0 ) = 3 (1 + cos ( 0 )) > 0: Ainsi la concavité de 1 en M0 est tournée vers
le pôle O ( ou contient le pôle O) .
u
4. On a la fonction est dérivable sur [0; ] et 0 ( ) = sin ( ) 0: Donc est décroissante sur [0; ] : elle
décroit de la valeur (0) =2 à la valeur ( ) =0.
uor
5. Le tracé est ci-contre :
P
e r
er

m
rm

or
Fo

La tangente à l’origine est horizontale


0
Le point M (2; 0) est paramétré par = 0; puisque (0) = 2 6= 0 et (0) = 0; la tangente est alors
portée par v (0) = ~j càd verticale.

✉: mamouni.myismail@gmail.com

1
✍ M AMOUNI M Y I SMAIL MAMOUNI . NEW. FR

0
Le point B (0; 1) est parmétré par = ; puisque = 1 et = 1: Si V =
2 2 2 2
. ~u
\
; T~ désigne l’angle que le vecteur ~u avec le vecteur tangent, alors ici tan V =
2 2 2 2
2
= 1; donc V = : Dans R O;~i;/~j cette tangente a pour équation : y = x + 1
ri
0 2 4
2

ir
Par symètrie, la tangente en C (0; 1) dans R O;~i;/~j a pour équation : y =
s
x 1:
Z 2 Z
s
1 0 0
6. L’arc 1 étant de classe C ; donc sa longueur l ( 1 ) est donnée par : l ( 1) = jj ( ) jjd = 2 jj ( ) jjd

s
0 0
par symétrie, donc
u Z q Z Z
2 2
p

s
l ( 1) = 2 sin ( ) + (1 + cos ( )) d = 2 2 (1 + cos ( ))d = 2 2 cos d =8
0 0 0 2
Z
é
1

u
2
7. L’aire cherché est donnée par la formule de Green-Reimman en polaire: Aire( 1 ) = d : il s’agit
2 @ 1

ic d’intégrale curviligne, où @ 1 désigne la fronière de 1 orienté dans le sens direct.


r
Z Z
1 2 1 2

é
2 1 + cos 2 1 3 3
Donc Aire( 1 ) = (1 + cos ( )) d = 1 + 2 cos ( ) + d = 2 = :
2 0 2 0 2 2 2 2
Po r
r
PARTIE II
u
A- Questions de cours
uor
1. Voir …gure ci-contre : (permettez mes outils de dessin vectoriel modestes!)
P
r
v(θ)
e
V
T α
u (θ)
er

m Μ(θ)

θ
rm

or
2. L’abscisse curvuligne est un paramétrage admissible de l’arc ; elle consiste à choisir une origine et de
paramétrer chaque point par la longueur (algébrique) de l’arc joignant ce point à l’origine. dans ce cas la
courbe est parcourue à vitesse uniforme valant 1.
Z
0
On choisit pour origine = 0; et on oriente dans le sens des croissant. Alors s ( ) = jj (f (t) ~u (t)) jjdt =
Fo

Z 0
p ds p
2 02
f (t) + f (t)dt et 2 02
( ) = f ( ) + f ( ):
0 d
3. D’abord la fonction angulaire V est dérivable selon le théorème de relévement, et en dérivant la relation :
f( )
tan (V ( )) = 0 ; on obtient :
f ( )
f 02 ( ) f ( ) f 00 ( ) f 02 ( ) f ( ) f 00 ( ) 1 f 02 ( ) f ( ) f 00 (
V 0 ( ) 1 + tan2 (V ( )) = : D’
où V 0
( ) = f 2(
=
f 02 ( ) f 02 ( ) 1+ ) f 02 ( ) + f 02 ( )
f 02 ( )

✉: mamouni.myismail@gmail.com

2
Par ailleurs ( à la physicienne, que l’on justi…e mathématiquement à l’aide de dérivée de composée), on a
1
ds ds d ds d d dV f 02 ( ) f ( ) f 00 ( )
R= = = , or = V + ; donc = 1+ = 1+ ; ainsi
d d d d d d d f 02 ( ) + f 02 ( )
3
d f 2 ( ) + 2f 02 ( ) f ( ) f 00 ( ) f 2 ( ) + f 02 ( ) 2
( )= 2 02
et par suite : R ( ) = 2 :
d (f ( ) + f ( )) f ( ) + 2f 02 ( ) f ( ) f 00 ( )
! !
4. On a M I = R N ; donc les coordonnées de M dans le repère mobile (M; !
u ( );!
v ( )) sont cos V + ; sin V +
2 2
\!
( sin V; cos V ) où V désigne l’angle !
u ( ); T :

B - Retour à l’arc 1

1. L’arc 1 privé de son pôle O est décrit lorsque parcourt l’intervalle ] ; [ ; il est un arc birégulier.
Commençons par déterminer l’abscisse curviligne orienté d’origine = 0 , puis la normale et le rayon de
courbure :
Z Z s
p t
s( ) = 2 02
f (t) + f (t)dt = 4 cos2 dt = 4 sin et s0 ( ) = 2 cos
0 0 2 2 2
! !
! dOM dOM d 1 sin
Vecteur tangent et normal : On a T = = = 1 + cos =
ds d ds 2 cos (!
u ( );!v ( ))
0 1 2

B sin 2 C
@ A ;
cos
2 (!
u ( );!
v ( ))
0 1
3 ! B cos
Ainsi V = + et = V + = + et N =
sin V
=@ 2 C
A
2 2 2 2 cos V (!u ( );!v ( ) ) sin
2 (!u ( );!v ( ))

ds ds d 2 4
Rayon de courbure : R = = = 2 cos = cos :
d d d 2 3 3 2
0 1
! ! ! ! ! 4 B cos
En…n M I = R N ; donc OI = OM +R N =
1 + cos
+ cos @ 2 C
A ;
0 (!
u ( );!
v ( )) 3 2 sin
2 (!
u( );!
v ( ))
d’où
! 1 1 + cos 1 2 + cos (1 cos )
OI ( ) = 2 sin = sin (1 cos )
3 (!
u( );!
v ( )) 3 !!
i;j

! cos (1 cos ) 1
2. On a OM ( + ) = ; alors le rapport de cette homothétie est = . Si
sin (1 cos ) !!
i;j 3
! ! !
= (a; b) désigne les coordonées de son centre dans le repère …xe O; i ; j ; alors on aura I ( ) =
1 ! ! ! 1 ! 1 !
M ( + ) ; donc O + OI ( ) = O OM ( + ) ; en identi…ant les coordonnées, en obtient
3 3 3
1 1
a = et b = 0: Ainsi le centre de cette homothétie est : = ;0 :
2 2
! 1 1 + cos ! 1
3. On a OI ( ) = 2 sin ; donc OH ( ) = (1 + cos ) !
u ( ) : Ainsi H ( ) est l’image de
3 (!
u ( );! 3
v ( ))
1
M ( ) par l’homothétie de centre O et de rapport :
3
4. Voir …gure ci-dessous (Merci Maple) : En noir la cardioïde 1; en rouge sa développée I et en bleu la
courbe H : ( Attention Daltoniens ...)

1
5. Puisque H se déduit de 1 par homothétie de centre O et de rapport ; alors selon I-(6) et I-(7) ; on a :
3
1 8 1
l( H) = l( 1) = et Aire( H) = Aire( 1) =
3 3 3 2
✍ M AMOUNI M Y I SMAIL MAMOUNI . NEW. FR

1
.

θ
ri
0,5 θ
I()
M (θ)

ir
s
H (θ)

1,5
2 0,5
1 0
s
0 Ω

s
u

s
-0,5
é

u
-1
r

é
Corrigé Exercice I : Pr. Deyris, CPGE France
Μ(θ + π)
Po r
r
1a) La fonction f est un polynôme en x et y, donc est de classe C∞ sur R2 .
u
uor ∂f ∂f
1b) On a ( x0 , y0 ) = 3x0 2 − 3(1 + y0 2 ) et ( x0 , y0 ) = −6x0 y0 .
∂x ∂y
1c) C’est immédiat.
1d) On a, pour tout (h, k) ∈ R2 : f (1 + h, k) = (1 + 3h + 3h2 + h3 ) − 3(1 + h)(1 + k2 ) =
P
−2 + 3h2 − 3k2 + h3 − 3hk2 .
Prenons pour norme sur R2 la norme définie par k( x, y)k = max{| x |, |y|} (les normes sur R2
étant toutes équivalentes, le choix de la norme n’influe pas sur le résultat). On a alors, pour
tout (h, k) ∈ R2 , |h| ≤ k(h, k)k et donc |h3 | ≤ k(h, k)k3 = o(k(h, k)k2 ) au voisinage de (0, 0)
r
; de même, hk2 = o(k(h, k)k2 ) au voisinage de (0, 0). Par suite, toujours au voisinage de (0, 0) :
f (1 + h, k) = −2 + 3(h2 − k2 ) + o(k(h, k)k2 )
e
La forme quadratique q : (h, k) 7−→ 3(h2 − k2 ) n’a pas un signe constant (par exemple q(1, 0) >
er

0 et q(0, 1) < 0) ; on sait qu’alors f ne présente pas d’extremum en (1, 0).


m1e) De même, on a f (−1 + h, k) = 2 − 3(h2 − k2 ) + o(k(h, k)k2 ) au voisinage de (0, 0). Pour
les mêmes raisons qu’en 1d), f ne présente pas d’extremum en (−1, 0).
rm

1f) La fonction f est de classe C1 sur l’ouvert R2 . On sait qu’alors, si elle présente un extremum
r
local en un point, ce point est un point critique de f . Or on a vu que ses seuls points critiques
sont (1, 0) et (−1, 0), et que f n’y a pas d’extremum ; par suite f n’a pas d’extremum local.
o
2a) La fonction g, restriction de f à D, est continue sur D. De plus, D est un fermé de R2 (image
réciproque du fermé [0, 1] par l’application continue ( x, y) 7−→ x2 + y2 ), et est clairement borné
; c’est donc un compact de R2 .
Fo

On sait que toute fonction continue sur un compact est bornée et atteint ses bornes ; donc g a
un maximum A en un point de D, et un minimum a en un point de D.
2b) Supposons que A soit atteint en un point ( x0 , y0 ) de D ′ = D \ C. Alors la restriction de g à
D ′ présenterait un maximum en ( x0 , y0 ). De plus, D ′ est un ouvert de R2 (c’est le disque unité
ouvert) et g est C1 sur D ′ ; donc ( x0 , y0 ) serait un point critique de g, donc de f .

✉: mamouni.myismail@gmail.com

3
✍ M AMOUNI M Y I SMAIL
MAMOUNI . NEW. FR

Or on a vu que les seuls points critiques de f sont (1, 0) et (−1, 0), qui n’appartiennent pas à
D ′ ; les extremums de g ne peuvent donc pas être atteints en un point de D ′ , ils le sont donc
forcément sur C.
ri
2c) Pour tout t ∈ R : g(cos t, sin t) = cos3 t − 3 cos t(2 − cos2 t) = 2 cos t(2 cos2 t − 3).

ir
s
Quand t décrit R, cos t décrit [−1, 1]. Étudions donc les variations de ϕ : u 7−→ 2u(2u2 − 3) sur
[−1, 1]. On a, pour tout u ∈ [−1, 1], ϕ′ (u) = 6(2u2 − 1) d’où le tableau de variations :
s
√ √
u −1 −1/ 2 1/ √2 1

s

u
ϕ (u) 2 ր 2 2 ց −2 2 ր −2
Puisque (cos t, sin t) décrit C quand t décrit R, et que g(cos t, sin t) = ϕ(cos t) pour tout t, le

s
maximum de g sur C est atteint en chaque
√ point (cos t, sin
√ t) pour lequel ϕ(cos t)√est maximal,

é

u
c’est à√dire pour
√ lequel cos t = − 1/ 2. Par suite A = 2 2, et est atteint en (− 1/ 2, 1/ 2) et
(−1/ 2, −1 2).
r
√ √ √ √ √
2d) De même, a = −2 2 et est atteint en (1/ 2, 1/ 2) et (1/ 2, −1 2).

é
Po r
r
Corrigé Exercice II : Pr. Lemaire, CPGE France
u
1.
uor ∂2f
(a) Tout d'abord le théorème de Schwarz permet avec des fonctions de classe C 2 de confondre (x, y) avec
∂x∂y
∂2f
(x, y).
∂y∂x
∂f
S'il existe a de classe C 1 telle que ∀(x, y) ∈ IR2 tel que (x, y) = a(x)f (x, y) alors
P
2 2
∂x
∂ f ∂f ∂ f ∂f ∂f ∂f
(x, y) = a(x) (x, y) d'où f (x, y) (x, y) = f (x, y) a(x) (x, y) = (x, y) (x, y).
∂x∂y ∂y ∂x∂y ∂y ∂x ∂y

On en déduit que f vérie (E).


r
Réciproquement, si f vérie (E) et qu'elle ne s'annule pas sur IR, considérons g dénie sur IR2 par g(x, y) =
e
∂f ∂2f ∂f ∂f
(x, y) (x, y)f (x, y) − (x, y) (x, y)
∂g ∂x∂y ∂x ∂y
er

∂x . On a g de classe C 1 par TG et ∀(x, y) ∈ IR2 , (x, y) = = 0 ,


f (x, y) ∂y f (x, y)2
m car f vérie (E). Comme IR2 est un ouvert convexe, il existe une fonction a de classe C 1 sur IR telle que ∀(x, y) ∈ IR2 ,
∂f
g(x, y) = a(x), soit (x, y) = a(x)f (x, y).
∂x
rm

Conclusion : On a bien l'équivalence


r
(b) Soit f solution de (E) et ne s'anulant pas sur IR. On a donc l'existence du a et du 1.(a)
o
∂f
Analyse : Si on pose f (x, y) = F (x) (y xé), alors (x, y) = a(x)f (x, y) s'écrit F 0 (x) = ax(x)F (x), soit à l'aide
∂x
des équations diérentielles du 1er ordre linéaires : F (x) = λeA(x) où A est une primitive de a.
Fo

Synthèse : Notons A une primitive de a sur IR (qui existe car a est continue sur IR) et considérons g dénie sur
IR2 par g(x, y) = f (x, y)e−A(x) . Comme a est C 1 sur IR, A est C 2 sur IR et donc g est a est C 2 sur IR2 . Dérivons par
∂g ∂f
rapport à x : (x, y) = (x, y)e−A(x) − f (x, y)a(x)e−A(x) = 0. Comme IR2 est convexe, il existe une fonction ψ de
∂x ∂x
1 2 A(x)

✉: mamouni.myismail@gmail.com

4
✍ M AMOUNI M Y I SMAIL
MAMOUNI . NEW. FR

classe C sur IR telle que ∀(x, y) ∈ IR , g(x, y) = ψ(y) et donc f (x, y) = ϕ(x)ψ(y) avec ϕ(x) = e .
.
Comme on a ψ(y) = f (x, y)e−A(x) , ψ est de classe C 2 sur IR par TG et ne s'anunle pas sur IR. On en déduit que
ri
∀(x, y) ∈ IR2 , f (x, y) = ϕ(x)ψ(y) avec ϕ et ψ de classe C 2 sur IR et ne s'anunlent pas sur IR.
s

ir
Réciproquement si ∀(x, y) ∈ IR2 , f (x, y) = ϕ(x)ψ(y) avec ϕ et ψ de classe C 2 sur IR et ne s'anunlent pas sur IR,
alors f est clairement C 2 , ne s'annule pas sur IR et vérie
s
∂2f ∂f ∂f

s
f (x, y) (x, y) − (x, y) (x, y) = ϕ(x)ψ(y)ϕ0 (x)ψ 0 (y) − ϕ0 (x)ψ(y)ϕ(x)ψ 0 (y) = 0.
u
∂x∂y ∂x ∂y
Conclusion : On a bien l'équivalence

s
1
On n'a pas l'unicité car si (ϕ, ψ) convient alors (2ϕ, ψ) convient aussi !
é
2

u
(c) Si f vérie E et ne s'annule pas alors il existe ϕ, ψ C 2 et ne s'annulant pas tel que ∀(x, y) ∈ IR2 ,
r
f (x, y) = ϕ(x)ψ(y), donc f (x, 0) = ϕ(x)ψ(0) et f (0, y) = ϕ(0)ψ(y).

é
Posons alors ϕ0 (x) = g(x) , ψ0 (y) = λh(y) et ∀(x, y) ∈ IR2 , f (x, y) = ϕ0 (x)ψ0 (y), donc
Po r
r
1
f (x, 0) = ϕ0 (x)ψ0 (0) = g(x)λh(0) et f (0, y) = ϕ0 (0)ψ0 (y) = g(0)λh(y). Comme g(0) = h(0), λ = convient et
g(0)
u
h(y)
∀(x, y) ∈ IR2 , f (x, y) = g(x)
uor g(0)

Pour l'unicité si f1 et f2 sont 2 telles fonctions, on a f1 (x, y) = ϕ1 (x)ψ1 (y), et f2 (x, y) = ϕ2 (x)ψ2 (y), d'où
f1 (x, 0) = ϕ1 (x)ψ1 (0) = f2 (x, 0) = ϕ2 (x)ψ2 (0) = g(x) et
P
f1 (0, y) = ϕ1 (0)ψ1 (y) = f2 (0, y) = ϕ2 (0)ψ2 (y) = h(y) d'où il existe 2 réels α et β tels que ϕ2 = αϕ1 et ψ2 = βψ1

On en déduit que f1 (0, 0) = f2 (0, 0) = ϕ1 (0)ψ1 (0) = αβϕ1 (0)ψ1 (0) = g(0) (= h(0)) comme tout est non nul, on a
donc αβ = 1 et donc f1 = f2 et l'on a l'unicité.
r
h(y)
Conclusion : ∀(x, y) ∈ IR2 , f (x, y) = g(x) convient et est unique
e
g(0)
2.
er

m
rm

or
Fo

✉: mamouni.myismail@gmail.com

5
(a) Petite erreur d'énoncé : lire y 7−→ f (x0 , y) et non y 7−→ f (x, y0 ).
Soit (x0 , y0 ) un maximum local de f . Soit r > 0 tel que ∀(x, y) ∈ [x0 − r, x0 + r] × [y0 − r, y0 + r], f (x, y) 6 f (x0 , y0 )
et donc ∀x ∈ [x0 − r, x0 + r], f (x, y0 ) 6 f (x0 , y0 ) d'où x0 est un maximum local de x 7−→ f (x, y0 ). On fait de même
pour y 7−→ f (x0 , y) qui admet un maximum local en y0 .
Réciproquement soient x0 un maximum local de x 7−→ f (x, y0 ) et y0 un maximum local de y 7−→ f (x0 , y).
Il existe ϕ et ψ telle que ∀(x, y) ∈ IR2 , f (x, y) = ϕ(x)ψ(y). Comme f ne s'annule pas, ϕ et ψ non plus. Par valeurs
intermédiaires elles ne changent donc pas de signe. Quitte à changer ϕ en −ϕ et ψ en et −ψ , on peut supposer que
ϕ > 0 et comme f est positive, on en déduit que ψ aussi.
f (x, y0 )
D'autre part, ϕ(x) = , donc ϕ admet un maximum en x0 . De même ψ admet un maximum en y0 . On a
ψ(y0 )
donc l'existence d'un r > 0 tel que ∀x ∈ [x0 − r, x0 + r] ϕ(x) 6 ϕ(x0 ) et ∀x ∈ [y0 − r, y0 + r] ψ(y) 6 ψ(y0 ) et
Comme tout est strictement positif, on en déduit que
∀x ∈ [x0 − r, x0 + r] × [y0 − r, y0 + r] f (x, y) = ϕ(x)ψ(x) 6 ϕ(x0 )ψ(y0 ) = f (x0 , y0 ).

Conclusion :
f admet un maximum local en (x0 , y0 ) SSI x0 est un max. local de x 7−→ f (x, y0 ) et y0 un max. local de y 7−→ f (x0 , y).

(b) Soit A = {x0 ∈ IR tel que x0 soit un maximum de x 7−→ f (x, y0 )} et B = {y0 ∈ IR tel que y0 soit un maximum
de y 7−→ f (x0 , y)}. On a alors
Conclusion : A × B = {(x0 , y0 ) tel que f admet un maximum local en (x0 , y0 ) }
3.
(a) Considérons
 h dénie sur IR par 
h(t) = t3 + |t|3 . On a donc 
 0 si t < 0
  0 si t < 0
  0 si t < 0

h(t) = d'où h (t) =
0
et h (t) =
00
 2t3 si t > 0
  6t2 si t > 0
  12t si t > 0

Les limites de h, h0 et h00 à gauche et à droite en 0 sont toutes égales à 0. Par le théorème de prolongement des
fonctions de classe C k , h est donc de classe C 2 sur IR. On en déduit par TG que f est C 2 sur IR2 ( f (x, y) = h(xy) ).
∂f ∂f ∂2f
(b) On a ∀(x, y) ∈ IR2 , (x, y) = yh0 (xy) (x, y) = xh0 (xy) et (x, y) = h0 (xy) + xyh00 (xy)
∂x ∂y ∂x∂y 
∂2f 0 si xy < 0

∂f ∂f 
f (x, y) (x, y)− (x, y) (x, y) = h(xy) h0 (xy)+xyh00 (xy)−yh0 (xy)xh0 (xy) =
∂x∂y ∂x ∂y  36x5 y 5 − 36x5 y 5 − = 0 si xy > 0

Conclusion : f vérie (E)


(c) Supposons qu'il existe ϕ et ψ telles que ∀(x, y) ∈ IR2 , f (x, y) = ϕ(x)ψ(y) f (1, 1) = ϕ(1)ψ(1) = 2 donc ϕ(1) 6= 0
et ψ(1) 6= 0, f (−1, −1) = ϕ(−1)ψ(−1) = 2 donc ϕ(−1) 6= 0 et ψ(−1) 6= 0 et enn f (−1, 1) = ϕ(−1)ψ(1) 6= 0 or
f (−1, 1) = 0 car (−1, 1) est dans la zone xy < 0 : Absurde !

Conclusion : Il n'existe pas ϕ et ψ telles que ∀(x, y) ∈ IR2 , f (x, y) = ϕ(x)ψ(y)


✍ M AMOUNI M Y I SMAIL
MAMOUNI . NEW. FR

Devoir Libre
14 Valeur moyenne d’une fonction
Enoncé : CNC 2006, MP

Définitions et notations
Dans ce problème, E désigne le R -espace vectoriel des applications continues de R+ dans R, et
E2 le sous ensemble de E formé des applications de carrés intégrables sur R+ .
À toute fonction f ∈ E on associe la fonction, notée ψ(f ), définie sur R+ par
Z
1 x
ψ(f )(0) = f (0) et ∀ x > 0, ψ(f )(x) = f (t) dt.
x 0

Si Φ est un endomorphisme de E, on dit que λ ∈ R est une valeur propre de Φ s’il existe f ∈ E
tel que Φ(f ) = λf et f 6= 0 ; dans ce cas, on dit que f est un vecteur propre de Φ associé à λ et
Ker (Φ − λidE ) s’appelle alors le sous-espace propre de Φ associé à la valeur propre λ.

Première partie
1. Soient a et b deux réels strictement positifs.

e−at − e−bt
1-1. Montrer que la fonction t 7−→ est intégrable sur ]0, +∞[.
Z t+∞ −at
e − e−bt
Dans la suite, on pose I(a, b) = dt.
0 t
1-2. Montrer que I(a, b) = −I(b, a) et que I(a, b) = I(1, b/a).
Z +∞ −t
e − e−xt
1-3. On note ϕ l’application définie, pour tout x > 1, par ϕ(x) = dt.
0 t
1-3-1. Montrer que ϕ est continue sur l’intervalle [1, +∞[.
1-3-2. Montrer que ϕ est de classe C 1 sur l’intervalle [1, +∞[ et calculer ϕ0 (x) pour x > 1.
1-3-3. Que vaut alors ϕ(x) pour x > 1 ?
1-4. En déduire soigneusement la valeur de l’intégrale I(a, b) en fonction de a et b.
ln(1 + t)
2. 2-1. Montrer que la fonction t 7−→ est intégrable sur l’intervalle ]0, 1].
t
X xn
2-2. Préciser le rayon de convergence et la somme de la série entière (−1)n .
n+1
n>0
2-3. Montrer que cette série entière converge uniformément sur le segment [0, 1].
+∞
X Z 1
1 π2 ln(1 + t) π2
2-4. On rappelle que = ; montrer alors que dt = .
n2 6 0 t 12
n=1

✉: mamouni.myismail@gmail.com
✍ M AMOUNI M Y I SMAIL
MAMOUNI . NEW. FR

. Deuxième partie
1. Soit f un élément de E ; on note g la fonction définie sur R+ par
Z x
∀ x > 0, g(x) = f (t) dt .
0

1-1. Justifier que g est de classe C 1 sur R+ et que la fonction ψ(f ) est un élément de E.
1-2. On suppose que la fonction f tend vers une limite finie λ lorsque x tend vers +∞ ;
montrer qu’il en est de même de la fonction ψ(f ). Étudier la réciproque.
1-3. Que peut-on dire dans le cas où cette limite est égale à +∞ ?
1-4. On pose h(x) = xf (x), x > 0.
1-4-1. Montrer que g − ψ(g) = ψ(h).
1-4-2. En déduire que si f est intégrable sur [0, +∞[ alors ψ(h) admet 0 comme limite en
+∞. Étudier la réciproque.
√ p
1-5. Montrer que si f est positive alors, 0 6 ψ( f ) 6 ψ(f ) ; dans quel cas y’a t-il égalité ?

2. 2-1. Montrer que ψ est un endomorphisme de l’espace vectoriel E.


2-2. Montrer que ψ est injectif.
2-3. L’endomorphisme ψ est-il surjectif ?

3. Soit λ un réel non nul.

3-1. Déterminer les applications f de ]0, +∞[ dans R dérivables et vérifiant

∀ x > 0, λxf 0 (x) + (λ − 1)f (x) = 0.

3-2. Pour quelles valeurs du réel λ ces fonctions sont-elles prolongeables à droite en 0 ?

4. 4-1. Est-ce que 0 est valeur propre de ψ ?


4-2. Montrer que si f ∈ E est un vecteur propre de ψ associé à une valeur propre µ alors f est
une fonction dérivable sur ]0, +∞[.
4-3. Déterminer l’ensemble des valeurs propres de ψ et préciser pour chacune d’elles le sous-
espace propre associé.

Troisième partie
1. 1-1. Montrer que si f et g sont deux éléments de E2 , leur produit f g est une fonction intégrable
sur R+ .
1-2. Montrer alors que E2 est un sous-espace vectoriel de E.
Z +∞
1-3. Montrer que l’application (f, g) 7−→ f (t)g(t) dt est un produit scalaire sur E2 .
0

Dans la suite, ce produit scalaire se notera (.|.) et k.k désignera la norme associée.

2. Soit f un élément de E2 ; on note toujours g la fonction définie sur R+ par


Z x
∀ x > 0, g(x) = f (t) dt .
0

✉: mamouni.myismail@gmail.com
✍ M AMOUNI M Y I SMAIL MAMOUNI . NEW. FR

g 2 (t)
2-1. Calculer la limite en 0+ de la fonction t 7−→ t .
g 2 (t)
. 2-2. Montrer que, pour tout réel b > 0, la fonction t 7−→ t2
est intégrable sur ]0, b] et que
Z b Z b Z b
2 g 2 (t)
ψ(f ) (t) dt = dt = −bψ(f )2 (b) + 2 f (t)ψ(f )(t) dt. (1)
0 0 t2 0

2-3. En déduire que, pour tout réel b > 0,


Z b µZ b ¶ 12 µZ b ¶ 12
ψ(f )2 (t) dt 6 2 f 2 (t) dt ψ(f )2 (t) dt .
0 0 0

2-4. Conclure que ψ(f ) ∈ E2 et que kψ(f )k 6 2kf k.


2-5. On note ψ2 l’endomorphisme induit par ψ sur E2 . Que peut-on alors dire de ψ2 en tant
qu’endomorphisme de l’espace vectoriel normé (E2 , k.k) ?

3. Soit f un élément de E2 .

3-1. En utilisant la formule (1) montrer que la fonction x 7−→ xψ(f )2 (x) tend vers 0 lorsque x
tend vers +∞.
3-2. Montrer alors que (ψ(f )|ψ(f )) = 2(f |ψ(f )).

4. Soit f ∈ E2 une fonction telle que kψ(f )k = 2kf k. Calculer kψ(f ) − 2f k2 et montrer que f est
la fonction nulle.

Quatrième partie
1. On considère un réel a > 0 et on note fa la fonction définie sur R+ par fa (x) = e−ax , x > 0.

1-1. Montrer que la fonction fa ∈ E2 et calculer kfa k2 .


kψ(fa )k
1-2. Calculer ψ(fa )(x) pour tout x > 0 puis donner les valeurs de (fa |ψ(fa )) et de .
kfa k
1
2. On considère la fonction f définie sur R+ par f (x) = , x > 0.
x+1
2-1. Calculer ψ(f )(x) pour tout x > 0.
Z 1³
ln(1 + t) ln t ´
2-2. Vérifier que f ∈ E2 et montrer que (f |ψ(f )) = − dt.
0 t 1+t
ln(1 + t) ln t kψ(f )k
2-3. Trouver une primitive de la fonction t 7−→ + puis calculer .
t 1+t kf k
3. Montrer plus généralement que si f ∈ E2 est positive, décroissante et non nulle, alors

kψ(f )k √
> 2.
kf k

kψ(f )k
4. 4-1. Montrer que l’application f 7−→ est continue sur E2 \ {0}.
kf k
n o
4-2. En déduire que kψ(f
kf k
)k
; f ∈ E2 \ {0} est un intervalle contenu dans ]0, 2[.

✉: mamouni.myismail@gmail.com
✍ M AMOUNI M Y I SMAIL
MAMOUNI . NEW. FR

5. Dans cette question, on va montrer ces deux ensembles coı̈ncident.

5-1. Pour tout s ∈] − 1, − 12 [ on note fs la fonction définie sur R+ par

fs (0) = 0, fs (t) = ts si t > 1 , et fs affine sur [0, 1].

5-1-1. Vérifier que fs ∈ E2 et calculer kfs k2 en fonction de s puis en donner un équivalent


lorsque s tend vers − 12 .
5-1-2. Calculer kψ(fs )k2 en fonction de s et en donner un un équivalent lorsque s tend vers
− 12 .
n o
5-1-3. En déduire que la borne supérieure de l’ensemble kψ(f kf k
)k
; f ∈ E2 \ {0} vaut 2.
5-2. Soit α > 0 ; on note f la fonction définie sur R+ par

f (t) = tα si t ∈ [0, 1], et f (t) = t−α−1 si t ∈ [1, +∞[.

5-2-1. Vérifier que f ∈ E2 et calculer kf k2 en fonction de α.


5-2-2. Calculer kψ(f )k2 en fonction de α et en donner un équivalent au voisinage de +∞.
n o
5-2-3. En déduire que la borne inférieure de l’ensemble kψ(f kf k
)k
; f ∈ E2 \ {0} est nulle.

i
F
nn

i
Á la prochaine

✉: mamouni.myismail@gmail.com
i

r
s
P ROBLÈMES C ORRIGÉS -MP ✍ M AMOUNI M Y I SMAIL
MAMOUNI . NEW. FR

i
s

s
u
Ibn Battuta, (1304 Tanger-1369 Marrakech)

s
Blague du jour

Mathématicien du jour
. Explorateur et voyageur marocain, parcourant 120 000 km en 28

u
Le professeur de chimie inscrit la formule HN03 sur le ans de voyages qui l’amènent à l’Inde, la Chine, au Kazakhstan,
tableau. Il interroge ensuite un élève : l’Andalousie, au Mali. Ses récits, compilés par Ibn Juzayy en un

r
- Que signifie cette formule ? livre appel Rihla (voyage) sont plus précis que ceux de Marco

é
- Heu, je l’ai sur le bout de la langue, monsieur ! Polo, mais contiennent plusieurs passages qui relèvent claire-
- Crachez-la tout de suite, c’est de l’acide nitrique ! ment de la pure imagination, notamment ceux décrivant des

Po r
r
êtres surnaturels.

u
 

uor
b
Corrigé : Pr. Mamouni, CPGE Rabat, Maroc I 1,
a
.

c
e−t − e− xt

P
1ère Partie i L’application : f : ( x, t) 7→ est continue sur

t
[1, +∞[×R en tant que somme, rapport de fonctions con-
① a Au voisinage de 0 : On sait que et = 1 + t + o(t), donc
tinue, qui ne s’annule pas. En ( x, 0) on a : f ( x, t) ∼ x − 1
e− at − e−bt continue, donc f est continue sur [1, +∞[×R.

r
= b − a + o(1) ∼ b − a intégrable au voisinage de
t D’autre part : pour x ∈ [ a, b] ⊂ [1, +∞[ on a :
0.   e−t − e− xt e−t − e− xt e−t − e−bt

e
− at 1 = ≤ qui est continue, inté-
Au voisinage de +∞ : On sait que e = o , donc t t t
er
t
  grable sur ]0, +∞[, donc ϕ est continue sur [1, +∞[.

m
e− at − e−bt 1
= o 2 intégrable au voisinage de +∞. ∂f
t t ii Pour x ∈ [ a, b] ⊂ [1, +∞[ on a : = e− xt ≤ e− at
∂x
b I (a, b) = − I (b, a), trés evident. continue, intégrable sur [0, +∞[. Donc ϕ est de classe C 1 sur
m

Z +∞
Posons : u = ta, donc : 1

r
Z +∞ − at Z +∞ −u b [1, +∞[, avec ϕ′ ( x ) = e− xt dt = .
e − e−bt e − e− a u 0 x

o
I (a, b) = dt = du = iii D’aés le raisonnement fait dans la question précé-
0 t 0 u
or

F
✉: mamouni.myismail@gmail.com

160
i

r
s
✍ M AMOUNI M Y I SMAIL
MAMOUNI . NEW. FR
P ROBLÈMES C ORRIGÉS -MP

i
s

s
u
1 (−1)n n
dente, on a : ϕ′ ( x ) = , donc ϕ( x ) = ln x + K, or ϕ(1) = 0, ∑ n + 1 x est une série alternée, donc vérifie le critère spé-

s
x n ≥0
d’où K = 0 et donc ϕ( x ) = ln x.
cial, en prticulier la majoration du reste par son 1ér terme,

u
b b (−1)k k (−1)n n 1
d Si b ≥ a, alors x = ≥ 1, donc I (a, b) = I (1, ) = donc ∑ x ≤ x ≤ , donc le reste con-
    a a k+1 n+1 n+1

r
b b k≥ n

é
ϕ = ln . verge uniformément vers 0, et par suite la convergence de la
a a
a série sur [0, 1] est uniforme.
Si b ≤ a, alors x = ≥ 1, donc :

Po r
r
b a a
a
I (a, b) = − I (b, a) = − I (1, ) = − ϕ = − ln =

u
  b b b
b

uor
ln .
a  
b
Conclusion : I (a, b) = ln .
a

P
② a Au voisinage de 0 : on sait que ln(1 + t) = t + o(t),
ln(1 + t)
d’où ∼ 1 intégrable au voisinage de 0, donc t 7→
t
ln(1 + t)

r
est intégrable sur ]0, 1].
t

e
(−1)n an +1
b Posons an = , on a lim = 1, donc
er
n+1 n →+ ∞ an

m
(−1)n n
le rayon de convergence de la série ∑ x est égal
n ≥0
n+1
ln(1 + x )
à 1, dont la somme est , puisqu’il s’agit de son
m

r
développement en série entière.

o
c Pour x ∈ [0, 1] fixé, on vérifie faciulement que la série
or

F
✉: mamouni.myismail@gmail.com

161
i

r
s
P ROBLÈMES C ORRIGÉS -MP ✍ M AMOUNI M Y I SMAIL
MAMOUNI . NEW. FR

i
s

s
u
Z 1 Z 1 +∞
ln(1 + t) (−1)n n ① a g est de classe C 1 , en tant que primitive de f qui est con-
∑ D’aprés 2.2

s
d dt = t dt
0 t 0 n =0 n+1 tinue.
+∞ Z 1 g( x )

é
(−1)n n On a ψ( f )( x ) = pour x > 0, donc ψ est continue sur

u
= ∑ t dt x
n =0 0 n + 1 R+.∗

r
Car la convergence est uniforme sur [0,1] Pour x 6= 0, le théorème des accroissement finie, donc g( x ) −

é
+∞
(−1)n g(0) = xg′ (c) avec c compris entre 0 et x, d’où ψ( f )( x ) =
= ∑ 2
n =0 ( n + 1) f (c) −→ f (0) = ψ( f )(0) car g(0) = 0 et g′ = f continue,

Po r
r
+∞
1 +∞
1 donc ψ( f ) est continue sur R + , autrement dit ψ( f ) ∈ E.
= ∑ 2
− ∑ 2
p=1 (2p + 1) p=0 (2p + 2)

u
On divise la somme en deux n = 2p, n = 2p + 1

uor
+∞ +∞ +∞
1 1 1
= ∑ 2−∑ 2
− ∑ 2
n =1
n p=1
(2p) p=0 (2p + 2)
+∞ +∞
1 1
= ∑ n2 − 2 ∑ (2p)2

P
n =1 p=1
+∞ +∞
1 1
Car ∑ (2p)2 = ∑ 2
p=1 p=0 (2p + 2)

r
+∞ +∞
1 1 1
= ∑ n2 − 2 ∑ p2
n =1 p=1

e
+∞
1 1
er
= ∑
2 n = 1 n2

m
+∞ +∞
1 1
Car ∑ n2 = ∑
n =1 p=1
p2
m

π2
=

r
12 b lim f (t) = λ =⇒ ∀ε > 0, ∃ A > 0 tel que | f (t) − λ| ≤
t→+ ∞
ε
2ème Partie

o
∀t ≥ A, donc pour x ≥ A on a :
2
or

F
✉: mamouni.myismail@gmail.com

162
i

r
s
✍ M AMOUNI M Y I SMAIL
MAMOUNI . NEW. FR
P ROBLÈMES C ORRIGÉS -MP

i
s

s
Z A 

u
Z Z x
1 x 1
| ϕ ( x ) − λ| = f (t)dt − λx ϕ( f )( x ) = f (t)dt + f (t)dt

s
x Z0 Z x x 0 A

1 x 1 B

é
= f (t)dt − λdt ≥ K + ( x − A)

u
x Z0 0 x 2
1 x K x−AB
= ( f (t) − λ)dt = +

r
xZ0 x x 2

é
1 x K x−AB B
≤ | f (t) − λ| dt ≥ B car lim + =
x Z0 x →+ ∞ x x 2 2
Z Donc lim ψ( f )( x ) = +∞.

Po r
1 A 1 A

r
= | f (t) − λ| dt + | f (t) − λ| dt x →+ ∞
x 0 Z x x

u
K 1 A d
= + | f (t) − λ| dt

uor
x x Zx
K 1 Aε i Dans ψ(h) on va utiliser une intégration par partie, en
≤ + dt posant u = x, v′ = f , donc u′ = 1, v = g,
x x x 2 Z  d’où : Z x 
K x−Aε 1 x 1 x
= + ψ(h)( x ) = t f (t)dt = [tg(t)] 0 − g(t)dt
x x 2 x 0 x 0 .

P
Z x
1
K ε x−A = g( x ) − g(t)dt = g( x ) − ψ( g)( x )
≤ + car ≤1 x 0
Z x
x 2 x
K ii f est intégrable sur [0, +∞[, donc g( x ) = f (t)dt
≤ ε car lim =0 0

r
x →+ ∞ x
admet une limite finie en +∞, d’aprés la question 1.2) ψ(h)
La réciproque est fausse, prenons pour contre-exemle la fonc-
sin x admet aussi la même limite en +∞, or ψ(h) = g − ψ( g), donc

e
tion f (t) = cos t, on a : ψ( f )( x ) = −→ 0 quand lim ψ(h)( x ) = 0.
x x →+ ∞
er
x −→ +∞, alors que lim cos x n’existe pas. La réciproque n’est pas toujours vraie, prenons pour contre-

m
x →+ ∞
e− x
exemple f ( x ) = , non intégrable au voisinage de 0, car
x Z
e− x 1 1 x −t 1
∼ , alors que ψ(h)( x ) = e dt = (1 − e− x ) −→
m

x x x 0 x
c lim f (t) = +∞ =⇒ ∀ B > 0, ∃ A > 0 tel que f (t) ≥

r
0, quand x −→ +∞.
t→+ ∞ Z q
B p p 1 x
e f ≥ 0 et x ≥ 0, donc ψ( f )( x ) = f (t) dt ≥ 0.

o
∀t ≥ A, donc
2 x 0
or

F
✉: mamouni.myismail@gmail.com

163
i

r
s
P ROBLÈMES C ORRIGÉS -MP ✍ M AMOUNI M Y I SMAIL
MAMOUNI . NEW. FR

i
s

s
u
D’autre part : en utilisant l’inégalité de r
Cauchy-schwarz pour b f est prolongeable en 0+ si et seulement si lim f ( x ) est
Z x rZ x

s
p Z xq x→
1 1 1−λ
1 et f , on aura : f (t) dt ≤ dt f (t)dt .
finie si et seulement si ≥ 0si et seulement si 0 < λ ≤ 1.

é
x 0 x 0 0

u
λ
r Z q ④ a 0 ne peut pas être une valeur propre de ψ car elle est
1 x

r
= f (t)dt = ψ( f ) injective.

é
x 0
On aura égalité, s’il
p y a égalité dans l’inégalité de Cauchy- b Soit f ∈ E non nulle telle que ψ( f ) = µ f , donc
schwarz pour 1 et f , donc s’ils sont proportionnels, c’est à 1

Po r
r
dire f est constante. f = ψ( f ) car µ 6= 0 d’aprés 4.1). De plus d’aprés 1.1) on
µ

u
② a Il est clair que ψ( f + λg) = ψ( f ) + λψ( g), n’oubliez pas peut affirmer que ψ( f ) est de classe C 1 sur R ∗+ , donc f aussi.

uor
de le mentionner pour x = 0, donc ψ est linéaire. c Soit λ valeur propre de ψ et f vecteur propr associé, donc
D’autre part d’aprés 1.1) ψ( f ) ∈ E, ∀ f ∈ E, donc ψ est un Z x
endomorphisme de E. ψ( f )( x ) = λ f ( x ), d’où f (t)dt = λx f ( x ), en dérivant cette
0
b f ∈ Ker (ψ) =⇒ ψ( f )( x )Z= 0, ∀ x > 0 égalité on obtient : λx f ′ ( x ) + (λ − 1) f ( x ) = 0, dont les solu-

P
x tions sont :
=⇒ g( x ) = f (t)dt = 0, ∀ x > 0 1−λ
0 f ( x ) = Kx λ , dérivables sur ]0, +∞[ pour tout λ ∈]0, 1].
=⇒ g′ ( x ) = f ( x ) = 0, ∀ x ≥ 0
Donc ψ est injective. 3ème partie

r
c D’aprés 1.1) on peut affirmer que ψ( f ) est de classe C 1
① a Pour tout segment [ a, b] ⊂ R + , on a d’aprés l’inégalité de
sur R ∗+ , donc toute fonction de E qui ne l’est pas ne peut pas

e
s s
être de la forme ψ( f ), c’est à dire n’admet pas d’antécédant, Z b Z b Z b
er
2
donc ψ n’est pas surjective. F( x ) = | x − 1| est un exemple Cauchy-Schwarz : f (t) g(t)dt ≤ f (t)dt g2 (t)dt

m
a a s a s
de fonction de E qui n’est pas de classe C 1 sur R ∗+ , car non Z +∞ Z +∞
2
dérivable en 1. ≤M= f (t)dt g2 ( t )
0 0
③ a Il s’agit d’une équation différentielle linéaire du 1ér or-
m

+
Donc f g est intégrable sur R
dre à coéfficients
Z x non constant, dont la solution est :

r
λ−1 b Il est clair que l’application nulle est de carré intégrable,
− tdt 1 − λ ln x 1−λ

o
f ( x ) = Ke 0 λ = Ke λ = Kx λ . donc appartient à E2 , d’autre part, soit ( f , g) ∈ E2 , λ ∈ R,
or

F
✉: mamouni.myismail@gmail.com

164
i

r
s
✍ M AMOUNI M Y I SMAIL
MAMOUNI . NEW. FR
P ROBLÈMES C ORRIGÉS -MP

i
s

s
u
alors : 1
v = − , d’où :

s
( f + λg)2 = f 2 + 2λ f g + g2 car f 2 , f g, g2 sont toutes inté- Z b 2
t  2 b Z b ′
grables, donc f + λg ∈ E2 et par suite E2 est un sous-espace g (t) g (t) g (t) g(t)

é
+2

u
2
dt = − dt
vectoriel de E. 0 t t 0 Z 0 t
c . g2 ( b ) b g′ (t) g(t)

r
=− +2 dt
Z +∞ Z +∞ b t

é
0
☛ Symétrie : ( f , g) = f (t) g(t)dt = g(t) f (t)dt = g2 ( t )
0 0 car : lim =0
t → 0+ t

Po r
( g, f ).

r
Z b
g2 ( b )
☛ Bilinéarité : ( f + λg, h) = ( f , h) + λ( g, h), car l’intégrale =− +2 f (t)ψ( f )(t)dt
b 0

u
est linéaire, d’où la linéarité à gauche, à l’aide de la symétrie g(t)
on conclut la bilinéarité. car : g′ (t) = f (t), = ψ( f )(t)

uor
Z +∞ t
☛ Positive : ( f , f ) = f 2 (t)dt ≥ 0. Z b Z b
0 c 2
Z +∞ ψ( f ) (t)dt ≤ 2 f (t)ψ( f )(t)dt D’aprés (1) ,
☛ Définie : ( f , f ) = 0 =⇒ f 2 (t)dt = 0 =⇒ f 2 = 0, car 0 s0
Z b
s
Z b
0

P
2
2
f continue positive, donc f = 0. ≤2 f (t)dt ψ( f )2 (t)dt
0 0
D’aprés l’inégalité de Cauchy-Shwarz.
g2 ( t ) Z b
② a = g(t)ψ( f )(t) −→ g(0)ψ( f )(0) = 0, quand
t Si ψ( f )2 (t)dt = 0, c’est terminé, sinon on peut simplifier

r
t −→ 0 , car g et ψ( f ) sont continues sur R + et g(0) = 0.
+ 0
avec et on obtient encore le résultat demandé.
g2 ( t )

e
b = (ψ( f )(t)) 2 −→ (ψ( f )(0)) 2 , quand t −→ 0+ , car d Découle immédiatement de 2-4) en faisant tendre b vers
t2
er
g2 ( t ) +∞.

m
ψ( f ) est continue sur R + , donc t 7→ est intégrable sur
t2 e D’aprés 2-5) on peut conclure que ψ2 est 2-lipshitzienne,
]0, b] car prolongeable par continuité en 0+ .
Z b Z b 2 donc continue.
2 g (t)
m

D’autre part : ψ( f ) (t)dt = dt, par définition de


0 0 t2 ③ a .

r
ψ( f ), pour l’autre égalité on va utiliser une intégration par
1
parties, avec u = g2 (t), v′ = 2 , donc u′ = 2g′ (t) g(t) et

o
b Faire tendre b vers +∞ dans (1), en utilisant 3-1).
t
or

F
✉: mamouni.myismail@gmail.com

165
i

r
s
P ROBLÈMES C ORRIGÉS -MP ✍ M AMOUNI M Y I SMAIL
MAMOUNI . NEW. FR

i
s

s
u
④ ||ψ( f ) − 2 f ||2 = (ψ( f ) − 2 f , ψ( f ) − 2 f ) ||ψ( f a )|| √
D’où : = 2 ln a.

s
= (ψ( f ), ψ( f )) − 4(ψ( f ), f ) + 4( f , f ) || f a ||
= ||ψ( f )||2 − 4(ψ( f ), f ) + 4|| f ||2

é
Z

u
1 x 1 ln(1 + x )
= −4(ψ( f ), f ) + 8|| f ||2 Car : ||ψ( f )|| = 2|| f || ② a Pour x 6= 0, on a : ψ( f )( x ) = dt = .
x 0 1+t x
= −4(ψ( f ), f ) + 2||ψ( f )||2 Car : ||ψ( f )|| = 2|| f ||

r
Pour x = 0, on a : ψ( f )(0) = f (0) = 1.
= 0 D’aprés 3-2)

é
Donc ψ( f ) − 2 f = 0, ainsi si f 6= 0, on aurait 2 est une valeur b Au voisiange de 0 : f 2 ( x ) ∼ 1
propre de ψ, impossible puisque les valeurs propres de ψ sont 1

Po r
r
les λ ∈]0, 1]. Au voisinage de +∞ : f 2 ( x ) ∼ 2 , donc f 2 est intégrable sur
x
R + , or f continue, donc f ∈ E2 .

u
4ème partie Z +∞

uor
( f |ψ( f )) = f (t)ψ( f )(t)dt
Z0 + ∞
① a f a2 ( x ) = e−2ax est évidement intégrable sur R + , avec : ln(1 + t)
= dt
Z +∞
1 Z0 1
t (1 + t ) Z +∞
2
|| f a || = e−2ax dx = . ln(1 + t) ln(1 + t)

P
0 2a = dt + dt
Z 0 t (1 + t ) 1 t(1 + t)
1 x − at 1 − e− ax
b Pour x 6 = 0, on a : ψ ( f a )( x ) = e dt = . Z 1 Z 1 ln 1 + u
x 0 ax ln(1 + t) u 1
Pour x = 0, on a : ψ( f a )(0) = f a (0) = 1. = dt + du Avec : u =
Z +∞ 0  t (1 + t ) 0  1 +u t
1

r
( f a , ψ( f a )) = f a ( x )ψ( f a )( x )dx Z 1 ln + t
0Z =  ln(1 + t) + t  dt On remplace u par t
1 +∞ e− ax − e−2ax t (1 + t ) 1+t

e
0
= dx Z 1
a 0 x (1 + t) ln(1 + t) − t ln t
er
1 = dt
= I (a, 2a)

m
Z0 1 
t (1 + t ) 
a
ln a ln(1 + t) ln t
= D’aprés 1-4 de la 1ère partie = − dt
0 t 1+t
 2 a
m

||ψ( f a )|| ln(1 + t) ln t


= 2a(ψ( f a ), ψ( f a ) D’aprés 1-1 c (ln t ln(1 + t)) ′ = , donc ln t ln(1 + t)

r
|| f a || +
. t 1+t
= 4a( f a , ψ( f a )) D’aprés 3-2, 3ème partie ln(1 + t) ln t
est une primitive de .

o
= 4 ln a +
t 1+t
or

F
✉: mamouni.myismail@gmail.com

166
i

r
s
✍ M AMOUNI M Y I SMAIL
MAMOUNI . NEW. FR
P ROBLÈMES C ORRIGÉS -MP

i
s

s
u
Z 1 Z 1
ln(1 + t) ln t f 2 est intégrable car son intégrale sur R + est égale à celui sur
Calculons d’abord : dt et dt, en effet :

s
Z a Z a+1
Z 1 0 t Z 1 0 1+t [0, a + 1], avec : || f ||2 = t2s dt − a2s (t − a − 1)2 dt
ln(1 + t) ln t
[ln t ln(1 + t)] 10

é
dt = − dt 0 a

u
0 t 1+t
0 a2s+1 a2s a2s+1
Intégration par parties avec : = − ∼
2s + 1 3 2s + 1

r
1
u = ln(1 + t) v′ =

é
t ii D’abord pour 0 ≤ x ≤ a, on a :
′ 1 Z Z
u = v = ln t 1 x 1 x s xs
Z 11 + t , car :

Po r
ψ( f )( x ) = f (t)dt = t dt =

r
ln t x 0 x 0 s+1
=− dt 1
0 1+t 2s + 1 > 0 =⇒ s > − =⇒ s + 1 > 0 =⇒ lim x s+1 = 0.

u
Car au voisinage de 0+ : ln t ln(1 + t) ∼ t ln t −→ 0 2 x → 0+

uor
D’autre part :
③ . Z +∞ Z a
2 2
||ψ( f )|| = ψ( f ) ( x )dx ≥ ψ( f )2 ( x )dx =
④ a les application f 7→ || f || et f 7→ ψ( f ) sont continue, or 0 0
Z a
||ψ( f )|| x2s a2s+1 2a2s+11
f 6= 0, donc l’application f 7→ est continue en tant 2
dx = 2
= . ≥

P
|| f || 0 ( s + 1) (s + 1) (2s + 1) (s + 1)(2s + 1) 2(s + 1)
que composée et rapport d’applications continues. 2a2s+1
  , car 2(s + 1) = 2s + 2 > 1.
||ψ( f )|| (s + 1)(2s + 1)
b tel que f ∈ E2 − 0 est un connexe dans R
|| f ||

r
en tant qu’image d’un connexe par une application continue, iii D’aprés les deux questions précèdentes, en faisant
 
||ψ( f )|| ||ψ( f )||2 2
d’autre part : 0 < < 2, puisque ψ( f ) est injective et tendre a vers +∞, on aura : Sup

e
2
≥ ∀s ∈
|| f || || f || s+1
er
d’aprés la question 2-4) 3ème partie, donc c’est un intervalle 1
R tel que 2s + 1 > 0, donc pour s ≥ − , en faisant ten-

m
contenu dans ]0, 2[.  2 
1 ||ψ( f )||2
⑤ a dre s vers − , on obtient : Sup ≥ 4, d’où :
 2
 || f ||2
m

i L’application f est définie ainsi : ||ψ( f )||


Sup ≥ 2, or d’aprés la question 4.2) on a :

r
f (t) = ts si : 0 ≤ t ≤ a  || f || 
= − as (t − a − 1) si : a ≤ t ≤ a + 1 ||ψ( f )||
Sup ≤ 2, d’où l’égalité.

o
=0 si : t ≥ a + 1 || f ||
or

F
✉: mamouni.myismail@gmail.com

167
i

r
s
P ROBLÈMES C ORRIGÉS -MP ✍ M AMOUNI M Y I SMAILMAMOUNI . NEW. FR

i
s

s
u
Z +∞
2
b ||ψ( f )|| = ψ( f )2 ( x )dx

s
0
Z 1 Z +∞  2
i x2α 2α + 1 1

é
= dx + − dx

u
1 0 ( α + 1)
2 xα(α + 1) αx α+1
ii Au voisinage de +∞ on a : f 2 (t) = 2α+2 est bien in- 1
t 1 (2α + 1)2 2(2α + 1)

r
tégrable car 2α + 2 > 1, avec : = 2
+ 2 2
− 2
Z +∞ Z 1 Z +∞ (2α + 1)(α + 1) α ( α + 1) α ( α + 1 )2

é
2 2 2α 1 1
|| f || = f (t)dt = t dt + dt + 2
0 0 1 t +2

α (2α + 1)
1 1 2

Po r
r
= + = 1 4α2 − 1 1
2α + 1 2α + 1 2α + 1 = + +
(2α + 1)(α + 1)2 α2 (α + 1)2 α2 (2α + 1)

u
iii Déterminons d’abord ψ( f )( x ) pour x ≥ 0. 4
1ér cas : 0 ≤ xZ ≤ 1, alors : Z ∼+∞ 2

uor
α
1 x 1 x α xα
ψ( f )( x ) = f (t)dt = t dt = . iv D’aprés les deux questions précèdentes, on aura :
x 0 x 0 α+1  
2ème cas : x ≥ 1, alors : Z 1  ||ψ( f )||2 2(2α + 1)
Z Z x Inf ≤ pour α > 0 assez grand, quand
1 x 1 || f || 2 α2

P
ψ( f )( x ) = f (t)dt = f (t)dt + f (t)dt  
x 0 x
Z x 0  1 ||ψ( f )||2
1
Z 1
1 α −→ +∞, on obtient Inf 2
≤ 0, or d’aprés la ques-
= tα dt + α +1
dt  || f ||
x 0 1 t  ||ψ( f )||
1 1 1 1 tion 4.2) on a : Inf ≥ 0, d’où l’égalité.
−1 || f ||

r
= −
x α + 1 α xα
2α + 1 1

e
= − α+1
xα(α + 1) αx
er
i

m
F n
i
n
m

r
Á la prochaine

o
or

F
✉: mamouni.myismail@gmail.com

168
i

r
s
✍ M AMOUNI M Y I SMAIL
MAMOUNI . NEW. FR
P ROBLÈMES C ORRIGÉS -MP

i
s

s
u

s
Devoir Libre
17 Transformée de Fourier, de Lebesgue. Séries de Dirichlet

u
r
Enoncé : CCP 2011, PSI Etude de la fonction L. Pour tout x réel tel que la série entière

é
!
k +∞ k
k−1 x k−1 x
Notations. On note ∑ (− 1 ) converge, on note L ( x ) = ∑ (− 1 ) sa
k k

Po r
r
k≥1 k = 1
|z| le module du nombre complexe z. somme.

u
J un intervalle de [0, +∞[. 1.1 Préciser le rayon de convergence de cette série entière, mon-

uor
f une fonction définie sur J à valeurs dans R ou C. trer que la fonction L est définie sur ] − 1, 1] et expliciter L( x )
pour x ∈] − 1, 1[.
g une fonction définie sur [0, +∞[ à valeurs dans R ou C.
Z 1.2 Montrer, avec soin, que la fonction L est continue sur l’inter-
Sous réserve de son existence, on note f˜g ( x ) = f (t) g( xt) dt pour valle [0, 1]. En déduire que L(1) = ln(2).
J

P
x > 0.   
1 2kπ
Chaque fois qu’aucune confusion ne sera possible, on notera f˜( x ) Etude de la série ∑ cos .
k 3
au lieu de f˜g ( x ). k≥1
On considère la suite
Objectifs. (ak )k∈N ∗ définie par

r
Pour différentes hypothèses sur la fonction f , sur l’in- 2 1 1
tervalle J et pour deux choix de g, on se propose de déterminer la ∀ p ∈ N ∗ , a3p = − ; ∀ p ∈ N, a3p+1 = et a3p+2 =
3p 3p + 1 3p + 2

e
limite de f˜g ( x ) lorsque le nombre réel x tend vers +∞.
2.1 Montrer que
er
Dans la partie 1, on étudie un exemple explicite avec application à
3p 3p 2p
1 1 1

m
des calculs de sommes de séries.
Dans la partie 2, on considère une fonction f définie sur [0, +∞[ à ∑ ak = ∑ k = p ∑ h
k=1 k= p+1 h=1 1 + p
valeurs réelles et l’objectif est d’obtenir la limite en +∞ de f˜g ( x )
3p
lorsque g(t) = | sin(t)|, lorsque f est de classe C 1 ou lorsque f est
m

2.2 déterminer la limite de ∑ ak lorsque p → +∞ (on pourra

r
continue par morceaux.
k=1
1
Partie I : Une étude de séries. considérer la fonction t 7→ sur un intervalle convenable).

o
1+t
or

F
✉: mamouni.myismail@gmail.com

169
i

r
s
P ROBLÈMES C ORRIGÉS -MP ✍ M AMOUNI M Y I SMAIL
MAMOUNI . NEW. FR

i
s

s
 

u
En déduire la convergence de la série ∑(ak )k≥1 et préciser sa cos(kα)
3.6 En déduire la convergence des séries ∑ et

s
somme.      kk≥ 1
1 2kπ sin(kα)

é
2.3 En déduire que la série ∑ cos converge et

u
∑ . Expliciter leur somme respective. Le résul-
k 3 k
  k≥1 k≥1
1 tat est-il conforme avec celui obtenu en 1.2.3 ?

r
montrer que sa somme est égale à ln √ .
3

é
    Partie II : Limite d’une intégrale.
cos(kα) sin(kα)
Etude des séries ∑ et ∑ .

Po r
r
k k≥1 k k≥1 Dans cette partie, on désigne par f une fonction continue par
Pour t ∈
1 n morceaux sur l’intervalle [0, +∞[ à valeurs réelles et telle que l’in-

u
]0, 2π [ et n ∈ N ∗ , on note ϕ(t) = it et Sn (t) = ∑ eikt . Z +∞
e −1

uor
k=1 tégrale généralisée | f (t)| dt soit convergente. On désigne
On désigne par α un nombre réel fixé dans l’intervalle ]0, 2π [. Pour 0
par g une fonction définie et continue sur l’intervalle [0, +∞[ à
simplifier l’écriture des démonstrations, on supposera π ≤ α <
valeurs complexes et (sous réserve d’existence) on note f˜g ( x ) =
2π. Z +∞
3.1 Montrer que Sn (t) = ϕ(t)(ei (n+1)t − eit ). f (t) g( xt) dt pour x > 0.

P
0
3.2 Montrer que ϕ ∈ C 1 ([π, α]). Existence de f˜g ( x ).
Z α On suppose que la fonction g est bornée sur
3.3 Montrer que l’intégrale ei (n+1)t ϕ(t) dt tend vers 0 quand +
R .
π

r
n → +∞ (on pourra utiliser une intégration par parties).
Z α
1. Justifier l’existence de f˜g ( x ) pour tout x > 0. Montrer que la
fonction f˜g est continue et bornée sur R +∗ .

e
3.4 Expliciter Sn (t) dt. Déduire de ce qui précède la conver-
π !
Limite de f˜g ( x ) lorsque g(t) = eit .
er
+ ∞ ikα
eikα e On suppose que f est de
gence de la série ∑ . Expliciter la somme ∑

m
k k Z +∞
Z αk≥1
k=1
classe C sur R et à valeurs réelles. Soit f˜g ( x ) =
1 +
f (t)eixt dt.
it 0
en fonction de ln(2) et de e ϕ(t) dt.
2.1. Justifier l’affirmation :
m

π
it Pour tout ε > 0, il existe un réel positif A tel que

r
e 2 Z +∞
3.5 Exprimer eit ϕ(t) en fonction de   où t ∈ [π, α]. | f (t)| dt ≤ ε
sin 2t

o
A
or

F
✉: mamouni.myismail@gmail.com

170
i

r
s
✍ M AMOUNI M Y I SMAIL
MAMOUNI . NEW. FR
P ROBLÈMES C ORRIGÉS -MP

i
s

s
u
2.2. Le nombre réel A étant fixé, montrer que l’intégrale 3.5. Expliciter Ẽ ( x ) pour x > 0. Déterminer la limite de Ẽ ( x )

s
Z A
lorsque x tend vers +∞.
f (t)eixt dt tend vers 0 lorsque x tend vers +∞ (on pourra

é
0 Etude générale.

u
utiliser une intégration par parties). On désigne de nouveau par f une fonction quel-
Z +∞ conque continue par morceaux sur l’intervalle [0, +∞[ et telle que

r
2.3. En déduire la limite de f˜g ( x ) = f (t)eixt dt lorsque x tend Z +∞

é
0 l’intégrale généralisée | f (t)| dt converge et on note
vers +∞. Z +0∞
f˜( x ) = f (t)| sin ( xt)| dt pour x > 0

Po r
r
Dans toute la suite, on suppose g(t) = | sin(t)| et on note 0
simplement 4.1. Lemme préliminaire.

u
 
cos(2kt)
Z +∞ Pour tout réel t tel que la série ∑ converge, on

uor
f˜( x ) = f (t)| sin ( xt)| dt 4k2 − 1 k≥1
0 +∞
cos(2kt)
pose h(t) = ∑ 2
. Montrer que la fonction h est définie
Etude pour une fonction f particulière. k=1 4k − 1
On suppose (dans cet
et continue sur R. Justifier l’égalité

P
exemple) que f désigne la fonction E définie par E(t) = e−t pour
Z +∞ 2 4
∀t ∈ R, | sin(t)| = − h(t)
t ≥ 0. et donc Ẽ ( x ) = e−t | sin( xt)| dt pour x > 0. π π
0
Z π 4.2. ˜ 1
Limite de f ( x ) dans le cas C .
3.1. Pour γ ∈ R, calculer l’intégrale θ (γ) = eγy sin(y) dy. On suppose de plus que f est une fonction de classe C 1 sur

r
0
R + . En utilisant les résultats obtenus dans la partie 2.2 et
3.2. Montrer que pour x > 0,
la question précédente, déterminer la limite de f˜( x ) lorsque

e
Z
1 +∞ − u
Ẽ ( x ) = e x | sin(u)| du x → +∞. Le résultat est-il conforme à celui obtenu pour la
er
x 0
fonction E ?

m
3.3. Exprimer pour k ∈ N et pour tout x ∈ R ∗ , l’intégrale
Z ( k + 1) π
u kπ 4.3. Cas d’une fonction continue par morceaux.
e− x | sin(u)| du en fonction de e− x et de θ (γ) pour 4.3.1 Une limite.

m

un γ convenable. Etant donnés deux nombres réels β et δ tels que 0 ≤

r
kπ β < δ, on considère, pour x > 0, l’intégrale F( x ) =
Z δ Z
3.4. Justifier, pour x > 0, la convergence de la série ∑(e− x )k≥0 . 1 δx
| sin( xt)| dt. Montrer que F( x ) = | sin(u)| du.

o
Préciser sa somme. β x βx
or

F
✉: mamouni.myismail@gmail.com

171
i

r
s
P ROBLÈMES C ORRIGÉS -MP ✍ M AMOUNI M Y I SMAILMAMOUNI . NEW. FR

i
s

s
Z

u
βx δx
On pose p la partie entière de et q celle de . Pour l’intégrale | f (t)| dt existe, on note toujours

s
π π J Z
π
x> , donner un encadrement de F( x ) en fonction f˜( x ) = f (t)| sin (tx )| dt

u
δ−β J
de p, q et x. Quelle est la limite de f˜( x ) lorsque x tend vers +∞ :
2

r
En déduire que F( x ) tend vers (δ − β) lorsque x → - lorsque J est un segment et f une fonction en es-

é
π
+∞. calier ?
Limite de f˜( x ) dans le cas d’une fonction continue par - lorsque J est un segment et f une fonction continue

Po r
r
4.3.2
morceaux. par morceaux ?

u
Si J est un intervalle de R + et si f est une fonction con- - lorsque J = R + et f une fonction continue par
tinue par morceaux sur J à valeurs réelles et telle que morceaux ?

uor
r P
e
er

m
m

or
or

F
✉: mamouni.myismail@gmail.com

172
i

r
s
✍ M AMOUNI M Y I SMAIL
MAMOUNI . NEW. FR
P ROBLÈMES C ORRIGÉS -MP

i
s

s
u
la série ∑ uk ( x ), sur [0, 1] , que l’on a
Pr. Verschueren, Lycée Daudet à Nîmes

s
déjà assurée, et le résultat complémentaire que le reste est majoré :
| L( x ) − Sn ( x )| = | Rn ( x )| 6 |un+1 ( x )|

é
Partie I Une étude de séries.

u
k= n
xk
Ainsi, pour n ≥ 1, en notant Sn ( x ) = ∑ (−1)k−1 , on a :
Question I. 1. 1. Pour k ≥ 1, considérons la série entière de terme

r
k=1
k
xk

é
général uk ( x ) = (−1)k−1 . ∀ x ∈ [0, 1] , | L( x ) − Sn ( x )| 6 | un+1 ( x )| 6
1
k n+1
uk+1 ( x ) kx

Po r
r
Pour x > 0, on a = → x quand k → ∞, donc.
uk ( x ) k+1 [0,1] 1
avec le critère de d’Alembert, si x > 1, on a Cette fonction est bornée, et N∞ ( L − Sn ) 6 Quand

u
n+1
absolue convergence de ∑ uk ( x ) et si x > 1, on a divergence [0,1]

uor
n → ∞, on a donc que N∞ ( L − Sn ) → 0,
grossière de cette même série.
et on a assuré la convergence uniforme de la suite (Sn )n∈N∗ sur
Ainsi R = 1 , et la somme totale de la série entière est définie et de [0, 1] vers L. Les Sn , qui sont des polynômes
classe C∞ sur ]−1, 1[ . (sommes partielles d’une série entière) sont continues, et par le thm
En x = 1, la série ∑ uk (1) est une série de Riemann alternée vérifi-

P
de continuité sous convergence uniforme,
ant le thm spécifique (car |uk (1)| → 0 en
L est continue sur [0, 1] Alors L(1) = lim L( x ) = lim ln(1 +
décroissant) donc convergente. En x = −1, la série ∑ uk (−1) est la x → 1− x → 1−
1 x ) donc L(1) = ln(2)
série harmonique ∑ divergente.
k

r
L est définie sur ]−1, 1] . On reconnaît le D.S.E. de ln(1 + x ), et k=3p q= p q = p−1

Question I. 2. 1. Pour p ∈ N , ∑ ak = ∑ a3q + ∑ a3q+1 +

e
L( x ) = ln(1 + x ) sur ]−1, 1[ k=1 q =1 q =0
er
q = p−1

m
Question I. 1. 2. Pour x ∈ [0, 1] , la série numérique ∑ uk ( x ) est ∑ a3q+2 en distinguant les entiers k modulo 3.
q =0
xk
une série alternée, car | uk ( x )| = . k=3p q= p q = p−1 q = p−1 q= p q= p
k 2 1 1 1 1 1
Donc ∑ ak = − ∑ + ∑ + ∑ = −∑ + ∑ +
m

Cette série alternée vérifie les hypothèses du thm spécifique car 3 q =1 q 3q + 1 3q + 2 q q=1 3q
k=1 q =0 q =0 q =1

r
| uk ( x )| → 0 quand k → ∞, et en décroissant k= p k=3p
u (x) kx 1 1
avec k+1 = < 1. On retrouve ainsi la convergence de = −∑ + ∑ en retrouvant, cette fois, tous les entiers k m

o
uk ( x ) k+1 k=1
k k=1
k
or

F
✉: mamouni.myismail@gmail.com

173
i

r
s
P ROBLÈMES C ORRIGÉS -MP ✍ M AMOUNI M Y I SMAIL
MAMOUNI . NEW. FR

i
s

s
u
k=3p k=3p h=2p h=2p la série ∑ an converge vers sa somme totale ln(3)
1 1 1 1

s
Et ∑ ak = ∑ k= ∑ = ∑ h
par changement
k=1 k= p+1 h=1
p+h p h=1 1+  
p 2k π

u
1 Question I. 2. 3. Le réel cos vaut selon la classe de k mod-
d’indice k = p + h, et factorisation de .   3
p  2k π

r

 si k = 3p : cos =1

 3

é
Question I. 2. 2. Une somme de Riemann de la fonction f sur le 
 
2k π 1
segment [ a, b] , qu’on partage en subdivision ulo 3 : si k = 3p + 1 : cos =−
    3  2

Po r

r
b − a h= n b−a 
 2k π 1
équidistante, est SRn ( f ) = , et on sait 
n h∑
f a+h 
 si k = 3p + 2 : cos =−
n
 3 2

u
=1 
que pour f continue sur [ a, b] , la suite 1 2k π 1

uor
Z b ∀k ∈ N ∗ , cos = − ak , série qui converge et
k 3 2
(SRn ( f )) n∈N∗ converge vers f (t) dt. Il en est de même de la    

 a 1 2k π 1 1
suite extraite SR2p ( f ) p∈N∗ . ∑ k cos 3 = − 2 ln(3) = ln √3
k=1
1

P
Pour f : t 7−→ le segment [ a, b] = [0, 2] et n = 2p, on a donc
1+t Question I. 3. 1. Pour t ∈ ]0, 2π [ , on a ei t 6= 1 et Sn (t) qui appa-
b−a 1 b−a h
que = et a + h = 1+ raît comme la somme des termes successifs de la
n p n p
n  k
k= n i t it ei t − e ( n + 1 ) i t

r
h=2p Z b Z 2 suite géométrique de raison e
Notons An la somme partielle de ∑ an : An = ∑ ak . , vaut donc S n ( t ) = ∑ e =
1 1 dt k=1
1 − ei t
lim ∑ = f (t) dt = = ln(3) k=1
p→ ∞ p
1 + hp 1+t

e
a 0
h=1 On vient de montrer que lim A3p = ln(3)
Question
p→ ∞ I. 3. 2. Sur ]0, 2π [ le dénominateur de ϕ est non nul et
er
1 de classe C∞ , avec e i t − 1 = cos(t) − 1 + i sin(t).
Les deux suites de termes généraux A3p+1 = A3p + et

m
3p + 1 La fonction ϕ est donc C1 sur ]0, 2π [ et sur le segment [ π, α] .
1 1 cos(t) − 1 − i sin(t) 1 sin(t)
A3p+2 = A3p + + ont même D’ailleurs ϕ(t) = = − −i
3p + 1 3p + 2 2 2 2 2(1 − cos(t))
  (cos(t) − 1) + sin (t)
m

limite, et les 3 suites extraites A3p p∈N∗ , A3p+1 p∈N et en séparant parties réelle et imaginaire.

r

A3p+2 p∈N tendent vers la même limite ln(3). Note : L’hypothèse α ∈ [π, 2π [ ne sert que pour l’écriture sans

o
Ce qui assure que la suite ( An )n∈N∗ converge, et que ambiguïté des segments [ π, α] , qu’on devrait
or

F
✉: mamouni.myismail@gmail.com

174
i

r
s
✍ M AMOUNI M Y I SMAIL
MAMOUNI . NEW. FR
P ROBLÈMES C ORRIGÉS -MP

i
s

s
" #

u
écrire, à strictement parler, [α, π ] si 0 < α < π, dans les ques- n
ekiα n
(−1)k−1

s
tions suivantes. −i ∑ +∑
k=1
k k=1
k

é
Z α Z α

u
Question I. 3. 3. Puisque ϕ est C1 sur le segment [π, α] , ainsi que Mais aussi avec I. 3. 1. on a Sn (t) dt = e(n+1) i t ϕ(t) dt −
Z α π π

r
la fonction t 7−→ e(n+1) i t , on peut effectuer une e i t ϕ(t) dt

é
Z α
" # t=α π
e ( n + 1 ) i t ϕ (t)
intégration par partie : e(n+1) i t ϕ(t) dt = − ekiα n n
(−1)k−1
Z α Z α

Po r
r
π ( n + 1) i Ainsi : ∀n ∈ N , ∑ ∗
= −∑ +i e ( n + 1) i t
ϕ(t) dt − i eit
t=π
Z α ( n + 1) i t k k π π
e k=1 k=1

u
ϕ′ (t) dt
π ( n + 1) i On a assuré l’existence des limites des termes de droite quand

uor
ϕ et ϕ′ sont continues sur le segment [π, α] donc bornées, respec- n → ∞, on assure donc la convergence de la série
tivement par M ϕ = Sup | ϕ(t)| et M ϕ′ = Sup ϕ′ (t)
∞ Z α
[ π,α] [ π,α] ekiα
" # t=α et la valeur de sa somme totale : ∑ = − ln(2) − i e i t ϕ(t) dt
Z α k
e ( n + 1) i t

P
ϕ (t) k=1 π
et ainsi : e(n+1) i t ϕ(t) dt 6 +
π ( n + 1) i
t=π Pour tout t ∈ ]0, 2π [ , on a ei t 6= 1 et e i t ϕ(t) =
Question I. 3. 5.
Z α ( n + 1) i t
e 2 Mϕ | α − π | M ϕ′ −i t it i t
ϕ′ (t) dt 6 + e it e 2e e 2
π ( n + 1) i n+1 n+1

r
= = 
e −1 −i t
t
i t 2i sin 2
Z α e 2 (e i t − 1 )

e
( n + 1) i t
Le numérateur est une constante réelle, donc lim e ϕ(t) dt = 0 
n→∞ π 1 cos 2t
er
it
Question I. 3. 6. Ainsi e ϕ(t) = +  et

m
Z α 2 2i sin 2t
Question I. 3. 4. Pour α ∈ ]0, 2π [ , Sn (t) dt = Z α
1  t=α
  " # α
π e i t ϕ(t) dt = (α − π ) − i ln sin 2t
n Z α n
ekit 1 n ekiα − ekiπ 2 t=π
kit Zπα
∑ π e dt = ∑ ki = ∑ 1 
m

it α
i k e ϕ ( t ) dt = ( α − π ) − i ln sin . En séparant parties

r
k=1 k=1 π k=1 2 2
π
 k Z α réelle et imaginaire, on a montré la
kiπ iπ k
et e e (−1) donc Sn (t) dt

o
= = = convergence des deux séries et calculé leurs sommes totales :
π
or

F
✉: mamouni.myismail@gmail.com

175
i

r
s
P ROBLÈMES C ORRIGÉS -MP ✍ M AMOUNI M Y I SMAILMAMOUNI . NEW. FR

i
s

s
u
Z ∞ Z ∞
∞ ∞
cos(k α)  sin(k α) π−α 0, feg ( x ) 6 | f (t) g( x t) | dt 6 Mg | f (t) | dt

s
α
∑ k = − ln(2) − ln sin 2 et ∑
k
=
2 0 0
k=1 k=1

é
donc feg est bornée sur ]0, ∞[

u
ce qui est vrai pour α ∈ ]0, 2π [ ,
en échangeant les bornes des intégrales.
Question II. 2. 1. Si g est la fonction t 7−→ e i t , on a les hypothèses,

r
√ ∞
2π  3 cos(k α)

é
En particulier pour α = , on a sin α
et de la question précédente, avec Mg = 1.
3 2 =
2 ∑ k
= Z ∞
! k=1 Par définition de la convergence d’une intégrale : | f (t) | dt =
√ √ 

Po r
r
3 Z A Z ∞ 0
− ln(2) − ln = − ln 3
2 lim | f (t) | dt donc lim | f (t) | dt = 0

u
A→∞ 0 A→∞ A
on retrouve ainsi le résultat de I. 2. 3. Note : pour α = π, on Z ∞

uor

(−1)k et pour tout ε > 0, il existe A > 0, tel que 0 6 | f (t) | dt 6 ε
retrouve ∑ = − ln(2). A
k=1
k
Question II. 2. 2. Pour A ainsi fixé, on peut effectuer une intégra-
Partie II Limite d’une intégrale. tion par partie, puisque les fonctions sont C1 sur

P
Z A  ixt  t= A
Question II. 1. Si g est bornée par Mg = Sup | g(t) | , alors pour ixt e
le segment [0, A] : e f (t) dt = f (t) −
[0,∞[ 0 ix t =0
Z A ixt
x > 0, on a | f (t) g( x t) | 6 Mg | f (t)| si t ∈ [0, ∞[ . e
Z ∞ Z ∞ f ′ (t) dt

r
Puisque l’intégrale | f (t)| dt converge, alors | f (t) g( x t) | dt 0 ix
Z A Z
0 0 | f ( A ) | + | f (0 ) | 1 A
Ainsi e ixt
f (t) dt 6 f ′ (t) dt et

e
converge et feg ( x ) existe pour tout x > 0 +
0 x x 0
er
De plus la fonction ϕ0 : t 7−→ Mg | f (t)| est positive, continue par K
puisque A est fixé, le majorant est de la forme

m
morceaux et intégrable sur [0, ∞[ , elle domine x
Z A
la fonction à deux variables : Fg : ( x, t) 7−→ f (t) g( x t), qui est
continue par rapport à x à t fixé et continue par où K est constante. Donc lim e i x t f (t) dt = 0
x→∞ 0
m

morceaux par rapport à t , à x fixé. On a ainsi les hypothèses du

r
thm de continuité des intégrales à paramètres Question II. 2. 3. Assurons que lim feg ( x ) = 0, en montrant
x→∞
que feg ( x ) peut être majoré par n’importe quel

o
sous domination : feg est continue sur ]0, ∞[ . Enfin : ∀ x >
or

F
✉: mamouni.myismail@gmail.com

176
i

r
s
✍ M AMOUNI M Y I SMAIL
MAMOUNI . NEW. FR
P ROBLÈMES C ORRIGÉS -MP

i
s

s
u
2ε > 0, pour x suffisamment grand. Considérons donc ε > 0, quel- fonctions étant C1 sur le segment [0, π ] :

s
conque mais fixé. Z π Z π
γy γy y= π
• Pour ce ε > 0, avec II. 2. 1. il existe A > 0, tel que e sin(y) dy = [ e (− cos(y)) ] y=0 + γ e γ y cos(y) dy =

u
Z ∞ Z ∞ 0  Z π 0 
y= π
∀ x > 0, e i x t f (t) dt 6 | f (t) | dt 6 ε e γπ γy
+ 1 + γ [ e (sin(y)) ] y=0 − γy
γ e sin(y) dy

r
A A 0
Z π Z π
• Pour un A ainsi  Zchoisi, il existe x0 > 0, tel que ∀ x ∈

é
γy γπ 2 γy
A d’où e sin(y) dy = e +1−γ e sin(y) dy et le
]0, ∞[ , ( x ≥ x0 ) =⇒ e i x t f (t) dt < ε 0 0
même résultat.

Po r
r
0
Alors, pour tout ε > 0, il existe x0 >  0, tel que ∀ x ∈ Z ∞
Z ∞

u
]0, ∞[ , ( x ≥ x0 ) =⇒ e i x t f (t) dt < 2ε . Question II. 3. 2. On a Ee ( x ) = e − t | sin( x t) | dt, intégrale
0

uor
0 convergente (II. 1.) dans laquelle on peut effectuer
  !
On a donc montré que lim feg ( x ) = 0 1
x→∞ u = xt t= u
le changement de variable ⇐⇒ x qui
t ∈ ]0, ∞[ u ∈ ]0, ∞[
Question II. 3. 1. Si g est la fonction t 7−→ | sin(t) | , on a les hy-
est C1 -difféomorphe entre ces intervalles,

P
pothèses de II. 1., avec Mg = 1.
donc conserve la convergence des intégrales et leur valeur.
Pour
Z f = E, la fonction E est C1 sur [0, ∞[ , à valeurs réelles et Z ∞
∞ 1 u
| E (t) | dt existe et vaut 1. Ainsi Ee ( x ) = e − x | sin(u) | du pour tout x > 0
0 x 0

r
La conclusion de II. 2. 3. est assurée. On peut calculer θ (γ) de
deux façons différentes : Question II. 3. 3. Avec le changement de variable (translation)

e
Z π
" #y= π v = u − k π , pour tout x > 0,
e (γ +i ) y
er
• En calculant e γ y e i y dy = car γ − i 6= 0. Z ( k + 1) π u
Z π v+ kπ

m
0 γ+i
y=0
on a e− x
| sin(u) | du = e − x | sin(v) | dv =
Z π h i Z kπ  0
1 i−γ kπ π v kπ 1
D’où e γ y e i y dy = e (γ +i ) π − 1 = 2 [e γ π + 1] et e − x
e −x
sin(v) dv = e − x
θ −
0 γ + i γ + 1 0 x
m

Z π
eγπ + 1  π k

r

donc e γ y sin(y) dy = 2 Question II. 3. 4. On note uk = e − x
= e − x , alors ∑ uk est
0 γ +1
π

o
• En effectuant deux intégrations par partie successives, les une série géométrique de raison e − x ∈ ]0, 1[
or

F
✉: mamouni.myismail@gmail.com

177
i

r
s
P ROBLÈMES C ORRIGÉS -MP ✍ M AMOUNI M Y I SMAIL
MAMOUNI . NEW. FR

i
s

s
u
donc absolument convergente, et sa somme totale est 1
de classe C∞ sur R et bornée : N∞ (hk ) = Sup | hk (t) | = .

s
∞ kπ 1 R 4k2−1
∑e− x = pour tout x > 0.

é
π
1

u
k=0 1 − e− x La série ∑ est convergente par équivalence à une série de
4k2−1
1

r
Question II. 3. 5. Avec la règle de Chasles, on peut décomposer Riemann ∑ 2 d’exposant 2 > 1,

é
l’intégrale sur [0, (n + 1)π ] en somme totale 4k
donc la série des fonctions ∑ hk est normalement convergente sur
d’une série d’intégrales sur les segments [kπ, (k + 1)π ] , ainsi , pour R, donc uniformément convergente :

Po r
r
tout n de N :
Z ( n + 1) π k = n Z ( k + 1) π par continuité de chaque hk , la somme totale h est définie et continue sur R

u
−t
e | sin( x t) | dt = ∑ e − t | sin( x t) | dt =
0

uor
k=0 kπ
Z ( k + 1) π  
1 k= n u 1 1 k=n − kπ
e− x | sin(u) | du = θ − e x
x k∑
=0 kπ x x k∑
=0
  Question II. 4. 1. b. La fonction g : t 7−→ | sin(t) | est continue, de

P
1 1 1
Quand n → ∞ : Ee ( x ) = θ − sur ]0, ∞[ ou classe C1 par morceaux, paire et π-périodique.
x x 1 − e − πx
On peut déterminer son développement en série de Fourier et ap-
π!
x 1 + e − x pliquer le thm complémentaire au thm de
Ee ( x ) = 2 sur ]0, ∞[

r
x + 1 1 − e − πx Dirichlet, qui assure la convergence normale de la série de Fourier
  de g vers g, sur R.

e
π
− x π 1
Quand x → ∞, e = 1− +O 2
et Ee ( x ) = Calculons les coefficients de Fourier de g. On a la période T = π,
er
 x x 2π
   la pulsation ω = = 2 et :

m
  1
x  2+O x  2 T
2 π    d’où lim Ee ( x ) = • Pour tout n ∈ N, bn ( g) = 0, par parité.
x +1 + O 12 x→∞ π
Z T
x x 2 2
m

• Pour tout n ∈ N, an ( g) = | sin(t) | cos(nωt)dt =


T −T

r
cos(2kt) 2
Question II. 4. 1. a. Notons hk la fonction t 7−→ . Pour Z π
4k2 − 1 4 2
sin(t) cos(2nt)dt, par parité.

o
k ≥ 1, la fonction hk est paire, π-périodique, π 0
or

F
✉: mamouni.myismail@gmail.com

178
i

r
s
✍ M AMOUNI M Y I SMAIL
MAMOUNI . NEW. FR
P ROBLÈMES C ORRIGÉS -MP

i
s

s
u
Z π | cos(2k x t) |
2 2 | f (t) | 6 | f (t) | intégrable sur [0, ∞[ et la

s
D’où an ( g) = [sin((2n + 1)t) − sin((2n − 1)t)] dt 2
π 0  4k − 1 Z ∞ Z ∞
  π   π 1

é
2  − cos((2n + 1)t) t= 2 − cos((2n − 1)t) t= 2 série des intégrales | f (t) hk (t) | dt 6 | f (t) | dt

u
= − 0 4k2 − 1 0
π 2n + 1 t =0 2n − 1 t =0 K
converge par comparaison à la série ∑ 2 .

r
    k
2 1 1 4 1

é
Ainsi an ( g) = − = − pour Les divers hypothèses du thm d’intégration terme à terme sont as-
π 2n + 1 2n − 1 π 4n2 − 1
tout n ∈ N. surées, et on a donc, chaque intégrabilité étant

Po r
r
Z ∞ ∞ Z ∞
Dans l’introduction, nous avons assuré les hypothèses du thm de 2 4 1
assurée, que fe ( x ) = f (t) dt − ∑ 4k2 − 1 f (t) cos(2k x t)
Dirichlet qui assure que la série de Fourier de g π π

u
0 k=1 0
a0 ( g ) Z
1 ∞

uor
converge sur R, de somme totale S(t) = +
2 Considérons la série de fonctions vk : x 7−→ 2 f (t) cos(2k x t) dt

2 4 ∞ cos(2nt) 4k − 1 0
∑ n cos sin définies et continues sur ]0, ∞[ .
π π n∑
a ( g ) ( nωt ) + b n ( g ) ( nωt ) = − 2
=1 4n − 1
Z ∞
n =1 1
Avec II. 2. 3. on sait que lim vk ( x ) = lim 2 f (t) cos(2k x t) dt =
x →∞ 4k − 1 0

P
2 4 Zx→ ∞
qui égale g(t) sur R, donc que ∀t ∈ R, | sin(t) | = − h(t) 1 ∞
π π 0, et N∞ (vk ) 6 2 | f (t) | dt
4k − 1 0
Question II. 4. 2. Si de plus f est de classe C1 sur [0, ∞[ , on a pour qui est le terme général d’une série convergente, donc la série des
tout x > 0 Z: fonctions ∑ vk converge normalement sur

r
Z ∞  
e
∞ 2 4 ]0, ∞[ , donc uniformément. On peut donc appliquer le thm de la
f (x) = f (t) | sin( x t) | dt = f (t) − h( x t) dt =
double limite à la suite des sommes partielles,

e
" 0 # 0 π π
Z ∞ ∞
2 4 ∞ cos(2k x t)
er
f (t) − ∑ dt et assurer que lim ∑ vk (x) = 0.
x→∞
π π k=1 4k2 − 1

m
0 k=1
Z ∞
2
où l’on voit apparaître l’intégrale d’une série des fonctions ∑ f hk . Enfin, on peut conclure que lim fe ( x ) = f (t) dt Ce qui
x→∞ π 0
Chacune de ces fonctions est C0 sur R,
m

est conforme au résultat obtenu pour E.


la série converge simplement (et même normalement) sur R vers

r
t 7−→ f (t) h( x t) qui est continue sur R. Question II. 4. 3. 1. Si on effectue le changement de variable

o
Chacune des fonctions est intégrable, car | f (t) hk (t) | = u = x t dans l’intégrale, pour 0 6 β < δ, on a :
or

F
✉: mamouni.myismail@gmail.com

179
i

r
s
P ROBLÈMES C ORRIGÉS -MP ✍ M AMOUNI M Y I SMAIL MAMOUNI . NEW. FR

i
s

s
 

u
Z δ Z δx
1 
 βx  −( p + 1) < − β x 6 − p

∀ x > 0, F( x ) = | sin( x t) | dt = | sin(u) | du. p6 < p+1

s
β βxx Mais π donc π et

 q6 δ x 
 q6 δ x
< q+1 < q+1

u
βx δx π δx  π π
et sont deux réels positifs et pour x > on a −  δ x β x
π π δ−β π  q−p < − +1

r
βx (δ − β) x π π
> 1.  δx − βx −1 < q− p

é
=
π π  π π
 β x β x    
 partie entière de donc 2 p+ δ x1 β x 2 δx βx

Po r
p p <

r

 6ainsi − − 2 6 F( x) 6 − + 2 pour x >
π π
Il existe donc deux entiers p et q tels que δx δx x π π x π π
q partie entière de donc q 6 π < q + 1

u

 et quand x → ∞, par le thm

 π π
et 0 6 p < q δ−β

uor
Ainsi p π 6 β x < ( p + 1) π et q π 6 δ x < (q + 1) π donc d’encadrement (thm "des gendarmes" ou "du sandwich"), on a
2
[( p + 1) π, q π ] ⊂ [ β x, δ x ] ⊂ [ p π, (q + 1) π ] lim F( x ) existe et vaut (δ − β)
x→∞ π
Puisque la fonction qu’on intègre est positive et continue sur R, on

P
a l’encadrement entre les intégrales : Question II. 4. 3. 2. Selon les cas, par extention de résultats suc-
Z qπ Z δx Z ( q + 1) π cessifs.
| sin(u) | du 6 | sin(u) | du 6 | sin(u) | du • Si f est une fonction en escalier et J un segment, alors il existe
( p + 1) π βx pπ
une subdivision ( β k )06k6n de J telle que f soit

r
Z ( k + 1) π constante égale à yk sur chacun des intervalles ] β k , β k+1 [ .
De plus pour tout k ∈ Z, | sin(u) | du = Z Z β

e
kπ k +1
Z π Z π e
Donc f ( x ) = f (t) | sin( x t) | dt = ∑ f (t) | sin( x t) | dt =
er
| sin(v) | dv = sin(v) dv = 2 avec v = u − k π J 06k6n −1 β k

m
0 0 Z β
Z qπ Z ( q + 1) π k +1
∑ yk | sin( x t) | dt
Ainsi | sin(u) | du = 2(q − p − 1) et | sin(u) | du = 06k6n −1 βk
( p + 1) π pπ
2( q + 1 − p ) Cette combinaison linéaire de fonctions ayant des limites quand
m

x → ∞ a une limite qui vaut :

r
2 2 π 2 2
d’où un encadrement de F( x ) : (q − p − 1) 6 F( x ) 6 (q − p + 1) pour x > lim fe ( x ) = ∑ yk ( β k+1 − β k ) =
π 06k∑
y k ( β k+1 − β k )

o
x x δx−
→∞β 06k6n −1
π 6n −1
or

F
✉: mamouni.myismail@gmail.com

180
i

r
s
✍ M AMOUNI M Y I SMAIL MAMOUNI . NEW. FR
P ROBLÈMES C ORRIGÉS -MP

i
s

s
u
Z Z
2 2
et lim fe ( x ) = f (t) dt que pour tout x ≥ x0 on ait g ϕ n0 ( x ) − ϕn (t) dt < ε.

s
x→∞ π J π J 0
Pour tout ε > 0, nous assurons qu’il existex0 > 0, t. q. ∀ x >

u
Z
e 2
0, ( x ≥ x0 ) =⇒ f (x) − f (t) dt < 2ε
π J

r
• Si f est continue par morceaux sur J un segment, alors il existe Z
2

é
e
donc lim f ( x ) = f (t) dt
une suite ( ϕn )n∈N de fonctions en escaliers x→∞ π J
J
qui converge uniformément sur J vers f . Ainsi N∞ ( f − ϕn ) =

Po r
r
Sup | f (t) − ϕn (t) | → 0 quand n → ∞.

u
J
Z
2 • Si f est continue par morceaux sur J = [0, ∞[ , on peut reprendre
Les applications f 7−→ f et f 7−→ e f (t) dt sont linéaires, donc

uor
π J pour un ε > 0 donné, le raisonnement de la
pour la différence (où x > 0) :    Z question Z ∞ II. 2. avec  un A suffisamment grand pour que
Z h i Z Z
2 2 2 2
fe ( x ) − f (t) dt = fe ( x ) − ϕ fn ( x ) + ϕ fn ( x ) − ϕn (t) dt + ϕn (t)|dt
f (−t) | dt f6(tε.) dt
Alors :
π J  Z
π J  Z
π J A π J Z Z ∞ Z

P
2 2 e 2 2 ∞
^
= f − ϕn ( x ) + ϕ fn ( x ) − ϕn (t) dt + ∀ x >
[ ϕn (t) − f (t)] dt0, f ( x ) − f ( t ) dt = f ( t ) | sin ( x t ) | dt − f (t) dt
π J π J π [0,∞[ 0 π 0
Z Z Z A Z
2 2 2 A
D’où fe ( x ) − J
f (t) dt 6 L( J ) N∞ ( f − ϕn ) + ϕ fn ( x ) − ϕn (t) dt + 6 f (t) | sin( x t) | dt − f (t) dt
π 0
π J π J Z0 A Z A

r
2 2
J
L( J ) N∞ ( f − ϕn ) 6 f (t) | sin( x t) | dt − f (t) dt
π 0 π 0

e
où L( J ) est la longueur du segment J. Par la convergence uniforme Pour ce A ainsi choisi, avec le segment J ′ = [0, A] et le résultat
er
sur J, pour tout ε > 0, il existe un entier n0 ci-dessus, il existe x0 tel que
! Z A 

m
Z A
ε 2
tel que : ∀n ∈ N, (n ≥ n0 ) =⇒ N∞ ( f − ϕn ) <
J
 =⇒ ∀ x > 0, ( x ≥ x0 ) =⇒ f (t) | sin( x t) | dt − f (t) dt < ε
L( J ) 1 + π 2 0 π 0
   Z   
2
 J 2 2
L( J ) 1 + π N∞ ( f − ϕn ) < ε et alors ∀ x > 0, ( x ≥ x0 ) =⇒ fe ( x ) − f (t) dt < 2 + ε .
m

π J π

r
Pour un ε > 0, Zfixons donc un tel n = n0 . Pour ce n0 , puisque Z
2 Et on a donc assuré de même que lim e ( x ) = 2 f (t) dt
f
lim g ϕ n0 ( x ) = ϕn (t) dt, il existe un x0 tel

o
x→∞ π J 0 x→∞ π J
or

F
✉: mamouni.myismail@gmail.com

181
i

r
s
P ROBLÈMES C ORRIGÉS -MP ✍ M AMOUNI M Y I SMAIL
MAMOUNI . NEW. FR

i
s

s
u

s
Devoir Libre
18 Suites et séries de fonctions

u
Georg Friedrich Bernhard Riemann (1826-1866)

r
Blague du jour

Mathématicien du jour
Cinq ingénieurs et cinq commerciaux se déplacent pour German mathematician who made lasting contributions to anal-
aller à un salon. Chacun des 5 commerciaux va acheter ysis and differential geometry, some of them enabling the

Po r
r
un billet de train. Les ingénieurs n’achètent qu’UN seul later development of general relativity. His father was a poor
Lutheran pastor who fought in the Napoleonic Wars. His

u
billet. Les 5 ingénieurs vont s’enfermer dans les toilettes
juste avant que le contrôleur n’arrive. En passant, le mother, died before he had reached adulthood. Riemann exhib-

uor
contrôleur voit que les toilettes sont occupées. (voir la ited exceptional mathematical skills, such as fantastic calcula-
suite en dessous) tion abilities, from an early age but suffered from timidity and a
fear of speaking in public.

P
Suite : Il frappe à la porte et demande : "Votre billet, s’il vous plaît !". Les ingénieurs glissent Le billet sous la porte. Le contrôleur est satisfait
et s’en va. Les commerciaux sont bien sûr extrêmement vexés que les ingénieurs leur ont encore une fois fait la leçon. Pour le retour (voir
prochain DL)

r
Problème I : Exemple d’étude a Soit u : ]0, +∞[ −→ R Montrer que la suite

e
x 7−→ u( x ) = 1
er
de fonctions (un )n≥0 converge simplement vers u sur ]0, +∞[
Soit (un )n la suite de fonctions définies sur ]0, +p∞[ par : u0 ( x ) = x

m
et que : ∀n ∈ N, ∀ x ∈ R, 0 < un ( x ) 6 1
2 un ( x )
pour tout réel x strictement positif. un ( x ) = pour tout a Soit (Un ) la suite de fonctions définies pour tout entier
1 + un ( x )
entier naturel n , pour tout rel x strictement positif. 1
m

naturel n sur ]0, +∞[ par : Un = .


un

r
① Étude de la convergence simple et uniforme de la suite de 1 + Un ( x )
i Montrer que : Un = p , ∀n ∈ N, ∀ x ∈]0, +∞[.

o
fonctions (un )n≥0 : 2 Un ( x )
or

F
✉: mamouni.myismail@gmail.com

182
i

r
s
✍ M AMOUNI M Y I SMAIL
MAMOUNI . NEW. FR
P ROBLÈMES C ORRIGÉS -MP

i
s

s
u
1 ③ Prouver que lim ζ ( x ) = 1
ii En déduire que : |Un+1 − 1| 6 | Un ( x ) − 1 | , ∀ n ∈

s
x →+ ∞
2
N, ∀ x ∈]0, +∞[. ∞
1

é
Indication : majorer ∑ par comparaison une intégrale.

u
iii Montrer que la suite de fonctions (Un ) converge sim- n =2 n
x

plement vers u sur ]0, +∞[ et que la suite de fonctions (Un )

r
converge uniformément vers u sur tout compact de ]0, +∞[ . ④ Prouver que lim ζ ( x ) = +∞
x →1

é
iv En déduire que la suite de fonctions (un ) converge  
1 1
uniformément vers u sur tout compact de . ⑤ Soit γ = lim + · · · + − ln(n) .
n →+ ∞ 1

Po r
r
n
② Soit (vn ) la suite de fonctions définies sur ]0, +∞[ par : ∞   
1 1
1 + vn ( x ) Montrer que γ = 1 + ∑ + ln 1 − puis que γ =

u
∀n ∈ N, ∀ x ∈]0, +∞[, v0 ( x ) = 1, vn ( x ) = vn ( x ) n =2 n n
2

uor

a Soit x un rel strictement positif Montrer que les suites ζ (k) − 1
1− ∑ .
k
(un ( x )vn ( x ))n et (vn ( x ))n sont adjacentes. k=2
On définit alors : f : ]0, +∞[ −→ R . a
⑥ Décomposer en éléments simples sur C la fractions ra-
x 7−→ f ( x ) = lim vn ( x )

P
1
tionnelle : Fn (X ) = .
b Montrer que les suites de fonctions (un vn )n et (vn )n con- (1 + X/n)n − 1
vergent uniformément vers f sur tout compact de ]0, ∞[ . 1
b En déduire pour x ∈ R ∗ : coth x = 2x −
e −1

r
c En déduire que f est continue sur ]0, ∞[. ∞
1 1 2x
2x
= + ∑ .
x k = 1 x + k2 π 2
2

e
e − −1
Problème II : Fonction ζ et η de Riemann.
er
c En déduire la valeur de ζ (2).

m

(−1)n−1
⑦ pour x > 0 : η ( x ) = ∑ .

1 n =1
nx
Soit ζ ( x ) = ∑ .
m

nx a Établir pour x > 1 : η ( x ) = (1 − 21− x )ζ ( x ).

r
n =1
① Déterminer le domaine de définition de ζ.
1
En déduire ζ ( x ) ∼ pour x −→ 1+ .

o
② Montrer que ζ est de classe C ∞ sur ce domaine. b
x−1
or

F
✉: mamouni.myismail@gmail.com

183
i

r
s
P ROBLÈMES C ORRIGÉS -MP ✍ M AMOUNI M Y I SMAIL
MAMOUNI . NEW. FR

i
s

s
u
1 ∞
ζ (k) − 1
c Montrer que ζ ( x ) = + γ + (1). On remarquera que 1− ∑ .

s
Z +∞ x−1 k
1 dt k=2
.

é
=

u
x−1 t =1 tx ❻ a Décomposer en éléments simples sur C la fractions ra-

(−1)n ln n 1
tionnelle : Fn (X ) = .

r
c En déduire la valeur de ∑ . (1 + X/n)n − 1
n

é
n =1
1

1 b En déduire pour x ∈ R ∗ : coth x = 2x −
⑧ Déterminer le domaine de définition de ζ ( x ) = ∑ . e −1

Po r
r
nx ∞
n =1 1 1 2x
2x
= + ∑ .
⑨ Soit s > 1. Exprimer après avoir justifié son existence, la e − −1 x k = 1 x + k2 π 2
2

u
+∞
1 c En déduire la valeur de ζ (2).

uor
somme ∑ s en fonction de ζ (s ).
n =0 (2n + 1)
❼ Fonction êta de Riemann : Pour x > 0 : η ( x ) =
⑩ Montrer que ζ est de classe C ∞ sur ce domaine, en déduire ∞
(−1)n−1
ses dérivées successives. ∑ nx .

P

1 n =1
❶ Prouver que lim ζ ( x ) = 1. Indication : majorer ∑ x par
a Montrer que η est définie et continue sur ]0, +∞[.
n =2 n
x →+ ∞
comparaison une intégrale.
b Établir pour x > 1 : η ( x ) = (1 − 21− x )ζ ( x ). En déduire
❷ Prouver que lim ζ ( x ) = +∞
1

r
x →1
ζ (x) ∼ pour x −→ 1+ .
❸ Montrer que la fonction ζ est décroissante et convexe sur x−1

e
]1, +∞[. Tracer la courbe de ζ. 1
c Montrer que ζ ( x ) = + γ + (1). On remarquera que
x−1
er
❹ Montrer qu’au voisinage de +∞, on a ζ ( x ) = 1 + 2− x + 1
Z +∞
dt

m
o (2 − x ). = .
x−1 t =1 t x
❺ Constante d’Euler : Soit γ définie par : γ = ∞
(−1)n ln n
  c En déduire la valeur de ∑
1 1 .
m

lim + · · · + − ln(n) n =1
n
n →+ ∞ 1 n

r
∞   
d Donner un développement asymptotique à deux termes
1 1
Montrer que γ existe et que γ = 1 + ∑ + ln 1 − =

o
n =2 n n de ζ (s) lorsque s → 1+ .
or

F
✉: mamouni.myismail@gmail.com

184
i

r
s
✍ M AMOUNI M Y I SMAIL
MAMOUNI . NEW. FR
P ROBLÈMES C ORRIGÉS -MP

i
s

s
!

u
+∞ +∞ +∞
s −1 1
e Montrer que ∀s > 1, ζ (s) = ∏ (1 − p−
n ) , où 2

s
= p1 < ∑ ∑ nq − 1 = 1.
n =1 p=1 n =1
p2 < .. < pn < .. les nombres premiers.

u
❿ Calculer les sommes suivantes, après avoir justifié leurs exis-
f Pour s > 1, la série ∑ p−n s est-elle convergente ? La
tences :

r
n ≥1
1
p−

é
série ∑ n est-elle convergente ?
n ≥1 1 1
a ∑ c ∑ .
❽ Pour tout entier naturel n on note ϕ(n) le nombre d’entiers p 2 q2 p 2 q2

Po r
r
( p,q )∈(N ∗ )2 ( p,q )∈(N ∗ )2 ,p∧q =1
naturels plus petits que n et premiers avec n, dite fonction
☛ Indication : A = ζ (2)2 ☛ Indication : C =

u
indicatrice d’Euler. Montrer que ∑ ϕ(d) = n puis que pour
A/ζ (4) = 5/2.
d| n 1

uor
+∞ +∞
+∞
ϕ (n) ζ ( x − 1) b ∑ p 2 q2 d ∑ ∑
(−1) p
.
tout réel x > 1 : ∑ x = . ( p,q )∈(N ∗ )2 ,p| q q3
n =1
n ζ (x) p=1 q = p
☛ Indication : B = 7
1 ☛ Indication : − ζ (3).
❾ Soit u définie par u p,q = q pour tout p ≥ 2 et q ≥ 2. ζ (2 ) ζ (4 ).

P
p 8
a Montrer que la suite u est sommable et calculer sa
sE( x )
somme. Soit s > 1. Montrer que la fonction f : x 7−→ est inté-
+∞
Z +∞ x s+1
Prouver l’identité suivante : − 1)

r
b ∑ (ζ (q) =
grable sur [1, +∞[ et que : f (t)dt = ζ (s)
q =2 1

e
er
i
F

m
nn
i
m

r
Á la prochaine

o
or

F
✉: mamouni.myismail@gmail.com

185
✍ M AMOUNI M Y I SMAIL
MAMOUNI . NEW. FR

11MPO1

Devoir Libres
15 Exemple de calcul de sommes
Énoncé : e3a 2011, MP

✉: mamouni.myismail@gmail.com
TOURNEZ LA PAGE S.V.P.
✍ M AMOUNI M Y I SMAIL
MAMOUNI . NEW. FR

✉: mamouni.myismail@gmail.com
✍ M AMOUNI M Y I SMAIL
MAMOUNI . NEW. FR

✉: mamouni.myismail@gmail.com
TOURNEZ LA PAGE S.V.P.
✍ M AMOUNI M Y I SMAIL
MAMOUNI . NEW. FR

FIN DE L©EPREUVE

✉: mamouni.myismail@gmail.com
i

r
s
P ROBLÈMES C ORRIGÉS -MP ✍ M AMOUNI M Y I SMAIL
MAMOUNI . NEW. FR

i
s

s
u
2. Montrons que f est intégrable sur [0, 1[. On√sait déjà qu’elle est con-
Corrigé : Pr. Deyris, CPGE France

s
√intervalle. D’autre part, f (t) 1 − t a pour limite 0 en
tinue sur cet
1 (puisque u ln u tend vers 0 en 0), donc f (t) est négligeable de-

u
tn 1
1. Le rapport tend vers 0 quand n tend vers +∞ si |t| < 1, et vant g(t) = √ au voisinage de 1. Puisque g est positive et
n+1
1−t

r
diverge si |t| > 1 : le rayon de convergence de la série entière est
intégrable sur [0, 1[, f est donc bien intégrable sur [0, 1[.

é
donc 1. Z 1
2. L’égalité est claire pour t = 0. D’autre part, on sait que, pour Par suite, L( x ) a pour limite − f (t) dt quand x tend vers 1 par
0

Po r
r
+∞
(−1)k+1 tk valeurs inférieures.
tout t ∈ ] − 1, 1[, ln(1 + t) = ∑ . On en déduit
k 3. On a vu que L a pour dérivée − f sur ] − ∞, 1[ (question II-1), et

u
k=1
+ ∞ k−1 que − f est strictement positive sur cet intervalle (question I-4.).
ln(1 − t) t

uor
=−∑ pour t non nul dans ] − 1, 1[, ce qui donne Puisque L est de plus continue en 1, elle est donc strictement crois-
t k=1
k
le résultat demandé en posant n = k − 1. sante sur ] − ∞, 1].
On peut aussi dériver tS(t) pour obtenir le résultat. 4. Puisque f est la somme d’une série entière sur ] − 1, 1[, on sait que
la primitive de f qui s’annule en 0 est la somme de la série entière

P
3. En tant que somme d’une série entière, la fonction f = −S est de
classe C∞ sur ] − 1, 1[. Les théorèmes usuels montrent que f est de obtenue en primitivant terme à terme celle de f , et que le rayon de
classe C∞ sur ] − ∞, 0[. L’intersection de ces deux intervalles n’é- convergence de cette nouvelle série entière est le même que celui
tant pas réduite à un point, cela montre que f est de classe C∞ sur de la série de f , c’est-à-dire ici 1.
+∞
x n +1 +∞ n

r
la réunion des deux, soit sur ] − ∞, 1[. x
On en déduit ∀ x ∈ ] − 1, 1[ L( x ) = ∑ 2
= ∑ 2
.
4. On a ln(1 − t) > 0 sur ] − ∞, 0[ et ln(1 − t) < 0 sur ]0, 1[. Dans tous n =0 ( n + 1) n =1 n

e
les cas (y compris en 0), on a donc f (t) < 0. 5. Soit x ∈ ] − 1, 1[. On peut appliquer la relation du II-4 à x et − x,
er
+∞ n
x + (−1)n x n

m
Partie II donc L( x ) + L(− x ) = ∑ 2
. Dans cette somme, les
n =1
n

1. Puisque f est de classe C donc continue sur ] − ∞, 1[, les résul- termes de rang impair sont tous nuls ; donc
tats sur l’intégrale
Z fonction de sa borne supérieure montrent que +∞
2x2p 1 +∞ ( x2 ) p 1
m

x
L( x ) + L(− x ) = ∑ = ∑ = L( x 2 )
L : x 7−→ − f (t) dt est de classe C1 sur cet intervalle, et que sa ( 2p ) 2 2 p 2 2

r
0 p=1 p=1
dérivée est − f . Puisque cette dérivée est de classe C∞ , L est elle Puisque L est en particulier continue en 1 et en −1, on peut faire

o
aussi de classe C∞ . tendre x vers 1 dans cette relation pour obtenir L(1) + 2L(−1) = 0.
or

F
✉: mamouni.myismail@gmail.com

190
i

r
s
✍ M AMOUNI M Y I SMAIL
MAMOUNI . NEW. FR
P ROBLÈMES C ORRIGÉS -MP

i
s

s
u
xn 1 2n − 1 − 1
1
6. Pour tout x ∈ [0, 1], on a ≤ 2 , et la série numérique

s
n 2 n ∑   . Multiplions par 4 et appliquons la question
2 (2k+1)π
1 xn k=0 sin 2n + 1
∑ n2 converge. La série entière ∑ n2 converge en fait normale-

u
précédente, en utilisant de plus sin(π − u) = sin u pour la deux-
ment, donc uniformément, sur [0, 1]. On peut donc lui appliquer le
ième somme :
théorème d’interversion de limites : compte tenu de la continuité

r
2n − 1 − 1 2n − 1 − 1
2n +1 1 1

é
de L en 1, on a 2 = ∑  + ∑  n +1 
+∞ n +∞ +∞ 2 ( 2k + 1 ) π 2 ( 2 −2k−1)π
x xn 1 k=0 sin k = 0 sin
L(1) = lim L( x ) = lim ∑ 2 = ∑ lim 2 = ∑ 2 2n + 2 2n + 2

Po r
r
x −→1− x −→1− n =1 n n =1 x −→1
− n
n =1
n On vérifie alors que les nombres 2n+1 − 2k − 1 sont les entiers im-
pairs entre 2n et 2n+1 − 1, donc les 2k + 1 quand k décrit l’intervalle

u
Partie III d’entiers [2n−1, 2n − 1]. On obtient donc bien la formule cherchée

uor
1. Il faut en fait supposer ici n ≥ 2. Posons a = 2n−1 ; soit ϕ : m 7−→ au rang n + 1.
m + a. Alors, a est pair (puisque n − 1 ≥ 1. . .) et 2n−1 + a = 2n , 4. (a) On a sin′′ ( x ) = − sin x ≤ 0 sur [0, π/2], donc la fonction si-
donc ϕ transforme les entiers impairs entre 0 et 2n−1 − 1 en entiers nus est concave sur cet intervalle. En particulier, la courbe est donc
impairs entre a = 2n−1 et 2n − 1. au-dessus de la corde passant par les points d’abscisses 0 et π/2,

P
De plus, chaque entier impair b entre 2n−1 et 2n − 1 a un unique qui est la droite d’équation y = 2x/π ; cela fournit la relation de-
antécédent par ϕ, b − a, qui est de même un entier impair entre 0 mandée.
et 2n−1 − 1, d’où le résultat. (b) Si 0 ≤ k ≤ 2n−1 − 1, alors (2k + 1)π/2n+1 est dans [0, π/2], on
2. Puisque x est dans ]0, π/2[, il en est de même pour x/2 et ( x + peut donc lui appliquer le (a). Puisque tout est positif :

r
π )/2 ; les différents sinus apparaissant dans la formule sont donc  (2k + 1)π  4 (2k + 1)2 π 2  π 2 1 π2
sin2 ≥ soit   ≤
bien non nuls. De plus, puisque sin(u + π/2) = cos u pour tout u,

e
2n +1 π 2 (2 n + 1 )2 2n+1 sin2 (2k+1)π 4 (2
on a 2n + 1
er
1 1 cos2 ( 2x ) + sin2 ( 2x ) 4 qui constitue le résultat demandé avec C = π 2 /4. (c) Il semble que

m
2 x
+ 2 x+π
= 2 x
=
sin ( 2 ) sin ( 2 ) 2 x
sin ( 2 ) cos ( 2 ) sin2 x le concepteur du sujet ait eu dans l’idée d’utiliser un “théorème de
en utilisant sin(2u) = 2 sin u cos u. convergence dominée pour les séries" (hors programme) : on trans-
1 forme la somme du 3. en somme infinie en rajoutant des termes
m

3. Pour n = 1, la relation à démontrer est 2 = 2


, donc est π2 +∞

r
sin (π/4) nuls et on multiplie par (π/2n+1 )2 pour obtenir = ∑ ak,n .
bien vérifiée. 8 k=0

o
Supposons établi à un rang n ≥ 1 la relation 22n−1 = On remplace ensuite chaque ak,n par sa limite quand n tend vers
or

F
✉: mamouni.myismail@gmail.com

191
i

r
s
P ROBLÈMES C ORRIGÉS -MP ✍ M AMOUNI M Y I SMAIL
MAMOUNI . NEW. FR

i
s

s
u
+∞, la majoration du (b) fournissant l’hypothèse de domination x ). Puisque L est dérivable sur ]0, 1[, de dérivée − f , et que 1 − x est

s
qui autorise ce passage à la limite. dans cet intervalle, M est dérivable sur ]0, 1[, et, pour tout x ∈ ]0, 1[,
À partir de 3., on peut déterminer la somme de la série de manière ln(1 − x ) ln x ln x ln(1 − x ) ln(1 −

é
M ′ ( x ) = − L ′ (1 − x ) + L ′ ( x ) +

u
− = − +
plus conforme au programme, de la manière suivante : on vérifie x 1−x 1−x x x
1 1 La fonction M est donc constante sur ]0, 1[ ; en prenant la limite

r
aisément que la fonction x 7−→ 2
− 2 est prolongeable par en 0, compte tenu de la continuité de L en 1, on voit que cette con-
sin x x

é
continuité en 0, donc bornée sur ]0, π/2]. Soit K majorant sa valeur stante vaut L(1), d’où le résultat.
absolue sur ]0, π/2]. 7. En prenant x = 1/2 dans la relation précédente, on obtient

Po r
r
Soit n ≥ 1. Notons An la somme donnée en 3. (qui vaut 22n−1 ). En π 2 (ln 2)2
posant xk = (2k + 1)π/2n+1 , on a alors 2L(1/2) = L(1) − (ln 2)2 d’où L(1/2) = − .
12 2

u
2n − 1 − 1 n + 1  2 2n − 1 − 1
2 1 1 1 8. Notons I l’intégrale proposée. On effectue le changement de vari-

uor
An − ∑ 2
≤ ∑ 2
− 2 ≤ K.2n−1 able Zu = 1 − e−t :
π (2k + 1) k=0 sin xk xk Z 1/2 
k=0
n +1 2
1/2 − ln(1 − u) du ln(1 − u) ln(1 − u) 
Multiplions tout par (π/2 ) ; en tenant compte de la valeur de I= =− + du
0 u 1−u 0 u 1−u
An , on obtient
n −1 À ce stade, on peut justifier l’existence de I : la dernière forme est
π 2 2 −1

P
1 Kπ 2 bien définie, puisque la fonction intégrée est prolongeable par con-
− ∑ ≤
8 k=0
(2k + 1)2 2n +3 tinuité au segment [0, 1/2] ; et le changement de variable effectué
On vient donc de trouver une suite extraite de la suite des sommes réalisait un C1 -difféomorphisme entre les intervalles ouverts con-
partielles de la série ∑ 1/(2k + 1)2 , qui converge vers π 2 /8 ; sidérés.

r
sachant que la série converge, π 2 /8 est donc la limite de la suite ln(1 − u)
Le calcul est maintenant immédiat. La fonction u 7−→ est
des sommes partielles, donc la somme de la série. (1 − u )

e
+∞ +∞ la dérivée de u 7−→ − ln2 (1 − u)/2, donc
1 1
er
5. Classiquement, on a L (1 ) = ∑ = ∑ + ln2 2 π2
n2 (2p)2 I = L(1/2) +

m
n =1 p=1 =
2 12
+∞ +∞
1 L (1 ) 1
∑ (2p + 1)2 = + ∑ 2
. La dernière somme valant Partie IV
p=0 4 p=0 (2p + 1)
m

1. Effectuons le changement de variable u = −t dans l’intégrale


π 2 /8, on en déduit que L(1) = π 2 /6.

r
définissant L( x ), pour éviter les problèmes de signes ; on obtient
La question II-5 donne alors L(−1) = −π 2 /12. Z −x
ln(1 + u)

o
6. Pour tout x ∈ ]0, 1[, posons M( x ) = L(1 − x ) + L( x ) + ln x ln(1 − L( x ) = − du.
0 u
or

F
✉: mamouni.myismail@gmail.com

192
i

r
s
✍ M AMOUNI M Y I SMAIL
MAMOUNI . NEW. FR
P ROBLÈMES C ORRIGÉS -MP

i
s

s
u
On
Z a ln(1 + u)/u ≥ 1/u pour u ≥ e − 1, et, pour tout a > 0, Partie V

s
y
du/u a pour limite +∞ quand y tend vers +∞. Donc, L( x ) a 1. L’équation homogène associée équivaut à xy′ + y = 0 sur J ; ses so-
lutions sont les fonctions f vérifiant x f ( x ) = K pour une certaine

é
a

u
pour limite −∞ quand x tend vers −∞.
constante K, ce sont les fonctions de la forme x 7−→ K/x où K est
2. La fonction x 7−→ 1 − 1/x est continue et strictement croissante une constante réelle.

r
sur ]0, 1[. L’intervalle image est donc l’ouvert borné par les limites, L’ensemble des solutions est donc la droite vectorielle engendrée

é
c’est-à-dire ] − ∞, 0[. par la fonction x 7−→ 1/x.
3. (a) Les fonctions x 7−→ 1 − x et x 7−→ 1 − 1/x sont dérivables sur 2. On applique la méthode de variation des constantes : on cherche

Po r
r
]0, 1[, à valeurs respectivement dans ]0, 1[ et ] − ∞, 0[ ; puisque L les solutions sous la forme x 7−→ K ( x )/x, où K est une fonction
dérivable sur J. Après calculs, on obtient l’équation (1 − x )K ′ ( x ) =

u
est en particulier dérivable sur ces deux intervalles, h2 est dériv-
able sur ]0, 1[. Pour tout x ∈ ]0, 1[ : 1.

uor
1 ln x 1 x ln x Les solutions sur J sont donc les fonctions de la forme x 7−→
h2′ ( x ) = − L′ (1 − x ) + 2 L′ (1 − 1/x ) = + 2 ln x = − ln(1 − x ) C C
x 1−x x x−1 x − + = − f (x) + où C est un réel quelconque.
(b) Puisque x 7−→ − ln2 x/2 a pour dérivée x 7−→ − ln x/x sur x x x
]0, 1[, il existe un réel C tel que, pour tout x ∈ ]0, 1[, h2 ( x ) = On peut remarquer qu’il est bien plus rapide de résoudre en

P
1
− ln2 x/2 + C. En prenant la limite en 1, on obtient C = 0, d’où le réécrivant dès le départ l’équation sous la forme xy′ + y = .
1−x
résultat demandé.
3. Les solutions de (F J ) sont évidemment les primitives des solutions
4. Dans la relation précédente, posons u = 1 − 1/x. Plus précisément, de (E J ), ce qui conduit directement à la formule donnée.

r
pour tout u < 0, le nombre x = 1/(1 − u) est dans ]0, 1[ ; on peut 4. Pour obtenir une solution sur ] − ∞, 1[, il faut raccorder une solu-
donc lui appliquer la relation précédente, d’où, en divisant par u, tion sur ] − ∞, 0[ à une solution sur ]0, 1[ en 0.
1  1  L(u)

e
1 1 1 Puisque L a une limite finie en 0, la seule manière d’obtenir une
∀u < 0 L 1− + = − ln2 = − ln2 (1 − u)
er
u 1−u u 2u 1−u 2u limite finie en 0 pour une telle solution est de choisir la constante
Dans cette relation, le membre de gauche a pour limite 0 en −∞

m
A nulle ; puis, pour que les deux solutions aient la même limite en
par croissances comparées. Dans le membre de droite, le terme 0, il faut prendre la même valeur de B de part et d’autre de 0.
L(1 − 1/(1 − u)) a pour limite L(1) quand u tend vers −∞ ; le pre- Réciproquement, toute fonction de la forme L + B, où B est un réel,
mier terme a donc pour limite 0 en −∞. Par suite, L(u)/u a pour est de classe C∞ sur ] − ∞, 1[ et vérifie l’équation sur cet intervalle
m

limite 0 en −∞. (puisqu’en particulier L′ (0) = − f (0) = 1). Les solutions cherchées

r
5. Puisque L tend vers −∞ en −∞, mais que L( x )/x y a pour limite sont donc les fonctions de la forme L + B, qui forment une droite

o
0, la courbe a une branche parabolique de direction horizontale. affine.
or

F
✉: mamouni.myismail@gmail.com

193
i

r
s
P ROBLÈMES C ORRIGÉS -MP ✍ M AMOUNI M Y I SMAIL
MAMOUNI . NEW. FR

i
s

s
u

s
Devoir Libre
19 Équations de Bessel et la fonction Gamma

u
r
Blague du jour Friedrich Wilhelm Bessel (1784-1846)

Mathématicien du jour
C’est un prof,pensant au glaçon qui passe à l’état de Astronome et mathématicien allemand, connu principalement
vapeu, demande à ses élèves de prépas si tout le monde pour avoir effectué en 1838 les premières mesures précises de

Po r
r
a bien compris ce que c’est que la sublimation. la distance d’une étoile et pour être le fondateur de l’école alle-
mande d’astronomie d’observation. Bessel est le premier à déter-

u
« Est-ce que vous pouvez me donner un exemple de
solide qui se transforme en gaz sans passer par l’état liq- miner avec succès la parallaxe, et par là même la distance d’une

uor
uide ? » étoile fixe. il émit, le premier, l’hypothèse que les queues de
Et du fond de la classe quelqu’un lance : « Les cigarettes comètes pouvaient être dues à une force répulsive et l’existence
m’sieur ! » d’une grande planète au-delà d’Uranus.

P
① Étudier, selon les valeurs de x, l’intégrabilité sur l’inter-
Énoncé : CNC 2007, MP valle ]0, 1] de la fonction
t 7−→ tx−1 e−t .

r
Définitions Pour tout le problème, on définit une famille d’équa-
② Montrer que cette même fonction est intégrable sur l’in-

e
tions différentielles ( Fλ )λ∈R + par : tervalle [1, +∞[.
1 λ2 
er
∀λ ∈ R + , y′′ + y′ − 1 + 2 y = 0. ( Fλ ) ② À quelle condition, nécessaire et suffisante, sur le complexe z

m
x x la fonction t 7−→ tz−1 e−t est-elle intégrable sur l’intervalle
par "solution d’une équation différentielle", on fait référence aux
solutions à valeurs réelles. ]0, +∞[ ?
Z +∞
tz−1 e−t dt,
m

La partie I du problème est largement indépendante des autres. ③ On pose Γ(z) = z ∈ C et Re(z) > 0.

r
0
PARTIE I
① Soit z un complexe tel que Re(z) > 0 ; montrer que
Γ(z + 1) = zΓ(z).

o
① Soit x un réel.
or

F
✉: mamouni.myismail@gmail.com

194
i

r
s
✍ M AMOUNI M Y I SMAIL
MAMOUNI . NEW. FR
P ROBLÈMES C ORRIGÉS -MP

i
s

s
u
② En déduire, pour tout réel α > −1 et tout p ∈ N ∗ , l’iden- Soient λ ≥ 0, α un réel et ∑ an zn une série entière, à coefficients

s
tité n ≥0
Γ(α + p + 1) = (α + p)(α + p − 1) . . . (α + 1)Γ(α + 1). réels et de rayon de convergence R > 0. Pour tout x ∈]0, R[, on

u
pose
③ Montrer que pour tout x > 0, Γ( x ) > 0. +∞
∑ an x n + α .

r
④ Calculer Γ(1) et en déduire la valeur de Γ(n + 1) pour yα ( x ) =

é
n =0
tout entier naturel n.
④ ① Soit z un complexe tel que Re(z) > 0 ; montrer ① On suppose que la fonction yα est solution de l’équation dif-

Po r
r
soigneusement que férentielle ( Fλ ) et que a0 6= 0. Montrer que 
+∞
(−1)n 1
Z +∞ α 2 = λ2 , (α + 1)2 − λ2 a1 = 0, et ∀ n ≥ 2, (α + n)2 − λ2 an = a

u
Γ(z) = ∑ + tz−1 e−t dt.
n! n + z 1 ② On suppose que α = λ et que la fonction yα est solution de

uor
n =0
+∞ l’équation différentielle ( Fλ ) avec a0 6= 0.
(−1)n 1
② Montrer que la fonction z 7−→ ∑ est
n =0
n! n + z ① Montrer que
définie sur la partie C \ {0, −1, −2, . . . } du plan com- a0 Γ ( α + 1 )
∀ p ∈ N, a2p+1 = 0 et a2p = .

P
2p
2 p!Γ(α + p + 1)
plexe et qu’elle y est continue.
La formule précédente permet de prolonger la fonction Γ à ② Les an étant ceux trouvés précédemment ; calculer le
C \ {0, −1, −2, . . . } . rayon de convergence de la série entière ∑ an zn .
⑤ Soient a et b deux réels avec 0 < a < b, et soit t > 0. n ≥0

r
① Déterminer max(ta−1 , tb−1 ) selon les valeurs de t. ③ Montrer que si a0 2λ Γ(λ + 1) = 1 alors

e
+∞
1  x 2p+λ
② Montrer que ∀ x > 0, yλ ( x ) = ∑ ,
er
∀ x ∈ [ a, b], 0 ≤ tx−1 ≤ max(ta−1 , tb−1 ). p=0 p!Γ(λ + p + 1) 2

m
puis donner un équivalent de la fonction yλ au voisinage
③ En déduire que la fonction Γ est de classe C1 sur R ∗+ et
de 0.
donner l’expression de sa dérivée sous forme intégrale.
③ On suppose que 2λ 6∈ N ; si p ∈ N ∗ , on note le pro-
m

④ Donner un équivalent de la fonction Γ au voisinage de


Γ ( α + p + 1)

r
0. duit (α + p)(α + p − 1) . . . (α + 1) par si α ∈
Γ ( α + 1)
PARTIE II

o
C \ {−1, −2, . . . }.
or

F
✉: mamouni.myismail@gmail.com

195
i

r
s
P ROBLÈMES C ORRIGÉS -MP ✍ M AMOUNI M Y I SMAIL
MAMOUNI . NEW. FR

i
s

s
u
① En reprenant la question précédente avec α = −λ, mon- tions a2p sont dérivables en 0 ; on pose alors

s
trer que la fonction da2p ′
p
1
+∞
1  x 2p−λ bp = (0) = a2p (0) et H p = ∑ .

é
dα k

u
x 7−→ ∑ k=1
p=0
p!Γ(−λ + p + 1) 2 Montrer que, pour tout entier naturel p ≥ 1,
est aussi solution, sur R ∗+ , de l’équation différentielle 1

r
bp = − p 2 Hp.

é
( Fλ ). (2 p!)
② Vérifier que la famille (yλ , y−λ ), d’éléments de C(R ∗+ , R ), ③ Calculer alors le rayon de convergence de la série entière

Po r
∑ b p z2p .

r
est libre et décrire l’ensembles des solutions, sur R ∗+ , de
p≥1
l’équation différentielle ( Fλ ).

u
② ① En utilisant la relation (1), montrer que , pour tout entier

uor
PARTIE III naturel p ≥ 1,
(2p)2 b p + 4pa2p (0) = b p−1 ,
Dans cette partie, on va construire, dans les cas λ = 0 ou 1, une so-
avec la convention b0 = 0.
lution zλ de l’équation différentielle ( Fλ ), définie sur R ∗+ , qui soit
linéairement indépendante de la solution yλ . ② En déduire que la fonction z0 : x 7−→ y0 ( x ) ln x +

P
+∞
A- Étude de ( F0 ) ∑ b p x2p est une solution de l’équation différentielle
p=1
On rappelle que ( F0 ) , définie sur R ∗+ .
+∞
1 ③ Vérifier que la famille (y0 , z0 ), d’éléments de C(R ∗+ , R ), est li-

r
∀ x > 0, y0 ( x ) = ∑ x2p .
(2 p p!)2 bre et décrire l’ensembles des solutions, sur R ∗+ , de l’équation
p=0

e
 différentielle ( F0 ).
Soit α > −1 ; on définit la suite a2p (α) p∈N par la donnée de
B- Étude de ( F1 )
er
a0 (α) = 1 et la relation

m
2 On rappelle que
a2p (α) α + 2p = a2( p−1) (α), p ≥ 1 (1 ). +∞  x 2p+1
1
① ① Montrer que, pour tout entier naturel p ≥ 1, ∀ x > 0, ∑ p!( p + 1)! 2
y1 ( x ) = .
p=0
m

p 
1
a2p (α) = ∏ . Soit α ∈]0, 2[ ; on définit la suite c2p (α) p∈N par la donnée de

r
2
k=1 (α + 2k) c0 (α) = 1 et la relation 

o
② On voit bien que, pour tout entier naturel p ≥ 1, les fonc- c2p (α) (α + 2p)2 − 1 = c2( p−1) (α), p ≥ 1 (2 ).
or

F
✉: mamouni.myismail@gmail.com

196
i

r
s
✍ M AMOUNI M Y I SMAIL
MAMOUNI . NEW. FR
P ROBLÈMES C ORRIGÉS -MP

i
s

s
u
① ① Montrer que, pour tout entier naturel p ≥ 1, avec la convention d0 = 0.

s
p
1 ② En déduire que la fonction u1 : x 7−→ 2y1 ( x ) ln x +
c2p (α) = ∏ 2
.

é
k=1 (α + 2k) − 1
+∞

u
∑ d p x2p+1 est une solution de l’équation différentielle
② On voit là aussi que, pour tout entier naturel p ≥ 1, les p=1

r
fonctions c2p sont dérivables en 1 ; on pose alors 1 1 2
y′′ + y′ − 1 + 2 y = ,

é
p (E1 )
dc2p ′ 1 x x x
dp = (1) = c2p (1) et H p = ∑ . définie sur R ∗+ .
dα k

Po r
r
k=1
Montrer que, pour tout entier naturel p ≥ 1, ③ ① Montrer que l’équation différentielle (E1 ) possède une
1  solution v1 , définie sur R ∗+ , qui est de la forme v1 ( x ) =

u
d p = − 2p+1 H p + H p+1 − 1 . +∞
2 p!( p + 1)!
∑ e n x n −1 .

uor
③ Calculer alors le rayon de convergence de la série entière n =0

∑ d p z2p . ② Vérifier que la fonction z1 = v1 − u1 est une solution de


p≥1 l’équation différentielle ( F1 ), définie sur R ∗+ .

P
② ① En utilisant la relation (2), montrer que , pour tout entier ④ Vérifier que la famille (y1 , z1 ), d’éléments de C(R ∗+ , R ), est li-
naturel p ≥ 1,  bre et décrire l’ensembles des solutions, sur R ∗+ , de l’équation
(1 + 2p)2 − 1 d p + 2(1 + 2p)c2p (1) = d p−1 , différentielle ( F1 ).

e r
i
F
er
nn

m
i
m

Á la prochaine

or
or

F
✉: mamouni.myismail@gmail.com

197
i

r
s
P ROBLÈMES C ORRIGÉS -MP ✍ M AMOUNI M Y I SMAIL
MAMOUNI . NEW. FR

i
s

s
u
+∞
Z
Corrigé : Pr. Taibi, CPGE Rabat, Maroc

s
z tz−1 e−t dt = zΓ(z) pour tout z tel que ℜ(z) > 0
0

u
Partie I
b. Pour tout z ∈ C tel que ℜ(z) > 0 et tout p ∈ N ∗ , on a :
1. L’application t 7→ tx−1 e−t est continue sur ]0, +∞[ pour tout Γ(z + p) = Γ((z + p − 1) + 1) = (z − p − 1)Γ(z − p − 1).

r
p p
réel x .

é
D’où : ∏ Γ(z + k) = ∏ ( z − k − 1) Γ ( z − k − 1) =
x −1 − t x −1 x −1 − t
a. On a : t e ∼ t , donc t 7→ t e est intégrable k=1 k=1

Po r
r
t → 0+ p−1 p−1
sur ]0, 1[ si, et seulement 1 − x < 1 soit x > 0. ∏ (z − k) ∏ Γ(z − k) et par suite :

u
1 k=1 k=1
b. On a aussi tx−1 e−t = O ( 2 ), donc t 7→ e−t tx−1 est p−1

uor
t→+ ∞ t
intégrable sur [1, +∞[ . Γ(z + p ) = ∏ (z − k)Γ(z)
k=1
2. L’applicationn t 7→ tx−1 e−t = e−t e(z−1) ln(t) est continue sur On prend z = α + 1, on a : ℜ(z) = ℜ(α + 1) = ℜ(α) +
]0, +∞[, et que pour tout t > 0, e t − t z−1
=e t − t ℜ(z)−1
, donc 1 > 0 et par suite

P
p−1
par la question 1◦ ), l’application t 7→ tz−1 e−t est intégrable Γ(α + 1 + p) = γ(α + 1) ∏ (α + 1 + k) = Γ(α + 1)(α + 1)...(α + p)
sur ]0, +∞[ si et seulement si ℜ(z) > 0 . k=1
3. Quelques formules utiles : c. Pour tout x > 0, la fonction t 7→ tx−1 e−t est continue et

r
+∞
Z
a. Les applications t 7→ tz et t 7→ e−t sont de classes C1 strictement positive, donc Γ( x ) = tx−1 e−t dt > 0 .
sur ]0, +∞[ et que pour tout z ∈ C tel que ℜ(z) > 0,

e
0
on a : e−t tz = e−t tℜ(z)−1 → 0. On applique alors
er
t7→+∞ d. Par un simple calcul, on a Γ(1) = 1 et par b) pour α = 0,

m
une intégration par parties à l’intégrale Γ(z + 1) = p = n, on a ::
+∞
Z n
z − z−1
t e dt : Γ ( n + 1) = ∏ k = n!
k=1
m

r
+∞
Z 4. Développement en série de Γ.
 +∞
Γ ( z + 1) = tz e−z−1 dt = −e− t tz t =0
+

o
0
a. Soit z ∈ C tel que ℜ(z) > 0, on a : Γ(z) =
or

F
✉: mamouni.myismail@gmail.com

198
i

r
s
✍ M AMOUNI M Y I SMAIL MAMOUNI . NEW. FR
P ROBLÈMES C ORRIGÉS -MP

i
s

s
Z Z

u
1 1 1 1 1
tz−1 e−t dt + tx−1 e−t dt donc | f n (z)| ≤ ≤ . Comme la série ∑

s
]0,1[ [1,+ ∞[ n! | n + z| n! α n!

(−1)n n converge, il en résulte que ∑ f n converge localement
∑ n! t , on a alors : tz−1 e−t =

é
Ecrivons e−t =

u
n =0 uniformément sur C8Z − , donc par le théorème de conti-
∞ +∞
(−1)n

r
∑ n! tz+n−1 nuité la fonction somme ∑ f n est continue sur C8Z − .

é
n =0 n =0

(−1)n z+n−1
Si l’on pose f n (t) = t pour t ∈]0, 1], on a : On peut aussi montrer que ∑ f n est continue en tout

Po r
r
n! n =0
f n est intégrable sur Z]0, 1] pour tout entier naturel n et
Z
1 1 point z0 de C8Z − en effet : Comme C8Z − est un ou-

u
que | f n (t)| dt ≤ dt = et puisque la série vert, on a pour tout z0 ∈ C8Z − , il existe r > 0 tel
]0,1] ]0,1] n! n!
B(z0 , r ) ⊂ C8Z − , on prend alors le compact K = B (z0 , α)

uor
1
∑ n! converge, il en résulte par le théorème d’intégra- et on termine comme avant .
tion terme à terme que 5. Soit 0 < a < b et t > 0, on a : ta−1 = e(a−1) ln(t) .
Z 1 +∞ Z +∞
(−1)n 1 z+n−1 (−1)n 1
tz−1 e−t dt = ∑ dt = ∑

P
t a. Si t ∈]0, 1], , alors ln(t) ≤ 0, donc (a − 1) ln(t) ≥ (b −
0 n =0 n! 0 n =0 n! z + n
1) ln(t) et comme x7→ e x est croissante, on déduit que
(−1)n 1 ta−1 ≥ tb−1 . Soit Max(ta−1 , tb−1 ) = ta−1 .
b. Posons f n (z) = pour n ∈ N et z ∈ C8Z − . Si t > 1, alors ln(t) > 0, donc ta−1 < tb−1 et par suite
n! z + n

r
Max(ta−1 , tb−1 ) = tb−1 .
Pour n ∈ N, la fonction f n est continue sur C8Z − ( Conclusion finale : Pour tous 0 < a < b et t > 0, on a :

e
fraction rationnelle en z ) Max(ta−1 , tb−1 ) ≤ ta−1 + tb−1 .
pour tout z ∈ C8Z − et tout n ∈ N, on a : | f n (z)| =
er
1 1 1 1 b. Pour t ∈]0, 1], on a d’après a) 0 < tx−1 ≤

m
6 car | n + ℜ(z)| ≤ |n + z| , Max(tx−1 , ta−1 ) = ta−1 = Max(ta−1 , tb−1 )
n! |n + z| n! | n + ℜ(z)|
donc ∑ f n (z) converge absolument et par suite ∑ f n de même si t > 1, on a : 0 < tx−1 ≤ Max(tx−1 , tb−1 ) =
converge simplement sur C8Z − . tb−1 = Max(ta−1 , tb−1 )
m

Soit K un compact inclu dans C8Z − , et α = d(Z− , K ), En conclusion : 0 < tx−1 ≤ Max(ta−1 , tb−1 ) pour tout

r
on a α > 0 car Z − fermé et K compact. On a alors pour t ∈]0, +∞[

o
tout z ∈ K, et tout n ∈ N, | n + z| = d(−n, z) ≥ α, c. La fonction f : ( x, t) 7→ tx−1 e−t . est continue sur
or

F
✉: mamouni.myismail@gmail.com

199
i

r
s
P ROBLÈMES C ORRIGÉS -MP ✍ M AMOUNI M Y I SMAIL
MAMOUNI . NEW. FR

i
s

s
u
R ∗+ × R ∗+

s
L’application x 7→ x α est de classe C∞ sur R∗+ et que
L’application x 7→ tx−1 e−t = e−t e(x−1) ln(t) est de classe ∞

é
∑ an xn est de classe C∞ sur ]0, R[ ( somme d’une série

u
d x 7→
C1 sur R ∗+ et f ( x, t) = ln(t) f ( x, t) pour tout ( x, t) ∈ n =0
dx entière ), donc yα est de classe C∞ sur ]0, R[ (produit de fonc-

r
R ∗+ × R ∗+ .
tions de classes C∞ ).

é
De plus pour tout compact K = [ a, b] ⊂ R ∗+ et tout ∞ ∞
d Par calculs : y′α ( x ) = αx α−1 ∑ an x n + x α ∑ nan xn−1 =
( x, t) ∈ K × R ∗+ , on a : f ( x, t) ≤ |ln(t)| e−t tx−1 ≤

Po r
r
dx n =0 n =1

|ln(t)| e−t Max(ta−1 , tb−1 ) ≤ |ln(t)| e−t (ta−1 + tb−1 ) ∑ (α + n) an x α+ n −1

u
et que la fonction ϕ : t 7→√|ln(t)| e−t (ta−√1 + tb−1 ) n =0

uor

est intégrable sur R ∗+ car tϕ(t) = t (t a−1 + y′′α ( x ) = ∑ (α + n)(α + n − 1)an xα+n−2
tb−1 )e−t |ln(t)| → 0. Pour t ≥ 1, ϕ(t) ≤ (ta−1 + n =1
t → 0+ Donc
tb−1 )te−t = (t + t )e .
a b −t
2

yα est solution sur ]0, R[ de ( Fλ ) ⇔ ∀ x ∈]0, R[, −( x + λ) ∑ an x α+

P
Donc par le théorème de dérivation sous le signe inté-
n =0
gral, il en résulte que Γ est de classe C1 sur l’ouvert R ∗+ ∞ ∞
et que
Z +∞ Z +∞
+ ∑ (α + n) an x α+ n + ∑ (α + n)
d n =0 n =1

Γ (x) = f ( x, t)dt = ln(t)tx−1 e−t dt. ⇔ ∀ x ∈]0, R[

r
0 dx 0 ∞ ∞
2 2 α+ n
d. On a Γ( x + 1) = xΓ( x ) pour tout x > 0, et comme Γ est ∑ ((n + α) − λ ) an x − ∑a

e
continue en 1, on a lim Γ( x + 1) = Γ(1) = 1, donc n =0 n =2
x → 0+ ⇔ ∀ x ∈]0, R[
er
1 ∞ ∞

m
Γ ( x ) ∼ x → 0+
x ∑ ((n + α)2 − λ2 )an xn − ∑ an −
n =0 n =2
:
Partie II : On fait tendre x vers 0+, obtenir α2 − λ2 = 0 car a0 6= 0 et
m

+∞
puis ((α + 1)2 − λ2 )a1 = 0 et une recurrence ((α + n)2 − λ
λ > 0, α ∈ R, yα ( x ) = ∑ an x n + α

r
2
n =0 )an = an−2 ..

o
1. a0 6= 0 et yα est solution sur ]0, R[ de l’équation ( Fλ ) . 2. α = λ, a0 6= 0 et yλ est solution sur ]0, R[ de ( Fλ ) .
or

F
✉: mamouni.myismail@gmail.com

200
i

r
s
✍ M AMOUNI M Y I SMAIL
MAMOUNI . NEW. FR
P ROBLÈMES C ORRIGÉS -MP

i
s

s
u
∞ ∞ +∞
a. On a : yλ ( x ) = ∑ an x λ+ n = x λ ∑ an x n . On sait que (1) On a : ∀ x > 0, ∑ a2p x2p+λ

s
yλ ( x ) =
n =0 n =0 p=0
((λ + n)2 − λ2 )an = an−2 pour tout n ≥ 2

é
+∞
Γ ( λ + 1)

u
a0
Puisque . (λ + 1)2 − λ2 6= 0 , on a a1 = 0 et par = ∑ 2p
x2p+λ
p=0 2 p! Γ ( λ + p + 1)
la relation (1), on a : a2p+1 = 0 pour tout p ∈

r
+∞
1 a0 Γ ( λ + 1 ) x
( )2p+λ 2λ

é
N .et a2p = a pour tout p ∈ = ∑
(λ + 2p)2 − λ2 2( p−1) p=0 p! Γ(λ + p + 1) 2
p p p +∞
1 1 1

Po r
x

r

N . Donc ∏ a2k = ∏ ∑ ( )2p+λ car a0 2λ
(λ + 2k)2 − λ2 k∏
a2( k − 1) = =
k=1 k=1 =1 p=0 p! Γ(λ + p + 1) 2

u
p
1
p−1 p
1 Equivalent au voisinage de 0 :
∏ (λ + 2k)2 − λ2 ∏ a2k soit : a2p = ∏ (λ + 2k)2 − λ2 a0 .

uor
D’après les propriétés des séries entières, on a :
k=1 k=0 k=1 ∞
1 1 x 1
2 2
Mais (λ + 2k) − λ = 4λk + 4k = 4k(λ + k), d’où
p p p
2
∑ p! Γ(λ + p + 1) ( 2 )2p x→∼0+ Γ(λ + 1)
1 1 1 1 p=0
∏ (λ + 2k)2 − λ2 = ∏ 4k(λ + k) = 4 p p! ∏ λ + k = Donc

P
k=1 k=1 k=1 1 x
yλ ( x ) ∼ ( )λ
1 Γ ( λ + 1) x → 0+ Γ ( λ + 1 ) 2
.
22p p! Γ(λ + p + 1) 3. On suppose ici que 2λ ∈
/N.
En conclusion :
a Γ ( λ + 1)

r
∀ p ∈ N, a2p = 2p0 a. D’après la question 1 et 2) la fonction y−λ est aussi solu-
2 p! Γ(λ + p + 1) tion sur R∗+ de (Fλ ) .

e
b. Montrons (yλ , y−λ ) est un système fondamental de solu-
er
a2p x2p a2p tions sur R∗+ de ( Fλ ) .
x2 =

m
b. Pour x > 0 , on a : = Soit (α, β) ∈ R2 tel que αyλ + βy−λ = 0.
a2( p − 1) x 2( p − 1) a2( p − 1)
1 x
1 Comme yλ ( x ) ∼ ( )λ et y−λ ( x ) ∼
2 2
x2 → 0, donc le rayon de convergence x → 0+ Γ ( λ + 1 ) 2 x → 0+
(λ + 2p) + λ p→+ ∞
m

1 x
R est infini . ( )−λ , on a : yλ ( x ) → 0 et y−λ ( x ) →

r
Γ(−λ + 1) 2 x → 0+ x → 0+
+∞, donc si l’on suppose α 6= 0, alors en faisant tendre

o
c. On suppose a0 2λ Γ(λ + 1) = 1. x vers 0, on aboutit à une contradiction.
or

F
✉: mamouni.myismail@gmail.com

201
i

r
s
P ROBLÈMES C ORRIGÉS -MP ✍ M AMOUNI M Y I SMAIL
MAMOUNI . NEW. FR

i
s

s
u
p
On conclut que α = 0 et puis β = 0, donc les solutions ′ 2
2k)), donc : a2p (α) et puis

s
yλ et y−λ sont linéairement indépendantes .
a2p (α ) = − ∑ α + 2k
k=1
( Fλ ) est une équation différentielle linéaire du second or-

é
p
1

u
dre à coefficients continus et sans second membre, son a’2p (0) = −2 ∑ a2p (0)
2k
ensemble de solutions est donc un espace vectoriel réel k=1

r
p
de dimension deux. En conséquence : ( yλ , y−λ ) est un 1
= − ∑ a2p (0)

é
système fondamental de solutions de ( Fλ ) et que toute k=1
k
solution sur R ∗+ de ( Fλ ) est de la forme : = − H p .a2p (0)

Po r
r
y = αyλ + βy−λ où α, β ∈ R p  2
1 1 1
Or a2p (0) = ∏ = 2p = , donc :
(2k)2 2 ( p!)2

u
k=1
2 p p!
Partie III.  2

uor
′ 1
b p = a2p (0) = − Hp
A- Etude de ( F ) : 2 p p!
0
∞ c. Calcul du rayon de convergence Rb :
1 1 1
Pour x > 0, on a : yλ ( x ) = ∑ p 2 x2p . On a b p ∼ − p 2 ln( p) = o( p ) car H p ∼ ln( p),
n =0 (2 p! )

P
p→ ∞ (2 p!) 2 p!
1. . donc le rayon de convergence de la série entière ∑ b p x p
p est infini :
a. Pour tout entier k ≥ 1 : ∏ a2k (α) = Rb = + ∞

r
k=1
p p p 2. .
1 1
∏ (α + 2k)2 ∏ a2(k−1) , donc a2p (α) = ∏ (α + 2k)2 a0 (α).

e
a. Pour tout p ∈ N ∗ , on a : (2p)2 b p + 4pa2p (0) = −(2p)2a2p (0) + 4
k1 k=1 k=1
er
Or a0 (α) = 1, d’où la formule cherchée : = a2p (0) −(2p)2 H
.

m
p
1 Mais (2p)2 a2p (0) = a2( p−1) (0), donc :
a2p (α) = ∏ (α + 2k)2 pour tout p ≥ 1. (2p)2 b p + 4pa2p (0) = − a2p (0) H p + 4pa2p (0)
k=1 1
m

b. D’après les notations de l’enoncé, pour tout p ∈ N ∗ , on = − a2( p−1) (0) H p−1 − a2( p−1) (0) + 4pa2p (0

r
p
p
1
p | {z
a : a2p (α) = exp( ∑ ln( )) = exp (− 2 ∑ ln(α + =0

o
( α + 2k ) 2
k=1 k=1 = b p−1
or

F
✉: mamouni.myismail@gmail.com

202
i

r
s
✍ M AMOUNI M Y I SMAIL
MAMOUNI . NEW. FR
P ROBLÈMES C ORRIGÉS -MP

i
s

s
u

D’où le résultat demandé .
x2 z0′′ ( x ) + xz0′ ( x ) − ( x2 + 0)z0 ( x ) = 2xy0′ ( x ) + ∑ b p (2p)2 x2

s
p=1
∞ ∞

u
= ∑ 4pa2p (0) x2p + ∑ bp
p=1 p=1
| {z

r

é
∑ b p−1 x2p
p =1

Po r
car xy0′ ( x )

r
b. L’application x 7→ y0 ( x ) ln( x ) est de classe C∞ sur R ∗+ (

u
Opérations ), donc z0 est de classe C∞ sur R ∗+ . Pour tout = b0 x2 = 0
x > 0, on a :

uor

Ce qui permet de conclure .
2p
z0 ( x ) = y0 ( x ) ln( x ) + ∑ bp x 1 x ∞
p=1 3. Comme y0 ( x ) ∼ ( )0 = 1, lim ∑ b p x2p = 0 et
∞ x → 0+ Γ (0 + 1 ) 2 x →0
p=1
1
z0′ ( x ) = y0 ( x ) + ln( x ).y0′ ( x ) + 2 ∑ pb p x2p−1 lim ln( x ) = −∞, on a :

P
x p=1 x → 0+
1 2 z0 ( x ) ∼ ln( x ).
z0′′ ( x ) = − 2 y0 ( x ) + y0′ ( x ) + ln( x ).y0′′ ( x ) + x → 0+
x x ceci permet de prouver ( comme à la question II 3.b) que les

solutions y0 et z0 sur R ∗+ de ( F0 ) sont linéairement indépen-
∑ p(2p − 1)b p x2p−2

r
2
p=1 dantes .et avec les mêmes raisons que dans I I I.3b), toute so-
lution de ( F0 ) est ∞ de la forme :

e
Donc x2 z0′′ ( x ) + xz0′ ( x ) − ( x2 + 0)z0 ( x ) = 2 ′′
−y0 ( x ) + 2xy0′ ( x ) + ln(yx )=.xαy
y0 + 2pβ(2p
∑ α,
( x )βz+0 où − 1des
sont
2p
)b p xconstantes réelles arbitraires .
er
p=1

m
B- Etude de ∞
( F1 ) :2p
+y0 ( x ) + ln( x ).xy0′ ( x ) + ∑ b p 2px
1. . p=1

1
m

− x2 ln( x )y0 ( x ) − a. x2p+2tout p ∈ N ∗ , on a : c2p (α) =


∑ b pPour c
(α + 2p)2 − 1 2( p−1)

r
p=1
p p p
En tenant compte du fait que y0 est solution sur R ∗+ de 1
, donc ∏ c2k (α) = ∏ (α + 2k)2 − 1 ∏ c2(k−1) et par

o
( F0 ) et de la question précédente, il vient : k=1 k= k=1
or

F
✉: mamouni.myismail@gmail.com

203
i

r
s
P ROBLÈMES C ORRIGÉS -MP ✍ M AMOUNI M Y I SMAIL
MAMOUNI . NEW. FR

i
s

s
u
p
1 1
suite c2p (α) = ∏ (α + 2k)2 − 1 c0 (α). dp = ( H p + H p +1 − 1) =

s
22p+1 p!( p + 1)
k=
1 1 1 1

é
et comme c0 (α) = 1, on déduite que : (2H p + − 1) ∼ 2p ln( p),

u
p 2 2p + 1 p!( p + 1)! p+1 p→∞ 2 p! ( p + 1)!
1
c2p (α) = ∏ donc le rayon de convergence demandé :

r
2
k= (α + 2k) − 1 Rd = + ∞

é
d
b. Pour tout p ∈ N∗ , d p = c2p (1). Comme

Po r
r
p
c2p (α) = exp(− ∑ ln((α + 2k)2 − 1), on a : c2p

(α ) =

u
k=1
p 2. .
2(α + 2k)

uor
−∑ c2p (α). D’où
k=1
(α + 2k)2 − 1
p p
2(1 + 2k) 1
dp = −∑ ∏ =
k=1
(1 + 2k) − 1 k= (α + 2k)2 − 1
2  

P
∗ 2
p p a. On a : Pour tout p ∈ N , (1 + 2p) − 1 d p + 2(1 +
2(1 + 2k) 1
− ∑ 4k(1 + k) ∏ (1 + 2k)2 − 1 . 2p)c2p (1) = d p−1 . En effet :
 
k=1 k= 2
p
1
p
1 1
p
1 1 1 1 par dérivation de l’identité c 2p ( α ) ( 1 + 2p ) − 1 =
Or ∏  
(1 + 2k)2 − 1 ∏ ∏
= = =

r
k= k=
4k(1 + k) 22p k= k(1 + k) 22p p! ( p + 1)! c2( p−1) (α), on a : c2p ′
(α) (α + 2p)2 − 1 + 2(α +
et)c (α) = c′
2p 2( p − 1) ( α )

e
2p
2(1 + 2k) 1 1 1 1 1 1 1 1 1
= − = − ( − )= ( + ) Pour α = 1, on a :
er
, 4k ( k + 1 ) k k ( k + 1 ) k 2 k k + 1 2 k k + 1 d p ((1 + 2p)2 − 1) + 2(1 + 2p)c2p (1) = d p−1

m
p
2(1 + 2k) 1
donc ∑ = ( H p + H p+1 − 1). D’où le résultat
k=1
4k(k + 1) 2

demandé :
m

1 b. Il est clair que les fonctions y 1 et x 7 → ∑ d p x2p+1 sont

r
d p = 2p+1 ( H p + H p +1 − 1) p=1
2 p!( p + 1) ∞ ∗
de classe C sur R + et par dérivation on obtient pout

o
c. On a : tout x > 0 :
or

F
✉: mamouni.myismail@gmail.com

204
i

r
s
✍ M AMOUNI M Y I SMAIL MAMOUNI . NEW. FR
P ROBLÈMES C ORRIGÉS -MP

i
s

s
!

u
∞ ∞
4 2 2 2
x2 u1′′ ( x ) + xu1′ ( x ) − (1 + x2 )u1 ( x )
′′
x2 2y1′ ( x ) ln( x ) + y1′ ( x ) − 2 y1 (xx )u+
1 (x
∑) +2pxu

+) 1−)d(p1x+
1 (x
2px
−1)u1 ( x ) = 4xy1′ ( x ) + ∑ (2p + 1)2

s
= (2p
x x p=1 p=0
! ∞

é
4(2p + 1)

u

2 = ∑ x2p+
+ x 2y1′ ( x ) ln( x ) + y1 ( x ) + ∑ (2p + 1)d p x2p p!( p + 1)!22p+1
x p=0
p=1

r
! ∞

− ∑ d p x 2(

é
2 2p+1
−(1 + x ) 2y1 ( x ) ln( x ) + ∑ d p x p=0
p=0 ∞
  1 4(2p +

Po r
r
2 ′′
= 2 ln( x ) x y1 ( x ) + xy1 ( x ) − (1 + x2 )y1 ( x ) + 4xy1′ ( x )
′ = ∑ p!( p + 1)!22p 2
∞ ∞ p=0 | {z }

u
+ ∑ (2p + 1)2 d p x2p+1 − (1 + x2 ) ∑ d p x2p+1 ∞
= c2p (1)

uor
p=0 p=0
Comme y1 est xsolution sur R ∗+ de ( F1 ), on a : + ∑ (2p + 1)2 − 1)
p=0
x2 y1′′ ( x ) + xy1′ ( x ) − (1 + x2 )y1 ( x ) = 0 et donc ∞ 
= ∑ 2(2p + 1)c2p (1) +
p=1 |

P
= 2x
On déduit alors que u1 est bien solution sur R ∗+ de (E1 )
.

r
3. .

e
er

e0
a. On pose u1 ( x ) = + ∑ e p x p−1 avec R =

m
x p=1
Rcv( ∑ e p x p−1 ) > 0.
p≥1
m

Sur ]0, R[, on a : x2 u1′′ ( x ) + xu1′ ( x ) − (1 + x2 )u1 (!


x) −

r

2e0
2x = x2 + ∑ ( p − 1)( p − 2)e p x p−2 +
x3

o
p=1
or

F
✉: mamouni.myismail@gmail.com

205
i

r
s
P ROBLÈMES C ORRIGÉS -MP ✍ M AMOUNI M Y I SMAIL
MAMOUNI . NEW. FR

i
s

s
!

u
∞ ∞ b. ( F1 ) est une équation différentielle linéaire sans second
− e0
∑ ( p − 1) e p x p −2

s
x + − 2x = ∑ ( p ( p − 1) e p − membre associée à (E1 ) et comme z1 et u1 sont solutions
x2 p=1 p=0
sur R ∗+ de (E1 ), il en résulte que z1 − u1 est solution sur

é
p−1

u
e p−2 ) x − (e0+ 2) x − e1 = 0. comme dans la question R ∗+ de ( F1 ) .
 e0 = −2

r
...., on déduit : e =0 , ce qui
 1

é
∀ p ≥ 3, p( p − 2)e p − e p−1 = 0 4. Comme dans la question....., en étudiant le comportement des
permet de conclure par une récurrence que : ∀ p ∈ N, solutions z1 et y1 au voisinage de 0+ , on déduit que (y1 , z1 )
est système fondamental de solutions sur R ∗+ de ( F1 ), donc

Po r
e0

r
e2p+1 = 0 et e2p = 2p car e0 = −2
2 p!( p + 1)! = −2c2p (1) toute solution sur R ∗+ de ( F1 ) est de la forme : y : x 7→

u
et par suite R est infini et que u1 est solution sur R ∗+ de αy1 ( x ) + βz1 ( x ) où α et β sont des constantes réelles arbi-
(E1 ). traires .

uor
r P
e
er

m
m

or
or

F
✉: mamouni.myismail@gmail.com

206
MAMOUNI . NEW. FR
✍ M AMOUNI M Y I SMAIL

Devoir Libres
16 Fonction Gamma et Psi d’Euler
ri

ir
s
Blague du jour
s
- Un élève se lève et va baisser le store, Le prof : T’arrivais pas à dormir ?

s
- Un prof M. à ses élèves : L’an prochain, quand on vous demandera le nom
u
de votre prof de maths de l’an dernier, ne dites surtout pas que c’était moi.

s
- Un élève : Pardon monsieur, je ne vois pas bien ce qui est écrit sur le
tableau ! Le prof : Désolé, ça fait longtemps que j’essaye de perdre du poids,
é

u
j’ai toujours pas réussi !
r

é
Mathématicien du jour
Leonhard Paul Euler (1707-1783)
Po r
r
Mathématicien et physicien suisse, il fit d’importantes découvertes dans le calcul infinitési-
u
mal et la théorie des graphes. Il introduisit également une grande partie de la terminologie
uor
et de la notation des mathématiques modernes. Il est également connu pour ses travaux
en mécanique, en dynamique des fluides, en optique et en astronomie.
P
Problème I : Extrait CCP 2006, MP
r
Si l’on rentre sur un logiciel de calcul formel l’instruction :
e
sum(1/(n^2-α ^2), n=1..infinity);
er

on obtient le résultat suivant :


m −
1 ψ (1 − α ) 1 ψ (1 + α )
+ .
2 α 2 α
rm

Un des objectifs du problème qui comporte trois paragraphes est de démontrer la


r
formule correspondante suivante :
+∞
1 ψ (1 + α ) − ψ (1 − α )
∑ n2 − α2 =
o
n =1

où α ∈ ]0,1[ et ψ est une fonction « d’Euler » que l’on définira ultérieurement.
Fo

Dans le paragraphe I, on définit la fonction Gamma d’Euler et on en montre quelques propriétés que
l’on utilisera dans le paragraphe II pour définir la fonction ψ d’Euler et démontrer la formule ci-
dessus.
Enfin, dans le paragraphe III, on étudie plus en détail la fonction Gamma, dans le but de la
représenter graphiquement.

Les paragraphes II et III sont indépendants mais utilisent des résultats du paragraphe I.
✉: mamouni.myismail@gmail.com
MAMOUNI . NEW. FR
✍ M AMOUNI M Y I SMAIL
1. Questions préliminaires
.
1
a. Soit α ∈ ]0,1[ , justifier la convergence de la série ∑ n2 − α2 .
n ≥1
ri
1
b. Pour quelle(s) valeur(s) de β l’intégrale
∫ t βdt est-elle convergente ? On ne demande

ir
s
0
pas de justifier.
s

s
I. La fonction Gamma d’Euler
u
2. Soit x un réel strictement positif, justifier l’existence d’un réel A strictement positif, tel que

s
1
pour tout réel t strictement supérieur à A, on ait e −t t x −1 < 2 , puis montrer que l’intégrale
é
t

u
+∞

∫ e −t t x −1 dt est convergente.
r
0

é
+∞
On peut donc définir sur ]0, +∞[ , la fonction Gamma d’Euler notée Γ par Γ( x) =
∫ 0
e −t t x −1 dt .
Po r
r
3. Calculer Γ(1) puis montrer la relation fondamentale suivante notée ( F ) :
u
∀x ∈ ]0, +∞[ , Γ( x + 1) = xΓ( x) (F ) .
uorEn déduire Γ(2) . Soit n un entier naturel non nul, donner une expression simple de Γ(n) .

4. Dérivabilité de la fonction Gamma


Soient a et b des réels tels que 0 < a < b .
P
a. Pour tout réel t strictement positif, on définit sur ]0, +∞[ , la fonction ht par ht ( x) = t x −1 .
Déterminer les variations de ht . On pourra discuter selon la valeur de t.
(
En déduire que ∀x ∈ [ a, b ] , ∀t ∈ ]0, +∞[ , 0 ≤ t x −1 ≤ t a −1 + t b −1 . )
r
b. Pour cette question, on pourra utiliser librement le résultat suivant : pour tout réel r
+∞


e
strictement supérieur à −1 , l’intégrale e −t t r ln t dt est convergente.
0
er

Montrer avec soin que la fonction Γ est dérivable sur [ a, b ] et déterminer Γ′( x) sous
m forme d’intégrale.

La fonction Γ est donc dérivable sur ]0, +∞[ . On montre de la même façon et on admet que Γ est
rm

r
de classe C ∞ sur ]0, +∞[ .
o
5. Soit p un entier naturel, écrire sans justifier Γ ( p ) ( x) sous forme d’intégrale. ( Γ ( p ) désigne la
dérivée p-ième de Γ ).
Fo

II. La fonction ψ d’Euler

On montre sans difficulté, que pour tout réel x strictement positif, Γ( x) > 0 . On peut donc définir
Γ′( x)
sur ]0, +∞[ , la fonction ψ d’Euler par ψ ( x) = .
Γ( x)
Soit α ∈ ]0,1[ .
✉: mamouni.myismail@gmail.com
MAMOUNI . NEW. FR
✍ M AMOUNI M Y I SMAIL
6. Relation entre ψ et des sommes.
. a. Quelle relation différentielle relie ψ et ln Γ ? En déduire que
1
∀x ∈ ]0, +∞[ , ψ ( x + 1) − ψ ( x) = (on pourra utiliser ( F ) ).
x
ri
b. Soit n un entier naturel non nul, déterminer en fonction de n et de α , quatre réels

ir
s
u1 , v1 , u2 , v2 tels que
n n
1 1
∑ k − α = ψ(u1 ) − ψ(v1 ) et ∑ k + α = ψ(u2 ) − ψ(v2 ) .
s
k =1 k =1

s
u
1

s
7. Décomposer en éléments simples la fraction rationnelle R ( X ) = puis en déduire que
X − α2
2
é
l’on a, pour tout entier naturel non nul n :

u
n
1 1 1
∑ k 2 − α 2 = 2α ( ψ(1 + α) − ψ(1 − α) ) + 2α ( ψ(n + 1 − α) − ψ(n + 1 + α) ) .
r
k =1

é
8. Enoncer avec soin l’inégalité de Cauchy-Schwarz pour le produit scalaire
Po r
r
+∞
( f , g ) 6 ∫0 f (t ) g (t ) dt , puis l’utiliser pour montrer que ( Γ′ ) ≤ Γ′′ × Γ où Γ′′ désigne la
2
u
dérivée seconde de Γ . En déduire que la fonction ψ est croissante.
uor
9. Montrer que pour tout entier naturel non nul n, on a
1 1
0 ≤ ψ(n + 1 + α) − ψ (n + 1 − α) ≤ + (penser que α ∈ ]0,1[ ).
n +1 n
+∞ ψ (1 + α ) − ψ (1 − α )
P
1
Conclure que ∑ 2 = .
n =1 n − α
2 2α
r
III. Etude de la fonction Gamma
e
10. Utiliser ( F ) pour déterminer un équivalent de Γ au voisinage de 0+ . En déduire la limite
er

de Γ en 0.
m
11.
a. Justifier que Γ′ est strictement croissante sur ]0; +∞[ .
rm

b. Justifier l’existence d’un réel c ∈ ]1, 2[ tel que Γ′(c) = 0 . En déduire le signe de Γ′ et
r
enfin, les variations de Γ .
o
12.
a. Déterminer la limite de Γ en +∞ . On pourra utiliser des résultats précédents.
b. Dessiner l’allure du graphe de Γ . On représentera, s’il y a lieu, les asymptotes et les
Fo

tangentes horizontales.

✉: mamouni.myismail@gmail.com
MAMOUNI . NEW. FR
✍ M AMOUNI M Y I SMAIL
Problème II : Extrait e3a 2011, MP

√ Z +∞ dt
 
1 π 2
ri
On admet que Γ = π, = .
2 1 + t4 4

ir
0
s
Z +∞
dt
① Pour tout x ∈]0, 2π [, on note I ( x ) = .
ch(t2 ) − cos2 x
s
0
Z +∞
dt

s
① Montrer que l’application x 7→ est bornée sur ]0, 2π [.
u
1 ch(t2 ) − cos2 x

s
t4
+ 1 − ch(t2 )
② Montrer que l’application t 7→ | 4 2 |, définie sur ]0, 1], est prolongeable
é

u
t 2
(ch(t ) − 1)
2
r
est prolongeable par continuité en 0.

é
Z 1 Z 1
dt dt
③ En déduire que l’application x 7→ 2 2
− 4
est bornée
0 ch(t ) − cos x 0 t
1 + − cos x 2
Po r
r
2
sur ]0, 2π [.
u
Z 1
dt
④ Calculer l’intégrale , en effectuant le changement de variable u =
uor 0 t 4
1 + − cos2 x
2
t
1 .
(2(1 − cos x )) 4
P
En déduire un équivalent de I ( x ) quand x tend vers 0 par valeurs positives.
② Montrer que, pour tout entier naturel n non nul, on a
Z +∞ −nt
1 1 e
√ =   √ dt.
n 1 t
r
0
Γ
2
e
N
sin nx
③ Pour tout entier N supérieur ou égal à 1 et pour tout réel x, on note S N ( x ) = ∑ √ .
er

n =1 n
m ① Calculer, pour tous réels x et t, la somme
N
∑ e−nt sin(nx). On pourra remarquer que
n =1
− nt
e sin(nx ) est la partie imaginaire du nombre complexe e(−t+ix)n .
rm

r
N
− nt
Z +∞ ∑ e sin(nx )
1
o
n =1
② En utilisant l’égalité S N ( x ) = √ √ dt, montrer que la suite
π 0 t
(S N ( x )) est convergente pour tout x réel élément de ]0, 2π [ et que l’on a l’égalité
suivante :
Fo

+∞
sin nx sin x +∞ dt
Z
∀ x ∈]0, 2π [, ∑ √ = √ 2
.
n =1 n π 0 ch(t ) − cos x
+∞
sin nx
④ En déduire un équivalent de la somme de la série ∑ √
n
quand x tend vers 0 par
n =1
valeurs positives.

✉: mamouni.myismail@gmail.com
i

r
s
P ROBLÈMES C ORRIGÉS -MP ✍ M AMOUNI M Y I SMAIL
MAMOUNI . NEW. FR

i
s

s
u
Corrigé Problème II : Pr. Patte

s
t4
Z 1
dt
Z 1
dt
Z 1 +1
2

é
1

u
① Soit x ∈]0, 2π [. La fonction t 7→ est continue − = 
ch(t ) − cos2 x
2 0 ch(t2 ) − cos2 x 0 t4 0 t4
1+ − cos2 x 1+ − cos2
2 2 2

r
sur [0, +∞[ et équivalente en +∞ à 2e−t , donc intégrable sur

é
[0, +∞[ et a fortiori sur ]0, +∞[. t4
1 Z 1 +1
① Soit x ∈]0, 2π [. Alors, sur [1, +∞[, 0 6 6 6  2

Po r
r
ch(t ) − cos2 x
2
0 t4
1 1 1+ − cos2
. Comme t 7→ est inté- 2

u
2
ch(t ) − 1 2
ch(t ) − 1
t4

uor
grable sur [1, +∞[ (même preuve que pour l’ex-
+1−c
Z 1
istence de I ( x )), on obtient l’encadrement 0 6 6  2 
Z +∞ Z +∞
dt dt 0 t4
2 2
6 . Donc la fonc- 1 + − 1 (c
1 ch(t ) − cos x 1 ch(t2 ) − 1 2
Z +∞
dt La majoration est possible du fait de la continuité de la

P
tion x 7→ est bornée sur ]0, 2π [.
1 ch(t2 ) − cos2 x fonction sous l’intégrale sur [0, 1] et montre le caractère
Z 1 Z 1
t4 t8 dt dt
② Au voisinage de 0, ch(t2 ) = 1 + + + o ( t4 ). borné de x 7→ 2 2
− 4
.
2 24 0 ch(t ) − cos x 0 t
1 + − cos x 2
t4

r
2
+ 1 − ch(t2 ) 1
On en déduit 4 2 → − . Donc t 7→
− ④ Avec le changement de variable affine proposé :

e
t 0 6
(ch(t2 ) − 1) 1
er
2 Z 1
dt 2
Z
t4 (2(1 − cos x ))1/4 du .

m
=
+ 1 − ch(t2 ) 0 t4 (2(1 − cos x ))3/4 0 1 + u4
2 est prolongeable en une fonction con- 1 + − cos2 x
t4 2
2
(ch(t ) − 1) La suite du calcul de l’intégrale est parfaitement inutile
m

2
tinue sur [0, 1]. pour obtenir un équivalent de I ( x ).

r
③ Les deux intégrales existent (par continuité des fonc- 1
Quand x tend vers 0+ , tend vers +∞,

o
tions sur [0, 1]). (2(1 − cos x ))1/4
or

F
✉: mamouni.myismail@gmail.com

212
i

r
s
✍ M AMOUNI M Y I SMAIL MAMOUNI . NEW. FR
P ROBLÈMES C ORRIGÉS -MP

i
s

s
u
1 √ Après simplification, la partie imaginaire vaut
Z 

s
(2(1 − cos x )) 1/4 du π 2
donc tend vers . N e−t sin x − e− Nt sin(( N + 1) x ) + e−( N +1)t
0 1 + u4 4 ∑e − nt
sin(nx ) =

u
n =1 (1 − e−t cos x )2 + e−2t sin2 x
2
De plus, 2(1 − cos x ) ∼ x2 , donc ∼ sin x − e− Nt sin(( N + 1) x ) + e−( N +1)t sin( N

r
0 (2(1 − cos x ))3/4 0 =
2(cht − cos x )

é
Z 1
2 dt π 1
. Donc ∼ √ . 3/2 . D’après N
x 3/2 0 t4 0 2 x − nt
1+ − cos2 Z +∞ ∑ e sin(nx )

Po r
x

r
Z 1 2 1 n =1
dt π 1 ② SN (x) = √ √ dt =
1.c, . 3/2 . Comme

u
∼ √ π 0 t
2 2
0 ch(t ) − cos x 0 2 x 1
Z +∞
sin x − e− Nt sin(( N + 1) x ) + e−( N +1)t sin( Nx )
Z +∞

uor
dt √ √ dt
est bornée sur ]0, +∞[, I ( x ) ∼ π 0 2(cht − cos x ) t
1 ch(t2 ) − cos2 x 0 Z +∞ √
π 1 1 sin x u= t
√ . 3/2 . semble converger vers √ √ dt =
π 0 2(cht − cos x ) t
2 x Z +∞
sin x du

P
√ 2
.
π 0 ch(u ) − cos x
② Par changement de variable affine sur une intégrale conver- On peut démontrer ce résultat à l’aide du théorème de
Z +∞ −nt Z +∞ −u
e u = nt 1 e 1 convergence dominée ou par simple majoration :
gente, √ dt = √ √ du = √ Γ(1/2),

r
Z +∞
0 t
Z +∞ −nt
n 0 u n −e− Nt sin(( N + 1) x ) + e−( N +1)t sin( Nx )
1 1 e √ dt 6
donc √ = √ √ dt. 0 2(cht − cos x ) t

e
n π 0 t Z +∞
−e− Nt sin(( N + 1) x ) + e−( N +1)t sin( Nx )
er
√ dt 6

m
0 2(cht − cos x ) t
③ ①   Z +∞ − Nt √  
N (− t + ix ) (− t − ix ) e + e −( N +1)t π 1 1
N
1 − e N (−t+ix) 1−e 1−e √ dt = √ +√
∑ en(−t+ix) = e−t+ix
1−e (− t + ix )
= e − t+ix
1−e (− t + ix )

1−e (− t − ix )
0 2(1 − cos x ) t 2(1 − cos x ) n n+1
m

n =1
   converge vers 0.

r
N (− t + ix ) (− t − ix ) r
1−e 1−e +∞
sin nx sin x x π 1 π
= e−t+ix . ④ Finalement, ∑ √ = √ I ( x ) ∼ √ . √ . 3/2 = .
(1 − e−t cos x )2 + e−2t sin2 x 2x

o
n =1 n π 0 π 2 x
or

F
✉: mamouni.myismail@gmail.com

213
i

r
s
P ROBLÈMES C ORRIGÉS -MP ✍ M AMOUNI M Y I SMAIL
MAMOUNI . NEW. FR

i
s

s
u

s
Devoir Surveillé
21 Transformée de Laplace

u
Pierre-Simon de Laplace (1749-1827)

r
Blague du jour

Mathématicien du jour
é
Il était une fois un explorateur qui tomba devant un lion. Mathématicien, astronome et physicien français.Il a contribué
L’explorateur apeuré dit : de façon décisive à l’émergence de l’astronomie mathématique

Po r
r
- Dieu, faites que ce lion est une pensée et foie en vous. et de théorie des probabilités. il était nommé ministre de
l’Intérieur, puis comte de l’Empire, en enfin marquis.

u
Et le lion répondis : Dieu, bénissez ce repas.

uor
EXERCICE 2

P
Énoncé : CCP 2011, MP

On considère l’équation différentielle (E); 2xy′ − 3y = x.
Le sujet est composé de deux exercices et d’un problème indépendants. ① Résoudre (E) sur ]0, +∞[.

r
EXERCICE 1 ② Déterminer l’ensemble des solutions de (E) sur l’intervalle
[0, +∞[.

e
2x n
On considère la série ∑ 2 .
er
n ≥2 n − 1

m
① Déterminer le rayon de convergence R de cette série entière. PROBLÈME : AUTOUR DE LA TRANSFORMATION DE LAPLACE
2x n
② On note S la fonction somme de la série ∑ 2 . Déter- Dans tout ce problème, on note :
n ≥2 n − 1 -F (R + , R ) l’ensemble des applications de R + dans R ;
m

miner S sur ] − R, R[. - E l’ensemble des fonctions f : R + −→ R, continues, telles que,

r
③ Démontrer que S( x ) admet une limite lorsque x tend vers 1 pour tout x > 0 réel, la fonction t 7−→ f (t)e− xt soit intégrable sur

o
par valeurs strictement inférieurs et déterminer cette limite. R+ ;
or

F
✉: mamouni.myismail@gmail.com

214
i

r
s
✍ M AMOUNI M Y I SMAIL
MAMOUNI . NEW. FR
P ROBLÈMES C ORRIGÉS -MP

i
s

s
u
-F l’ensemble des fonctions continues et bornées sur R + . ③ ① On considère la fonction U : R + −→ R définie par

s
Pour tout f dans E, on appelle transformée de L APLACE de f et on U (t) = 1. déterminer L(U ).
note L( f ) la fonction définie pour tout x > 0 réel par :

é
② Soit λ ≥ 0 réel. on considère la fonction hλ : [0, +∞[−→

u
Z +∞
L( f )( x ) = f ( t )e − xt
dt. R définie pour tout t ≥ 0 par : hλ (t) = e−λt . Démontrer
0 que hλ est dans E et déterminer L(hλ ).

é
① Questions préliminaire ④ Soient f dans E et n dans N. On considère gn : t 7−→ tn f (t)
de [0, +∞[ dans R.
Soient a ∈ R et f : [ a, +∞[−→ R une fonction continue par

Po r
r
morceaux. Pour tout x dans [ a, Pour x > 0, justifier de l’existence de A > 0 tel que tn e− xt 6
Z +∞[, on pose :
x − xt
e 2 pour tout t ≥ A.

u
G(x) = f (t)dt.
a
En déduire que gn est un élément de E.

uor
On considère les propositions suivantes :
(i) f est intégrable sur [ a, +∞[ ; ⑤ Transformée de Laplace d’une dérivée
Soit f dans E de classe C 1 , croissante et bornée sur [0, +∞[.
(ii) G admet une limite finie en +∞.
Démontrer que f ′ est encore dans E et que l’on a : ∀ x ∈

P
Donner, sans démonstration, toutes les implications possibles ]0, +∞[, L( f ′ )( x ) = x L( f )( x ) − f (0).
entre (i) et (ii) lorsque :
⑥ Régularité d’une transformée de Laplace
(a) f est positive sur [ a, +∞[ ;
(b) f n’est pas positive sur [ a, +∞[. ① Démontrer que pour tout f dans E, la fonction L( f ) est
de classe C 1 sur ]0, +∞[ et que l’on a L( f )′ = −L( g1 ) où

r
g1 a été définie à la question 4.
PARTIE I : Exemples et propriétés

e
② Démontrer que pour tout f dans E, la fonction L( f )
er
est de classe C ∞ sur ]0, +∞[ et pour tout x > 0 et

m
② ① Démontrer que E est un sous-espace vectoriel de n ∈ N, déterminer L( f )(n) ( x ) à l’aide d’une transformée
F ( R + , R ). de Laplace.
② Démontrer que F est un sous-espace vectoriel de E.
m

PARTIE II : Comportements asymptotiques

r
③ Justifier que L est une application linéaire de E dans
F (R +
∗ , R ), espace vectoriel des applications de ]0, + ∞[

o
dans R. Dans toute cette partie, f est un élément de E
or

F
✉: mamouni.myismail@gmail.com

215
i

r
s
P ROBLÈMES C ORRIGÉS -MP ✍ M AMOUNI M Y I SMAIL
MAMOUNI . NEW. FR

i
s

s
u
⑦ On suppose dans cette question que f est dans F. ② On fixe ε > 0.

s
① Déterminer la limite en +∞ de L( f ). Justifier de l’existence de A réel positif tel que pour tout
t ≥ A, on ait | R(t)| 6 ε.

é
② Théorème de la valeur initiale

u
On suppose, de plus, que f est de classe C 1 et croissante En déduire que, pour tout x > 0, on a : |L( f )( x ) −
Z A
sur R + , avec f ′ bornée sue R + .

r
R(0)| 6 x | R(t)|dt + ε

é
Démontrer que lim x L( f )( x ) = f (0). 0
x →+ ∞ ③ Démontrer que L( f ) se prolonge par continuité en 0 (on
⑧ Théorème de la valeur finale précisera la valeur en 0 de ce prolongement).

Po r
r
On suppose dans cette question que lim f ( x ) = ℓ où ℓ est
x →+ ∞
PARTIE III : Application

u
un réel. Soit (an )n∈N une suite de réels strictement positifs
qui converge vers 0.

uor
① Démontrer que f appartient à F. ⑩ Calcul de l’intégrale de Direchlet
② Soit n un entier naturel. Démontrer que an L( f )(an ) = sin t
Z +∞ Ici f la fonction définie par : f (0) = 1 et f (t) = pour
t
hn ( x )dt[ x ] où hn est la fonction définie sur [0, +∞[ t > 0 réel.

P
0   +
−x
par hn ( x ) = e f
x
. Z que la fonction G : R −→ R définie par
① Démontrer
x
an G(x) = f (t)dt admet une limite réelle ℓ en +∞.
0
③ En déduire à l’aide du théorème de la convergence dom- Z ( n + 1) π

r
inée, que lim an L( f )(an ) = ℓ. ② En considérant la série un où un = | f (t)|dt,
n →+ ∞ ∑ nπ
n ≥0
④ Lorsque ℓ 6= 0, déterminer un équivalent de L( f )( x ) en

e
+
démontrer que f n’est pas intégrable sur R .
0.
er
③ Soit x > 0. Démontrer, en détaillant les calculs, que pour
⑨ Dans cette question, on suppose que f est intégrable sur R +

m
Z +∞ tout
Z X X > 0 on a :
et on pose R( x ) = f (t)dt pour tout x dans [0, +∞[. 1
x (sin t)e− xt dt = − (e− xX ( x sin X + cos X ) − 1).
0 1 + x2
① Démontrer que R est une fonction de classe C 1 sur
m

Démontrer que la fonction t 7−→ (sin t)e− xt est inté-


[0, +∞[ et déterminer R′ .

r
grable sur R + .
En déduire que, pour tout x > 0 réel, on a : L( f )( x ) = Z +∞
Déterminer alors (sin t)e− xt dt.

o
R(0) − x L( R)( x ). 0
or

F
✉: mamouni.myismail@gmail.com

216
i

r
s
✍ M AMOUNI M Y I SMAIL
MAMOUNI . NEW. FR
P ROBLÈMES C ORRIGÉS -MP

i
s

s
u
④ Déterminer, pour x > 0, une expression simple de lim L( f )( x ) = ℓ.

s
x →0
L( f )( x ) et en déduire ℓ.
On notera que, par rapport à la question 9), on a rem-
Pour cela, on pourra utiliser le résultat suivant(La dé-

é
placé Zl’hypothèse f intégrable sur R + par l’hypothèse

u
marche de la preuve étant identique à celle de la ques- x
tion 9)) : lim f (t)dt = ℓ ∈ R.

r
Z x x →+ ∞ 0

é
Lorsque f dans E vérifie lim f (t)dt = ℓ ∈ R, alors
x →+ ∞ 0

Po r
r
u
uor
i
F

nn
i

P
Á la prochaine

e r
er

m
m

or
or

F
✉: mamouni.myismail@gmail.com

217
i

r
s
P ROBLÈMES C ORRIGÉS -MP ✍ M AMOUNI M Y I SMAIL
MAMOUNI . NEW. FR

i
s

s
u

s
Devoir Surveillé
22 Transformée de Laplace

u
Pierre-Simon de Laplace (1749-1827)

r
Blague du jour

Mathématicien du jour
é
Il était une fois un explorateur qui tomba devant un lion. Mathématicien, astronome et physicien français.Il a contribué
L’explorateur apeuré dit : de façon décisive à l’émergence de l’astronomie mathématique

Po r
r
- Dieu, faites que ce lion est une pensée et foie en vous. et de théorie des probabilités. il était nommé ministre de
l’Intérieur, puis comte de l’Empire, en enfin marquis.

u
Et le lion répondis : Dieu, bénissez ce repas.

uor
EXERCICE 2

P
Énoncé : CCP 2011, MP

On considère l’équation différentielle (E); 2xy′ − 3y = x.
Le sujet est composé de deux exercices et d’un problème indépendants. ① Résoudre (E) sur ]0, +∞[.

r
EXERCICE 1 ② Déterminer l’ensemble des solutions de (E) sur l’intervalle
[0, +∞[.

e
2x n
On considère la série ∑ 2 .
er
n ≥2 n − 1

m
① Déterminer le rayon de convergence R de cette série entière. PROBLÈME : AUTOUR DE LA TRANSFORMATION DE LAPLACE
2x n
② On note S la fonction somme de la série ∑ 2 . Déter- Dans tout ce problème, on note :
n ≥2 n − 1 -F (R + , R ) l’ensemble des applications de R + dans R ;
m

miner S sur ] − R, R[. - E l’ensemble des fonctions f : R + −→ R, continues, telles que,

r
③ Démontrer que S( x ) admet une limite lorsque x tend vers 1 pour tout x > 0 réel, la fonction t 7−→ f (t)e− xt soit intégrable sur

o
par valeurs strictement inférieurs et déterminer cette limite. R+ ;
or

F
✉: mamouni.myismail@gmail.com

218
i

r
s
✍ M AMOUNI M Y I SMAIL
MAMOUNI . NEW. FR
P ROBLÈMES C ORRIGÉS -MP

i
s

s
u
-F l’ensemble des fonctions continues et bornées sur R + . ③ ① On considère la fonction U : R + −→ R définie par

s
Pour tout f dans E, on appelle transformée de L APLACE de f et on U (t) = 1. déterminer L(U ).
note L( f ) la fonction définie pour tout x > 0 réel par :

é
② Soit λ ≥ 0 réel. on considère la fonction hλ : [0, +∞[−→

u
Z +∞
L( f )( x ) = f ( t )e − xt
dt. R définie pour tout t ≥ 0 par : hλ (t) = e−λt . Démontrer
0 que hλ est dans E et déterminer L(hλ ).

é
① Questions préliminaire ④ Soient f dans E et n dans N. On considère gn : t 7−→ tn f (t)
de [0, +∞[ dans R.
Soient a ∈ R et f : [ a, +∞[−→ R une fonction continue par

Po r
r
morceaux. Pour tout x dans [ a, Pour x > 0, justifier de l’existence de A > 0 tel que tn e− xt 6
Z +∞[, on pose :
x − xt
e 2 pour tout t ≥ A.

u
G(x) = f (t)dt.
a
En déduire que gn est un élément de E.

uor
On considère les propositions suivantes :
(i) f est intégrable sur [ a, +∞[ ; ⑤ Transformée de Laplace d’une dérivée
Soit f dans E de classe C 1 , croissante et bornée sur [0, +∞[.
(ii) G admet une limite finie en +∞.
Démontrer que f ′ est encore dans E et que l’on a : ∀ x ∈

P
Donner, sans démonstration, toutes les implications possibles ]0, +∞[, L( f ′ )( x ) = x L( f )( x ) − f (0).
entre (i) et (ii) lorsque :
⑥ Régularité d’une transformée de Laplace
(a) f est positive sur [ a, +∞[ ;
(b) f n’est pas positive sur [ a, +∞[. ① Démontrer que pour tout f dans E, la fonction L( f ) est
de classe C 1 sur ]0, +∞[ et que l’on a L( f )′ = −L( g1 ) où

r
g1 a été définie à la question 4.
PARTIE I : Exemples et propriétés

e
② Démontrer que pour tout f dans E, la fonction L( f )
er
est de classe C ∞ sur ]0, +∞[ et pour tout x > 0 et

m
② ① Démontrer que E est un sous-espace vectoriel de n ∈ N, déterminer L( f )(n) ( x ) à l’aide d’une transformée
F ( R + , R ). de Laplace.
② Démontrer que F est un sous-espace vectoriel de E.
m

PARTIE II : Comportements asymptotiques

r
③ Justifier que L est une application linéaire de E dans
F (R +
∗ , R ), espace vectoriel des applications de ]0, + ∞[

o
dans R. Dans toute cette partie, f est un élément de E
or

F
✉: mamouni.myismail@gmail.com

219
i

r
s
P ROBLÈMES C ORRIGÉS -MP ✍ M AMOUNI M Y I SMAIL
MAMOUNI . NEW. FR

i
s

s
u
⑦ On suppose dans cette question que f est dans F. ② On fixe ε > 0.

s
① Déterminer la limite en +∞ de L( f ). Justifier de l’existence de A réel positif tel que pour tout
t ≥ A, on ait | R(t)| 6 ε.

é
② Théorème de la valeur initiale

u
On suppose, de plus, que f est de classe C 1 et croissante En déduire que, pour tout x > 0, on a : |L( f )( x ) −
Z A
sur R + , avec f ′ bornée sue R + .

r
R(0)| 6 x | R(t)|dt + ε

é
Démontrer que lim x L( f )( x ) = f (0). 0
x →+ ∞ ③ Démontrer que L( f ) se prolonge par continuité en 0 (on
⑧ Théorème de la valeur finale précisera la valeur en 0 de ce prolongement).

Po r
r
On suppose dans cette question que lim f ( x ) = ℓ où ℓ est
x →+ ∞
PARTIE III : Application

u
un réel. Soit (an )n∈N une suite de réels strictement positifs
qui converge vers 0.

uor
① Démontrer que f appartient à F. ⑩ Calcul de l’intégrale de Direchlet
② Soit n un entier naturel. Démontrer que an L( f )(an ) = sin t
Z +∞ Ici f la fonction définie par : f (0) = 1 et f (t) = pour
t
hn ( x )dt[ x ] où hn est la fonction définie sur [0, +∞[ t > 0 réel.

P
0   +
−x
par hn ( x ) = e f
x
. Z que la fonction G : R −→ R définie par
① Démontrer
x
an G(x) = f (t)dt admet une limite réelle ℓ en +∞.
0
③ En déduire à l’aide du théorème de la convergence dom- Z ( n + 1) π

r
inée, que lim an L( f )(an ) = ℓ. ② En considérant la série un où un = | f (t)|dt,
n →+ ∞ ∑ nπ
n ≥0
④ Lorsque ℓ 6= 0, déterminer un équivalent de L( f )( x ) en

e
+
démontrer que f n’est pas intégrable sur R .
0.
er
③ Soit x > 0. Démontrer, en détaillant les calculs, que pour
⑨ Dans cette question, on suppose que f est intégrable sur R +

m
Z +∞ tout
Z X X > 0 on a :
et on pose R( x ) = f (t)dt pour tout x dans [0, +∞[. 1
x (sin t)e− xt dt = − (e− xX ( x sin X + cos X ) − 1).
0 1 + x2
① Démontrer que R est une fonction de classe C 1 sur
m

Démontrer que la fonction t 7−→ (sin t)e− xt est inté-


[0, +∞[ et déterminer R′ .

r
grable sur R + .
En déduire que, pour tout x > 0 réel, on a : L( f )( x ) = Z +∞
Déterminer alors (sin t)e− xt dt.

o
R(0) − x L( R)( x ). 0
or

F
✉: mamouni.myismail@gmail.com

220
i

r
s
✍ M AMOUNI M Y I SMAIL
MAMOUNI . NEW. FR
P ROBLÈMES C ORRIGÉS -MP

i
s

s
u
④ Déterminer, pour x > 0, une expression simple de lim L( f )( x ) = ℓ.

s
x →0
L( f )( x ) et en déduire ℓ.
On notera que, par rapport à la question 9), on a rem-
Pour cela, on pourra utiliser le résultat suivant(La dé-

é
placé Zl’hypothèse f intégrable sur R + par l’hypothèse

u
marche de la preuve étant identique à celle de la ques- x
tion 9)) : lim f (t)dt = ℓ ∈ R.

r
Z x x →+ ∞ 0

é
Lorsque f dans E vérifie lim f (t)dt = ℓ ∈ R, alors
x →+ ∞ 0

Po r
r
u
uor
i
F

nn
i

P
Á la prochaine

e r
er

m
m

or
or

F
✉: mamouni.myismail@gmail.com

221
i

r
s
P ROBLÈMES C ORRIGÉS -MP ✍ M AMOUNI M Y I SMAIL
MAMOUNI . NEW. FR

i
s

s
u

s
Devoir Surveillé
23 Ev. Euclidiens : Les "must" Classiques

u
André-Louis Cholesky (1875-1918

r
Blague du jour

Mathématicien du jour
Polytechnicien et officier français, ingénieur topographe et
• Une puce et un labrador discutent : géodésien. Il est célèbre pour sa méthode de résolution des

Po r
r
-Le chien : Qu’est-ce que tu a regardé hier soir la télé ? systèmes d’équations linéaires, toujours intensément utilisée de
-La puce : La deuxième chienne, et toi ? nos jours. Durant la Première Guerre mondiale, il participe à

u
- Moi, canal puce ... l’amélioration d’une cartographie nécessaire à la préparation

uor
• Qu’est-ce qu’un dromadaire ? des tirs. il décède des suites de blessures reçues sur le champ
Réponse : c’est un chameau qui bosse double ! de bataille.

P
rg( x1 , . . . , x p ) = rg Gram( x1 , . . . , x p ).
Matrice de Gram.
② ① Montrer que det G est inchangé si on remplace xk par
Soient C = ( x1 , . . . , x p ) une famille de vecteurs d’un
x k − ∑ λi x i .
espace vectoriel euclidien E de dimension n, et Gram( x1 , . . . , x p )

r
i 6=k
leur matrice de Gram  de type p × p, dont les coefficients
sont Vect xi | Vect x j . On pose Γ( x1 , . . . , x p ) = det ≤ ② Soit F = Vect( x1 , . . . , xn ) et x ∈ E.

e
 Γ ( x1 , . . . , x n , x )
Gram( x1 , . . . , x p ) Montrer que d( x, F)2 =
er
.
Soit B = (e1 , . . . , en ) une base orthonormale directe de E, et Γ ( x1 , . . . , x n )

m
A = MB (C). ③ On suppose dans cette question que B une famille quel-
① ① Comparer rg A et rg( x1 , . . . , x p ). conque de E, vérifiant la relation suivante :
n
m

② Préciser le type de la matrice A, ainsi que ses coefficients. 2


∀ cx ∈ E, kx k = ∑ ( x | e i )2

r
③ Montrer que t AA = Gram( x1 , . . . , x p ). i =1

o
④ Montrer que Ker t AA = Ker A, en déduire que ① Démontrer que (e1 , . . . , en ) est une base de E.
or

F
✉: mamouni.myismail@gmail.com

222
i

r
s
✍ M AMOUNI M Y I SMAIL
MAMOUNI . NEW. FR
P ROBLÈMES C ORRIGÉS -MP

i
s

s
u
n
② Démontrer que : ∀ x, y ∈ E, ( x | y) = ∑ (x | ei )(y | ei ). Décomposition polaire

s
i =1

é
Soit E un espace vectoriel euclidien. Un endomorphisme

u
③ On note G la matrice de Gram de e1 , . . . , en .
Démontrer que G2 = G et conclure. symétrique u ∈ S(E) est dit positif si pour tout x de E, (u( x ), x ) ≥ 0.

r
Il est dit défini positif si pour tout x de E non nul, (u( x ), x ) > 0. On
④ Soit u ∈ L(E), on suppose dans cette question que B une base

é
notera S+ (E) l’ensemble des endomorphismes symétriques posi-
quelconque de E. tifs, et S++ (E) l’ensemble des endomorphismes symétriques défi-
Montrer que Γ(u(e1 ), . . . , u(en )) = (det u)2 Γ(e1 , . . . , en ). nis positifs.

Po r
r
⑤ Soit A ∈ matn, pR. Montrer que det(t AA) ≥ 0. 1. Soit u ∈ S(E). Montrer que u appartient S+ (E) si et seule-

u
ment si ses valeurs propres sont positives ou nulles. Donner
⑥ Soit un tétraèdre ABCD tel que AB = AC = AD = 1 et

uor
π π π une condition nécessaire et suffisante sur les valeurs propres
( AB, AC) ≡ , ( AB, AD ) ≡ , ( AC, AD ) ≡ . Calculer son de u ∈ S(E) pour que u ∈ S++ (E).
4 3 2
volume.
2. Soit u ∈ S+ (E), λ1 , . . . , λ p ses valeurs propres (distinctes), et
p
⑦ Soient B , B ′ deux bases quelconques de E. On note P la ma- Ei = Ker(u − λi Id E ). On définit vi par vi ( x ) = λi x si x ∈ Ei ,

P
trice de passage de B B ′ , et G, G ′ les matrices de Gram de B et vi ( x ) = 0 si x ∈ Ei⊥ . On note enfin v = v1 + · · · + v p . Justi-
et B ′ . Quelle relation y a-t-il entre P, G et G ′ ? fier que v2 = v ◦ v = u, et que v est positif.
⑧ Soit (~e1 , . . . , ~en ) une base de E, G sa matrice de Gram et 3. Soit w un autre élément de S+ (E) tel que w2 = u.

r
G −1 = (aij ).
a. Montrer que wu = uw. En déduire que w(Ei ) ⊂ Ei .
Montrer que : ∀ ~x ∈ E, ∑ aij (~ei | ~x)(~e j | ~x) = k~x k2 .

e
i,j b. Soit wi l’endomorphisme induit par w sur Ei . Vérifier
er
que wi est symétrique positif, puis diagonaliser wi .
⑨ Soit B = (~e1 , . . . , ~en ) une base non orthonormée de E, G sa

m
matrice de Gram f ∈ L(E) et M sa matrice dans B . c. En déduire que w = v.

① Montrer que f est auto-adjoint si et seulement si t MG = 4. Soit f ∈ Gl (E).


m

GM. a. Montrer que f ∗ ◦ f ∈ S++ (E).

r
② Montrer que f est orthogonal si et seulement si t MGM = b. Montrer qu’il existe un unique couple (h, g) ∈ O(E) ×

o
G. S++ (E) tel que f = h ◦ g.
or

F
✉: mamouni.myismail@gmail.com

223
i

r
s
P ROBLÈMES C ORRIGÉS -MP ✍ M AMOUNI M Y I SMAIL
MAMOUNI . NEW. FR

i
s

s
u
① Montrer qu’il existe une matrice T triangulaire
Endomorphismes normaux.

s
supérieure telle que A = t TT. Montrer que T est unique
si on impose la condition : ∀ i, Tii > 0.

é
Soit E un espace vectoriel hermitien. Un endomorphisme u ∈ L(E)

u
n
est dit normal si u et u∗ commutent. ② Application : Montrer que det A ≤ ∏ aii .

r
① Soit u normal, montrer que si F est un sous-espace propre i =1

é
de u alors F⊥ est stable par u. ② Décomposition QR :
En déduire que u est diagonalisable dans base orthonormale. ① Soit M ∈ Mn (R ) inversible. Montrer qu’il existe

Po r
r
La réciproque est-elle vraie ? une matrice orthogonale, P, et une matrice triangulaire

u
② Soit u ∈ L(E). Montrer l’équivalence entre les propriétés supérieure à coefficients diagonaux positifs, T, uniques
telles que M = PT.

uor
suivantes :
(1) u est normal. ② Inégalité de Hadamard : Soit E un espace vecto-
(2) ∀ x ∈ E, ku( x )k = ku∗ ( x )k. riel euclidien, B = (~e1 , . . . , ~en ) une base orthonormée, et
(3) Tout sous-espace vectoriel stable par u est stable par u∗ . C = (~u1 , . . . , ~un ) des vecteurs quelconques.

P
(4) Si un sous-espace vectoriel F est stable par u alors F⊥ est Démontrer que | det(C)| ≤ ∏ k~u j k. Étudier les cas d’-
B j
stable par u.
galité.
(5) Il existe P ∈ C [ X ] tel que u∗ = P(u).

r
③ Soit f ∈ L(E) tel que f ◦ f ∗ = f ∗ ◦ f et f 2 = −id. Montrer
que f est orthogonal. F 6= E mais F ⊥ = {0E }.

e
④ Soit A ∈ Mn (C ) de valeurs propres λ1 , . . . , λn . Montrer que
er
① Soit E unespace vectoriel réel muni d’un produit scalaire
AA∗ = A∗ A ⇐⇒ Tr( AA∗ ) = |λ1 |2 + · · · + |λn |2 .

m
(Vect | Vect ), et F un sous-espace vectoriel de E.
① Montrer que l’application ( x, y) 7→ (Vect x | Vect y) est
Décomposition de Cholesky, QR. continue sur E2 .
m

② Soit x ∈ E fixé, montrer que l’application y 7→

r
① Décomposition de Cholesky : Soit A ∈ M n (R ) (Vect x | Vect y) est continue sur E.

o
symétrique définie positive. ③ En déduire que F = F⊥ .
or

F
✉: mamouni.myismail@gmail.com

224
i

r
s
✍ M AMOUNI M Y I SMAIL
MAMOUNI . NEW. FR
P ROBLÈMES C ORRIGÉS -MP

i
s

s
u
④ On suppose que F est dense dans E, montrer que F⊥ = R f (x) =
( f (~x ) | ~x )
où x 6= 0E . Soit λ1 ≤ λ2 ≤ · · · ≤ λn les

s
{0 E }. k~x k
Si de plus F 6= E, montrer que F n’admet pas de supplé- valeurs propres de f .

u
mentaires orthogonal. ① Montrer que : ∀ ~x ∈ E, λ1 k~x k2 ≤ ( f (~x ) | ~x ) ≤ λn k~x k2 .

r
② Soit E = C([0, 1], R ) muni du produit scalaire usuel, F le ② Montrer que si l’une de ces deux inégalités est une égal-

é
sous-espace vectoriel des fonctions polynomiales et g la fonc- ité pour un vecteur ~x 6= ~0, alors ~x est vecteur propre
tion exponentielle sur [0, 1]. de f .

Po r
r
① Montrer que g ∈
/ F. ③ Soit (~e1 , . . . , ~en ) une base orthonormée de E telle que
② Montrer qu’il existe une suite ( f n ) de fonctions polyno- pour tout i : ( f (~ei ) | ~ei ) = λi .

u
miales convergeant vers g pour la norme euclidienne. Montrer que : ∀ i, f (~ei ) = λi~ei .

uor
③ En déduire que F n’a pas de supplémentaire orthogonal. ④ En déduire que le quotient de Rayleigh de f atteint ses
extremums, préciser ces extremums et en quels vecteurs
③ Soit E = C([0, 1], R ) muni du produit scalaire usuel. Pour ils sont atteints.
Z 1
f ∈ E, on pose ϕ( f ) = 2 f (t) dt. Théorème de Courant-Fischer Soit E un espace vecto-

P

0 riel euclidien.
① Montrer que ϕ est continue. ① Soit v ∈ S(E), (i.e : auto-adjoint) tel que
② Montrer que H = Ker ϕ est fermé. (Vect v( x )| Vect x ) = 0 pour tout x. Montrer que v = 0.

r
③ Montrer que H ⊥ = {0}. ② Soient u1 , . . . , u p ∈ S(E). On suppose que rg(u1 ) + · · · +
rg(u p ) = n, et que  ∀ x ∈ E, (Vect u1 ( x )| Vect x ) + · · · +

e
Vect u p ( x )| Vect x = (Vect x | Vect x ) .
er
Rayleigh, Courant Fischer ① Montrer que u1 + · · · + u p = Id E .

m
. ② Montrer que E = Im(u1 ) ⊕ · · · ⊕ Im(u p ).
① Quotient de Rayleigh Soit f ∈ L(E) auto-adjoint, on ③ Montrer que pour tout i, ui est la projection orthogo-
m

se propose d’étudier les extremum du quotient de Rayleigh nale sur Im(ui ).

or
or

F
✉: mamouni.myismail@gmail.com

225
i

r
s
P ROBLÈMES C ORRIGÉS -MP ✍ M AMOUNI M Y I SMAIL
MAMOUNI . NEW. FR

i
s

s
u

s
Devoir Libre
24 Réduction simultanée et décomposition de Cholesky

u
Jorgen Pedersen Gram (1850-1916)

Mathématicien du jour
Blague du jour

é
Mathématicien danois, il est le fils d’un agriculteur. Il travailla
☛ Une logicienne (spécialiste en logie) vient d’avoir un la majeure partie dans le domaine des assurances, c’est ainsi

Po r
r
enfant. Une de ses amies lui téléphone, et lui demande : qu’il fonda son entreprise, et il fut le président du conseil
C’est une fille ou un garçon ? Oui, répond la logicienne. danois de l’assurance. Il décéda après avoir été heurté par un

u
☛ La vie est complexe, elle a une face réelle et une autre vélo.

uor
imaginaire.

Question préliminaire

P
Enoncé : Extrait CCP 2011, MP ①
On rappelle qu’une matrice S appartient à Sn+ , si S appartient
à Sn et si, pour toute matrice X ∈ Mn,1 (R ), on a t XSX ≥ 0.
Dans ce problème, on note pour n entier naturel non nul :
Démontrer qu’une matrice S de Sn est élément de Sn+ si et

r
① Sn l’ensemble des matrices symétriques de Mn (R ), seulement si toutes les valeurs propres de S sont positives.
② Sn+ l’ensemble des matrices symétriques positives de Partie I

e
M n (R ), √ 1
er
② Soit S ∈ Sn+ . Démontrer que det S ≤ trace S.
n
③ Sn++ l’ensemble des matrices symétriques définies positives

m
n
de Mn (R ). ③ Application : soit M ∈ Mn (R ).
1 n ① Démontrer que t MM ∈ Sn+ .
On admet que, si x1 , x2 , . . . , xn sont n réels positifs,
n i∑
xi ≥ 
m

=1 ② Si M = mi,j , en déduire l’inégalité (det M)2 ≤


!1

r
 n n n !n
n n 1
∏ xi . ∑ ∑ m2i,j .

o
n i =i j =1
i =1
or

F
✉: mamouni.myismail@gmail.com

226
i

r
s
✍ M AMOUNI M Y I SMAIL
MAMOUNI . NEW. FR
P ROBLÈMES C ORRIGÉS -MP

i
s

s
u
Partie II : héorème de réduction simultanée ② A ∈ Sn+ et B ∈ Sn+ , en démontrant d’abord que A + B ∈

s
Sn+ et en considérant les cas où les matrices sont dans Sn+
④ On se donne deux matrices A ∈ Sn++ et B ∈ Sn . On note
sans être dans Sn++ .

é
B la base canonique de R n et, dans cette base, A est la ma-

u
trice d’un produit scalaire ϕ. On note l’espace euclidien E = ⑥ Soient A et B deux matrices de Sn++ et t ∈ [0, 1]. On note P
une matrice inversible et D = diag (λ1 , λ2 , . . . , λn ) une ma-

r
(R n , ϕ). Soit B ′ une base orthonormée de E et R la matrice de
trice diagonale dans le théorème de réduction simultanée.

é
passage de la base B vers la base B ′ .
① Justifier que In = t RAR. ① Exprimer det (tA + (1 − t) B) en fonction de det P, t et

Po r
r
les λi .
② On note C = t RBR, justifier qu’il existe une matrice
② En utilisant la fonction ln, démontrer que,
orthogonale Q et une matrice diagonale D telles que

u
t pour tout i entier compris entre 1 et n,
QCQ = D.

uor
t + (1 − t) λi ≥ λ1i −t .
③ Déterminer, en fonction des matrices R et Q, une matrice
inversible P telle que : ③ Démontrer que det (tA + (1 − t) B) ≥ (det A)t (det B)1−t .
A = t PP et B = t PDP (théorème de réduction simultanée). ⑦ Si A est une matrice de Sn++ et B une matrice de Sn+ , on dé-
④ Danscettequestion, on prend l’exemple de la matrice montre de même par le théorème de réduction simultanée

P
1 1 (par la convexité de la fonction x 7→ ln (1 + e x )) le résultat
B= . suivant qui est admis :
1 1
Démontrer qu’une matrice inversible P telle que la ma- 1 1 1
(det ( A + B)) n ≥ (det A) n + (det B) n .
trice t PBP soit diagonale n’est pas nécessairement une

r
matrice orthogonale. ① Démontrer que Sn++ est dense dans Sn+ .
② Démontrer l’inégalité ci-dessus pour A et B deux matri-

e
On pourra, par exemple, utiliser la forme quadratique
canoniquement associée à la matrice B. ces de Sn+ .
er

m
⑤ Démontrer l’inégalité « det ( A + B) ≥ det A + det B » dans Partie III : Théorème de Choleski
les deux cas suivants :
⑧ Si A est une matrice de Sn++ , il est possible, par le procédé
① A ∈ Sn++ et B ∈ Sn+ , en utilisant le théorème de réduc- d’orthonormalisation de Schmidt, de trouver une matrice tri-
m

tion simultanée. On pourra remarquer ! ici que, avec tous angulaire supérieure inversible à coefficients diagonaux posi-

r
n n
tifs T, vérifiant A = t TT (décomposition de Choleski).
les λi ≥ 0, ∏ (1 + λi ) ≥ 1 + ∏ λi .

o
i =1 i =1 On ne demande pas de prouver ce résultat.
or

F
✉: mamouni.myismail@gmail.com

227
i

r
s
P ROBLÈMES C ORRIGÉS -MP ✍ M AMOUNI M Y I SMAIL
MAMOUNI . NEW. FR

i
s

s
u
① On se propose de démontrer que cette matrice T est le programme sur la copie) puis, pour chacun des cas suiv-

s
unique. ants, donner la matrice T :  
1
Si on pose A = t T1 T1 = t T2 T2 , démontrer que T1 T2−1 =

é
  1 0

u
49 14 −14  2
In et conclure.  1 
A1 =  14 20 −8  
, A2 =  0 0
On pourra admettre que, si T est l’ensemble des ma- 

r
−14 −8 21 1 2 3

é
trices triangulaires supérieures inversibles de Mn (R ), 0
(T , ·) est un groupe.   2 4 
 1 0 −2 1 2 3

Po r
r
② Exemple : si A = ai,j , où pour tout
A3 =  0 1 −1 et A4 = 2 20 26 .
couple (i, j) d’entiers compris entre 1 et n,
−2 −1 6 3 26 70

u
ai,j = Min (i, j), donner la décomposition de Choleski

uor
de la matrice A. ⑩ Inégalité d’Hadamard
On ne demande pas de vérifier que A est une matrice de
 n
Sn++ .
① Soit S = si,j ∈ Sn++ , démontrer que det S ≤ ∏ si,i .
i =1

P
⑨ Un peu d’informatique
② Application : démontrer
 que, pour toute matrice M ∈
Pour une matrice A de S3++ , écrire un algorithme en français Mn (R ), M = ai,j ,
permettant de trouver la matrice T de la décomposition de !! 1
Choleski. n n 2
|det M| ≤ ∏ ∑ a2k,i

r
.
Entrer cet algorithme dans la calculatrice (on ne demande pas i =1 k=1

e
i
F
er
n

m
i
n
m

Á la prochaine

or
or

F
✉: mamouni.myismail@gmail.com

228
Concours Communs Polytechniques - Session 2011
Corrigé de l’épreuve d’algèbre- Filière MP
Commutant d’une matrice, inégalités sur les déterminants de matrices symétriques
Corrigé par M.TARQI http://alkendy.x10.mx

Exercice
Commutant d’une matrice

1. Il suffit de montrer que C(A) est un sous-espace vectoriel de M3 (R), en effet, C(A) est non vide
puisqu’il contient A et si M et N sont dans C(A) et λ ∈ R, alors

A(M + λN ) = AM + λAN = M A + λN A = (M + λN )A,

et donc M + λN est dans C(A).


2. Le polynôme caractéristique de A est χA (x) = −(x − 3)(x − 2)2 , donc on peut trigonaliser la
matrice A. Cherchons d’abord les sous-espaces propres associés à 3 et 2. Après calculs on trouve
E3 = Vect(1, 1, 1) = Vect(v1 ) et E2 = Vect(4, 3, 4) = Vect(v2 ). On choisit alors un vecteur v3
quelconque vérifiant les conditions suivantes : les vecteurs (v1 , v2 , v3 ) sont linéairement indé-
pendants et Av3 − 2v3 = v2 .
Posons v3 = (x, y, z), alors :
µ ¶
−x + 4y − 2z = 4
Av3 − 2v3 = v2 ⇐⇒
4y − 3z = 3
Choisissons, par exemple, v3 = (−2, 0, −1).  
1 4 −2
La famille {v1 , v2 , v3 } est bien libre et si P = 1 3 0  ( la matrice de passage de la base
1 4 −1
   
3 4 −6 3 0 0
canonique à la base (v1 , v2 , v3 ) ) alors P −1 = −1 −1 2 , et P −1 AP = 0 2 1  = T.
−1 0 1 0 0 2
 
a b c
3. M =  a0 b0 c0  ∈ C(T ) si et seulement si AM = M A ou encore
a00 b00 c00
   
3a 3b 3c 3a 2b b + 2c
 2a0 + q 00 2b0 + b00 3c0 + c00  =  3a0 2b0 b0 + 2c0 
2a00 2b00 2c00 3a00 2b00 b00 + 2c00
donc
       
a 0 0 1 0 0 a 0 0 0 0 0
M =  0 c00 c0  = a  0 0 0  + c00  0 1 0  + c0  0 0 1 
0 0 c00 0 0 0 0 0 1 0 0 0
     
1 0 0 0 0 0 0 0 0
Donc M ∈ Vect(J, K, L) où J =  0 0 0  , K =  0 1 0  et L =  0 0 1 
0 0 0 0 0 1 0 0 0
Réciproquement, les matrices J, K, L sont linéairement indépendantes et sont dans C(T ), donc
C(T ) = Vect(J, K, L) et par suite dim C(A) = 3.

CCP11Maths2MP.tex - page 1
4. (a) Il est clair que l’application est un endomorphisme de M3 (R) et que P −1 M P = 0 si et
seulement si M = 0, donc l’application M 7−→ P −1 M P est un automorphisme de M3 (R).
L’image de C(A) n’est autre que C(T ) et par suite dim C(A) = dim C(T ) = 3.
(b) Non, si oui, la matrice A serait diagonalisable.
(c) Il est clair que Vect(I, A, A2 ) ⊂ C(A) et toute relation de la forme aI +bA+cA2 = 0 entraîne
a = b = c = 0 ( d’après 5.(b) ), donc la famille {I, A, A2 } est libre et comme dim C(A) = 3,
alors C(A) = Vect(I, A, A2 )
(d) Il est clair que tout polynôme en A est dans C(A). Inversement soit M = P (A) un poly-
nôme en A et soit R le reste de la division euclidienne de P par χA , d’après le théorème de
Cayly-Hamilton χA (A) = 0 et donc M = R(A), avec deg R ≤ 2, ainsi

C(A) = {P (A)/P ∈ R[X]}.

Le résultat n’est vrai en général, il suffit considérer une matrice nilpotente d’indice 2.
Problème
I NÉGALITÉS SUR LES DÉTERMINANTS DE MATRICES SYMÉTRIQUES

1. Question préliminaire S étant symétrique réelle, donc elle est orthogonalement diagonalisabe.
Soit X ∈ Mn,1 (R)\{0} un vecteur propre associé à une valeur propre λ de A, puisque X 6= 0,
t XX > 0, d’où :
t
XAX = λt XX
ou encore
t XAX
λ= t XX
≥ 0.

Inversement si les valeurs propres de A sont positives, alors, dans une base de diagonalisation
de A :
Xn
t
XAX = λi x2i ≥ 0,
i=1

où les xi désignent les composantes de X dans cette base.


PARTI I
2. Soit S ∈ Sn+ et soit Sp(A) = {λ1 , λ2 , ..., λn } l’ensemble des valeurs propres de A, d’après 1.
Sp(A) ⊂ R+ et donc
n
à n !1
1X Y n

λi ≥ λi ,
n
i=1 i=1

inégalité qui s’écrit encore sous la forme



n 1
det S ≤ traceS.
n

3. (a) Il est clair que t (t M M ) =t M M et pour tout X ∈ Mn,1 (R), on a :


t
X t M M X =t (M X)(M X) ≥ 0,

donc t M M ∈ Sn+ .
P
n P
n
(b) Posons t M M = (cij ). On a cii = mki mki = m2ki , donc
k=1 k=1

n
X n X
X n
trace(t M M ) = cii = m2ki .
i=1 i=1 k=1

CCP11Maths2MP.tex - page 2
n n
tM M
p
n t
1 XX 2
L’inégalité de la question 2. appliquée à entraîne det( M M ) ≤ mki et
n
i=1 k=1
comme det(t M ) = det M , alors
µ ¶n ÃX
n X
n
!n
1
(det M )2 ≤ m2ki .
n
i=1 k=1

PARTIE II : T HÉORÈME DE RÉDUCTION SIMULTANÉE


4. (a) La matrice de ϕ dans la base B0 est In puisque B0 est une base orthonormée de E et dans la
base canonique est B, donc d’après la formule de changements de bases, on a In =t RAR.
(b) C étant symétrique réelle, donc elle est orthogonalement diagonalisable, c’est-à-dire il
existe une matrice diagonale D et une matrice orthogonale Q telles que C = QDQ−1 =
QDt Q ou encore D =t QCQ.
(c) La relation C =t RBR est équivalent encore à

B = (t R)−1 CR−1 =t (R−1 )CR−1 =t (RQ)−1 D(RQ)−1

et la relation In =t RAR est équivalent à

A =t (R−1 )R−1 =t (RQ)−1 (RQ)−1 ,

il suffit donc de prendre P = (RQ)−1 .


µ ¶ µ ¶
1 −1 t 4 0
(d) La matrice P = vérifie P BP = , cependant la matrice P n’est pas
1 1 0 0
orthogonale.
5. (a) Posons D = diag(λ1 , λ2 , ..., λn ) ⊂ R+ . On a A + B =t P (In + D)P et donc

det(A + B) = (det P )2 det(In + D)


Yn
= (det P )2 (1 + λi )
i=1
à n
!
Y
2
≥ (det P ) 1+ λi
i=1
n
Y
2 t
= (det P ) + det( P ) λi det P = det A + det B
i=1

(b) Pour tout X ∈ Mn,1 (R), t X(A + B)X =t XAX +t XBX ≥ 0 et donc A + B ∈ Sn+ , ainsi
det(A + B) ≥ 0.
Si A ∈ Sn++ ou B ∈ Sn++ , alors l’inégalité est toujours vérifie. Maintenant soit A et B dans
Sn+ \Sn++ , alors det A = det B = 0 et dans ce cas aussi on a det(A + B) ≥ det A + det B.
6. (a) On a tA + (1 − t)B =t P (tIn + (1 − t)D)P et donc
n
Y
det(tA + (1 − t)B) = (det P )2 (t + (1 − t)λi ) .
i=1

(b) La fonction ln étant concave, donc pour tout i, ln(t + (1 − t)λi ) ≥ t ln 1 + (1 − t) ln λi et donc

ln(λ1−t
i ) ≤ ln(t + (1 − t)λi )

et par conséquent
t + (1 − t)λi ≥ λ1−t
i .

CCP11Maths2MP.tex - page 3
(c) D’après l’inégalité précédente, on a :

n
Y
det(tA + (1 − t)B) = (det P )2 (t + (1 − t)λi )
i=1
n
Y
≥ (det P )2 λ1−t
i
i=1
n
Y
2t
≥ (det P ) (det P ) 2(1−t)
λ1−t
i
i=1
= (det A)t (det B)1−t

1
7. (a) Pour S ∈ Sn+ , on définie la suite (Sp )p∈N∗ par Sp = S + In qui tend vers S quand p tend
p
1 t
vers l’infini, avec Sp ∈ Sn++ car t XSp X =t XSX + XX > 0 pour tout X ∈ Mn,1 (R)\{0}.
p
++
(b) Soient (Ap )p∈N et (Bp )p∈N deux suites de Sn telles que lim Ap = A et lim Bp = B, alors
p→∞ p→∞
pour tout p ∈ N, on a :
1 1 1
(det(Ap + Bp )) n ≥ (det Ap ) n + (det Bp ) n

et comme l’application det est continue, alors on obtient, par passage à la limite dans
l’inégalité précédente :
1 1 1
(det(A + B)) n ≥ (det A) n + (det B) n

PARTIE III : T HÉORÈME DE C HOLESKI


8. (a) L’égalité t T1 T1 =t T2 T2 entraîne T1 T2−1 =t (T2 T −1 ) et comme T est un groupe, alors T1 T2−1
est triangulaire supérieure et inférieure à la fois, donc nécessairement T1 T2−1 = In , donc
T2 = T1 est par conséquent T est unique.
   
1 1 ... 1 1 2 ... n
 0 1 1   2 2 n 
  tT T =  
(b) Soit T =  . . . ..  , on a bien  . .. . =A
 .. . .  .. . .. 
0 0 ... 1 n n ... n
Remarque : Notons q la forme quadratique canoniquement associée à A. On a :
 
1 2 ... n
 2 2 n 
 
A= . . ..  ;
 .. . . . 
n n ... n
½
1 si i, j > r
A est somme des matrices Ar définie par Ar (i, j) = , pour tout 1 ≤ i, j ≤ n et
0 sinon
1 ≤ r ≤ n. On en déduit la relation :
 2
n
X Xn
∀x ∈ Rn , q(x) =  xp  .
i=1 p=i

Les formes linéaires du membre de droite étant linéairement indépendantes, on peut affirmer
que q est définie positive. Autrement dit, la matrice A ∈ Sn++

CCP11Maths2MP.tex - page 4
9. Un peu d’informatique La matrice T est obtenue par la méthode qui consiste à résoudre le système
n2 × n2 :
n min(i,j)
X X
aij = tik tjk = tik tjk , 1 ≤ i, j ≤ n.
k=1 k=1

en calculant d’abord la première colonne de T ( correspondant à i = 1 dans le système ) :


 √

 t11 = a11 ,

 t21 = a12 ,
t11
 ..

 .

tn1 = at11
1n

puis la seconde colonne ( en fixant i = 2 ), et ainsi de suite jusqu’à déterminer tnn .


10. Inégalité d’Hadamard
(a) Puisque S ∈ Sn++ , alors t Ei SEi = aii > ( où Ei désigne la matrice du ième vecteur de la
base canonique ), donc on peut définir la matrice B = DSD avec

1 1 1
D = diag( √ , √ , ..., √ ),
a11 a22 ann

il est clair que B ∈ Sn++ et donc


p √
n 1
n
det S(det D)2 = det B ≤ trace(B),
n
Yn
1
d’où det S(det D)2 ≤ 1 car bii = 1 et donc det S ≤ 2
= aii .
(det B)
i=1
n
Y n
X n
X
t
(b) On a det(t M M ) ≤ ci,i où ci,i = ( A)i,k ak,i = a2k,i , mais :
i=1 k=1 k=1

det(t M M ) = det(t M ) det(M ) = det(M )2 ,

n
à n !1
Y X 2

on a donc | det(M )| 6 a2k,i (inégalité d’Hadamard).


i=1 k=1

• • • • • • • • • • ••

CCP11Maths2MP.tex - page 5
✍ M AMOUNI M Y I SMAIL
MAMOUNI . NEW. FR

Devoir Libre
16 Phénomène de Gibbs
ri

ir
s
Blague du jour
s
Dans une voiture, quatre ingénieurs. Tout coup la voiture s’arrête..

s
- Le Mécanicien : Je le savais, c’est un problème de transmission.
u
- Le chimiste : c’est la faute des acides de la batterie !

s
- L’électronicien : c’est le circuit électronique qui ne marche plus !.
- L’Informaticien en dernier : ... et si on essayait de fermer toutes les fenêtres
é

u
ouvertes, de quitter, et redémarrer à nouveau ?
r

é
David Hilbert (1862-1943)
Po r

Mathématicien du jour
r
Mathématicien allemand. Il est considéré comme un des plus grands mathématiciens du
XX ème siècle, au même titre que Henri Poincaré. Il a développé la théorie des invariants,
u
l’axiomatisation de la géométrie, les fondements de l’analyse fonctionnelle, la mécanique
uor
quantique et la relativité générale. Il a adopté et défendu avec vigueur les idées de Georg
Cantor. Il est aussi connu comme l’un des fondateurs de la théorie de la démonstration, de
la logique mathématique et a clairement distingué les mathématiques des métamathéma-
tiques.
P
Énoncé : CCP 2006, MP
e r
EXERCICE 1
er

On considère la fonction f de R2 dans R dénie par :


m
y4
f (x, y) = si (x, y) 6= (0, 0) et f (0, 0) = 0.
x2 + y 2
rm

1. Démontrer que la fonction f admet des dérivées partielles premières en (0, 0) que l'on déter-
r
minera.
o
2. Démontrer que la fonction f est diérentiable en (0, 0).
Fo

EXERCICE 2

1. Rappeler la dénition (par les suites) d'une partie compacte d'un espace vectoriel normé.
2. Soit E et F deux espaces vectoriels normés, et f une application continue de E dans F .
Si A est une partie compacte de E , démontrer que f (A) est une partie compacte de F .
L'image réciproque par f d'une partie compacte de F est-elle nécessairement une partie
compacte de E ?
✉: mamouni.myismail@gmail.com

1
✍ M AMOUNI M Y I SMAIL
MAMOUNI . NEW. FR

.
PROBLÈME : PHÉNOMÈNE DE GIBBS

Partie préliminaire
ri
sin t
1. (a) Justier que la fonction t 7→ est intégrable sur l'intervalle ]0; π].

ir
s
t
Z π
sin t
On pose I = dt.
s
0 t

s
(b) Rappeler le développement en série entière en 0 de la fonction sinus et déterminer, avec
u

s
soin, une suite (uk )k≥0 vériant I =
X
(−1)k uk .
k=0
é

u
n
πn
   
π
2. (a) Démontrer que la suite converge et que la suite est décroissante.
r
n! n≥0 n.n ! n≥1

é
+∞
(b) Si Rn = (−1)k uk , majorer |Rn |, en utilisant la question (a).
X
Po r
r
k=n+1
2
En déduire, en précisant la valeur de n utilisée, une valeur approchée du réel I à 10−2
u
π
près.
uor
Première partie : Phénomène de Gibbs
P
On considère la fonction f dénie sur R impaire et de période 2π vériant :
f (t) = 1 pour t ∈ ]0; π[ et f (0) = f (π) = 0.
r
3. On pose pour tout entier naturel n non nul et t réel,
e
n−1
4 X sin[(2k + 1)t]
Sn (t) = .
er

π k=0 2k + 1
m Démontrer, à l'aide d'une série de Fourier, que la suite de fonctions (Sn )n≥1 converge sim-
plement vers la fonction f sur R. La convergence est-elle uniforme sur R ?
rm

h π i
4. Sur un même graphique, uniquement à l'aide d'une calculatrice, tracer sur l'intervalle − ; π
r
2
la courbe de la fonction f et l'allure
h π de i la courbe de la fonction S10 . Puis sur un autre
o
graphique, tracer sur l'intervalle − ; π la courbe de la fonction f et l'allure de la courbe
2
de la fonction S20 .
Que constate-t-on sur les courbes des fonctions Sn lorsque t se rapproche de 0 par valeurs
Fo

supérieures ou par valeurs inférieures ?


Cette particularité est appelée phénomène de Gibbs.
5. On pose pour n entier naturel non nul et t réel,
n−1
X
Tn (t) = sin[(2k + 1)t].
k=0

✉: mamouni.myismail@gmail.com

2
✍ M AMOUNI M Y I SMAILMAMOUNI . NEW. FR

(a) Démontrer que ∀n ∈ N∗ , ∀t ∈ R r πZ,


. sin2 (nt)
Tn (t) = .
sin t
ri
Dans la suite
 de cette question 5., on considère deux nombres réels a et b tels que a < b
et [a, b] ⊂ 0; π2 .

ir
s
(b) Justier qu'il existe une constante M telle que pour tout entier naturel n non nul et
tout t ∈ [a, b], Tn (t) ≤ M .
s

s
(c) Démontrer que l'on peut trouver une suite de réels (wn ) convergeant vers 0 et telle que
u
pour tout entier naturel n non nul et tout t ∈ [a, b], |f (t) − Sn (t)| ≤ wn .

s
En commençant par observer que sin[(2k + 1)t] = Tk+1 (t) − Tk (t), on pourra chercher à
majorer, pour tout couple (n, p) d'entiers naturels non nuls et tout t ∈ [a, b], |Sn+p (t) − Sn (t)|.
é

u
Que peut-on en déduire concernant la série de Fourier de la fonction f ?
r
6. (a) Calculer Sn0 (t)pour tout t ∈ 0; π2 et déterminer la plus petite valeur αn qui annule
 

é
Sn0 (t) sur 0; π2 .
(b) Démontrer que, pour x ∈ 0; π2 et n entier naturel non nul,
 
Po r
r
Z x Z π
2 sin(2nt) 1 sin u
Sn (x) = dt puis que Sn (αn ) = u du.
u π sin t nπ 0 sin 2n
0
(c) Démontrer que la suite (Sn (αn ))n≥1 converge et préciser sa limite.
uor

On pourra utiliser sans démonstration : pour θ ∈ 0; π2 , sin θ ≥ .
 
π
 
P
7. Démontrer que la suite  sup |Sn (x) − f (x)| ne converge pas vers 0.
x∈]0; π2 [
n
r
Deuxième partie : Démonstration du théorème de convergence normale
e
Pour une fonction f continue par morceaux de R dans C et de période 2π , on note pour tout
er

n∈Z:
m cn (f ) =
1
Z 2π
f (t)e−int dt.
2π 0
8. Rappeler le théorème de Parseval (avec les coecients cn (f )) pour une fonction continue par
rm

morceaux de R dans C et de période 2π .


r
Dans le cas où la fonction f est de plus continue sur R, justier que si pour tout n∈ Z,
cn (f ) = 0 alors f est la fonction nulle.
o
Ce résultat reste-t-il valable si la fonction f est seulement continue par morceaux de R dans
C et de période 2π ?
Fo

9. Soit f une fonction continue de R dans C et de période 2π dont la série de Fourier converge
uniformément sur R vers une fonction g :
+∞
X
∀t ∈ R, g(t) = c0 (f ) + (c−p (f )e−ipt + cp (f )eipt ).
p=1

(a) Justier que l'application g est continue sur R puis pour tout entier n ∈ Z, exprimer,
avec soin, cn (g) en fonction de cn (f ).
✉: mamouni.myismail@gmail.com

3
✍ M AMOUNI M Y I SMAIL
MAMOUNI . NEW. FR

. (b) Démontrer que f = g .


10. Dans cette question, f est une fonction continue de R dans C, de période 2π et de classe C1
ri
par morceaux.

ir
s
On pose pour n entier naturel non nul et t réel,
s
un (f )(t) = cn (f )eint + c−n (f )e−int .

s
(a) Déterminer une relation entre cn (f 0 ) et cn (f ).
u
(b) Démontrer que pour tout t réel,

s
é
1 1

u
2 2
|un (f )(t)| ≤
2
+ (|cn (f 0 )| + |c−n (f 0 )| ).
n 2
r
(c) Démontrer, avec soin, que la série de fonctions un (f ) converge normalement sur R
P

é
et préciser vers quelle fonction.
(d) Énoncer le théorème que l'on vient de démontrer.
Po r
r
Le phénomène de Gibbs peut-il se produire pour cette fonction f ?
u
uor
r P
e
er

m
rm

or
Fo

✉: mamouni.myismail@gmail.com

4
i

r
s
✍ M AMOUNI M Y I SMAIL
MAMOUNI . NEW. FR
P ROBLÈMES C ORRIGÉS -MP

i
s

s
u
Corrigé : Pr. Boujaida, CPGE Rabat, Maroc

s
é
Problème

u
② ① Utiliser de D’Alembert, ou alors mentionner le fait que
Partie préliminaire xn
la série entière ∑ a un rayon de convergence infini

r
n!
sin t

é
① ① la fonction t 7−→ est continue sur ]0, π ] prolonge- (c’est du cours).
t πn an +1
able par continuité en 0. Elle est donc intégrable sur Si on pose an = alors an > 0 et =

Po r
r
]0, π ]. n.n! an
π π
+∞ ≤ ≤ 1 si n ≥ 1.
(−1)n 2n+1

u
( n + 1) ! ( n + 1) 2.2
② Pour tout t ∈ R, sin t = ∑ (2n + 1)! t . et donc si donc (an )n≥1 est décroissante. Elle converge vers 0

uor
n =0
t 6= 0, πn
+∞ puisque an ≤ .
sin t (−1)n 2n n!
= ∑ t . .
t n =0 (2n + 1)! ② On a pour tout n ∈ N, un = a2n+1 donc (un )n est décrois-

P
Soit la fonction f somme de la série entière sante et converge vers 0. D’après le critère spécial de
(−1)n 2n convergence des séries alternées, la série ∑ (−1)n un est
∑ (2n + 1)! t . f est naturellement DSE sur R, chacune +∞
de ses primitives l’est aussi, si onZ pose pour tout x ∈ R convergente et si on pose Rn = ∑ (−1)k uk alors
x k= n +1

r
F( x) = f (t)dt π 2n+3
0 | Rn | ≤ un +1 ≤
alors (2n + 3)! (2n + 3)

e
+∞ +∞
(−1)n (−1)n 2
x2n+1 = ∑ x2n+1
er
F ( x ) = F (0 ) + ∑ Nous allons maintenant approcher le réel I par les ter-
( 2n + 1 ) ! ( 2n + 1 ) ( 2n + 1 ) ! ( 2n + 1 ) π

m
n =0 n =0
sin t 2
Comme f (t) = pour tout t ∈]0, π ] alors mes Sn où Sn est la somme partielle d’ordre n de la
t π
Z π
sin t +∞
(−1)n série ∑(−1)n un .
π 2n+1
m

F (π ) = dt = ∑ 2 2 2 π 2n+2 10n+1
0 t ( 2n + 1 ) ! ( 2n + 1 ) | Sn − I | = | R n | ≤ 2 ≤2

r
n =0
π π π (2n + 3)! (2n + 3) (2n + 3)! (2n +
+∞
π 2n+1 2 2
Soit I = ∑ (−1)n un avec un = Pour que Sn approche I à 10−2 près, il suffit que

o
n =0 (2n + 1)!(2n + 1) π π
or

F
✉: mamouni.myismail@gmail.com

237
i

r
s
P ROBLÈMES C ORRIGÉS -MP ✍ M AMOUNI M Y I SMAIL
MAMOUNI . NEW. FR

i
s

s
u
10n+1
2 ≤ 10−2

s
(2n + 3)! (2n + 3)

é
soit (2n + 3)! (2n + 3) ≥ 2.10n+3.

u
1

r
n = 3 suffit et dans ce cas Sn ≃ 1, 17 π
π

é

2
Phénomène de Gibbs
π

Po r
r
③ Vu la parité de la fonction f , pour tout n ∈ N, an ( f ) = 0, et

u
4
par un calcul simple b2n = 0 et b2n+1 = . −1
S10
π (2n + 1)

uor
S20
f est de classe C 1 par morceaux sur R. Ses points de disconti-
nuité sont ceux de la forme kπ où k ∈ Z.
D’après le théorème de Dirichlet, pour tout point x ∈ R \πZ,

P
point où f est continue  
+∞   4 +∞ sin (2k + 1)t
f (t) = ∑ a2k+1 sin (2k + 1)t =
k=0
π k∑
=0
2k + 1

r
égalité
 encore valable lorsque t ∈ ∗πZ puisque dans ce cas
sin (2k + 1)t = 0 pour tout k ∈ N et f (t) = 0.

e
La suite de fonction (Sn )n converge donc simplement vers f
er
sur R.

m
La convergence n’est pas uniforme puisque les fonctions Sn
sont toutes continues sur R est que f ne l’est pas.
m

r
④ Les graphes des fonctions S10 , S20 et séparèment celui de S200
pour une meilleure illustration du phénomène, sur l’inter-

o
valle [−π/2, π ].
or

F
✉: mamouni.myismail@gmail.com

238
i

r
s
✍ M AMOUNI M Y I SMAIL
MAMOUNI . NEW. FR
P ROBLÈMES C ORRIGÉS -MP

i
s

s
u
⑤ ① Soit n ∈ N ∗ , pour tout t ∈ R \πZ !

s
1 n −1   n −1 n −1 
Tn (t) = ∑ sin (2k + 1)t = ℑ ∑ ei (2k+1)t = ℑ eit ∑ e

u
k=0 k=1 k=0
 2int 
π it 1 − e eit − ei (2n

r
− = ℑ e =ℑ
2 1 − e2it 1 − e2

é
! 
π
−2i sin (nt) ei (n+1)t s
= ℑ =ℑ

Po r
r
−2i sin(t)e it

sin2 (nt)

u
=
sin t

uor
−1

② Soit un segment [ a, b] ⊂ ]0, π/2[. Pour tout n ∈ N ∗ et


pour tout t ∈ [ a, b]
Graphe de S200 avec zoom sur une partie 1

P
Tn (t) ≤
sin a
1
On pose alors M = .
sin a

r
③ On exécute la transformation dite d’Abel, pour tout
n, p ∈ N ∗ et pour tout t ∈ [ a, b]

e
 
er
n+ p n+ p
4 sin (2k + 1)t 4 Tk+1 (t) −
∑ ∑

m
Sn + p ( t ) − Sn ( t ) = =
π k= n +1 2k + 1 π k= n +1 2k +
!
n + p+1 n+ p
4 Tk (t) Tk (t)
= ∑ − ∑
m

Au voisinage de 0, que ce soit à droite ou à gauche, la conver- π k=n+2 2k − 1 k=n+1 2k + 1

r
gence de Sn vers f n’est pas uniforme. On perçoit de fortes n+ p
perturbations de la fonction Sn au voisinage de 0, au fur est à 4 Tn+ p+1 Tn+1
= − + ∑

o
mesure que n grandit. C’est le phénomène de Gibbs. π 2( n + p + 1) − 1 2( n + 1) − 1 k = n +
or

F
✉: mamouni.myismail@gmail.com

239
i

r
s
P ROBLÈMES C ORRIGÉS -MP ✍ M AMOUNI M Y I SMAIL
MAMOUNI . NEW. FR

i
s

s
u
De quoi on déduit le segment [0, π/2] avec Sn (0) = 0. D’après le théorème
!

s

4M 1 1
n+ p
1 1 fondamental du calcul intégral,
Z
pour toutZ x ∈ [0, π/2]
|Sn+ p (t) − Sn (t)| ≤ + + ∑ − 2 x 2 x sin 2nt

é
π 2n + 2p + 1 2n + 1 k=n+1 2k − 1 2k + 1 Sn ( x ) = sn (t)dt =

u
dt
   π 0 π 0 sin t
4M 1 1 1 1 l’intégrale du dernier terme de cette égalité se faisant sur
≤ + + −

r
π 2n + 1 2n + 1 2(n + 1) − 1 2n + 2p +l’intervalle
1 ]0, x ]. De là

é
Z Z π
4M 3 12M 2 π/2n sin 2nt u=2nt 1 sin u
≤ . = S (
n nα ) = dt = du
π 2n + 1 π (2n + 1) π 0 sin t nπ 0 sin(u/2n)

Po r
r
12M
Soit alors ε > 0 et soit un entier N tel que ≤ ε. sin u sin u
π (2N + 1) ③ En observant que ∼ au

u
sin(u/2n) u/2n
Pour tous n ≥ N, p ∈ N et pour tout t ∈ [ a, b] on a alors
voisinage de Z 0, on peut faire l’hypothèse que

uor
|Sn+ p (t) − Sn (t)| ≤ ε Z π
1 π sin u 2 sin u
Ceci démontre que la suite de fonctions (Sn ) converge du −→ du (∗)
nπ 0 sin(u/2n) π 0 u
uniformément sur le segment [ a, b]. Convergence qui se chose qu’on va s’atteler à démontrer.
fait vers f puisque on a déjà vu qu’il y’a convergence
On va utiliser les inégalités connues :

P
simple vers f sur R.
⑥ ① Soit t ∈]0, π/2]. 2
  ∀t ∈ [0, π/2], sin t ≥ t (concavité de sin sur [0.π/2])
4 n −1  4 n−1 i (2k+1)t 4 sin nt int 2 sin 2nt π
Sn′ (t) = cos 2k 1 e e
π k∑ π k∑
( + ) t ] = = ℜ = 1
π sin t π sin∀tt ∈ R, | sin t − t| ≤ |t|3 (inégalité de Taylor–

r
=0 =1
6
k Lagrange).
Sn′ (t) = 0 ⇐⇒ sin 2nt = 0 ⇐⇒ ∃k ∈ Z; t = π

e
2n
La plus petite valeur qui annule Sn′ sur ]0, π/2] est donc On a alors pour tout t ∈]0, π/2]
er
π
le réel αn = . 1 t3

m
2n 1 1 |t − sin t| π
sin 2nt | − |= ≤ 6 ≤ t
② On a vu que pour tout t ∈]0, π/2], Sn′ (t) = . La sin t t t sin t t. π2t 12
sin t u
sin 2nt Si maintenant n ∈ N ∗ et u ∈]0, π ] alors
m

∈ [0, π/2] et
fonction sn : t 7−→ se prolonge par continuité en 2n

r
sin t donc
0 en posant sn (0) = 2n. sin u sin u π u π2
| − | ≤ . | sin u | ≤

o
1
Sn , qui est de classe C , est alors une primitive de sn sur sin(u/2n) u/2n 12 2n 24n
or

F
✉: mamouni.myismail@gmail.com

240
i

r
s
✍ M AMOUNI M Y I SMAILMAMOUNI . NEW. FR
P ROBLÈMES C ORRIGÉS -MP

i
s

s
u
On intègre sur ]0, π ] ⑧ La formule de Parseval pour une fonction f : R −→ C con-

s
Z π Z Z π
1 sin u 2 π sin u 1 sin u π 2 morceaux 2π-périodique
sin u tinue par
| du − du| ≤ | − |du ≤ +∞ Z 2π
nπ 0 sin(u/2n) nπ 0 sin(u/2n) u/2n 24n2 1

é
π 0 u
|c0 ( f )|2 + ∑ |cn ( f )|2 + |c−n ( f )|2 = | f (t)|2 dt

u
Ce qui démontre l’assertion (∗), assertion qui signifie 2π 0
n =1
que Si f est continue et tous ses coefficients de Fourier sont nuls

r
2
. alors

é
Sn (αn ) −→ I
π
Z 2π
⑦ αn ∈]0.π/2[ donc f (αn ) = 1 et donc
| f (t)|2 dt = 0 (∗∗)

Po r
r
0
Sup |Sn ( x ) − f ( x )| ≥ |Sn (αn ) − f (αn )| ≥ |Sn (αn ) − 1| La fonction | f |2 étant continue positive sur le segment [0, 2π ],

u
x ∈]0,π/2[ cela implique qu’elle est nulle sur ce segment et par périodic-

uor
2 2 ité, sur R tout entier. f est donc la fonction nulle.
Puisque (Sn (αn )) converge vers I est que I ≃ 1, 17
π π Si maintenant f était seulement continue par morceaux, l’é-
d’après la question (2.b.) alors la quantité
Sup |Sn ( x ) − f ( x )| galité (∗∗) est toujours valable, elle implique que f s’annule
en tout point où elle est continue, et donc pas forcément

P
x ∈]0,π/2[
ne peut converger vers 0, car sinon (Sn (αn )) convergerait vers partout nulle, sauf si ... elle était continue.
1. (Précisons, si f est 2π-périodique, continue par morceaux et
vérifie l’égalité (∗∗) alors f = 0 ⇐⇒ f est continue)
N.B : La question revient à démontrer que la suite de fonc-

r
tions (Sn )n ne converge pas uniformément vers f sur l’inter- ⑨ f est continue 2π-périodique et sa série de Fourier converge
valle ]0, π/2[. Tout le brique à braque mis en œuvre pour y uniformément sur R. On pose pour tout t ∈ R.

e
arriver, en dehors de son aspect sportif, est inutile. En effet il +∞
∑ c p ( f )eipt + c− p ( f )e−ipt
er
suffisait de mettre en défaut le théorème d’interversion g (t) = c0 ( f ) +

m
p=1
lim lim Sn ( x ) 6= lim lim Sn ( x )
n →+ ∞ x →0+ x →0+ n →+ ∞
La première double limite valant 0, la seconde 1. ① les fonctions t 7−→ c p ( f )eipt + c− p ( f )e−ipt sont contin-
La question aurait eu plus d’intérêt si elle s’était orien- ues sur R est la convergence est uniforme sur R, donc g
m

tée vers une minoration effective de Sup |Sn ( x ) − f ( x )|. est continue sur R.

r
x ∈]0,π/2[ Maintenant, grace à la convergence uniforme sur le seg-

o
Démonstration du théorème de la convergence normale ment [0, 2π ] on peut intégrer terme à terme l’expression
or

F
✉: mamouni.myismail@gmail.com

241
i

r
s
P ROBLÈMES C ORRIGÉS -MP ✍ M AMOUNI M Y I SMAIL
MAMOUNI . NEW. FR

i
s

s
u
g(t)e−int , ce qui donne pour tout n ∈ Z termes réels positifs ∑ |cn ( f ′ )|2 et ∑ |c−n ( f ′ )|2 sont con-

s
Z 2π  Z Z 
1 +∞ −i (vergentes. La majoration obtenue dans la question précé-
c0 ( g ) 2π 2π
−int i ( p− n )t p+ n )t
cn ( f ) = e dt + ∑ cp( f ) e dt + c− p ( f ) e dentedtachève de démontrer que la série de fonctions
2π 0 2π p=1

é
0 0

u
1
Z 2π ∑ un ( f ) converge normalement sur R. Elle converge
Comme pour tout (k, h) ∈ Z , 2
e i (k− h)t
dt = δkh vers la fonction g définie dans la question (9.), toujours

r
2π 0 d’après cette question, on a forcément g = f .

é
une seule des intégrales figurant dans cette expression
Ainsi ∑ un ( f ) converge normalement vers f sur R.
est non nulle, elle est obtenue pour p = n si n ≥ 0, pour
④ Le phénomène de Gibbs ne subsiste plus pour une

Po r
r
p = −n si n < 0. Ce qui nous mène à l’égalité
cn ( g) = cn ( f ) fonction 2π périodique, continue et de classe C 1 par

u
morceaux sur R.
② f − g est continue 2π-périodique sur R et par linéarité

uor
des coefficients de Fourier
∀n ∈ Z, cn ( f − g) = cn ( f ) − cn ( g) = 0
D’après la question (8.) on a donc g = f .
⑩ ① f étant continue et C 1 par morceaux, une intégration par

P
parties donne  
Z 2π Z 2π
′ 1 ′ −int 1  
−int 2π −int
cn ( f ) = f (t)e dt = f (t)e 0
+ in f (t)e dt = incn ( f )
2π 0 2π 0

r
② Pour deux nombres complexes a et b, | ab| ≤
1 2 
| a| + |b| (découle tout bêtement de (| a| − |b|)2 ≥
2

e
2
0).
er
∈ N ∗ et t ∈ R alors un ( f )(t) =

m
Si n
1  
′ −int int
c n ( f )e − c − n ( f )e et donc
in
|cn ( f )| |c−n ( f ′ )|
′ 1 1 
m

|un ( f )(t)| ≤ + ≤ 2+ |cn ( f ′ )|2 + |c−n ( f ′ )|2


n n n 2

r
③ La fonction f ′ étant continue par morceaux 2π-

o
périodique, d’après la formule de Parseval les séries à
or

F
✉: mamouni.myismail@gmail.com

242
i

r
s
✍ M AMOUNI M Y I SMAIL
MAMOUNI . NEW. FR
P ROBLÈMES C ORRIGÉS -MP

i
s

s
u

s
Devoir Libre
26 Équations différentielles : Exemple d’étude.

u
r
Blague du jour La famille des Riccati

é
☛ Que signifie le sigle B.U.S.H ? Jacopo Francesco Riccati (1676-1754) était un physicien et math-

Mathématicien du jour
-Réponse : Bombarder Uniquement Saddam Hussein .... ématicien italien, père de Vincenzo Riccati et de Giordano Ric-

Po r
r
☛ Les Irakiens sont dans la rue et crient : A bas Clinton, cati. Ses travaux en hydraulique (canaux de Venise) et en acous-
tique le conduisent à résoudre des équations différentielles du

u
à bas Clinton !
Un autre Irakien intervient et dit aux manifestants : Ce second ordre en les réduisant au 1er ordre et plus générale-

uor
n’est plus Clinton le président, c’est Bush. ment à rechercher des méthodes de séparation des variables
Les manifestants répondent : Mais ça n’a aucun sens afin d’obtenir les solutions par simples quadratures. Ses travaux
! On ne va tout de même pas crier "A babouche ! A furent publiés après sa mort par ses fils à partir de 1764 sous le
babouche !" titre Opere del conte Jacopo Riccati.

P
ditions sont remplies, certaines propriétés des ces solutions sont
Énoncé : CNC 2004, MP ensuite étudiées.

r
Définitions et notations I. E XEMPLES

e
ET RÉSULTATS GÉNÉRAUX
er
① Un premier exemple

m
Dans tout le problème, par "solution d’une équation différentielle",
Soient α un réel et f α la fonction x 7−→ eαx .
on fait référence aux solutions à valeurs réelles définies sur R.
Si f est une fonction continue sur R à valeurs réelles, on lui associe ① Résoudre l’équation différentielle (E f 1 ). Cette équation
l’équation différentielle possède-t-elle des solutions bornées au voisinage de
m

y′ − y + f = 0. +∞ ?

r
(E f )
Le but du problème est d’étudier des conditions d’existence de so- ② Ici on suppose que α 6= 1.

o
lutions bornées de l’équation différentielle (E f ), et lorsque ces con- ① Résoudre l’équation différentielle (E f α ).
or

F
✉: mamouni.myismail@gmail.com

243
i

r
s
P ROBLÈMES C ORRIGÉS -MP ✍ M AMOUNI M Y I SMAIL
MAMOUNI . NEW. FR

i
s

s
u
② À quel condition nécessaire et suffisante sur α cette ② La solution Yf est-elle nécessairement bornée au

s
équation admet-elle des solutions bornées au voisi- voisinage de +∞ ?
nage de +∞ ? Lesquelles ?

u
⑥ On suppose ici que f est bornée.
③ L’équation différentielle (E f α ) admet-elle des solutions
bornées sur R ? ① Montrer que Yf est bien définie et que c’est l’unique

r
solution bornée, sur R, de l’équation différentielle

é
② Résultats généraux (E f ).

Po r
r
② Si en outre f tend vers 0 en +∞, montrer que Yf pos-
① Quelle est la structure de l’ensemble des solutions de
sède une limite nulle en +∞.
l’équation différentielle (E f ) ?

u
② Montrer que les solutions de l’équation différentielle ③ Si maintenant f tend vers 0 en −∞, montrer que Yf

uor
(E f ) sont de la forme possède une limite nulle en −∞.
 Z x 
x
yλ : x 7−→ e λ − e−t f (t) dt , λ ∈ R. ③ Un autre exemple
0

P
③ On suppose que la solution yλ est bornée au voisinage On pose
Z +∞
+ 2) n x n
p (2p
de +∞. Montrer alors que l’intégrale e −t
f (t) dt est un,p ( x ) = (−1) , x∈R et (n, p) ∈ N 2 .
0 (2p + 1)! n!
convergente et vaut λ.
① Montrer que, pour tout réel x, la suite double

r
④ Combien de solutions bornées au voisinage de +∞ un,p ( x ) (n,p)∈N 2 est sommable.
l’équation différentielle (E f ) peut-elle avoir au maxi-

e
mum ? ② En déduire le rayon de convergence et la somme de la
Z +∞
er
xn
⑤ On suppose maintenant que l’intégrale e−t f (t) dt série entière ∑ an , où

m
0 n ≥0 n!
est convergente et on pose +∞
Z +∞ (2p + 2)n
λf = e −t
f (t) dt et Yf = yλ f . an = ∑ (−1) p (2p + 1)! , n ∈ N.
m

0 p=0
Dans la suite on pose u( x ) = e x sin(e x ), x ∈ R.

r
① Vérifier que, pour tout réel x, Yf ( x) =
Z +∞ Z +∞
x −t
③ Montrer que l’intégrale e−t u(t) dt est convergente.

o
e e f (t) dt.
x 0
or

F
✉: mamouni.myismail@gmail.com

244
i

r
s
✍ M AMOUNI M Y I SMAIL
MAMOUNI . NEW. FR
P ROBLÈMES C ORRIGÉS -MP

i
s

s
u
Z +∞
④ Montrer que, pour tout réel x, e−t u(t) dt = ⑥ Montrer alors que Y| f | est intégrable sur R.

s
Z +∞ x ⑦ En déduire que Yf est intégrable sur R et montrer que
sin θ Z +∞ Z +∞

é

u
ex θ Yf (t) dt = f (t) dt.
−∞ −∞
⑤ En faisant une intégration par partie dans l’intégrale du ⑧ On désigne par E l’espace vectoriel des fonctions réelles con-

r
second membre de l’égalité précédente, montrer que la tinues et intégrables sur R ; on le muni de la norme N1

é
solution Yu de l’équation différentielle (Eu ) est bornée définie, pour tout élément g de E , par
sur R. Z +∞

Po r
r
N1 ( g) = | g(t)| dt.
−∞
II. C AS D ’ UNE FONCTION INTÉGRABLE
Montrer que l’application Φ : g 7−→ Yg est un endomor-

u
phisme continu de E et calculer sa norme.

uor
A- Cas où f est intégrable sur R
B- Cas où l’intégrale de f sur R converge
On suppose que f est intégrableZ sur R et on pose, pour tout réel x, On suppose ici que f possède une intégrale convergente sur R et
x
G(x) = f (t) dt. on pose, pour tout réel x,
Z +∞

P
−∞
① Montrer que la fonction G est continue, bornée et tend vers 0 F( x) = f (t) dt.
x
en −∞. ① Montrer que la fonction F est continue, bornée et tend vers 0
② Montrer que, pour tout réel x, la fonction t 7−→ e −t
f (t) est en +∞.
Z +∞

r
intégrable sur [ x, +∞[. ② Montrer que, pour tout réel x, l’intégrale e−t f (t) dt est
③ Montrer alors que la solution Yf de l’équation différentielle x

e
convergente et que
(E f ) vérifie Z +∞
er
Z +∞ e x
e−t f (t) dt = F( x ) − YF ( x ).

m
∀ x ∈ R, Yf ( x) ≤ | f (t)| dt, x
x (on pourra faire une intégration par partie)
puis en déduire que Yf est bornée sur R et tend vers 0 en +∞. ③ En déduire que la solution Yf de l’équation différentielle (E f )
④ Montrer que Yf = − G + YG et conclure que Yf tend vers 0 en est bornée et tend vers 0 en +∞.
m

−∞. ④ Montrer que Yf tend vers 0 en −∞.

r
⑤ Justifier que la solution Y| f | de l’équation différentielle (E| f | ) ⑤ Montrer alors que Yf possède une intégrale convergente sur

o
est bornée et tend vers 0 en ±∞. R, égale à celle de f .
or

F
✉: mamouni.myismail@gmail.com

245
i

r
s
P ROBLÈMES C ORRIGÉS -MP ✍ M AMOUNI M Y I SMAIL
MAMOUNI . NEW. FR

i
s

s
u
III. C AS D ’ UNE FONCTION PÉRIODIQUE ② Montrer que la série de F OURIER de f 1 est normalement

s
convergente.
On suppose ici que f est 2π-périodique. 

u
③ En déduire la convergence de la série ∑ |c−k ( f 1 )| +
① Montrer que l’équation différentielle (E f ) possède une k∈N

unique solution bornée qui est la fonction Yf .

r
|ck ( f 1 )| .

é
② Montrer que Yf est 2π-périodique et de classe C 1 .
④ En utilisant le théorème de D IRICHLET, montrer que
③ Calculer les coefficients de F OURIER complexes de Yf en fonc-  
1 n −1 + ∞ 

Po r
r
tion de ceux de f . ∗
∀n ∈ N , ∀ x ∈ R, | f n ( x ) − c0 ( f )| ≤ √ ∑ |c−k ( f1 )| +
④ On pose f 0 = f et f n+1 = Yf n , n ≥ 0. 2 k=1

u
① Pour tout n ∈ N, exprimer les coefficient de F OURIER ⑤ Quelle conclusion concernant le mode de convergence

uor
complexes de f n en fonction de ceux de f 1 . de la suite ( f n )n∈N peut-on tirer de ce qui précède ?

P
i
F

r
i
n

e
er
Á la prochaine

m
m

or
or

F
✉: mamouni.myismail@gmail.com

246
i

r
s
✍ M AMOUNI M Y I SMAIL
MAMOUNI . NEW. FR
P ROBLÈMES C ORRIGÉS -MP

i
s

s
u
au voisinage de +∞ est que α < 0, en prenant λ = 0
Corrigé : Pr. Mamouni, CPGE Rabat, Maroc

s
mais cette solution n’est pas bornée sur R.

u
② Résultats généraux

Partie I. E XEMPLES ① L’ensemble des solutions de l’équation différentielle (E f )

r
ET RÉSULTATS GÉNÉRAUX
est un espace affine de dimension 1.

é
① Un premier exemple ② Soit y une solution de l’équation différentielle (E f ), donc

Po r
r
(y( x )e− x )′ = (Zy′ ( x ) − y( x ))e− x = − f ( x )e− x , d’où
① l’équation différentielle en question est : (E f 1 ) : y′ − y = x
y( x )e − x = λ − e−t f (t) dt et donc y = yλ : x 7−→

u
−e x , dont la solution s’écrit y = y H + y0 où y H solution 0
 Z x 
générale de l’équation homogène : (E H) : y′ − y = 0

uor
ex λ − e−t f (t) dt , λ ∈ R.
et y0 solution particulière de (E f 1 ). On a y H ( x ) = λe x 0
et à l’aide de la méthode de la variation de la constante, ③ Si on suppose que Zla solution yλ est bornée au voisinage
on pose y0 ( x ) = λ( x )e x , on injecte cette solution dans x
l’équation et on trouve λ′ ( x ) = −1, d’où y0 ( x ) = − xe x . de +∞, alors λ − e−t f (t) dt = yλ ( x )e− x → 0 et

P
x →+ ∞
Z +0∞
Donc y( x ) = (− x + λ)e x . Cette équation ne possède au-
donc l’intégrale e−t f (t) dt est convergente et vaut
cune solution bornée au voisinage de +∞. 0
λ.
② ① De même on a : l’équation différentielle en question
④ L’équation différentielle (E f ) peut avoir au maximum

r
est : (E f α ) : y′ − y = −eαx , dont la solution s’écrit
y = y H + y0 où y H solution générale de l’équation une solution bornée au voisinage de +∞, en prenant
Z +∞

e
homogène : (E H) : y′ − y = 0 et y0 solution par- λ = e−t f (t) dt, à condition que l’intégrale
ticulière de (E f α ). On a y H ( x ) = λe x et à l’aide
er
Z +∞ 0
de la méthode de la variation de la constante, on

m
e−t f (t) dt soit convergente.
pose y0 ( x ) = λ( x )e x , on injecte cette solution 0

dans l’équation et on trouve λ′ ( x )e x = −eαx , d’où ⑤ ① Pour tout réel x, on a : Yf ( x) = ex λ f −
1 αx 1 αx
m

Z x  Z 0
y0 ( x ) = − e . Donc y( x ) = − e + λe x . −t
α−1 α−1 e f (t) dt = x
e−t f (t) dt +

r
e
② Donc une condition nécessaire et suffisante sur α Z0 + ∞  Z +∞ x
−t x −t

o
pour que cette équation admet des solutions bornées e f (t) dt = e e f (t) dt.
0 x
or

F
✉: mamouni.myismail@gmail.com

247
i

r
s
P ROBLÈMES C ORRIGÉS -MP ✍ M AMOUNI M Y I SMAIL MAMOUNI . NEW. FR

i
s

s
u
Z +∞ Z A
② La solution Yf n’est pas nécessairement bornée au e x −t
e | f (t) |dt = e x
e − t | f (t) |dt +

s
t Z x+ ∞ Z +∞ x
voisinage de +∞ si on prend par exemple f (t) = e 2 ,
e−t | f (t) |dt, or e x e−t | f (t) |dt → 0

é
ex

u
x
dans ce cas Yf ( x ) = 2e 2 → +∞ A Z +∞ A
x →+ ∞ −t
quand x → −∞, car e | f (t) |dt est une con-

r
⑥ ① Si f est bornée par une constante M, alors A Z A

é
Z +∞ Z +∞
| e − t f (t) dt| 6 e−t | f (t)| dt 6 stante qui ne dépond pas x et e x
e−t | f (t) |dt ≤
Z A x
xZ x

Po r
r
+∞
M e−t dt = Me −x
, donc l’intégrale e x
e−t εdt = εe x (e− x − e− A ) = ε(1 − e x− A ) ≤ ε, et
x x

u
Z +∞ donc Yf possède une limite nulle en −∞.
e−t f (t) dt est bien définie donc Yf ( x ) =

uor
xZ
+∞
e x −t
e f (t) dt est bien définie et bornée aussi ③ Un autre exemple
x
par M, comme l’équation admet au maximum une
solution bornée alors c’est l’unique solution bornée, 1 ((2p + 2) x )n

P
sur R, de l’équation différentielle (E f ).
① ∑ |un,p ( x )| =
(2p + 1)! n∑ n!
=
n ≥0 ≥0

② Si f tend vers 0 en +∞, alors ∀ε > 0 ∃ A > 0 e(2p+2)x e(2p+2)x


finie ∑∑ |un,p ( x )| = ∑ =
tel que ∀ x ∈ R : x > A ⇒ | f ( x )| < ε. Ainsi (2p + 1)! p≥0 n ≥0 p≥0 (2p + 1)!

r
∀ x > A on a : | f (t| < ε. ∀t ≥ x donc |Yf ( x )| = (e x )2p+1
Z +∞ Z +∞ ex ∑ (2p + 1)! = ex sh(x) aussi finie et donc, pour tout
|e x e−t f (t) dt| ex e−t | f (t) |dt

e
≤ ≤ p≥0
Zx x 
réel x, la suite double un,p ( x ) (n,p)∈N 2 est sommable.
er
+∞
εe x e−t dt = ε, d’où Yf possède aussi une lim-

m
x
ite nulle en +∞. xn
② Le rayon de convergence de la série entière ∑ an ,
n ≥0 n!
③ Si maintenant f tend vers 0 en −∞, alors ∀ε >
est alors infinie et sa somme est ∑ ∑ un,p (x) =
m

0 ∃ A < 0 tel que ∀ x ∈ R : x < A ⇒


p≥0 n ≥0

r
| f ( x )| < ε. Ainsi ∀ x < A on a : | f (t| < ε.
Z +∞ (2p + 2)n x n
∀ x ≤ t ≤ A, donc |Yf ( x )| = |e x −t ∑ ∑ (−1) p =

o
e f (t) dt| ≤ p≥0 n ≥0 (2p + 1)! n!
x
or

F
✉: mamouni.myismail@gmail.com

248
i

r
s
✍ M AMOUNI M Y I SMAIL MAMOUNI . NEW. FR
P ROBLÈMES C ORRIGÉS -MP

i
s

s
u
(−1) p ((2p + 2) x )n (−1) p (2p+2)x A- Cas où f est intégrable sur R
∑ (2p + 1)! ∑ = ∑ (2p + 1)! e =

s
p≥0 n ≥0
n! p≥0
(−1) p (e x )2p+1 ① La fonction G est continue, car primitive, bornée et tend vers

u
ex ∑ (2p + 1)! = ex sin(ex ). 0 en −∞ car f intégrable sur R.
p≥0

r
Z A Z A ② f est intégrable sur R, donc sa limite en +∞ ne peut qu’être

é
−t t
③ l’intégrale e u(t) dt = sin(e )dt = finie et donc f ne peut qu’être bornée par une constante M,
0 0 Z A
Z B=e A
sin( x ) d’où ∀ A ≥ x, on a : e−t | f (t)dt| ≤ Me− x . Donc, pour tout

Po r
r
dx est alors une intégrale classique conver- x
1 x réel x, la fonction t 7−→ e−t f (t) est intégrable sur [ x, +∞[.
(−1)k

u
Z +∞
gente car de même nature que la série alternée ∑ .
k ③ Et dans ce cas : ∀ x ∈ R, Yf ( x ) = |e x
e−t f (t) dt| ≤

uor
t
On a effectué le changement de variable x = e . Z +∞ Z +∞ x Z +∞
Z +∞ Z +∞ x −t x −x
sin θ e e | f (t)| dt ≤ e e | f (t)| dt = | f (t)| dt,
④ Pour tout réel x, on a : e−t u(t) dt = dθ x x Z +∞ x
x ex θ
en effectuant le changement de variable θ = et . donc Yf est bornée sur R par | f (t)| dt et tend vers 0 en

P
Z +∞ Z +∞ −∞
⑤ Yu ( x ) = e x e−t u(t) dt est déja bornée en +∞ +∞ car | f (t)| dt tend vers 0 en +∞.
Z A x x
Z +∞
car e−t u(t) dt converge, il reste donc à l’étudier en
0 ④ D’autre part ∀ x ∈ R, Yf ( x ) = e e−t f (t) dt = x

r
−∞. faisons une intégration par partie dans l’intégrale Z +∞  t→+∞
x Z +∞
Z +∞   Z +∞ −t ′ x −t
sin θ cos θ +∞ cos θ e x
e G (t) dt = e e G (t) x +e x
e−t G (t) dt =

e
| dθ | = | − + 2
dθ | = x x
ex θ θ x ex θ − G( x ) + YG ( x )
er
Z +∞ e Z +∞
cos e x cos θ 1 1 2 car lim e−t G (t) = 0, puisque G est bornée et donc Yf tend

m
| x + 2
dθ | ≤ x + 2
dθ = , d’où
e ex Z θ e ex Z θ ex t→+ ∞
+∞ + ∞ sin θ vers 0 en −∞ car G et YG tendent vers 0 en −∞.
|Yu ( x )| = |e x e−t u(t) dt| = |e x dθ | ≤ 2
x ex θ ⑤ On a f intégrable R ⇒ | f | intégrable R, donc de façon
m

donc la solution Yu de l’équation différentielle (Eu ) est pareille on montre que la solution Y| f | de l’équation différen-

r
bornée sur R. tielle (E| f | ) est bornée et tend vers 0 en ±∞.
Partie II. C AS D ’ UNE FONCTION INTÉGRABLE

o
⑥ On a : Y| f | (t) = Y|′f | (t) + | f (t)|, or Y|′f | intégrable car Yf tend
or

F
✉: mamouni.myismail@gmail.com

249
i

r
s
P ROBLÈMES C ORRIGÉS -MP ✍ M AMOUNI M Y I SMAIL
MAMOUNI . NEW. FR

i
s

s
u
vers 0 en ±∞ et | f | intégrable donc Y| f | intégrable sur R et B- Cas où l’intégrale de f sur R converge

s
par suite Yf est aussi intégrable sur R puisque |Yf | ≤ Y| f | .
① La fonction F est continue sur R, car c’est une primitive, et en

u
Z +∞
plus admet une limite nulle en +ß par construction de F et
⑦ Effectuons une intégration par parties, donc : Yf ( x ) dx =
une limite finie en −ß car l’intégrale converge, donc bornée

r
 −∞ 
Z +∞ Z +∞ Z +∞ x →+ ∞ et tend vers 0 en +∞.

é
ex e−t f (t) dt = ex e−t f (t) dt +
−∞ x x ② Même raisonnement que celui de la question II.A.4) En dé-
Z +∞ Z +∞ Z +∞ x→−∞
duire que la solution Yf de l’équation différentielle (E f ) est

Po r
r
e x e− x f ( x )dx = f ( x ) dx, car lim e x e−t f (t) dt =
−∞ −∞ x →− ∞ x bornée et tend vers 0 en +∞.
−t
0, puisque la la fonction t 7−→ e f (t) est intégrable et

u
Z +∞ Z +∞ ③ Ainsi on a : Yf = F − YF , or F bornée et tend vers 0 en −∞,
donc YF aussi et donc Yf vérifie la même chose.

uor
|e x e−t f (t) dt| ≤ | f (t)| dt → 0 quand x → +∞
x x
④ Même raisonnement que celui de la question II.A.7).
⑧ ∀( f , g) ∈ E2 , ∀λ ∈ R, on a : Φ( f + λg)( x ) =
Z +∞ Z +∞ Partie III. C AS D ’ UNE FONCTION PÉRIODIQUE
−t −t

P
Yf +λg ( x ) = e ( f (t) + λg(t)) dt = e f (t) dt +
Z +∞ x x
① f est 2π–périodique continue, donc bornée sur R, d’où Yf
λ e−t f (t) dt = Yf ( x ) + λYg ( x ) = Φ( f )( x ) + λΦ( g)( x ), aussi, or l’équation différentielle (E f ) possède au maximum
x
d’où une solution bornée qui est donc la fonction Yf .

r
Φ( f + λg) = Φ( f ) + λΦ( g) et par suite Φ est linéaire, de plus ② On effectue le changement de variable u = t − 2π donc y F ( x +
d’après les questions précédentes si g est une fonction réelle Z +ß Z +ß
x +2π x +2π
2π ) = e −t
e−t−2π f (t −

e
continue et intégrable sur R, alors Yg = Φ( g) l’est aussi, donc e f (t) dt = e
Φ : g 7−→ Yg est un endomorphisme de E, d’autre part : Z x++ß2π x
er
Z +∞ Z +∞ Z +∞ x −t
2π ) dt = e e f (t) dt = Yf ( x ), donc Yf est 2π–

m
N1 (Yg ) = |Yg (t)| dt ≤ Y| g(t)| dt = | g(t)| dt = x
−∞ −∞ −∞ périodique et de classe C 1 , comme produit de deux fonction
N1 (Φ( g)) de classe C 1 .
N1 ( g), d’où Φ est continue avec kΦk = Sup ≤
m

g6 =0 N1 ( g)
③ Les coefficients de F OURIER complexes de Yf sont donnés par

r
1, de plus, pour g ≥ 0 on a : Yg ≥ 0, d’où N1 (Yg ) = la formule :
Z +∞ Z +∞ Z 2π
1
g(t) dt, d’où kΦk ≥ 1 et donc kΦk = 1. ∀k ∈ Z : Yf ( x )e−ikx dx =

o
Yg (t) dt = ck (Yf ) =
−∞ −∞ 2π 0
or

F
✉: mamouni.myismail@gmail.com

250
i

r
s
✍ M AMOUNI M Y I SMAIL MAMOUNI . NEW. FR
P ROBLÈMES C ORRIGÉS -MP

i
s

s
 

u
 Z +ß  " #2π Z
e(1=−ik| )f 1x ( x−0x)| = Max | f 1 ( x )| .
Z 2π (1−ik)t Z +ß 2πM
1 1 e

s
e(1−ik)x e−t f (t) dt dx =  e−t f (t) dt + ∈R
e f ( x ) xdt =
2π 0 x 2π
1 − ik x 1 − ik le théorème de D IRICHLET , on a : ∀n
0 D’aprés
④ ∈
0

é
 

u
1
Z +ß Z +ß
c (f) N ∗ , ∀ x ∈ R on a :

e −t
e f (t) dt − e f (t) dt + k
−t
= | f n ( x ) − c0 ( f n )| = | ∑ ck ( f n )eikx |
2π (1 − ik) 2π 0 1 − ik

r
k∈Z ∗
ck ( f )

é
car ≤ ∑∗ |ck ( f n )|
1 −Zik Z +ß k∈Z

2π −t −t = ∑ |c−k ( f n )| + |ck ( f n )|

Po r
f (t) dt en effectuant le change-

r
e e f (t) dt = e
2π 0 k∈N ∗
ment de variable u = t − 2π et utilisant le fait que f est |c
−k ( f n )| |c ( f n )|
+ − k n −1

u
c (f)
= ∑ ∗ |1 + ik| n − 1 |1 − ik|
2π–périodique. D’où ∀k ∈ Z : ck (Yf ) = k . k∈N

uor
1 − ik  
1 n −1 + ∞  
ck ( f 1 ) ≤ √
2
∑ −k 1
| c ( f )| + | c (
k 1f )|
④ ① Pour tout n ∈ N, on a : ck ( f n ) = . p k=1 p
(1 − ik)n−1
car |1 + ik| = 1 + k2 ≥ 2 et |1 − ik| = 1 + k2 ≥ 2. de
② Parceque Yf de classe C1 bornée.

P
plus c0 ( f n ) = c0 ( f ) d’où le résultat.
   |c ( f )| + |c ( f )| 
③ ∑ |c−k ( f 1 )| + |ck ( f 1 )| = M ∑ −k 1 k 1 ⑤ Le mode de convergence de la suite ( f n )n∈N est le même
 
k∈N k∈N
M 1 n −1
est finie car c’est la série de FOURRIER de f 1 en x0 où que celui de la suite géométrique √
2

e r
er
i
F

m
nn
i
m

r
Á la prochaine

o
or

F
✉: mamouni.myismail@gmail.com

251
i

r
s
P ROBLÈMES C ORRIGÉS -MP ✍ M AMOUNI M Y I SMAIL
MAMOUNI . NEW. FR

i
s

s
u

s
Devoir Libre
27 Fonctions holomorphes : Série de Fourier lacunaire quadratique

u
Blague du jour John Edensor Littlewood (1885-1977)

Mathématicien du jour
é
☛ Comment appelle t-on un chien sans pattes ? On ne Mathématicien anglais. Il a surtout travaillé en analyse sur le su-
l’appelle pas, on va le chercher ! jet des fonctions entières. Il a collaboré pendant de nombreuses

Po r
r
☛ Un vieux rat rencontre une petite taupe. Curieux, il lui années avec Hardy et ils ont formulé ensemble deux conjectures.
demande : Il a aussi travaillé sur la théorie de Fourier. Il est lauréat de la

u
- Que veux-tu faire plus tard, ma petite ? Médaille Sylvester, de la Royal Medal et de la médaille Copley

uor
- Taupe-modèle ! ! en 1958.

Le sujet a pour objet d’établir la dérivabilité de la fonction q au

P
Énoncé : CNC MP, 2011 point 1 ; il utilise des outils de l’analyse complexe.
Le problème est composé de cinq parties ; les deux premières par-
La fonction q définie, pour tout x ∈ R, par ties ont pour objectif d’établir la formule (2) qui sera utile dans la
+ ∞ iπn2 x cinquième partie. Les trois dernières parties du problème s’enchaî-

r
e
q( x ) = ∑ nent entre elles.
n =1
iπn2

e
est étudie par Riemann il y a environ 150 ans, avec l’idée que cette
1ère partie : Formule sommatoire de Poisson
er
fonction est continue sur R mais nulle part dérivable. L’étude est

m
poursuivie par Hardy qui prouve en 1916 que cette fonction est
non dérivable en tout x ∈ R \{Q }. Il reste à étudier la dérivabil- Soit g : R −→ C une application de classe C 1 telle que les applica-
ité en x rationnel et c’est Gerver qui en 1970 réussit à trouver le tions t 7−→ t2 g(t) et t 7−→ t2 g′ (t), définies sur R, soient bornées à
m

résultat assez inattendu : la fonction est dérivable en tout x ∈ Q. l’infini, ce qui revient à dire que

r
Depuis, d’autres propriétés de cette fonction ont été étudiées, pro-
   
priétés qui analysent plus finement la régularité de cette fonction : 1 ′ 1
g(t) = O 2 et g (t) = O 2

o
en particulier son ordre de Holder local et son spectre multifractal. t−→±+ ∞ t t−→±+ ∞ t
or

F
✉: mamouni.myismail@gmail.com

252
✍ M AMOUNI M Y I SMAIL
MAMOUNI . NEW. FR
P ROBLÈMES C ORRIGÉS -MP

is

r
s

i
On lui associe la suite ( gn )n∈N de fonctions définies par

s
u
2ème partie : Application de la formule sommatoire de Poisson
g0 ( t ) = g ( t ) , gn (t) = g(t + 2nπ ) + g(t − 2nπ ) t ∈ R, n ∈ N ∗

s
Pour tout réel α > 0, on note hα la fonction définie, pour tout t ∈ R,

é
① Montrer que pour tout réel x, la fonction t 7−→ g(t)e−ixt est 2 2

u
intégrable sur R. par hα (t) = e−α t .

r
Par définition, la transformée de Fourier de g est la fonction notée ① Vérifier que, pour tout réel α > 0, la fonction hα satisfait les
gb définie sur R par hypothèses faites sur la fonction g dans la partie précédente.

é
Z ++∞
Dans la suite, on notera hbα la transformée de Fourier de hα , α > 0 ;
gb( x ) = g(t)e−ixt dt, x ∈ R. √

Po r
r
−+ ∞ on admettra que hb1 (0) = π.
② Montrer que la fonction hb1 est dérivable sur R et qu’elle satis-

u
② Montrer que la série de fonctions ∑ gn converge uniformé-
n∈N ∗ fait l’équation différentielle

uor
ment sur tout segment de R. x
y′ + y = 0 (2)
③ On note ge la fonction définie, pour tout t ∈ R, par ge(t) = 2
+∞ ③ Résoudre l’équation différentielle (1) et donner l’expression
∑ g n ( t ).

P
de hb1 .
n =0 √
π −x22
① Montrer que la fonction ge est de classe C 1 sur R. ④ Montrer, pour tout α > et tout réel x, hbα ( x ) = e 4α .
α
② Justifier que la fonction ge est 2π-périodique et que ses ⑤ Montrer, pour tout réel a > 0, ! la relation

r
coefficients de Fourier complexes sont donnés par √ +∞
2
+ ∞ −πn2
1 a 1 + 2 ∑ e−πn a = 1+2 ∑ e a . (3)
gb(k), k ∈ Z

e
ck ( ge) = n =1 n =1

On remarquera que, pour tout t ∈ R, ge(t) =
er
3ème partie : Un résultat général sur les fonctions holomorphes

m
p= n
lim ∑ g(t + 2pπ ).
n −→+ ∞
p=− n Si a et b sont des nombres complexes, γa,b désigne le chemin du
plan complexe C défini, pour tout t ∈ [0, 1], par γa,b (t) = (1 − t)a +
③ Montrer que les familles ( g(2nπ )n∈N et ( gb(n)n∈N sont
m

r
tb ; son image γa,b [0, 1]) = {(1 − t)a + tb, 0 ≤ t ≤ 1} est notée
sommables et que leurs sommes vérifient la relation
[ a, b] ; c’est le segment du complexe d’extrémités a et b.
suivante, dite formule sommatoire de Poisson,

o
Pour la suite du problème, on notera Ω la partie de C définie par
2π ∑ g(2nπ ) = ∑ gb(n). Ω = {z ∈ C, Im(z) > 0}
or

n∈Z n∈Z

F
✉: mamouni.myismail@gmail.com

253
i

r
s
P ROBLÈMES C ORRIGÉS -MP ✍ M AMOUNI M Y I SMAIL
MAMOUNI . NEW. FR

i
s

s
u
① Vérifier que pour tout (a, b) ∈ Ω2 , [ a, b] ⊂ Ω, puis justifier orientée dans le sens direct, de la plaque triangulaire T

s
que Ω est un ouvert connexe par arcs de C. du plan R 2 dont les sommets Z sont les affixes
Z des com-
Soit f : Ω −→ C une application continue, si (a, b) ∈ Ω2 , on

é
plexes a, b et c. En déduire f (z)dz + f (z)dz =

u
définit
Z l’intégarle curviligne de f le long du chemin γa,b , notée Z γ a,c γc,b

f (z)dz.

r
f (z)dz ou simplement Φ(a, b), par
γ a,b γ a,b

é
Z Z 1
Φ(a, b) = f (z)dz := (b − a) f ((1 − t)a + tb)dt. ③ Soit a ∈ Ω fixé ; déduire de ce qui précède que la fonction
γ a,b 0 Φa est holomorphe sur Ω et que sa dérivée au sens com-

Po r
r
② Soit a fixé dans Ω ; montrer que l’application Φa : Ω −→ C, plexe, noté Φ′a , vérifie Φ′a (b) = f (b) pour tout b ∈ Ω ; on
b 7−→ Φa (b) = Φ(a, b), est continue sur Ω. rappelle que

u
③ Soit ψ la fonction définie, pour tout z ∈ Ω, par ψ(z) = z ; Φ a (c ) − Φ a (b)

uor
Φ′a (b) = lim
soient a et b deux éléments distincts de Ω. Montrer que c−→b c−b
c∈Ω\{b }
l’ensemble des c ∈ Ω tels que
④ On suppose que, pour tout b ∈ Ω, la fonction r 7−→
Z Z Z
Φ(ir, b), définie sur ]0, + + ∞[, admet une limite dans C

P
ψ(z)dz + ψ(z)dz = ψ(z)dz
γ a,c γc,b γ a,b lorsque r tend vers 0+ ; on note F(b) cette limite. Montrer
est soit une droite soit une demi-droite du plan complexe à que, pour tout (b, c) ∈ Ω2 , F(c) − F(b) = Φ(b, c), puis
préciser. en déduire que F est holomorphe sur Ω et que F′ = f
④ Dans la suite de cette partie, f est supposée holomorphe sur sur Ω.

r
Ω. Si x et y sont des réels tels que x + iy ∈ Ω, on pose
P( x, y) = Re( f ( x + iy)), Q( x, y) = Im( f ( x + iy)). 4ème partie : Étude d’un exemple

e
∂P
er
① Rappeler les relations reliant les dérivées partielles
∂x

m
∂Q ∂P ∂Q On note exp la fonction exponentielle complexe. Si z ∈ C \R − ,
et puis et . on note Arg(z) l’élément de l’intervalle ] − π, π [ tel que z =
∂y ∂y ∂x
|z| exp(iArg(z)) ; on pose alors Log(z) = ln(|z|) + iArg(z) et, pour
② Soit (a, b, c) ∈ Ω3 . Montrer à l’aide des résultats
Z du pro-
m

tout λ ∈ R,
zλ = exp(λLog(z))

r
gramme sur les formes différentielles que Pdx −
Z ∂T +
La fonction Log est le logarithme principal défini sur C \R − ; on
Qdx + Pdy, où ∂T + désigne la frontière,

o
Qdy = rappelle que les fonctions exp et Log sont holomorphes sur C et
∂T +
or

F
✉: mamouni.myismail@gmail.com

254
i

r
s
✍ M AMOUNI M Y I SMAIL
MAMOUNI . NEW. FR
P ROBLÈMES C ORRIGÉS -MP

i
s

s
u
C \ R− respectivement ; leur dérivées au sens complexes vérifiant Pour tout entier naturel non nul n, on note un la fonction définie,

s
1 pour tout z ∈ C, par
Log′ (z) = , z ∈ C \R − et exp′ (z) = exp(z), z ∈ C.
z un (z) = exp(iπn2 z).

u
Soit λ un réel fixé dans l’intervalle ] − 1, 0[ ; on note f λ la fonction
définie, pour tout z ∈ C \R − , par ① Soit z ∈ C ; montrer que la série ∑ un (z) converge ⇐⇒ z ∈

r
i n ≥1
f λ (z) = zλ exp(− ).

é
z Ω.
+∞
① Justifier que f λ est holomorphe sur Ω.
Dans la suite, on pose u(z) = ∑ un (z), z ∈ Ω.

Po r
r
② Soit b un complexe fixé dans Ω. On Znote Jλ,b la fonction, n =1

définie pour tout r > 0, par Jλ,b (r ) = f λ (z)dz. Montrer

u
② Montrer que, pour tout z ∈ Ω, u(z + 1) + u(z) = 2u(4z).
γir,b

uor
que la fonction Jλ,b admet une limite, notée Fλ,b , lorsque r tend ③ Pour tout n ∈ N ∗ et tout ( x, y) ∈ R ×]0, + + ∞[, on pose
vers 0+ et que Z uen ( x, y) = un ( x + iy) et ue( x, y) = u( x + iy)
Fλ (b) = bλ+1 tλ exp(−i tb)dt.
]0,1] ① Montrer que pour tout k ∈ N, la série de fonctions

P
On pourra utiliser le théorème de convergence dominée après en
avoir vérifié les conditions de validité.
∑ nk uen converge normalement sur R ×]0, + + ∞[ pour
n ≥1
③ On note Gλ lafonction, définie pour tout z ∈ C, par Gλ (z) =
tout a > 0.
i
z−λ−2 exp Fλ (z). ② Montrer soigneusement que la fonction ue, définie ci-

r
z dessus, possède en tout point de R ×]0, + + ∞[ une
① Justifier que les fonctions Fλ et Gλ sont holomorphes sur ∂ue

e
Ω et que Fλ′ = f λ sur Ω. dérivée partielle par rapport à x et exprimer ( x, y),
∂x
er
② Montrer pour ( x, y) ∈ R ×]0, + + ∞[, sous la forme de la somme
 que, pour tout z ∈
 Ω, Gλ (z) =

m
Z ++∞ d’une série.
1 i iu
exp u−λ−2 exp − du.
z z 1 z ③ Montrer de même que la fonction ue possède en tout
③ Montrer que, pour tout z ∈ Ω, | Gλ (z)| ≤ 2 et que point de R ×]0, + + ∞[ une dérivée partielle par rapport
m

3
| F−1/2 (z)| ≤ 2|z| 2 . ∂ue

r
à y et l’exprimer en fonction de ( x, y).
∂x
5ème partie : Démonstration de la propriété proposée

o
④ Montrer que la fonction u est holomorphe sur Ω.
or

F
✉: mamouni.myismail@gmail.com

255
i

r
s
P ROBLÈMES C ORRIGÉS -MP ✍ M AMOUNI M Y I SMAIL
MAMOUNI . NEW. FR

i
s

s
u
④ Partant de la formule (2) de la deuxième partie et moyennant ⑧ Pour tout z ∈ Ω, on pose

s
   
un résultat sur les zéros d’une fonction holomorphe, montrer +∞
un (z) (iπ )1/2 +∞
4z
que pour tout z ∈ Ω, v1 ( z ) = ∑ et w ( z ) = ∑ nF−1/2 − 2nF−1/2
iπn2 2 πn2

u
 1/2   n =1 n =1
i 1
(1 + 2u(z)) = 1 + 2u − . ① Montrer que la fonction v1 est holomorphe sur Ω et que
z z

r
v1′ = u.

é
1 ② On admet que la fonction w est holomorphe sur Ω ; cal-
⑤ En déduire que, pour tout z ∈ Ω, u(1 + z) + = culer sa dérivée w′ , au sens complexe, en admettant que
2

Po r
r
 1/2 +∞    l’on puisse dériver terme à terme la série définissant w.
i −iπn2 −iπn2
∑ exp( 4z ) − exp .
③ Montrer que, pour tout z ∈ Ω,

u
z n =1
z
   
−z (iπ )1/2 +∞
4z

uor
un v 1 ( z + 1 ) − v (1 ) = + ∑ nF−1/2 − 2nF−1/2
⑥ Montrer que la série de fonctions converge normale- 2 2 πn2 π

iπn2
n =1
n ≥1 ⑨ Montrer qu’il existe une constante c positive telle que, pour
ment sur {z ∈ C, Im(z) ≥ 0} et que sa somme, notée v, est tout z ∈ Ω, on ait
continue sur cette ensemble. z

P
v ( z + 1 ) − v (1 ) + ≤ c|z|3/2 .
2
⑦ Montrer que,
 αzpour
 tout z ∈ Ω et tout α > 0, la série
x
⑩ Montrer soigneusement que q( x + 1) − q(1) + =
∑ nF−1/2 πn2 est convergente, où F−1/2 est la fonction 2 x−→0
O( x3/2 ). En déduire que la fonction q est dérivable en 1 et

r
n ≥1
définie dans la quatrième partie. préciser q′ (1).

e
i
er
F

m
n
i
n
m

r
Á la prochaine

o
or

F
✉: mamouni.myismail@gmail.com

256
i

r
s
✍ M AMOUNI M Y I SMAIL
MAMOUNI . NEW. FR
P ROBLÈMES C ORRIGÉS -MP

i
s

s
u
Soient U un ouvert de C, et f : U → C une application.
Corrigé : Pr. Hfa, CPGE Agadir, Maroc

s
On pose : U e = {( x, y) ∈ R2 tq x + iy ∈ U } et ∀( x, y) ∈ U e ;
fe( x, y) = f ( x + iy).

é
Rappels du cours :

u
En plus du programme Français, ce problème utilise ce petit rappel On pose aussi : ∀( x, y) ∈ U e ; P( x, y) = Re( f ( x + iy)) et
du cours, qui fait partie Q( x, y) = Im( f ( x + iy)).

r
e est un ouvert de R2 et les assertions suivantes sont équiv-

é
Alors U
du programme Marocain. alentes :
Définition :

Po r
(i) f est holomorphe sur U.

r
Soient U un ouvert de C, a ∈ U, et f : U → C une application. e e
(ii) fe est de classe C1 sur U e ; ∂ f ( x, y) + i ∂ f ( x, y) =
e et ∀( x, y) ∈ U

u
f (z) − f ( a)
(i) On dit que f est holomorphe en a si lim existe dans ∂x ∂y
z−a

uor
z→ a
6= 0.
1 e e
C. (iii)
 P et Q sont de classe C sur U et ∀( x, y) ∈ U ;
f (z) − f ( a)  ∂P ∂Q
Dans ce cas cette limite est notée : f ′ (a) = lim , dite 
 ( x, y) = ( x, y)
z→ a
6=
z − a ∂x ∂y

P
dérivée de f en a. 
 ∂Q ∂P
 ( x, y) = − ( x, y)
(ii) On dit que f est holomorphe sur U si f est holomorphe en ∂x ∂y
tout point de U, et dans ce Remarque :
Sous les notations du théorème précèdente, si f est holomorphe
sur U, alors :

r
cas, l’application f ′ : U → C est dite la dérivée de f sur U.
e e
Remarques : ( Opérations sur les dérivées ) ∀( x, y) ∈ U e ; f ′ ( x + iy) = ∂ f ( x, y) = −i ∂ f ( x, y)

e
(i) On dérive de la même manière que pour les fonctions d’une vari- ∂x ∂y
er
able réelle, le produit, le

m
rapport ( quand il est défini ), les combinaisons linéaires, de deux Théorème : ( Principe des zéros isolés )
fonctions holomorphes. Soient U un ouvert connexe par arcs de C, et f : U → C une
(ii) Si U est un ouvert connexe par arcs de C, et f : U → C une application holomorphe.
On suppose que f n’est pas identiquement nulle, et qu’il existe
m

application holomorphe sur U,


a ∈ U tel que : f (a) = 0.

r
alors f est constante sur U si et seulement si ∀z ∈ U ; f ′ (z) = 0. alors il existe r > 0, tel que D (a, r ) = {z ∈ C tq |z − a| < r } ⊂ U

o
Théorème : ( Équations de Cauchy Riemann ) et ∀z ∈ D (a, r )\{ a} ; f (z) 6= 0.
or

F
✉: mamouni.myismail@gmail.com

257
i

r
s
P ROBLÈMES C ORRIGÉS -MP ✍ M AMOUNI M Y I SMAIL
MAMOUNI . NEW. FR

i
s

s
u
Autrement dit ( par contraposée ) si {z ∈ U tq f (z) = 0} admet segment [− A, A], donc bornée sur ce segment, et par suite bornée

s
un point d’accumulation, sur R.
alors f est identiquement nulle. Il existe alors M > 0 ; ∀t ∈ R ; t2 g(t) ≤ M.

u
∞ 2 Soit [ a, b] un segment de R, Puisque : lim (b − 2nπ ) = −∞ et
eiπn x n→∞

r
Dans tout ce problème, on pose : q( x ) = ∑ 2 lim (a + 2nπ ) = +∞
n =1 iπn n→∞

é
∃ N ∈ N ∗ ; ∀n ≥ N ; ∀t ∈ [ a, b] ; (t − 2nπ ) ≤ (b − 2nπ ) < 0 et

Po r
(t + 2nπ ) ≥ (a + 2nπ ) > 0.

r
M M
∀n ≥ N ; ∀t ∈ [ a, b] ; | gn (t)| ≤ + ≤

u
ère 2
1 Partie : Formule sommatoire de Poisson (t − 2nπ ) (t + 2nπ )2
Soit g : R → C une application de classe C1 telle que : g(t) =

uor
1 1 M M
O ( 2 ) et g′ (t) = O ( 2 ). 2
+ .
| t|→+ ∞ t |t|→+ ∞ t (b − 2nπ ) (a + 2nπ )2
M M M M
On pose : ∀t ∈ R ; g0 (t) = g(t) et ∀n ∈ N ∗ ; gn (t) = ∼ et ∼ alors la série
(b − 2nπ )2 n →∞ 4π 2 n2 ( a + 2nπ )2 n→∞ 4π 2 n2

P
g(t + 2nπ ) + g(t − 2nπ ). !
h
1.1) L’application [t 7−→ g(t)e−ixt ] est continue sur R, et d’aprés M M
∑ + converge.
l’hypothèse ci-dessus : (b − 2nπ )2 ( a + 2nπ )2
1

r
h(t) = O ( 2 ), alors h est intégrable sur R. !
|t|→+ ∞ t
Par conséquent : la série ∑ gn ( t ) converge normalement pour

e
Z +∞ n≥ N
er
On pose alors : ∀ x ∈ R ; gb( x ) = g(t)e−ixt dt. gb est la trans- t ∈ [ a, b].

m
−∞
formée de Fourier de !
h g. i
1.2) L’application t 7−→ t2 g(t) est continue sur R et g(t) = Conclusion : la série ∑ gn ( t ) converge uniformément sur tout
1 n ≥0
m

O ( 2 ), alors il existe A > 0, segment de R.

r
| t|→+ ∞ t
1.3) On note ge la fonction définie, pour tout t ∈ R, par : ge(t) =
tel que cette application est bornée sur R \[− A, A], cette application
∑ g n ( t ).

o
est continue sur le n ≥0
or

F
✉: mamouni.myismail@gmail.com

258
i

r
s
✍ M AMOUNI M Y I SMAIL MAMOUNI . NEW. FR
P ROBLÈMES C ORRIGÉS -MP

i
s

s
u
1.3.1) g est de classe C1 sur R, alors ∀n ∈ N ; gn est de classe C1 sur
n
Alors, d’aprés 1.2) la série ∑ g p (t)e−ikt converge uniformément

s
R. p=0
Notons que : ∀t ∈ R ; g0′ (t) = g′ (t) et ∀n ∈ N ∗ ; gn′ (t) =

é
sur tout segment de R vers

u
g′ (t + 2nπ ) + g′ (t − 2nπ ). [t 7−→ ge(t)e−ikt ], en particulier sur [0, 2π ]. On déduit alors que :
On remplace g par g′!dans la question précèdente, on obtient alors

r
Z 2π
1
gn (t)e−ikt dt

é
∀k ∈ Z ; ck ( ge) =
que la série : ∑ gn′ 2π n∑
≥0 0
n ≥0
converge uniformément sur tout segment de R. Z 2π Z 2π Z

Po r
r
1 −ikt 1 −ikt
∀k ∈ Z ; ck ( ge) = g(t + 2nπ )e
D’où ge est de classe C1 sur R, et ∀t ∈ R ; ge′ (t) = 2π n∑
∑ gn′ (t). g(t)e dt + dt +
2π 0 0 0

u
n ≥0 ≥1
1.3.2) Remarquons que d’aprés le raisonnement de la question 1.2) On pose : u = t + 2nπ et v = t − 2nπ.
 Z 2( n + 1) π

uor
Z 2π Z 2
on a : ! ! 1 −ikt 1 −iku
∀k ∈ Z ; ck ( ge) =
2π n∑
g(t)e dt + g (u)e du +
2π 0 ≥1 2nπ −2
∀t ∈ R ; les séries ∑ g(t + 2nπ ) et ∑ g(t − 2nπ ) conver-
n ≥1 n ≥1
De 1.1) l’application [t 7−→ g(t)e−ikt ] est intégrable sur R, la rela-

P
gent. tion de Chasle donne alors :
Z 2π Z +∞
∀t ∈ R ; ge(t + 2π ) = ∑ gn (t + 2π ) = g(t + 2π ) + ∑ g(t + 2(n + 1 −ikt 1
∀k ∈ Z ; ck ( ge) = g(t)e dt + g(u)e−iku du +
n ≥0 n ≥1 2π 0 2π 2π
Z 0
1) π ) + ∑ g ( t − 2 ( n − 1 ) π ). 1

r
g(v)e−ikv dv.
n ≥1 2π −∞ Z +∞
ge(t + 2π ) = g(t + 2π ) + ∑ g(t + 2nπ ) + ∑ g(t − 2nπ ). 1 1

e
n ≥2 n ≥0
∀k ∈ Z ; ck ( ge) = g(t)e−ikt dt = gb(k).
2π −∞ 2π
er
  !
ge(t + 2π ) = g(t + 2π ) + g(t) + g(t − 2π ) + ∑ ( g(t + 2nπ ) + g(t − 2nπ )) =

m
1
n ≥2 1.3.3) g(2nπ ) = O alors les séries ∑ g(2nπ ) et
ge(t). | n |→∞ n2 n ≥0
Z 2π !
1
m

∀k ∈ Z ; ck ( ge) = ge(t)e−ikt dt. ∑ g(−2nπ ) sont


2π 0

r
n n n ≥1
∀n ∈ N ; ∀t ∈ R ; ge(t)e −ikt −ikt
. absoluments convergentes, et donc la famille ( g(2nπ )) n∈Z est
− ∑ g p (t)e = ge(t) − ∑ g p (t)

o
p=0 p=0 sommable.
or

F
✉: mamouni.myismail@gmail.com

259
i

r
s
P ROBLÈMES C ORRIGÉS -MP ✍ M AMOUNI M Y I SMAIL
MAMOUNI . NEW. FR

i
s

s
u
Z +∞ ϕ 2
∀n ∈ Z ; gb(n) = g(t)e−int dt = 2πcn ( ge). L’application [( x, t) 7−→ e−t −ixt ] est de classe C1 sur R2 .

s
−∞ ∂ϕ 2 2
ge est 2π-périodique de classe C1 sur R. ∀n ∈ Z ; cn ( ge′ ) = incn ( ge). ∀( x, t) ∈ R2 ; ( x, t) = |t| e−t et l’application [t 7−→ | t| e−t ]

é
∂x

u
1 1 1 2 est continue intégrable sur R.
∀n ∈ Z∗ ; |cn ( ge)| = cn ( ge′ ) ≤ ( 2 + cn ( ge′ ) ).
| n| 2 n ′
hb1 est donc de classe C1 sur R et ∀ x ∈ R ; hb1 ( x ) =

r
1 Z +∞ Z +∞

é
La famille ( 2 )n∈Z ∗ est sommable, et d’aprés la formule de Parse- ∂ϕ 2
n ( x, t)dt = −i te−t −ixt dt.
2 − ∞ ∂x −∞
val, la famille ( cn ( ge′ ) )n∈Z Z

Po r
i +∞

r
b ′ x b 2
est aussi sommable, donc la famille (| cn ( ge)|)n∈Z est sommable, et ∀ x ∈ R ; h1 ( x ) + h1 ( x ) = (−ix − 2t)e−t −ixt dt =
2 2 −∞
par suite la famille ( gb(n)) n∈Z i h −t2 −ixt i+∞

u
est aussi sommable. e = 0.
2

uor
−∞
D’autre part, en utilisant le théorème de Direchlet, on a :
hb1 est donc solution sur R de l’équation différentielle : (1)
∑ gb(n) = 2π ∑ cn (ge) = 2π ge(0) = 2π ∑ g(2nπ ).
n ∈Z n ∈Z n ∈Z
x
y′ + y = 0.

P
2ème Partie : Application de la formule sommatoire de 2
2
Poisson −x
2.3) La solution génèrale de (1) est y( x ) = λe 4 . avec λ constante
réelle.
Pour tout réel α > 0, on note hα la fonction définie, pour tout t ∈ R, 2
−x √

r
2 2
par hα (t) = e−α t . Posons alors : ∀ x ∈ R ; hb1 ( x ) = λe 4 . hb1 (0) = λ = π.
2.1) hα est de classe C1 sur R, et lim t2 hα (t) = lim t2 h′α (t) = 0,

e
| t|→∞ | t|→∞ 2
er
alors les applications √ −x
Conclusion : ∀ x ∈ R ; hb1 ( x ) = πe 4 .

m
[t 7−→ t2 hα (t)] et [t 7−→ t2 h′α (t)] sont bornées sur R, et donc hα Z +∞
2 t2 −ixt
2.4) ∀ x ∈ R ; hbα ( x ) = e−α dt. On pose : u = αt.
−∞
vérifie les hypothèses de g. √ 2
Z
π − x 2.
Z +∞
m

− x2 − t2 −itx 1 +∞ −u2 −ix u 1b x


2.2) ∀ x ∈ R ; h1 ( x ) = e et hb1 ( x ) = e e dt = b
∀ x ∈ R ; hα ( x ) = e α du = h1 ( ) = e 4α

r
Z +∞ −∞ α −∞ α α α
e−t
2 −ixt
2.5) Maintenant, appliquons 1.3.3) à hα . 2π ∑ hα (2nπ ) = ∑ hbα (n).

o
dt.
−∞ n ∈Z n ∈Z
or

F
✉: mamouni.myismail@gmail.com

260
i

r
s
✍ M AMOUNI M Y I SMAIL MAMOUNI . NEW. FR
P ROBLÈMES C ORRIGÉS -MP

i
s

s
 

u
! √ 2 Z Z Z 1
∞ n . ∞
π − ψ(z)dz + ψ(z)dz = (c − a) ((1 − t)a + tc )dt + (b −

s
2 π 2 α2
2π 1 + 2 ∑ e−4n = 1 + 2 ∑ e 4α2  . γ a,c γc,b 0
n =1
α n =1 Z 1

é
r

u
a c) ((1 − t)c + tb)dt.
Soit a > 0, on applique la formule ci-dessus pour α = . On Z 0 Z
4π 1
(ca − | a|2 ) + (bc − |c|2 ) + (| c|2 − ac ) + (| b|2

r
obtient : ψ(z)dz + ψ(z)dz =
! ! 2

é
γ a,c γc,b
√ ∞
2
∞ πn2 Z
1 
a 1 + 2 ∑ e−πn a = 1 + 2 ∑ e− a . . ψ(z)dz = (ba − | a|2 ) + (| b|2 − ab ) .
2

Po r
n =1 n =1

r
γ a,b
Z Z Z
3ème
Partie : Un résultat général sur les fonctions ψ(z)dz + ψ(z)dz si et seulement si
ψ(z)dz =

u
γ a,c γc,b γ a,b
holomorphes   
(ca − | a| 2 ) + (bc − | c|2 ) + (| c| − ac ) + (| b|2 − cb) = (ba − | a|2 ) + (| b|2
2

uor
Pour tous a, b ∈ C, on pose : ∀t ∈ [0, 1] ; γa,b (t) = (1 − t)a + tb.
si et seulement si ca + bc − ac − cb = ba − ab
On note aussi : Ω = {z ∈ C tq Im(z) > 0}.
si et seulement si c(a − b) − (a − b)c = ba − ab
3.1) Ω est un demi plan, alors ∀ a, b ∈ Ω ; [ a, b] ⊂ Ω.
si et seulement si Im(c(a − b) − ba ) = 0

P
Ω est donc convexe, et par suite Ω est connexe par arcs.
λ + ba
si et seulement s’il existe un réel λ, tel que : c =
Soit f : Ω → C une application continue. Z a−
Zb
Z Z 1 L’ensemble des c ∈ Ω tels que : ψ(z)dz + ψ(z)dz =
2
On pose : ∀(a, b) ∈ Ω ; Φ(a, b) = f ((1 −

r
f (z)dz = (b − a) Z γ a,c γc,b
γ a,b 0
ψ(z)dz
t)a + tb)dt.

e
γ a,b
er
3.2) Soit a fixé dans Ω, et Φa l’application définie sur Ω par :
est une droite horizontale si (a − b) ∈ R et c’est une demi-droite

m
∀b ∈ Ω ; Φa (b) = Φ(a, b). h i
̥a ouverte si non.
f est continue, alors l’application (t, b) 7−→ f ((1 − t)a + tb) est
3.4) Dans la suite de cette partie, f est supposée holomorphe sur
continue sur [0, 1] × Ω, alors Ω, et si ( x, y) ∈ R2 tel que :
m

Z 1
̥a (t, b)dt définit une applicaton continue sur Ω. ( x + iy) ∈ Ω, on pose : P( x, y) = Re( f ( x + iy)) et Q( x, y) =

r
Φ a (b) =
0

o
3.3) Soient a, b, c ∈ Ω. Im( f ( x + iy)).
or

F
✉: mamouni.myismail@gmail.com

261
i

r
s
P ROBLÈMES C ORRIGÉS -MP ✍ M AMOUNI M Y I SMAIL MAMOUNI . NEW. FR

i
s

s
u
Z 1
∂P ∂Q +i (Re(c) − Re(a)) Q( x (t), y(t)) +

s
3.4.1) Les équations de Cauchy Riemann : = et
∂x ∂y 0
(ZIm(c) − Im(aZ)) P( x (t), y(t)))dt. Z

u
∂Q ∂P
=− f (z)dz = Pdx − Qdy + i Qdx + Pdy.
∂x ∂y γ a,c γ a,c γ a,c

r
3.4.2) Soit (a, b, c) ∈ Ω3 . D’aprés la formule de Green-Riemann on

é
aZ : D’où d’aprés les
Z formules (∗)Z et (∗∗) ci-dessus :
Z Z   Z Z
∂Q ∂P f (z)dz + f (z)dz + f (z)dz = 0 et f (z)dz =
Pdx − Qdy = − − dxdy = 0.

Po r
r
∂T + T ∂x ∂y γ a,c γc,b γb,a γ a,b
Z Z Z  
∂P ∂Q

u
Qdx + Pdy = − dxdy = 0. D’autre part, on a Z Z
∂T + T ∂x ∂y f (z)dz + f (z)dz.

uor
γ a,c γc,b
: Z 3.4.3)
 a est toujours fixé dans Ω. Soit b ∈ Ω et c ∈ Ω\{b}.
Z Z Z Z Z
Φ a (c ) − Φ a (b) 1
0= Qdx + Pdy = ± Qdx + Pdy + Qdx + Pdy + Qdx + Pdy . = ( f (z)dz − f (z)dz) =
∂T + γ a,c γc,b γb,a c−b c − b γa,c γ a,b

P
Z Z 1
1
f (z)dz = f ((1 − t)b + tc)dt.
(∗) c − b γb,c 0
Z Z Z Z 
L’application [(t, c) 7−→ f ((1 − t)b + tc)] est continue sur [0, 1] × Ω,
0= Pdx − Qdy = ± Pdx − Qdy + Pdx − Qdy + Pdx − Qdy .
∂T + γ a,c γc,b γb,a alors l’application

r
Z 1
[c 7−→ f ((1 − t)b + tc)dt] est continue sur Ω, donc

e
(∗∗) 
0
x = (1 − t)Re(a) + tRe(c)
er
γa,c est paramétré par : ; t ∈ [0, 1] ; Z 1
y = (1 − t)Im(a) + tIm(c) Φ a (c ) − Φ a (b)
lim f (b)dt = f (b).

m
 ′ =
x = Re(c) − Re(a) c→ b c−b 0
. 6=
y′ = Im(c) − Im(a) 3.4.4) On suppose que, pour tout b ∈ Ω, la fonction [r 7−→ Φ(ir, b],
Z Z 1
définie sur ]0, +∞[,
m

f (z)dz = (b − a) ( P( x (t), y(t)) + iQ( x (t), y(t))) dt


admet une limite dans C lorsque r tend vers 0+ . On note F(b) cette

r
γ a,c 0
Z Z 1
limite.
f (z)dz = (Re(c) − Re(a)) P( x (t), y(t)) − ( Im(c) − Im(a)) Q( x (t), y(Soit
t)))dt

o
γ a,c 0 (b, c) ∈ Ω2 et soit r > 0.
or

F
✉: mamouni.myismail@gmail.com

262
i

r
s
✍ M AMOUNI M Y I SMAIL MAMOUNI . NEW. FR
P ROBLÈMES C ORRIGÉS -MP

i
s

s
Z 

u
Z
i
F(c) − F(b) = lim (Φ(ir, c) − Φ(ir, b)) = lim f (z)dz + f (z)dz ∀r > 0 ; [t 7−→ ((1 − t)ir + tb) λ exp(−
= )] est con-

s
r →0
>
r →0
>
γir,c γb,ir ((1 − t)ir + tb)
Z tinue sur le segment [0, 1].

u
f (z)dz = Φ(b, c). i
γb,c ∀r > 0 ; ∀t ∈ [0, 1] ; exp(− ) =
Fixons b ∈ Ω et soit c ∈ Ω\{b}. ((1 − t)ir + tb)

r
Z 1 i (tb − (1 − t)ir ) tIm(b) + (1 − t)r
F (c ) − F (b)

é
= f ((1 − t)b + tc)dt, alors d’aprés la question exp(− 2
) = exp(− )≤1
c−b 0 (|(1 − t)ir + tb|) (|(1 − t)ir + tb|)2
F (c ) − F (b)

Po r
i

r
précèdente : lim = f ( b ). ((1 − t)ir + tb) λ exp(− ) ≤ ((1 − t)ir + tb) λ =
c→ b c−b (( 1 − t ) ir + tb )
6=

u
|(1 − t)ir + tb| λ

uor
On déduit alors que F est holomorphe sur Ω et que F ′ = f sur Ω. 
i
((1 − t)ir + tb) λ exp(− ) ≤ ((1 − t) r + tIm(b))2 + t2 Re(b
4ème Partie : Étude d′ un exemple ((1 − t)ir + tb)
λ λ
Soit λ un réel fixé dans l’intervalle ] − 1, 0[ ; et f λ la fonction définie < 0, alors [t 7−→ t 2 ] est décroissante sur R ∗+ , alors :

P
sur C \R − par : 2
i i | b|λ
f λ (z) = zλ exp(− ) ((1 − t)ir + tb) λ exp(− ) ≤ |b| λ tλ = −λ avec
z ((1 − t)ir + tb) t
−λ < 1,

r
i |b| λ
4.1) Les fonctions [z 7−→ zλ = exp(λ log(z))] et [z 7−→ − ] sont L’application [t 7−→ −λ ] est continue intégrable sur ]0, 1].
z t

e
holomorphes sur Ω, alors la i
∀t ∈]0, 1] ; lim ((1 − t)ir + tb) λ exp(− ) =
er
r →0
>
((1 − t)ir + tb)

m
fonction f λ est produit deux fonctions holomorphes sur Ω, f λ est −i
donc holomorphe sur Ω. bλ tλ exp( ).
tb
4.2)
Z Soit b un complexe fixé dans Ω. On pose : ∀r > 0 ; Jλ,b (r ) =
−i
En plus l’application [t 7−→ bλ tλ exp( )] est continue sur ]0, 1].
m

f λ (z)dz. tb
Alors d’aprés le théorème de convergence dominée, on a :

r
γir,b Z 1 Z 1
Z 1 λ i −i
i lim ((1 − t)ir + tb) exp(− )dt = bλ tλ exp( )dt.
Jλ,b (r ) = (b − ir ) ((1 − t)ir + tb) λ exp(− )dt.

o
0 ((1 − t)ir + tb) r →0 0
>
((1 − t)ir + tb) 0 tb
or

F
✉: mamouni.myismail@gmail.com

263
i

r
s
P ROBLÈMES C ORRIGÉS -MP ✍ M AMOUNI M Y I SMAIL
MAMOUNI . NEW. FR

i
s

s
u
Z 1 Z +∞
−i −iv
D’où lim Jλ,b (r ) = Fλ (b) = b λ+1 λ
t exp( )dt. Gλ ( z ) = − i + i ( λ + 2 ) (1 + v)−λ−3 exp( )dv.

s
r →0 0 tb 0 z
>
−iv

é
Im(z) > 0 et v > 0, alors exp( ) ≤ 1.

u
4.3) On note Gλ la fonction, définie pour tout z ∈ Ω, par Gλ (z) = z
i
z−λ−2 exp( ) Fλ (z). Z +∞

r
z | Gλ (z)| ≤ 1 + ( λ + 2) (1 + v)−λ−3 dv = 1 +

é
4.3.1) Fλ est holomorphe sur Ω et Fλ′ = f λ d’aprés 3.4.4), Gλ est 0
produit de trois fonctions h i+∞

Po r
r
−(1 + v)−λ−2 = 2.
holomorphes sur Ω, alors Gλ est aussi holomorphe sur Ω. 0
Z 1
3  

u
i −i −i
4.3.2) ∀z ∈ Ω ; Gλ (z) = z−λ−2 exp( )zλ+1 tλ exp( )dt = 2
D’aprés 4.3) et l’étape précèdente : F 1 (z) = G 1 (z) |z| exp ≤
z tz

uor
0 − − z
1 i
Z 1
− i 2 2
exp( ) tλ exp( )dt. 3
z z 0 tz 2 |z| 2 .
1 −du
On pose : u = ; dt = . ∀ z ∈ Ω ; Gλ ( z ) = 5ème Partie : Démonstration de la propriété proposée

P
Z +∞ t u2 Pour tout entier naturel non nul n, on note un la fonction définie,
1 i −iu
exp( ) u−λ−2 exp( )du. pour tout z ∈ C, par :
z z 1 Z z Z
1 + ∞ − λ−2 −i ( u − 1) 1 +∞
4.3.3) Gλ (z) = u exp( )du = (1 + un (z) = exp(iπn2 z)

r
z 1 z z 0
−iv
v)−λ−2 exp( )dv

e
z
5.1) Soient z ∈ C et n ∈ N ∗ . |un (z)| = exp(−πn2 Im(z)).
−iv −ivz
er
( où l’on a posé v = u − 1 ) ; exp( ) = exp( 2 ) = Si Im(z) ≤ 0 alors la série ∑ un (z) diverge grossièrement.
z  

m
|z| 1 
−vIm(z) Si Im(z) > 0 alors |un (z)| = O 2
alors ∑ un (z) converge
exp( )≤1 n→∞ n
| z|2 absolument.
  Z +∞
m

− λ−2 −iv +∞ 
Gλ ( z ) = i (1 + v ) exp( ) + i ( λ + 2) (1 +

r
z Finalement la série ∑ un (z) converge si et seulement si z ∈ Ω.
0 0
−iv 5.2) Séparons les indices paires et impaires dans la somme suivante
v)−λ−3 exp(

o
)dv :
z
or

F
✉: mamouni.myismail@gmail.com

264
i

r
s
✍ M AMOUNI M Y I SMAIL
MAMOUNI . NEW. FR
P ROBLÈMES C ORRIGÉS -MP

i
s

s
u
∞ ∞
2 D’où en appliquant à y fixé le théorème de dérivation sous le signe
u ( z + 1) = ∑ exp(iπn2 (z + 1)) = ∑ (−1)n exp(iπn2 z) =

s
∂ue

n =1

n =1
∑, la fonction ∂x (x, y)

u
2 2
∑ exp(iπ4n z) − ∑ exp(iπ (2n − 1) z)
existe en tout point de R ×]0, +∞[, avec
∂ue
( x, y) =
n =1 n =1 ∂x

r
iπ ∑ n2 exp(−πn2 y + iπn2 x ).
u(z + 1) = u(4z) − u(z) + ∑ exp(iπ4n2 z). D’où : ∀z ∈ Ω ;

é
n =1 n ≥1
5.3.3) ∀n ∈ N ∗ ; uen est de classe C1 sur R ×]0, +∞[.

Po r
r
u(z + 1) + u(z) = 2u(4z). En effet : ∀( x, y) ∈ R ×]0, +∞[ ; uen ( x, y) = exp(−πn2 y + iπn2 x ).
5.3) Pour tout n ∈ N ∗ et tout ( x, y) ∈ R ×]0, +∞[, on pose : ∂uen ∂uen
( x, y) = −πn2 exp(−πn2 y + iπn2 x ) et

u
uen ( x, y) = un ( x + iy) et ue( x, y) = u( x + iy). ( x, y) =
∂y ∂x

uor
πn2 exp(−πn2 y).
5.3.1) Soient a > 0 et k ∈ N, Soit ( x, y) ∈ R × [ a, +∞[. !
∀n ∈ N ∗ ; nk uen ( x, y) = nk exp(−πn2 y) ≤ nk exp(−πn2 a). ∂uen
La série ∑ ( x, y) converge normalement sur R × [ a, +∞[
    n ≥1
∂y
k 2 1 k 2
n exp(−πn a) = O alors la série ∑ n exp(−πn a) pour tout a > 0.

P
n → ∞ n2
converge et D’où en appliquant à x fixé le théorème de dérivation sous le signe
! par suite la série ∂ue
∑, la fonction ∂y (x, y)
∑ nk uen converge normalement sur R × [ a, +∞[ pour tout a >

r
n ≥1

0. ∂ue
existe en tout point de R ×]0, +∞[, avec

e
( x, y) =
5.3.2) ∀n ∈ N ∗ ; uen est de classe C1 sur R ×]0, +∞[. ∂y
er
En effet : ∀( x, y) ∈ R ×]0, +∞[ ; uen ( x, y) = exp(−πn2 y + iπn2 x ). ∂ue
−π ∑ n2 exp(−πn2 y + iπn2 x ) = i ( x, y).

m
∂uen ∂uen n ≥1
∂x
( x, y) = iπn2 exp(−πn2 y + iπn2 x ) et ( x, y) =
∂x ∂x ∂ue
5.3.4) ∀( x, y) ∈ R ×]0, +∞[ ; ( x, y) = iπ ∑ n2 exp(−πn2 y +
πn2 exp(−πn2 y). ∂x
m

! n ≥1
∂uen iπn2 x ).

r
La série ∑ ( x, y) converge normalement sur R × [ a, +∞[
n ≥1
∂x Cette série converge normalement sur R × [ a, +∞[ pour tout a > 0,

o
pour tout a > 0. et pour tout n ≥ 1, la
or

F
✉: mamouni.myismail@gmail.com

265
i

r
s
P ROBLÈMES C ORRIGÉS -MP ✍ M AMOUNI M Y I SMAIL
MAMOUNI . NEW. FR

i
s

s
h i

u
fonction ( x, y) 7−→ n2 exp(−πn2 y + iπn2 x ) est continue sur D’où d’aprés le principe des zéros isolés, h est nulle Ω, ce qui établit

s
R ×]0, +∞[. :
 − 1

é
∂ue

u
est donc continue sur R ×]0, +∞[ et d’aprés la question précè- i 2 −1
∂x ∀z ∈ Ω ; (1 + 2u(z)) = 1 + 2u( )
∂ue z z

r
dente, est aussi continue

é
∂y
sur R ×]0, +∞[, alors ue est de classe C1 sur R ×]0, +∞[ et vérifie : 1 1
∂ue ∂ue 5.5) D’aprés 5.2), u(z + 1) + = 2u(4z) − u(z) + = (2u(4z) +

Po r
r
( x, y) + i ( x, y) = 0 2 2
∂x ∂y 1
1) − (2u(z) + 1).

u
2
D’où u est holomorphe sur Ω. Alors en appliquant la question précèdente :

uor
i  1  1
5.4) Soit z ∈ Ω, on a bien ∈ / R − car Im(z) > 0. 1 i 2 −1 1 i 2 −1
z u ( z + 1) + = (1 + 2u( )) − (1 + 2u( ))
2 4z 4z 2 z z
( Dans cette question, une faute de frappe à l’énoncé, un signe -
oublié )  1    1 

P
 − 1  − 1 ! 1 i 2 −1 −1 i 2 −iπn2
i 2 i 2 u ( z + 1) + = u( ) − u( ) = ∑ exp( ) − exp(
(1 + 2u(z)) = 1 + 2 ∑ exp(iπn2 z) et 1 + 2 z 4z z z n ≥1
4z
z z n ≥1  
un (z) 1
5.6) ∀n ≥ 1 ; ∀z ∈ C ; Im(z) ≥ 0 =⇒ |un (z)| ≤ 1 =⇒ ≤ 2 .

r
2
−1 −iπn2   iπn n
2u( ) = 1 + 2 ∑ exp( ). u n
z z Alors la série ∑ converge normalement sur {z ∈ C tq

e
n ≥1 2
D’aprés la question précèdente : n ≥1 iπn
er
 − 1  − 1 Im!(z) ≥ 0}.

m
i 2 −1 i 2
h(z) = (1 + 2u(z)) − (1 + 2u( )) = 1 + 2 ∑ exp(iπn2 zEn − : ∀n ≥ 1 ; l’application [z 7−→ un (z) ] est continue sur
) plus
z
!
z z n ≥1 iπn2
{z ∈ C tq Im(z) ≥ 0}.
−iπn2
m

1 + 2 ∑ exp( ) .  
z un (z)

r
n ≥1 D’où v(z) = ∑ définit une application continue sur cet
définit une fonction holomorphe sur l’ouvert connexe par arcs Ω. n ≥1 iπn2

o
D’aprés la formule (2), si z = ia avec a > 0, alors h(z) = 0. ensemble.
or

F
✉: mamouni.myismail@gmail.com

266
i

r
s
✍ M AMOUNI M Y I SMAIL
MAMOUNI . NEW. FR
P ROBLÈMES C ORRIGÉS -MP

i
s

s
u
3 1
5.7) D’aprés 4.3.3) : ∀z ∈ Ω ; F 1 (z) ≤ 2 |z| 2 . alors : (iπ ) 2 4 4z − 1 −iπn2 2 z −1 −iπn2

s


w (z) = ∑ ( 2 ) 2 exp( )− ( 2 ) 2 exp( )
2 2 n≥1 πn πn 4z πn πn z

u
αz αz 1 !
∀z ∈ Ω ; ∀α > 0 ; ∀n ≥ 1 ; ∈ Ω et donc : nF 1 ( 2 ) ≤ 2 1 2 1 2
πn 2 ( iπ ) 2 − − iπn 2 − − iπn
2 πn

w′ (z) = √ z 2 exp( ) − √ z 2 exp( ) .
2 n∑

r
3 ≥1 π 4z π z
2 α |z| 3

é
αz 2
2n = ( )2 .  1  
πn2 n2 π ! ′ i 2 −iπn2 −iπn2 1
w (z) = ∑ exp( ) − exp( ) = u (1 + z ) + .

Po r
r
αz z n ≥1
4z z 2
∀z ∈ Ω ; ∀α > 0 ; ∀n ≥ 1 ; La série ∑ nF 1 ( 2 ) converge
2 πn

u
n ≥1
absolument, donc converge. ( selon 5.5 )

uor
5.8) Pour tout z ∈ Ω, on pose : 5.8.3) Posons : ∀z ∈ Ω ; w1 (z) = v1 (z + 1) − v(1). w1 est holomor-
1 ! phe sur l’ouvert connexe
un (z) (iπ ) 2 4z z 1
v1 ( z ) = ∑ et w(z) = nF 1 ( 2 ) − 2nF 1 ( 2 ) . par arcs Ω, et ∀z ∈ Ω ; w1′ (z) = v1′ (z + 1) = u(z + 1) = w′ (z) − .
n ≥1 iπn
2 2 n∑ 2
2 πn 2 πn
− −
≥1 z

P
Alors l’application [z 7−→ w1 (z) − w(z) + ] est constante sur Ω.
2 !
5.8.1) On reprend un raisonnement identique à celui de 5.3) pour
4z z
établir que v1 est D’aprés 4.3.3) la série ∑ nF 1 ( 2 ) − 2nF 1 ( 2 ) converge
2 πn 2 πn
− −
holomorphe sur Ω. n ≥1

r
∂ev1 normalement pour z ∈ Ω,
∀( x, y) ∈ R ×]0, +∞[ ; v1′ ( x + iy) = ( x, y) = |z| ≤ 1, on peut alors intervertir les signe ∑ et lim , la constante
∂x z →0

e
1 ∂uen n ≥1 z ∈Ω
∑ iπn2 ∂x (x, y) = ∑ uen (x, y) = u(x + iy). cherchée est alors nulle !
er
n ≥1 n ≥1

m
z
D’où : ∀z ∈ Ω ; v1′ (z) = u(z), c’est à dire : v1′ = u. D’où : ∀z ∈ Ω ; w1 (z) − w(z) + = 0 ; c’est à dire :
2
5.8.2) Vu ce qui est admis dans l’énoncé : pour tout z ∈ Ω, on a : 1 !
1 ! z (iπ ) 2 4z z
m

∀ z ∈ Ω ; v 1 ( z + 1 ) − v (1 ) = − + nF 1 ( 2 ) − 2nF 1 ( 2 )
( iπ ) 2 4 4z 2 z 2 2 n∑

r
2 πn 2 πn
− −
w′ (z) = F ′1( 2) − F ′ 1 ( 2 ) . Utilisons
2 n∑
≥1
πn − πn πn − πn
≥1 2 2

o
4.3.1) : 5.9) D’aprés la question précèdente, pour tout z ∈ Ω, on a :
or

F
✉: mamouni.myismail@gmail.com

267
i

r
s
P ROBLÈMES C ORRIGÉS -MP ✍ M AMOUNI M Y I SMAIL
MAMOUNI . NEW. FR

i
s

s
!

u

z π 4z z 5.10) Soient x ∈ R et y > 0, d’aprés la question précèdente :

s
v ( z + 1 ) − v (1 ) + = ∑ nF 1 ( 2 ) − 2nF 1 ( 2 ) .
2 2 x + iy 3
2 πn 2 πn
− −
n ≥1
v( x + iy + 1) − v(1) + ≤ c | x + iy| 2 . On applique 5.6), on

é
2

u
4z z
et selon 4.3.3), ∀n ≥ 1 ; nF 1 ( 2 ) − 2nF 1 ( 2 ) ≤ fixe x, et on fait tendre y vers 0.
2 πn 2 πn
− −
3

r
3 3 x
3 θ On obtient : ∀ x ∈ R ; q( x + 1) − q(1) + ≤ c | x | 2 , d’où

é
4z 2 z 2 ! 2
2n + 4n = | z | 2 .  
πn2 πn2 n2 x 3
q ( x + 1 ) − q (1 ) + = O | x| 2 .

Po r
où θ est un réel strictement positif √ indépendant de z.

r
2 x →0
θ π 1
On pose alors : c = ∑ ∈ R ∗+ . ∀z ∈ Ω ; x

u
2 n ≥1 n 2 Finalement : q( x + 1) = q(1) − + o ( x ) .
2 x →0

uor
z 3 1
v ( z + 1 ) − v (1 ) + ≤ c |z| 2 . On déduit alors que q est dérivable en 1 et q′ (1) = − .
2 2

i
F

P
n
i

r
Á la prochaine

e
er

m
m

or
or

F
✉: mamouni.myismail@gmail.com

268
i

r
s
✍ M AMOUNI M Y I SMAIL
MAMOUNI . NEW. FR
P ROBLÈMES C ORRIGÉS -MP

i
s

s
u

s
Devoir Libre
28 Formule de Green-Rieman & Intégrale de Dirichlet

u
George Green (1793-1841)

Mathématicien du jour
Blague du jour
Physicien britannique, auteur d’un Essai sur l’application de l’-
On raconte qu’une secrétaire d’une société dont je tairais analyse mathématique aux théories de l’électricité et du mag-

Po r
r
le nom avait insérer le cd dans son pc avec la pochette en nétisme en introduisant plusieurs concepts importants, parmi
lesquels les fonctions potentielles. Sa particularité est qu’il était

u
plastique transparent.
Elle pensait que ça protégeait des virus ... presque totalement autodidacte, il n’a passé qu’un an environ à

uor
l’école, entre 8 et 9 ans et a travaillé longtemps dans le moulin
qu’il a hérité de son père.

P
Suite Il l’intégra l’université de Cambridge comme étudiant à l’âge de 40 ans. Il écrivit des publications dans le domaine de l’optique, de
l’acoustique et de l’hydrodynamique. C’est surtout à Lord Kelvin, qui le fit connaître, qu’il doit sa renommée.

des t croissants. Z
Énoncé : e3a 2009, PSI

r
On rappelle que la circulation de V le long de γ, notée V se cal-
γ
cule par la formule

e
Applications simples du cours. Z Z
V= ( P( x, y) dx + Q( x, y) dy)
er
γ γ

m
Rappels. On suppose P et Q de classe C1 sur U. L’arc γ est supposé fermé,
2
Soit I = [ a, b] (avec a < b) un intervallede R et U un
 ouvert de R . sans point double et parcouru dans le sens trigonométrique. Il
P( x, y) délimite un domaine G d’un seul tenant, inclus dans U.
On considère V : ( x, y) ∈ U 7→ un champ de
m

Q( x, y) On rappelle la formule de Green-Riemann : 


ZZ 

r
vecteurs Z
  et γ un arc orienté plan de paramétrage : t ∈ I 7→ ∂Q ∂P
x (t) V= ( x, y) − ( x, y) dxdy
, de classe C1 par morceaux sur I, parcouru dans le sens ∂x ∂y

o
γ G
y(t)
or

F
✉: mamouni.myismail@gmail.com

269
i

r
s
P ROBLÈMES C ORRIGÉS -MP ✍ M AMOUNI M Y I SMAIL
MAMOUNI . NEW. FR

i
s

s
u
1. Dans cette question seulement, on prend G = {( x, y) ∈ dans un repère orthonormé (O,~i,~j) direct.

s
R 2 , y ≥ x2 et y2 ≤ x } et pour γ l’arc frontière délimitant On précisera les tangentes aux points singuliers.
ce domaine, parcouru dans le sens trigonométrique.

é
3.3. Déterminer l’aire délimitée par la courbe γ.

u
1.1. Représenter le domaine G et γ.
Z

r
1.2. Calculer directement, en paramétrant l’arc : V avec Problème.

é
γ
P : ( x, y) 7→ 2xy − x2 et Q : ( x, y) 7→ x + y2
Préliminaires.

Po r
r
1.3. Retrouver le résultat précédent en utilisant la formule
de Green-Riemann. 1. Illustrer graphiquement la double inégalité : ∀t ∈

u
π 2
2. On suppose que les deux fonctions P et Q vérifient : pour [0, ], t ≤ sin(t) ≤ t.
2 π

uor
∂Q ∂P
tout ( x, y) ∈ R 2 , ( x, y) = ( x, y). Z +∞
sin(t)
∂x ∂y 2. On veut montrer que l’intégrale dt est conver-
Z 0 t
2.1. Que vaut V ? gente.
γ sin(t)

P
2.2. Donner un exemple de champ de vecteur V, non iden- On pose alors, pour tout t > 0, ϕ(t) = .
t
tiquement nul, et vérifiant la propriété : 2.1. Vérifier que ϕ est prolongeable par continuité sur l’inter-
∂Q ∂P
∀( x, y) ∈ R 2 , ( x, y) = ( x, y) valle [0, 1].
∂x ∂y Z x

r
3. 2.2. Pour tout x > 1, on définit φ : x 7→ φ( x ) = ϕ(t) dt.
1

e
3.1. Démontrer
Z
que les intégrales
Z
curvilignes suivantes
Z
: cos( x )
1 Montrer que l’on a : ∀ x > 1, φ( x ) = cos(1) − −
er
A1 = x dy, A2 = − y dx et A3 = ( x dy − Z x x
2 γ cos(t)

m
γ γ
dt.
y dx ) sont égales. Interpréter géométriquement le résul- 1 t2
tat obtenu. 2.3. Prouver que φ( x ) tend vers une limite finie quand x tend
3.2. Représenter graphiquement l’arc orienté γ d’équations
m

vers +∞.
Z +∞

r
paramétriques   sin(t)
x (t) = cos3 (t) 2.4. Déduire de ces résultat que l’intégrale dt est
t ∈ [−π, π ] 7→ 0 t

o
y(t) = sin3 (t) convergente.
or

F
✉: mamouni.myismail@gmail.com

270
i

r
s
✍ M AMOUNI M Y I SMAILMAMOUNI . NEW. FR
P ROBLÈMES C ORRIGÉS -MP

i
s

s
u
Z +∞ γ1 : le segment [ A1 , A2 ] où A1 = (r, 0)et A2 = ( R, 0),
sin(t)

s
Une première façon de calculer I = dt. Soient
0 t γ2 : le quart de cercle de centre O et de rayon R reliant
A2 à A3 = (0, R),

é
les deux fonctions :

u
e−y γ3 : le segment [ A3 , A4 ] où A4 = (0, r ),
P : ( x, y) 7→ P( x, y) = [ x sin( x ) − y cos( x )]
x 2 + y2 γ4 : le quart de cercle de cercle de centre O et de rayon r

r
e−y reliant A4 à A1 .

é
Q : ( x, y) 7→ Q( x, y) = 2 [ x cos( x ) + y sin( x )]
x + y2  4.1. Représenter graphiquement l’arc orienté γ dans un

Po r
P( x, y) repère orthonormé (O,~i,~j) direct.

r
et V le champ de vecteurs : ( x, y) 7→ .
Q( x, y) Z Z R
sin(t)

u
4.2. Montrer que V= dt.
1. Justifier le fait que P et Q sont deux fonctions de classe C1 sur γ1 r t

uor
Z
tout domaine U de R 2 ne cotenant pas l’origine.
4.3. Montrer que V dt = 0.
2. Soit γ un arc paramétré sans point double, n’entourant pas γ1
l’origine Zet parcouru dans le sens trigonométrique. Démon- 4.4. En utilisant les résultats précédents, déterminer la
Z +∞
trer que V = 0. sin(t)
valeur de I =

P
γ
dt.
0 t
3. On considèreΓ l’arc de cercle derayon ρ > 0 paramétré par Z +∞
sin(t)
π x (θ ) = ρ cos(θ ) Une deuxième façon de calculer I = dt. On
θ ∈ [0, ] 7→ et on note Aρ l’intégrale 0 t
2 y(θ )Z= ρ sin(θ ) Z π

r
Aρ = ( P( x, y) dx + Q( x, y) dy) pose, pour tout n ∈ N ∗ , un = 2 cos(t) sin (2nt) dt et v =
n
Γ 0 sin(t)

e
Z π/2 Z π
3.1. Montrer que Aρ = − ρ sin(θ )
cos(ρ cos(θ )) dθ. 2 sin(2nt) dt.
er
e
0 0 t

m
3.2. Calculer lim Aρ . 1.
ρ → 0+
1.1. Vérifier que un et vn existent pour tout n ∈ N ∗ .
3.3. Montrer que lim Aρ = 0. On pourra, par exemple,
1.2. En calculant un+1 − un , montrer que un est indépen-
m

ρ→+ ∞
utiliser les préliminaires. dante de n et donner sa valeur.

r
4. Soient r et R deux réels tels que 0 < r < R. On considère l’arc 2. Soit h une fonction de classe C1 sur un segment [α, β] à valeurs

o
γ constitué par : dans R.
or

F
✉: mamouni.myismail@gmail.com

271
i

r
s
P ROBLÈMES C ORRIGÉS -MP ✍ M AMOUNI M Y I SMAIL
MAMOUNI . NEW. FR

i
s

s
u
Z β
2. En utilisant l’intégrale J, montrer que
On pose, pour tout m ∈ N, Hm = h(t)eimt dt.

s
Z u Z u
α sin( x ) ux 1 − eyu (cos(u) + y sin(u))
Montrer, en utilisant une intégration par parties, que (1 − e ) dx = dy
x 1 + y2

é
0 0

u
lim Hm = 0. Z u
m→+ ∞ sin( x )
Ce résultat est connu sous le nom de Lemme de Riemann- 3. On note alors K1 = e− xu dx et K2 =

r
0 x
Lebesgue. Z u

é
y sin(u) + cos(u) −yu
1 cos(t) 2
e dy.
3. Montrer que la fonction h : t 7→ h(t) = − est pro- 0 1+y
t sin(t)

Po r
r
π 3.1. Prouver que lim K1 = 0.
longeable en une fonction de classe C1 sur [0, ]. u →+ ∞
2

u
4. 3.2. En utilisant une majoration, déterminer lim K2 .
u →+ ∞

uor
4.1. Calculer lim (vn − un ). Z +∞
sin(t)
n →+ ∞
Z +∞ 3.3. Retrouver alors la valeur de I = dt.
sin(t) 0 t
4.2. En déduire la valeur de I = dt.
0 t 4.

P
Z +∞ Z +∞
sin(t) sin2 (t)
Une troisième façon de calculer I = dt. Soit u ∈ 4.1. Montrer que l’intégrale dt est convergente.
ZZ
0 t t2
0

R +∗ . On note ∆ = [0, u] × [0, u] et J = sin( x )e− xy dxdy. 4.2. En utilisant ce qui précède, calculer la valeur de
Z +∞
∆ sin2 (t)

r
1. Donner une primitive sur R de x 7→ sin( x )e−αx où α ∈ R ∗ . dt.
0 t2

e
er
i
F

m
n
i
n
m

r
Á la prochaine

o
or

F
✉: mamouni.myismail@gmail.com

272
i

r
s
✍ M AMOUNI M Y I SMAIL
MAMOUNI . NEW. FR
P ROBLÈMES C ORRIGÉS -MP

i
s

s
u
2.2. Soit f : ( x, y) 7→ cos( xy). f étant de classe C 2 sur R 2 , on a
Corrigé : Pr. Devulder, CPGE France

s
∂2 f ∂2 f
∀( x, y) ∈ R 2 , ( x, y) = ( x, y)

é
∂x∂y ∂y∂x

u
Applications simples du cours. ∂f ∂f
En posant P( x, y) = ( x, y) = −y sin( xy) et Q = ( x, y) =
∂x ∂y

r
1.1. G est l’intersection de "l’intérieur" de deux paraboles.
− x sin( xy) on a donc la relation voulue.

é
3.1. En utilisant la formule de
Z Green-Riemann
ZZ on a

Po r
r
A1 = x dy = dxdy
γ
Z ZZ

u
A2 = − y dx = dxdy
γ

uor
Z ZZ
1
A3 = ( x dy − y dx ) = dxdy
1.2. On peut paramétrer γ par 2 γ
 et ces trois quantités sont égales à l’aire délimitée par γ.
(t2 , −t) si t ∈ [−1, 0]
( x (t), y(t)) =
(t, t2 ) si t ∈]0, 1] 3.2. Notons M(t) = ( x (t), y(t)) = (cos3 (t), sin3 (t) ; on a

P
On en déduit que M(t + 2π ) = M(t), M(t + π ) = sym0 ( M(t)), M(−t) = sym(Ox) ( M(t)
Z Z 0   Z 1  1 donc faire une étude sur [0, π/4] puis une symétrie
V= 3 4 2
(−2t − t )2t) + 2t (−1) dt + (2t − t ) + (t + t )(2t) On
3 2 4
dt =peut
γ −1 0 30 à la première bissectrice (courbe sur [0, π/2]) puis
par rapport
1.3. Si on utilise la formule deZZGreen-Riemann, on obtient une symétrie d’axe Ox (courbe sur [−π/2, π/2]) puis une

r
Z
V= (1 − 2x ) dxdy symétrie par rapport à l’origine (courbe que [−π/2, 3π/2] et

e
γ G √ donc en entier puisqu’il y a 2π-périodicité). On a
2
On a G = {( x, y)/ x ∈ [0, 1], y ∈ [ x , x ]}. Par théorème de ∀t ∈ [0, π/4], x ′ (t) = −3 sin(t) cos2 (t) ≤ 0, y′ (t) = 3 cos(t) sin2 (t) ≥
er
Fubini (appliqué à une fonction continue sur un compact) on et x décroît sur [0, π/4] alors que y croît. Le seul point d’an-

m
a donc ! nulation commun est en 0 (point stationnaire). Comme
Z Z 1 Z √x Z 1 √  3
1 t2 3 2
V= (1 − 2x ) dy dx = (1 − 2x )( x − x2 ) dx = x ( t ) = 1 − + o ( t 2
) = 1 − t + o0 ( t2 )
0 2 0 30 2
0
2
m

γ x
Z
y(t) = (t + o0 (t))3 = o0 (t2 )

r
2.1. D’après la formule de Green-Riemann, on V = 0 dans le
γ la tangente en M(0) est portée par ( x ′′ (0), y′′ (0)) = (−3, 0) et

o
cas envisagé. est horizontale. D’après les symétries détectées, on a un point
or

F
✉: mamouni.myismail@gmail.com

273
i

r
s
P ROBLÈMES C ORRIGÉS -MP ✍ M AMOUNI M Y I SMAIL
MAMOUNI . NEW. FR

i
s

s
u
de rebroussement de première espèce. On obtient la courbe cos( x ) 1 cos(t)
2.3. ≤ est de limite nulle quand x → +∞. t 7→

s
suivante x x t2
est continue sur [1, +∞[ et dominée par 1/t2 ; elle est donc

u
intégrable sur [1, +∞[. A fortiori, sont intégrale entre 1 et x
admet une limite quand x → +∞. On peut donc écrire que

r
Z +∞
cos(t)

é
lim φ( x ) = cos(1) − dt
x →+ ∞ 1 t2
Z x Z 1
sin(t)

Po r
r
3.3. On peut utiliser l’une des formules de 3.1 pour obtenir l’aire. 2.4. Pour tout x > 0, on a dt = ϕ + φ( x ) admet une
0 t 0
Elle vaut limite quand x → +∞ et on peut écrire que

u
Z Z 2π Z +∞ Z 1 Z +∞
1 2π 3 sin(t) sin(t) cos(t)
(3 cos4 (t) sin2 (t) + 3 sin4 (t) cos2 (t)) dt = cos2 (t) sin2 (t) dt

uor
A= dt = dt + cos(1) − dt
2 0 2 0 0 t 0 t 1 t2
AVec la formule
Z
du sinus double, Z
3 2π 2 3 2π 3π Z +∞
sin(t)
A= sin (2t) dt = (1 − cos(4t)) dt = Une première façon de calculer I = dt.
8 0 16 0 8 t
0

P
Problème. Préliminaires.
1. Par théorèmes généraux, P et Q sont de classe C ∞ sur l’ouvert
1. La fonction t 7→ sin(t) est concave sur [0, π/2] (sa dérivée R 2 \ {(0, 0)} et donc, en particulier, de classe C 1 sur tout do-
seconde, − sin, est négative sur cet intervalle). Son graphe
maine U (ouvert selon le programme) de R 2 qui ne contient
est donc situé au dessus de la corde [(0, 0), (π/2, 1)] et en

r
pas (0, 0).
dessous de la première bissectrice (tangente à l’origine). Ceci

e
s’écrit h πi 2 2. Avec les hypothèses faites, on peut utiliser le théorème de
Green-Riemann. Or, un calcul donne
er
∀t ∈ 0, , t ≤ sin(t) ≤ t
2 π ∂Q ∂P

m
2.1. ϕ est continue sur ]0, 1] et de limite 1 en 0. Elle est donc pro- ( x, y) = ( x, y)
∂x ∂y
longeable par continuité en 0 (avec ϕ(0) = 1). e−y 
1
2.2. Une intégration par parties (les fonctions étant C sur [1, +∞[, = 2 2 2
(− x2 + y2 + yx2 + y3 ) cos( x ) − ( x3 + xy2 + 2x
m

(x + y )
celle-ci est licite)donne  On en déduit donc que Z

r
Z x Z x
cos(t) x cos(t) cos( x ) cos(t)
∀ x ≥ 1, φ( x ) = − − dt = cos(1) − − dt V=0
t2 t2

o
t 1 1 x 1 γ
or

F
✉: mamouni.myismail@gmail.com

274
i

r
s
✍ M AMOUNI M Y I SMAIL
MAMOUNI . NEW. FR
P ROBLÈMES C ORRIGÉS -MP

i
s

s
u
3.1. On a On en déduit que

s
e − y(θ ) Z π/2
" #π/2
P( x (θ ), y(θ )) = (sin( x (θ )) cos (θ ) − cos( x (θ )) sin (θ )) 2ρ π − 2ρ θ π
− θ
ρ | Aρ | ≤ e π dθ = − e π ≤ → 0

é
2ρ 2ρ ρ→+∞

u
0
e − y(θ ) 0
P( x (θ ), y(θ )) = (cos( x (θ )) cos (θ ) + sin( x (θ )) sin (θ ))
ρ 4.1. Le dessin est fait ici avec r = 1 et R = 3.

r
On multiplie la première quantité par −ρ sin(θ ) et la seconde

é
par ρ cos(θ ) pour obtenir
Z π/2 −y(θ ) Z π/2
e

Po r
r
Aρ = cos( x (θ )) dθ = e−ρ sin(θ ) cos(ρ cos(θ )) dθ
0 ρ 0

u
3.2. On veut utiliser le théorème de continuité des intégrales à

uor
paramètres.
4.2. γ1 est paramétré par t ∈ [r, R] 7→ (t, 0) et ainsi
Z Z R Z R
- +
∀ρ ∈ R , θ 7→ e − ρ sin(θ )
cos(ρ cos(θ )) est continue sur sin(t)
V= P(t, 0) dt = dt
[0, π/2]. γ1 r r t

P
4.3. γ3 est paramétré par t ∈ [− R, −r, ] 7→ (0, −t) et ainsi
- ∀θ ∈ [0, π/2], ρ 7→ e−ρ sin(θ ) cos(ρ cos(θ )) est continue Z Z −r
sur R + . V=− Q(0, −t) dt = 0
γ1 −R
Z
- ∀ρ ≥ 0, ∀θ ∈ [0, π/2], e−ρ sin(θ ) cos(ρ cos(θ )) ≤ 1. Le
4.4. D’après la question 2, V = 0. Par ailleurs (en prenant garde

r
majorant est une fonction intégrable sur [0, π/2] (con- Γ
au fait que γ4 estZ parcouru dansZle sens horaire)
tinue sur ce segment).

e
V = A R et V = − Ar
er
Le théorème s’applique et indique que ρ 7→ Aρ est continue γ2 γ4

m
Avec les deux questions précédentes, on a donc
sur R + . En particulier, Z R
sin(t)
Z π/2
π 0= dt + A R − Ar
lim Aρ = A0 = dθ = r t
ρ → 0+ 0 2 Il reste à faire tendre r vers 0+ et R vers +∞ (on a vu l’exis-
m

tence des limites) pour obtenir

r
3.3. En utilisant la première question des préliminaires, on a Z +∞
h πi 2ρ sin(t) π
dt =
, e−ρ sin(θ ) cos(ρ cos(θ )) ≤ e−ρ sin(θ ) ≤ e− π θ

o
∀θ ∈ 0, 0 t 2
2
or

F
✉: mamouni.myismail@gmail.com

275
i

r
s
P ROBLÈMES C ORRIGÉS -MP ✍ M AMOUNI M Y I SMAIL
MAMOUNI . NEW. FR

i
s

s
u
Z +∞ alors immédiatement
sin(t)

s
Une deuxième façon de calculer I = dt. lim Hm = 0
0 t m→+ ∞

u
cos(t) sin (2nt) sin(2nt) 3. h est une fonction continue sur [0, π/2] et
1.1. f n : t 7→ et gn : t 7→ sont con-
sin(t) t sin(t) − t cos(t) (t + o(t2 )) − (t + o(t2 )) o (1 )

r
tinues sur ]0, π/2] et prolongeables par continuité en 0 (par h(t) = = 2 2
=

é
t sin(t) t + o (t ) 1 + o (1 )
f n (0) = 2n et gn (0) = 2n). Ce sont donc des fonctions inté- On en déduit que
grables sur le segment [0, π/2] et un et vn existent a fortiori. lim h(t) = 0

Po r
r
t →0
a+b a−b et h est prolongable par continuité en posant h(0) = 0. On
1.2. La formule sin(a) − sin(b) = 2 cos( ) sin( ) indique

u
2 2 a alors une fonction continue sur [0, 1] et de classe C 1 sur
que

uor
Z π/2 ]0, π/2] avec
un +1 − un = 2 cos((2n + 1)t) cos(t) dt t2 − sin2 (t) (t − sin(t))(t + sin(t) (t3 /6)(2t) 1

0 ∀t > 0, h (t) = 2 2 = ∼ →
1 t sin (t) t2 sin2 (t) t →0 t4 t →0 3
La formule cos(a) cos (b) = (cos(a + b) + cos(a − b)) in-
2 Par un corollaire des accroissements finis, on en déduit que h
dique alors que

P
Z π/2 est de classe C 1 sur [0, π/2] avec h′ (0) = 1/3.
un +1 − un = (cos((2n + 2)t) + cos(2nt)) dt Z π/2
0
On en déduit que 4.1. On remarque que vn − un = h(t) sin (2nt) dt. D’après le
∗ 0
∀ n ∈ N , un +1 − un = 0

r
lemme de Riemann-Lebsque (en passant la partie imaginaire)
La suite (un )n≥1 est donc constante et on a donc
Z π/2

e
π lim (vn − un ) = 0

∀ n ∈ N , u n = u1 = 2 cos2 (t) dt = n →+ ∞
2
er
0

m
2. Une intégration par parties donne 4.2. Le changement de variable x = 2nt donne
 β Z β Z nπ
eimt 1 sin( x )

∀ m ∈ N , Hm = h ( t ) − h′ (t)eimt dt vn = dx
im α im α 0 x
En faisant tendre n vers +∞ (on a existence des différentes
m

Ainsi, on a
quantités avec ce qui précède) on en déduit que

r
|h(α) + h( β)| | β − α| ′ Z +∞ Z nπ
∀ m ∈ N ∗ , | Hm | ≤ + kh k∞,[α,β] sin( x ) sin( x ) π
m m dx = lim dx = lim vn = lim un =

o

ceci étant licite car h est continue sur le segment [α, β]. On a 0 x n →+ ∞ 0 x n →+ ∞ n →+ ∞ 2
or

F
✉: mamouni.myismail@gmail.com

276
i

r
s
✍ M AMOUNI M Y I SMAIL
MAMOUNI . NEW. FR
P ROBLÈMES C ORRIGÉS -MP

i
s

s
u
Z +∞ donc une fonction bornée sur R + . Soit M un majorant de son
sin(t)

s
Une troisième façon de calculer I = dt. module.
0 t Z u 2
1 − e−u

é
1. Un simple calcul de dérivée montre que − yu
→ 0

u
| K2 | ≤ M e dy = M
e−αx 0 u u →+ ∞
x 7→ − (cos( x ) + α sin( x )) 3.3. La question 2 donne

r
1 + α2 Z u Z u
sin( x )

é
− αx
est une primitive sur R de x 7→ sin( x )e . dy
dx = K1 (u) + 2
− K2 ( u )
2. Avec le théorème de Fubini (sur un pavé et utilisé avec une 0 x 0 1+y
Z u

Po r
fonctionde classe C 1 ) on a  dy

r
Z u Z u Z u  y= u Z u En faisant tendre u vers +∞ ( 2
= arctan(u)) on a
− xy sin( x ) − xy sin( x ) ux 0 1+y
J= sin( x )e dy dx = − dx = 1 − e ) dx
(alors

u
e
0 0 0 x y=0 0 x Z +∞
sin( x ) π

uor
maisZaussi Z  Z   dx =
u u u e−yx
x = u 0 x 2
J= sin( x )e− xy dx dy = − ( cos ( x ) + y sin ( x )) dy
0 0 0 1 + y2 x =0 sin2 ( x )
Z u yu 4.1. x 7 → 2
est continue sur R +∗ , prolongeable par continu-
1 − e (cos(u) + y sin(u)) x

P
= 2
dy ité en 0 (par la valeur 1) et dominée par 1/x2 (donc intégrable
0 1+y
L’égalité demandée s’en déduit. au voisinage de +∞). C’est donc une fonction intégrable sur
3.1. Comme | sin( x )/x | ≤ 1, on a R + et son intégrale sur R + existe a fortiori.
Z u 2
1 − e−u 4.2. Par un calcul similaire à celui de la question 2.2 des prélimi-

r
− xu
K1 ≤ e dx = → 0
0 u u →+ ∞ naires, on a (en primitivant sin en 1 − cos)
Z +∞ Z +∞
3.2. On a aussi sin( x ) 1 − cos( x )

e
Z u dx =
1 + y −yu dx
| K2 | ≤ e dy 0 x 0 x2
er
2
0 1+y En posant t = x/2, on en déduit que

m
1+y Z +∞ Z +∞
est continue sur R +
et de limite nulle en + ∞. C’est π 1 − cos(2t) sin2 (t)
= 2 dt = dt
1 + y2 2 0 4t2 0 t2
m

or
or

F
✉: mamouni.myismail@gmail.com

277
i

r
s
P ROBLÈMES C ORRIGÉS -MP ✍ M AMOUNI M Y I SMAIL
MAMOUNI . NEW. FR

i
s

s
u

s
é

u
r

é
Po r
r
u
uor
i
F

nn
i

P
Á la prochaine

e r
er

m
m

or
or

F
✉: mamouni.myismail@gmail.com

278

Vous aimerez peut-être aussi